Download as docx, pdf, or txt
Download as docx, pdf, or txt
You are on page 1of 92

LAGUNA LAKE DEVELOPMENT AUTHORITY vs.

COURT OF Government of Caloocan sought to be declared as the sole authority


APPEALS empowered to promote the health and safety and enhance the right of the
G.R. No. 110120 | 1994-03-16 people in Caloocan City to a balanced ecology within its territorial
ROMERO, J.: jurisdiction. 9

The clash between the responsibility of the City Government of Caloocan to On September 25, 1992, the Executive Judge of the Regional Trial Court of
dispose of the 350 tons of garbage it collects daily and the growing concern Caloocan City issued a temporary restraining order enjoining the LLDA from
and sensitivity to a pollution-free environment of the residents of Barangay enforcing its cease and desist order. Subsequently, the case was raffled to the
Camarin, Tala Estate, Caloocan City where these tons of garbage are dumped Regional Trial Court, Branch 126 of Caloocan which, at the time, was
everyday is the hub of this controversy elevated by the protagonists to the presided over by Judge Manuel Jn. Serapio of the Regional Trial Court,
Laguna Lake Development Authority (LLDA) for adjudication. Branch 127, the pairing judge of the recently-retired presiding judge.

The instant case stemmed from an earlier petition filed with this Court by The LLDA, for its part, filed on October 2, 1992 a motion to dismiss on the
Laguna Lake Development Authority (LLDA for short) docketed as G.R. No. ground, among others, that under Republic Act No. 3931, as amended by
107542 against the City Government of Caloocan, et al. In the Resolution of Presidential Decree No. 984, otherwise known as the Pollution Control Law,
November 10, 1992, this Court referred G.R. No. 107542 to the Court of the cease and desist order issued by it which is the subject matter of the
Appeals for appropriate disposition. Docketed therein as CA-G.R. SP No. complaint is reviewable both upon the law and the facts of the case by the
29449, the Court of Appeals, in a decision 1 promulgated on January 29, Court of Appeals and not by the Regional Trial Court. 10
1993 ruled that the LLDA has no power and authority to issue a cease and
desist order enjoining the dumping of garbage in Barangay Camarin, Tala On October 12, 1992 Judge Manuel Jn. Serapio issued an order consolidating
Estate, Caloocan City. The LLDA now seeks, in this petition, a review of the Civil Case No. C-15598 with Civil Case No. C-15580, an earlier case filed
decision of the Court of Appeals. by the Task Force Camarin Dumpsite entitled "Fr. John Moran, et al. vs. Hon.
Macario Asistio." The LLDA, however, maintained during the trial that the
The facts, as disclosed in the records, are undisputed. foregoing cases, being independent of each other, should have been treated
separately.
On March 8, 1991, the Task Force Camarin Dumpsite of Our Lady of
Lourdes Parish, Barangay Camarin, Caloocan City, filed a letter-complaint 2 On October 16, 1992, Judge Manuel Jn. Serapio, after hearing the motion to
with the Laguna Lake Development Authority seeking to stop the operation dismiss, issued in the consolidated cases an order 11 denying LLDA's motion
of the 8.6-hectare open garbage dumpsite in Tala Estate, Barangay Camarin, to dismiss and granting the issuance of a writ of preliminary injunction
Caloocan City due to its harmful effects on the health of the residents and the enjoining the LLDA, its agent and all persons acting for and on its behalf,
possibility of pollution of the water content of the surrounding area. from enforcing or implementing its cease and desist order which prevents
plaintiff City of Caloocan from dumping garbage at the Camarin dumpsite
On November 15, 1991, the LLDA conducted an on-site investigation, during the pendency of this case and/or until further orders of the court.
monitoring and test sampling of the leachate 3 that seeps from said dumpsite
to the nearby creek which is a tributary of the Marilao River. The LLDA On November 5, 1992, the LLDA filed a petition for certiorari, prohibition
Legal and Technical personnel found that the City Government of Caloocan and injunction with prayer for restraining order with the Supreme Court,
was maintaining an open dumpsite at the Camarin area without first securing docketed as G.R. No. 107542, seeking to nullify the aforesaid order dated
an Environmental Compliance Certificate (ECC) from the Environmental October 16, 1992 issued by the Regional Trial Court, Branch 127 of
Management Bureau (EMB) of the Department of Environment and Natural Caloocan City denying its motion to dismiss.
Resources, as required under Presidential Decree No. 1586, 4 and clearance
from LLDA as required under Republic Act No. 4850, 5 as amended by The Court, acting on the petition, issued a Resolution 12 on November 10,
Presidential Decree No. 813 and Executive Order No. 927, series of 1983. 6 1992 referring the case to the Court of Appeals for proper disposition and at
the same time, without giving due course to the petition, required the
After a public hearing conducted on December 4, 1991, the LLDA, acting on respondents to comment on the petition and file the same with the Court of
the complaint of Task Force Camarin Dumpsite, found that the water Appeals within ten (10) days from notice. In the meantime, the Court issued
collected from the leachate and the receiving streams could considerably a temporary restraining order, effective immediately and continuing until
affect the quality, in turn, of the receiving waters since it indicates the further orders from it, ordering the respondents: (1) Judge Manuel Jn.
presence of bacteria, other than coliform, which may have contaminated the Serapio, Presiding Judge, Regional Trial Court, Branch 127, Caloocan City
sample during collection or handling. 7 On December 5, 1991, the LLDA to cease and desist from exercising jurisdiction over the case for declaration
issued a Cease and Desist Order 8 ordering the City Government of of nullity of the cease and desist order issued by the Laguna Lake
Caloocan, Metropolitan Manila Authority, their contractors, and other Development Authority (LLDA); and (2) City Mayor of Caloocan and/or the
entities, to completely halt, stop and desist from dumping any form or kind City Government of Caloocan to cease and desist from dumping its garbage
of garbage and other waste matter at the Camarin dumpsite. at the Tala Estate, Barangay Camarin, Caloocan City.

The dumping operation was forthwith stopped by the City Government of Respondents City Government of Caloocan and Mayor Macario A. Asistio,
Caloocan. However, sometime in August 1992 the dumping operation was Jr. filed on November 12, 1992 a motion for reconsideration and/or to
resumed after a meeting held in July 1992 among the City Government of quash/recall the temporary restraining order and an urgent motion for
Caloocan, the representatives of Task Force Camarin Dumpsite and LLDA reconsideration alleging that ". . . in view of the calamitous situation that
at the Office of Environmental Management Bureau Director Rodrigo U. would arise if the respondent city government fails to collect 350 tons of
Fuentes failed to settle the problem. garbage daily for lack of a dumpsite (i)t is therefore, imperative that the issue
be resolved with dispatch or with sufficient leeway to allow the respondents
After an investigation by its team of legal and technical personnel on August to find alternative solutions to this garbage problem."
14, 1992, the LLDA issued another order reiterating the December 5, 1991
order and issued an Alias Cease and Desist Order enjoining the City On November 17, 1992, the Court issued a Resolution 13 directing the Court
Government of Caloocan from continuing its dumping operations at the of Appeals to immediately set the case for hearing for the purpose of
Camarin area. determining whether or not the temporary restraining order issued by the
Court should be lifted and what conditions, if any, may be required if it is to
On September 25, 1992, the LLDA, with the assistance of the Philippine be so lifted or whether the restraining order should be maintained or
National Police, enforced its Alias Cease and Desist Order by prohibiting the converted into a preliminary injunction.
entry of all garbage dump trucks into the Tala Estate, Camarin area being
utilized as a dumpsite. The Court of Appeals set the case for hearing on November 27, 1992, at
10:00 in the morning at the Hearing Room, 3rd Floor, New Building, Court
Pending resolution of its motion for reconsideration earlier filed on of Appeals. 14 After the oral argument, a conference was set on December
September 17, 1992 with the LLDA, the City Government of Caloocan filed 8, 1992 at 10:00 o'clock in the morning where the Mayor of Caloocan City,
with the Regional Trial Court of Caloocan City an action for the declaration the General Manager of LLDA, the Secretary of DENR or his duly authorized
of nullity of the cease and desist order with prayer for the issuance of a writ representative and the Secretary of DILG or his duly authorized
of injunction, docketed as Civil Case No. C-15598. In its complaint, the City representative were required to appear.
Republic Act No. 4850 and its amendatory laws, Presidential Decree No. 813
It was agreed at the conference that the LLDA had until December 15, 1992 and Executive Order No. 927, series of 1983, it is invested with the power
to finish its study and review of respondent's technical plan with respect to and authority to issue a cease and desist order pursuant to Section 4 par. (c),
the dumping of its garbage and in the event of a rejection of respondent's (d), (e), (f) and (g) of Executive Order No. 927 series of 1983 which provides,
technical plan or a failure of settlement, the parties will submit within 10 days thus:
from notice their respective memoranda on the merits of the case, after which
the petition shall be deemed submitted for resolution. 15 Notwithstanding "SECTION 4. Additional Powers and Functions. The Authority shall have
such efforts, the parties failed to settle the dispute. the following powers and functions:

On April 30, 1993, the Court of Appeals promulgated its decision holding xxx xxx xxx
that: (1) the Regional Trial Court has no jurisdiction on appeal to try, hear
and decide the action for annulment of LLDA's cease and desist order, (c ) Issue orders or decisions to compel compliance with the provisions of
including the issuance of a temporary restraining order and preliminary this Executive Order and its implementing rules and regulations only after
injunction in relation thereto, since appeal therefrom is within the exclusive proper notice and hearing.
and appellate jurisdiction of the Court of Appeals under Section 9, par. (3),
of Batas Pambansa Blg. 129; and (2) the Laguna Lake Development (d) Make, alter or modify orders requiring the discontinuance of pollution
Authority has no power and authority to issue a cease and desist order under specifying the conditions and the time within which such discontinuance
its enabling law, Republic Act No. 4850, as amended by P.D. No. 813 and must be accomplished.
Executive Order No. 927, series of 1983.
(e) Issue, renew, or deny permits, under such conditions as it may determine
The Court of Appeals thus dismissed Civil Case No. 15598 and the to be reasonable, for the prevention and abatement of pollution, for the
preliminary injunction issued in the said case was set aside; the cease and discharge of sewage, industrial waste, or for the installation or operation of
desist order of LLDA was likewise set aside and the temporary restraining sewage works and industrial disposal system or parts thereof. . . .
order enjoining the City Mayor of Caloocan and/or the City Government of
Caloocan to cease and desist from dumping its garbage at the Tala Estate, (f) After due notice and hearing, the Authority may also revoke, suspend or
Barangay Camarin, Caloocan City was lifted, subject, however, to the modify any permit issued under this Order whenever the same is necessary
condition that any future dumping of garbage in said area, shall be in to prevent or abate pollution.
conformity with the procedure and protective works contained in the
proposal attached to the records of this case and found on pages 152-160 of (g) Deputize in writing or request assistance of appropriate government
the Rollo, which was thereby adopted by reference and made an integral part agencies or instrumentalities for the purpose of enforcing this Executive
of the decision, until the corresponding restraining and/or injunctive relief is Order and its implementing rules and regulations and the orders and
granted by the proper Court upon LLDA's institution of the necessary legal decisions of the Authority."
proceedings.
The LLDA claims that the appellate court deliberately suppressed and totally
Hence, the Laguna Lake Development Authority filed the instant petition for disregarded the above provisions of Executive Order No. 927, series of 1983,
review on certiorari, now docketed as G.R. No. 110120, with prayer that the which granted administrative quasi-judicial functions to LLDA on pollution
temporary restraining order lifted by the Court of Appeals be re-issued until abatement cases.
after final determination by this Court of the issue on the proper interpretation
of the powers and authority of the LLDA under its enabling law. In light of the relevant environmental protection laws cited which are
applicable in this case, and the corresponding overlapping jurisdiction of
On July 19, 1993, the Court issued a temporary restraining order 16 enjoining government agencies implementing these laws, the resolution of the issue of
the City Mayor of Caloocan and/or the City Government of Caloocan to whether or not the LLDA has the authority and power to issue an order which,
cease and desist from dumping its garbage at the Tala Estate, Barangay in its nature and effect was injunctive, necessarily requires a determination
Camarin, Caloocan City, effective as of this date and continuing until of the threshold question: Does the Laguna Lake Development Authority,
otherwise ordered by the Court. under its Charter and its amendatory laws, have the authority to entertain the
complaint against the dumping of garbage in the open dumpsite in Barangay
It is significant to note that while both parties in this case agree on the need Camarin authorized by the City Government of Caloocan which is allegedly
to protect the environment and to maintain the ecological balance of the endangering the health, safety, and welfare of the residents therein and the
surrounding areas of the Camarin open dumpsite, the question as to which sanitation and quality of the water in the area brought about by exposure to
agency can lawfully exercise jurisdiction over the matter remains highly open pollution caused by such open garbage dumpsite?
to question.
The matter of determining whether there is such pollution of the environment
The City Government of Caloocan claims that it is within its power, as a local that requires control, if not prohibition, of the operation of a business
government unit, pursuant to the general welfare provision of the Local establishment is essentially addressed to the Environmental Management
Government Code, 17 to determine the effects of the operation of the Bureau (EMB) of the DENR which, by virtue of Section 16 of Executive
dumpsite on the ecological balance and to see that such balance is Order No. 192, series of 1987, 18 has assumed the powers and functions of
maintained. On the basis of said contention, it questioned, from the inception the defunct National Pollution Control Commission created under Republic
of the dispute before the Regional Trial Court of Caloocan City, the power Act No. 3931. Under said Executive Order, a Pollution Adjudication Board
and authority of the LLDA to issue a cease and desist order enjoining the (PAB) under the Office of the DENR Secretary now assumes the powers and
dumping of garbage in the Barangay Camarin over which the City functions of the National Pollution Control Commission with respect to
Government of Caloocan has territorial jurisdiction. adjudication of pollution cases. 19

The Court of Appeals sustained the position of the City of Caloocan on the As a general rule, the adjudication of pollution cases generally pertains to the
theory that Section 7 of Presidential Decree No. 984, otherwise known as the Pollution Adjudication Board (PAB), except in cases where the special law
Pollution Control law, authorizing the defunct National Pollution Control provides for another forum. It must be recognized in this regard that the
Commission to issue an ex-parte cease and desist order was not incorporated LLDA, as a specialized administrative agency, is specifically mandated
in Presidential Decree No. 813 nor in Executive Order No. 927, series of under Republic Act No. 4850 and its amendatory laws to carry out and make
1983. The Court of Appeals ruled that under Section 4, par. (d), of Republic effective the declared national policy 20 of promoting and accelerating the
Act No. 4850, as amended, the LLDA is instead required "to institute the development and balanced growth of the Laguna Lake area and the
necessary legal proceeding against any person who shall commence to surrounding provinces of Rizal and Laguna and the cities of San Pablo,
implement or continue implementation of any project, plan or program Manila, Pasay, Quezon and Caloocan 21 with due regard and adequate
within the Laguna de Bay region without previous clearance from the provisions for environmental management and control, preservation of the
Authority." quality of human life and ecological systems, and the prevention of undue
ecological disturbances, deterioration and pollution. Under such a broad
The LLDA now assails, in this partition for review, the abovementioned grant of power and authority, the LLDA, by virtue of its special charter,
ruling of the Court of Appeals, contending that, as an administrative agency obviously has the responsibility to protect the inhabitants of the Laguna Lake
which was granted regulatory and adjudicatory powers and functions by region from the deleterious effects of pollutants emanating from the
discharge of wastes from the surrounding areas. In carrying out the waters of the Philippines cannot be made to wait until protracted litigation
aforementioned declared policy, the LLDA is mandated, among others, to over the ultimate correctness or propriety of such orders has run its full
pass upon and approve or disapprove all plans, programs, and projects course, including multiple and sequential appeals such as those which Solar
proposed by local government offices/agencies within the region, public has taken, which of course may take several years. The relevant pollution
corporations, and private persons or enterprises where such plans, programs control statute and implementing regulations were enacted and promulgated
and/or projects are related to those of the LLDA for the development of the in the exercise of that pervasive, sovereign power to protect the safety, health,
region. 22 and general welfare and comfort of the public, as well as the protection of
plant and animal life, commonly designated as the police power. It is a
In the instant case, when the complainant Task Force Camarin Dumpsite of constitutional commonplace that the ordinary requirements of procedural due
Our Lady of Lourdes Parish, Barangay Camarin, Caloocan City, filed its process yield to the necessities of protecting vital public interests like those
letter-complaint before the LLDA, the latter's jurisdiction under its charter here involved, through the exercise of police power. . . ."
was validly invoked by complainant on the basis of its allegation that the
open dumpsite project of the City Government of Caloocan in Barangay The immediate response to the demands of "the necessities of protecting vital
Camarin was undertaken without a clearance from the LLDA, as required public interests" gives vitality to the statement on ecology embodied in the
under Section 4, par. (d), of Republic Act. No. 4850, as amended by P.D. No. Declaration of Principles and State Policies or the 1987 Constitution. Article
813 and Executive Order No. 927. While there is also an allegation that the II, Section 16 which provides:
said project was without an Environmental Compliance Certificate from the
Environmental Management Bureau (EMB) of the DENR, the primary "The State shall protect and advance the right of the people to a balanced and
jurisdiction of the LLDA over this case was recognized by the Environmental healthful ecology in accord with the rhythm and harmony of nature."
Management Bureau of the DENR when the latter acted as intermediary at
the meeting among the representatives of the City Government of Caloocan, As a constitutionally guaranteed right of every person, it carries the
Task Force Camarin Dumpsite and LLDA sometime in July 1992 to discuss correlative duty of non-impairment. This is but in consonance with the
the possibility of re-opening the open dumpsite. declared policy of the state "to protect and promote the right to health of the
people and instill health consciousness among them." 28 It is to be borne in
Having thus resolved the threshold question, the inquiry then narrows down mind that the Philippines is party to the Universal Declaration of Human
to the following issue: Does the LLDA have the power and authority to issue Rights and the Alma Conference Declaration of 1978 which recognize health
a "cease and desist" order under Republic Act No. 4850 and its amendatory as a fundamental human right. 29
laws, on the basis of the facts presented in this case, enjoining the dumping
of garbage in Tala Estate, Barangay Camarin, Caloocan City. The issuance, therefore, of the cease and desist order by the LLDA, as a
practical matter of procedure under the circumstances of the case, is a proper
The irresistible answer is in the affirmative. exercise of its power and authority under its charter and its amendatory laws.
Had the cease and desist order issued by the LLDA been complied with by
The cease and desist order issued by the LLDA requiring the City the City Government of Caloocan as it did in the first instance, no further
Government of Caloocan to stop dumping its garbage in the Camarin open legal steps would have been necessary.
dumpsite found by the LLDA to have been done in violation of Republic Act
No. 4850, as amended, and other relevant environment laws, 23 cannot be The charter of LLDA, Republic Act No. 4850, as amended, instead of
stamped as an unauthorized exercise by the LLDA of injunctive powers. By conferring upon the LLDA the means of directly enforcing such orders, has
its express terms, Republic Act No. 4850, as amended by P.D. No. 813 and provided under its Section 4 (d) the power to institute "necessary legal
Executive Order No. 927, series of 1983, authorizes the LLDA to "make, proceeding against any person who shall commence to implement or
alter or modify orders requiring the discontinuance or pollution." 24 continue implementation of any project, plan or program within the Laguna
(Underscoring for emphasis) Section 4, par. (d) explicitly authorizes the de Bay region without previous clearance from the LLDA."
LLDA to make whatever order may be necessary in the exercise of its
jurisdiction. Clearly, said provision was designed to invest the LLDA with sufficiently
broad powers in the regulation of all projects initiated in the Laguna Lake
To be sure, the LLDA was not expressly conferred the power "to issue an ex- region, whether by the government or the private sector, insofar as the
parte cease and desist order" in a language, as suggested by the City implementation of these projects is concerned. It was meant to deal with
Government of Caloocan, similar to the express grant to the defunct National cases which might possibly arise where decisions or orders issued pursuant
Pollution Control Commission under Section 7 of P.D. No. 984 which, to the exercise of such broad powers may not be obeyed, resulting in the
admittedly was not reproduced in P.D. No. 813 and E.O. No. 927, series of thwarting of its laudable objective. To meet such contingencies, then the
1983. However, it would be a mistake to draw therefrom the conclusion that writs of mandamus and injunction which are beyond the power of the LLDA
there is a denial of the power to issue the order in question when the power to issue, may be sought from the proper courts.
"to make, alter or modify orders requiring the discontinuance of pollution" is
expressly and clearly bestowed upon the LLDA by Executive Order No. 927, Insofar as the implementation of relevant anti-pollution laws in the Laguna
series of 1983. Lake region and its surrounding provinces, cities and towns are concerned,
the Court will not dwell further on the related issues raised which are more
Assuming arguendo that the authority to issue a "cease and desist order" were appropriately addressed to an administrative agency with the special
not expressly conferred by law, there is jurisprudence enough to the effect knowledge and expertise of the LLDA.
that the rule granting such authority need not necessarily be express. 25
While it is a fundamental rule that an administrative agency has only such WHEREFORE, the petition is GRANTED. The temporary restraining order
powers as are expressly granted to it by law, it is likewise a settled rule that issued by the Court on July 19, 1993 enjoining the City Mayor of Caloocan
an administrative agency has also such powers as are necessarily implied in and/or the City Government of Caloocan from dumping their garbage at the
the exercise of its express powers. 26 In the exercise, therefore, of its express Tala Estate, Barangay Camarin, Caloocan City is hereby made permanent.
powers under its charter as a regulatory and quasi-judicial body with respect
to pollution cases in the Laguna Lake region, the authority of the LLDA to SO ORDERED.
issue a "cease and desist order" is, perforce, implied. Otherwise, it may well
be reduced to a "toothless" paper agency.

In this connection, it must be noted that in Pollution Adjudication Board v.


Court of Appeals, et al., 27 the Court ruled that the Pollution Adjudication
Board (PAB) has the power to issue an ex-parte cease and desist order when
there is prima facie evidence of an establishment exceeding the allowable
standards set by the anti-pollution laws of the country. The ponente,
Associate Justice Florentino P. Feliciano, declared:

"Ex parte cease and desist orders are permitted by law and regulations in
situations like that here presented precisely because stopping the continuous
discharge of pollutive and untreated effluents into the rivers and other inland
"KAISAHAN NG MGA MANGGAGAWA SA KAHOY SA express authority of the court and in violation of section 19 of
PILIPINAS" vs. GOTAMCO SAW MILL Commonwealth Act No. 103, as amended. It is also recited in the said order
G.R. No. L-1573 | 1948-03-29 of March 28, 1947, that on that same date, January 9, 1947, respondent filed
HILADO, J: with the court another urgent motion for contempt against the petitioning
union for picketing on the premises of the respondent's saw mill and for grave
In its petition for a writ of certiorari, the "Kaisahan ng mga Manggagawa sa threats which prevented the remaining laborers from working.
Kahoy sa Pilipinas" prays, for the reasons therein set forth, that we reverse
and vacate the orders of the Court of Industrial Relations dated September Upon request of both parties, the court required the presentation of evidence
23, 1946 (Annex A) and March 28, 1947 (Annex B) and its resolution of July pertinent to the incidents thus raised. Thereafter, the said order of March 28,
11, 1947 (Annex C). 1947, was entered, and the court stated therein the three questions to be
determined as follows: first, if there was violation by the petitioning union of
In the order of September 23, 1946, it is recited that the laborers in the main the order of said court of September 23, 1946, which would warrant the
case (case No. 31-V of the Court of Industrial Relations) declared a strike on commencement of contempt proceedings; second, whether the facts and
September 10, 1946, "which suspended all the work in the respondent circumstances attending the picketing constitute contempt of court; third,
company"; that on September 19, 1946 (presumably after the case had been whether there was violation by the respondent of section 19 of
brought to the Court of Industrial Relations) said court informed the parties Commonwealth Act No. 103, as amended, in taking in four Chinese laborers
that the continuation of the strike would necessarily prejudice both parties, pending the hearing and without express authority of the court; and fourth,
and that a temporary solution, satisfactory to both parties, must be found to whether the dismissal of Maximino Millan was with or without just cause.
put an end to it, at the same time, urging both parties to be reasonable in their The court, passing upon these questions, found and held:
attitude towards each other; that ample opportunity was given to both parties
to iron out their differences until September 21, 1946, when the court "(1) That there was a violation of the order of the court dated September 23,
continued the conference at which, among other things, the leader of the 1946, by the petitioning union and thereby ordered Atty. Pastor T. Reyes,
laborers informed the court that, although said laborers were not exactly special agent of the court, to take such action as may be warranted in the
satisfied with the arrangement, in order to cooperate with the court and with premises against the person or persons responsible therefor for contempt;
the parties so that the laborers could return to work and the company resume
its operation, they had no objection to accepting a temporary settlement of "(2) That the question of picketing being closely and intimately related to the
P3.50 without meal, as against the proposal of the company of P2.00 without strike which had been found illegal, did not need to be passed upon, it being
meal; that after a series of conferences held on September 23, 1946, the date imbibed by question No. 1;
of the order now under consideration, the labor leader decided to accept a
temporary arrangement of the wage problem as proposed by the "(3) That there being no strong and clear proof on the question of respondent
management, that is, P2.00 over-all increase without meal to all striking having violated section 19 of Commonwealth Act No. 103, as amended,
laborers; that Francisco Cruz, President of the Union, manifested that he respondent was thereby exonerated from any liability in connection with the
would have a hard time convincing the laborers, but in view of their desire to alleged employment of four Chinamen;
preserve the harmony which used to exist between the parties, they were
going to accede to this proposition, provided that the management would "(4) That Maximino Millan being of troublesome nature and unworthy to
permit the laborers to bring with them home, if available, small pieces of work among his fellow laborers, his petition for reinstatement contained in
lumber to be utilized as firewood; that the negotiations culminated in an demand No. 5 of the main case was thereby denied."
agreement by which the laborers would return to their work on Tuesday, The above cited resolution of July 11, 1947, was entered by the Court of
September 24, 1946, at 7:00 o'clock in the morning, and the respondent Industrial Relations, sitting in banc, and denied reconsideration of its order
company would resume its operation on said date under the following of March 28, 1947, as requested by the petitioning union. In the course of
conditions: said resolution, the union's contention is recited that the provisions of section
19 of Commonwealth Act No. 103, as amended, upon which the order of
"(1) That all the laborers and workingmen will receive an overall increase of September 23, 1946, was based, had not been complied with; in other words,
P2.00 daily, without meal, over the wages received by them before the strike; that the said order was not issued in conformity with the requisites of the said
section, because, it was said, before its issuance there had been no proper
"(2) That the management will permit the laborers to bring with them home, hearing and there was no express finding by the court that public interest
if available, small pieces of lumber to be utilized as firewood; and required the return of the striking workers. The further contention is therein
recited that, granting that the order of September 23, 1946, was issued in
"(3) That the foregoing increase and privilege will take effect upon the return conformity with said section 19, said provision is unconstitutional for being
of the workingmen to work until the final determination of the present in violation of the organic proscription of involuntary servitude. Passing
controversy." upon these contentions, the Court of Industrial Relations said:

The same order then proceeds as follows: "The order of September 23, 1946, was issued in conformity with the
provisions of section 19. Said order was proposed and issued on the basis of
"Finding the above temporary agreement between the parties to be reasonable the agreement entered into by the parties after the preliminary hearings and
and advantageous to both, the court approves the same and orders the striking conferences. While it is true that the order of the Court now in question did
laborers of the respondent company to return to their work on Tuesday, not make any express finding as to whether public interest required the return
September 24, 1946 at 7:00 o'clock in the morning, and the respondent of the striking workers, it is undeniable, however, that until the present the
company to resume its operation and admit the striking laborers. The main case has not been decided or settled in view of the numerous incidents
respondent company is enjoined not to lay-off, suspend or dismiss any arising therefrom since the certification of the dispute by the Department of
laborer affiliated with the petitioning union, nor suspend the operation of the Labor to the Court on September 14, 1946. The fact that the Court did not
temporary agreement, and the labor union is enjoined not to stage a walk-out decide nor settle the dispute promptly, need not be stated in the said order
or strike during the pendency of the hearing." because it is a fact which is borne out by the entire record of the case. If the
petitioner was aggrieved by the terms of the order, it could have objected
From the order of March 27, 1947, it appears that on January 7, 1947, the right then and there and could have appealed said order within the period
respondent Gotamco Saw Mill filed with the Court of Industrial Relations an prescribed by law, and not to wait after it had become final, definite, and
urgent motion asking that the petitioning union be held for contempt of court conclusive. The record shows that the petitioner in its answer and counter-
for having staged a strike during the pendency of the main case "in violation petition for contempt based its complaint upon section 19 (Incidental Case
of the order of this court dated September 23, 1946"; that on January 9, 1947, No. 31-V [4]). It is, indeed, strange that after taking advantage of this order
petitioner filed an answer with a counter-petition alleging, among other and enjoyed (enjoying) the benefits thereunder, the petitioner now comes to
things, that a representative of petitioner conferred with respondent regarding impugn and challenge the validity. The second motion for reconsideration is
certain discriminations obtaining in the respondent's saw mill, but instead of a sad instance where the petitioner attacks the validity of an order under
entertaining their grievances said respondent in a haughty and arbitrary which it once took shelter.
manner ordered the stoppage of the work and consequently the workers did
then and there stop working; and in the counter-petition said petitioner asked "The court believes that section 19 is constitutional. To start with, this section
that the respondent be held for contempt for having employed four new is presumed to be constitutional. Several laws promulgated which apparently
Chinese laborers during the pendency of the hearing of the main case, without infringe the human rights of individuals were `subjected to regulation by the
State basically in the exercise of its paramount police power'. The provisions Commonwealth Act No. 103, in providing for an order of the court for the
of Act No. 103 were inspired by the constitutional injunction making it the return of striking workers, authorizes such order, among other cases, "when
concern of the State to promote social justice to insure the well being and the dispute can not, in its opinion, be promptly decided or settled". The
economic security of all the people. In order to attain this object, section 19 provision says: ". . . and if he has already done so (struck or walked out), that
was promulgated which grants to labor what it grants to capital and denies to he shall forthwith return to it, upon order of the court, which shall be issued
labor what it denies to capital. Section 19 complements the powers of the only after hearing when public interest so requires or when the dispute
Court to settle industrial disputes and renders effective such powers which cannot, in its opinion, be promptly decided or settled" (Italics supplied). In
are conferred upon it by the different provisions of the Court's organic law, other words, the order to return, if the dispute can be promptly decided or
more particularly, sections 1 and 4, and 'other plenary powers conferred upon settled, may be issued "only after hearing when public interest so requires",
the Court to enable it to settle all questions matters, controversies or disputes but if in the court's opinion the dispute cannot be promptly decided or settled,
arising between, and/or affecting employers and employees', 'to prevent non- then it is also authorized after hearing to issue the order: we construe the
pacific methods in the determination of industrial or agricultural disputes' provision to mean that the very impossibility of prompt decision or
(International HardWood and Venser Co. vs. The Pañgil Federation of settlement of the dispute confers upon the court the power to issue the order
Laborers, G. R. No. 47178, cited in the case of Mindanao Bus Co. vs. for the reason that the public has an interest in preventing undue stoppage or
Mindanao Bus Co. Employees' Association, 40 Off. Gaz., 115). Section 4 has paralyzation of the wheels of industry. And, as well stated by the court's
been upheld in the case aforecited. It appearing that the power of this Court resolution of July 11, 1947, this impossibility of prompt decision or
to execute its orders under section 19 is also the same power it possesses settlement was a fact which was borne out by the entire record of the case
under section 4 of the same act, it inferentially follows that section 19 is and did not need express statement in the order.
likewise valid. (Manila Trading and Supply Co. vs. Philippine Labor Union,
G. R. No. 47796.)" Finally, this Court is not authorized to review the findings of fact made by
the Court of Industrial Relations (Commonwealth Act No. 103, section 15,
In Manila Trading and Supply Company vs. Philippine Labor Union, supra, as amended by Commonwealth Act 559, section 2; Rule 44, Rules of Court;
this Court said: National Labor Union vs. Phil. Match Co., 40 Off. Gaz. 8th Supp. p. 134,
Bardwell Brothers vs. Phil. Labor Union, 39 Off. Gaz. 1032; Pasumil
"In the first place, the ultimate effect of petitioner's theory is to concede to Workers' Union vs. Court of Industrial Relations, 40 Off. Gaz. 6th Supp., p.
the Court of Industrial Relations the power to decide a case under section 19 71).
but deny it, the power to execute its decisions thereon. The absurdity of this
proposition is too evident to require argument. In the second place, However, Mr. Justice Briones thinks that we should expressly reserve our
considering that the jurisdiction of the Court of Industrial Relations under opinion on the constitutionality of the above statutory and reglementary
section 19 is merely incidental to the same jurisdiction it has previously provisions should it, in the future, become necessary to decide it.
acquired under section 4 of the law, if follows that the power to execute its For all these considerations, the orders and resolution of the Court of
orders under section 19 is also the same power that it possesses under section Industrial Relations assailed by the instant petition are hereby affirmed, with
4." (40 Off. Gaz., [14th Supp.], No. 23, p. 178.) costs against petitioner-appellant. So ordered.

Among the powers thus conferred is that to punish a violation of an order


such as those now under consideration as for contempt of court.

We agree with the Court of Industrial Relations that section 19 of


Commonwealth Act No. 103 is constitutional. It does not offend against the
constitutional inhibition prescribing involuntary servitude. An employee
entering into a contract of employment after said law went into effect,
voluntarily accepts, among other conditions, those prescribed in said section
19, among which is the "implied condition that when any dispute between
the employer or landlord and the employee, tenant or laborer has been
submitted to the Court of Industrial Relations for settlement or arbitration,
pursuant to the provisions of this Act, and pending award or decision by it,
the employee, tenant or laborer shall not strike or walk out of his employment
when so enjoined by the court after hearing and when public interest so
requires, and if he has already done so, that he shall forthwith return to it,
upon order of the court, which shall be issued only after hearing when public
interest so requires or when the dispute can not, in its opinion, be promptly
decided or settled . . .". (Italics supplied.) The voluntariness of the employee's
entering into such a contract of employment - he has a free choice between
entering into it or not - with such an implied condition, negatives the
possibility of involuntary servitude ensuing. The resolution of July 11, 1947,
states that the order of - September 23, 1946, was issued after a series of
preliminary hearings or conferences, and we are satisfied that these were
"hearings" within the meaning of the above mentioned section 19 of the law.
The record certainly reveals that what was done during and what resulted
from said preliminary hearings or conferences were reported to the court at a
formal hearing. As to public interest requiring that the court enjoin the strike
or walk out, or the return of striking laborers, aside from the legal
presumption that the Court of Industrial Relations complied with the
provisions of the law in this respect, we think that, considering the
universally known fact, of which this Court takes judicial notice, that as a
result of the destructions wrought by the late war, the economic and social
rehabilitation of this country urgently demands the reconstruction of
industrial, commercial and residential buildings, which in turn necessitates
building materials, in which lumber figures prominently among the most
vital, public interest of a most real and positive character has attached to the
lumber business. It is obvious that any undue stoppage or diminution in the
production of lumber or allied products so sorely needed in reconstruction
work will inevitably tend to paralyze, impede or slow down the country's
program of rehabilitation which, for obvious and natural reasons, the
government is striving to accelerate as much as is humanly possible.
Besides, the order of the court was for the striking workers to return to their
work. And that order was made after hearing, and, moreover, section 19 of
BOROVSKY vs. THE COMMISSIONER OF IMMIGRATION Aliens illegally staying in the Philippines have no right of asylum therein
G.R. No. L-4352 | 1951-09-28 (Soewapadji vs. Wixon, Sept. 13, 1946, 157 F. ed., 289, 290), even if they
TUASON, J.: are "stateless," which the petitioner claims to be. It is no less true however as
impliedly stated in this Court's decision, supra, and numerous American
This is a second petition for habeas corpus filed by the petitioner with this decisons, that foreign nationals, not enemy, against whom no criminal
Court, the first having been denied in a decision promulgated on June 30, charges have been formally made or judicial order issued, may not
1949. indefinitely be kept in detention. The protection against deprivation of liberty
without due process of law and except for crimes committed against the laws
Victor A. Borovsky, the petitioner, claims to be a stateless citizen, born in of the land is not limited to Philippine citizens but extends to all residents,
Shanghai, China, of Russian parentage. He came to the Philippines in 1936 except enemy aliens, regardless of nationality. Whether an alien who entered
and had resided therein ever since, if the period of his detention be included. the country in violation of its immigration laws may be detained for as long
as the Government is unable to deport him, is beside the point and we need
On June 24, 1946, by order of the Commissioner of Immigration, the not decide. There is no allegation that the petitioner's entry into the
petitioner was arrested for investigation as to his past activities. Following Philippines was not lawful; on the contrary, the inference from the pleadings
his arrest, a warrant for his deportation was issued by the Deportation Board, and the Deportation Board's findings is that he came to and lived in this
which is said to have found him an undesirable alien, a vagrant and habitual country under legal permit.
drunkard. The petitioner protests that he was not given a hearing, nor
informed of the charges preferred against him. This point however is Moroever, by its Constitution (Art. II, sec. 3) the Philippines "adopts the
unimportant in this proceeding. generally accepted principles of international law as part of the law of
Nation." And in a resolution entitled "Universal Declaration of Human
In May, 1947, the petitioner was put on board a ship which took him to Rights" and approved by the General Assembly of the United Nations of
Shanghai, but he was not allowed to land there because he was not a national which the Philippines is a member, at its plenary meeting on December 10,
of China and was not provided with an entry visa. He was therefore brought 1948, the right to life and liberty and all other fundamental rights as applied
back to Manila and was confined to the new Bilibid Prison in Muntinlupa to all human beings were proclaimed. It was there resolved that "All human
until December 8, 1947, when he was granted provisional release by the beings are born free and equal in degree and rights" (Art. 1); that "Everyone
President through the Secretary of Justice for a period of six months. Before is entitled to all the rights and freedom set forth in this Declaration, without
the expiration of that period, namely, on March 20, 1948, the Commissioner distinction of any kind, such as race, colour, sex, language, religion, political
of Immigration caused his rearrest and he has been in confinement in the or other opinion, nationality or social origin, property, birth, or other status
abovementioned prison ever since. (Art. 2); that "Everyone has the right to an effective remedy by the competent
national tribunals for acts violating the fundamental rights granted him by
In his return to the writ, the Solicitor General in behalf of the respondents the Constitution or by law" (Art. 8); that "No one shall be subjected to
alleges that the Commissioner of Immigration "has availed of every arbitrary arrest, detention or exile" (Art. 9); etc.
opportunity presented to carry out the deportation order as shown by the fact
that when the petitioner was enjoying his provisional release after the In U. S. vs. Nichols, 47 Fed. Supp., 201, it was said that the court "has the
unsuccessful attempt to deport him to Shanghai, China, he was again re- power to release from custody an alien who has been detained an
arrested and flown to Cebu for the purpose of placing him on board a Russian unreasonably long period of time by the Department of Justice after it has
vessel which had called at the port, with a view to carrying out the deportation become apparent that although a warrant for his deportation has been issued,
order issued against him, but said deportation was not carried out for the the warrant can not be effectuated;" that "the theory on which the court is
reason that the captain of the said boat refused to take on board the herein given the power to act is that the warrant of deportation, not having been able
petitioner on the ground that he had no permission from the Russian to be executed, is functus officio and the alien is being held without any
government to take on board the petitioner." It is further alleged that "the authority of law." The decision cited several cases which, it said, settled the
immigration officials have taken steps regarding the disposition of those matter definitely in that jurisdiction, adding that the same result had been
foreigners subject to deportation while awaiting availability of transportation reached in innumerable cases elsewhere. The cases referred to were United
or arrangements to the place where they may be sent." States ex rel. Ross vs. Wallis, 2 Cir. 279 F. 401, 404; Caranica vs. Nagle, 9
Cir., 28 F. 2d 955; Saksagansky vs. Weedin, 9 Cir., 53 F. 2d 13, 16 last
In this Court's majority decision on the first application it was observed that paragraph; Ex parte Matthews, D.C.W.D. Wash., 277 F. 857; Moraitis vs.
the applicant's detention was temporary, and it was held that "temporary Delany, D.C. Md. Aug. 28, 1942, 46 F. Supp. 425.
detention is a necessary step in the process of exclusion or expulsion of
undesirable aliens and that pending arrangements for his deportation, the The most recent case, as far as we have been able to find, was that of
Government has the right to hold the undesirable alien under confinement for Staniszewski vs. Watkins (1948), 80 Fed. Supp. 132, which is nearly
a reasonable length of time." It took note of the fact that "this Government foursquare with the case at hand. In that case a stateless person, formerly a
desires to expel the alien, and does not relish keeping him at the people's Polish national, resident in the United States since 1911 and many times
expense . . . making efforts to carry out the decree of exclusion by the highest serving as a seaman on American vessels both in peace and in war, was
officer of the land." No period was fixed within which the immigration ordered excluded from the United States and detained at Ellis Island at the
authorities were to carry out the contemplated deportation beyond the expense of the steamship company, when he returned from a voyage on
statement that "The meaning of 'reasonable time' depends upon the which he had shipped from New York for one or more European ports and
circumstances, specially the difficulties of obtaining a passport, the return to the United States. The grounds for his exclusion were that he had
availability of transportation, the diplomatic arrangements with the no passport or immigration visa, and that in 1937 had been convicted of
governments concerned and the efforts displayed to send the deportee away," perjury because in certain documents he represented himself to be an
but the Court warned that "under established precedents, too long a detention American citizen. Upon his application for release on habeas corpus, the
may justify the issuance of a writ of habeas corpus." Court released him upon his own recognizance. Judge Leibell, of the United
States District Court for the Southern District of New York, said in part:
Mr. Justice Paras, now Chief Justice, Mr. Justice Feria, Mr. Justice Perfecto,
and the writer of this decision dissented. Mr. Justice Feria and Mr. Justice "When the return to the writ of habeas corpus came before this court, I
Perfecto voted for outright discharge of the prisoner from custody. Mr. suggested that all interested parties . . . make an effort to arrange to have the
Justice Paras qualified his dissent by stating that he might agree "to a further petitioner ship out to some country that would receive him as a resident. He
detention of the herein petitioner, provided that he be released if after six is a native-born Pole but the Polish Consul has advised him in writing that he
months, the Government is still unable to deport him." This writer joined in is no longer a Polish subject. This Government does not claim that he is a
the latter dissent but thought that two months constituted reasonable time. Polish citizen. His attorney says he is stateless. The Government is willing
that he go back to the ship, but if he were sent back aboard ship and sailed to
Over two years having elapsed since the decision aforesaid was promulgated, the port (Cherbourg, France) from which he last sailed to the United States
the Government has not found ways and means of removing the petitioner he would probably be denied permission to land. There is no other country
out of the country, and none are in sight, although, it should be said in justice that would take him, without proper documents.
to the deportation authorities, it was through no fault of theirs that no ship or
country would take the petitioner. "It seems to me that this is a genuine hardship case and that the petitioner
should be released from custody on proper terms . . .
"What is to be done with the petitioner? The government has had him in implies a more than negligible risk of reversal. Indeed this experience lies
custody almost seven months and practically admits it has no place to send back of our rule permitting and practice of allowing bail where such
him out of this country. The steamship company, which employed him as questions exist, to avoid the hazard of unjustifiably imprisoning persons with
one of a group sent to the ship by the Union, with proper seaman's papers consequent reproach to our system of justice. If that is prudent judical
issued by the United States Coast Guard, is paying $3.00 a day for petitioner's practice in the ordinary case, how much more important to avoid every
board at Ellis Island. It is no fault of the steamship company that petitioner chance of handing to the Communist world such an ideological weapon as it
is an inadmissible alien as the immigration officials describe him . . . would have if this country should imprison this handful of Communist
leaders on a conviction that our own highest Court would confess to be
"I intend to sustain the writ of habeas corpus and order the release of the illegal. Risks, of course, are involved in either granting or refusing bail. I am
petitioner on his own recognizance. He will be required to inform the not naive enough to underestimate the troublemaking propensities of the
immigration officials at Ellis Island by mail on the 15th of each month stating defendants. But, with the Department of Justice alert to the dangers, the worst
where he is employed and where he can be reached by mail. If the they can accomplish in the short time it will take to end the litigation is
government does succeed in arranging for petitioner's deportation to a preferable to the possibility of national embarrassment from a celebrated case
country that will be ready to receive him as a resident, it may then advise the of unjustified imprisonment of Communist leaders. Under no circumstances
petitioner to that effect and arrange for his deportation in the manner must we permit their symbolization of an evil force in the world to be
provided by law." hallowed and glorified by any semblance of martyrdom. The way to avoid
that risk is not to jail those men until it is finally decided that they should stay
Although not binding upon this Court as a precedent, the case aforecited jailed."
offered a happy solution to the quandary in which the parties here find
themselves, solution which we think is sensible, sound and compatible with If that case is not comparable with ours on the issues presented, its underlying
law and the Constitution. For this reason, and since the Philippine law on principle is of universal application. In fact, its ratio decidendi applies with
immigration was patterned after or copied from the American law and greater force to the present petition, since the right of accused to bail pending
practice, we choose to follow and adopt the reasoning and conclusions in the appeal of his case, as in the case of the ten Communists, depends upon the
Staniszewski decision with some modifications which, it is believed, are in discretion of the court, whereas the right to be enlarged before formal charges
consonance with the prevailing conditions of peace and order in the are instituted is absolute. As already noted, not only are there no charges
Philippines. pending against the petitioner, but the prospects of bringing any against him
are slim and remote.
It was said or insinuated at the hearing of the petition at bar, but not alleged
in the return, that the petitioner was engaged in subversive activities, and fear Premises considered, the writ will issue commanding the respondents to
was expressed that he might join or aid the disloyal elements if allowed to be release the petitioner from custody upon these terms: The petitioner shall be
at large. Bearing in mind the Government's allegation in its answer that "the placed under the surveillance of the immigration authorities or their agents
herein petitioner was brought to the Philippines by the Japanese forces," and in such form and manner as may be deemed adequate to insure that he keep
the fact that Japan is no longer at war with the United States or the Philippines peace and be available when the Government is ready to deport him. The
nor identified with the countries allied against those nations, the possibility surveillance shall be reasonable and the question of reasonableness shall be
of the petitioner's entertaining or committing hostile acts prejudicial to the submitted to this Court or to the Court of First Instance of Manila for decision
interest and security of this country seems remote. in case of abuse. He shall also put up a bond for the above purpose in the
amount of P5,000.00 with sufficient surety or sureties, which bond the
If we grant, for the sake of argument, that such a possibility exists, still the Commissioner of Immigration is authorized to exact by Section 40 of
petitioner's unduly prolonged detention would be unwarranted by law and the Commonwealth Act No. 613. No costs will be charged.
Constitution, if the only purpose of the detention be to eliminate a danger that
is by no means, actual, present, or uncontrollable. After all, the Government
is not impotent to deal with or prevent any threat by such measure as that just
outlined. The thought eloquently expressed by Mr. Justice Jackson of the
United States Supreme Court in connection with the application for bail of
ten Communists convicted by a lower court of advocacy of violent overthrow
of the United States Government is, in principle pertinent and may be availed
of at this juncture. Said the learned Jurist:

"The Government's alternative contention is that defendants, by misbehavior


after conviction, have forfeited their claim to bail. Grave public danger is
said to result from what they may be expected to do, in addition to what they
have done since their conviction. If I assume that defendants are disposed to
commit every opportune disloyal act helpful to Communist countries, it is
still difficult to reconcile with traditional American law the jailing of persons
by the courts because of anticipated but as yet uncommitted crimes.
Imprisonment to protect society from predicted but unconsummated offenses
is so unprecedented in this country and so fraught with danger of excesses
and injustice that I am loath to resort to it, even as a discretionary judicial
technique to supplement conviction of such offenses as those of which
defendants stand convicted.

xxx xxx xxx

"But the right of every American to equal treatment before the law is wrapped
up in the same constitutional bundle with those of these Communists. If in
anger or disgust with these defendants we throw out the bundle, we also cast
aside protection for the liberties of more worthy critics who may be in
opposition to the government of some future day.

xxx xxx xxx

"If, however, I were to be wrong on all of these abstract or theoretical matters


of principle, there is a very practical aspect of this application which must
not be overlooked or underestimated - that is the disastrous effect on the
reputation of American justice if I should now send these men to jail and the
full Court later decide that their conviction is invalid. All experience with
litigation teaches that existence of a substantial question about a conviction
ASSOCIATION OF SMALL LANDOWNERS IN THE PHILIPPINES, enactment of R.A. No. 6657, otherwise known as the Comprehensive
INC. vs. HONORABLE SECRETARY OF AGRARIAN REFORM Agrarian Reform Law of 1988, which President Aquino signed on June 10,
G.R. Nos. 78742, 79310, 79744, and 79777 | 1989-07-14 1988. This law, while considerably changing the earlier mentioned
CRUZ, J.: enactments, nevertheless gives them suppletory effect insofar as they are not
inconsistent with its provisions. 4
In ancient mythology, Antaeus was a terrible giant who blocked and
challenged Hercules for his life on his way to Mycenae after performing his The above-captioned cases have been consolidated because they involve
eleventh labor. The two wrestled mightily and Hercules flung his adversary common legal questions, including serious challenges to the constitutionality
to the ground thinking him dead, but Antaeus rose even stronger to resume of the several measures mentioned above. They will be the subject of one
their struggle. This happened several times to Hercules' increasing common discussion and resolution. The different antecedents of each case
amazement. Finally, as they continued grappling, it dawned on Hercules that will require separate treatment, however, and will must be explained
Antaeus was the son of Gaea and could never die as long as any part of his hereunder.
body was touching his Mother Earth. Thus forewarned, Hercules then held
Antaeus up in the air, beyond the reach of the sustaining soil, and crushed G.R. No. 79777
him to death.
Squarely raised in this petition is the constitutionality of P.D. No. 27, E.O.
Mother Earth. The sustaining soil. The giver of life, without whose Nos. 228 and 229, and R.A. No. 6657.
invigorating touch even the powerful Antaeus weakened and died.
The subjects of this petition are a 9-hectare riceland worked by four tenants
The cases before us are not as fanciful as the foregoing tale. But they also tell and owned by petitioner Nicolas Manaay and his wife and a 5-hectare
of the elemental forces of life and death, of men and women who, like riceland worked by four tenants and owned by petitioner Augustin Hermano,
Antaeus, need the sustaining strength of the precious earth to stay alive. Jr. The tenants were declared full owners of these lands by E.O. No. 228 as
qualified farmers under P.D. No. 27.
"Land for the Landless" is a slogan that underscores the acute imbalance in
the distribution of this precious resource among our people. But it is more The petitioners are questioning P.D. No. 27 and E.O. Nos. 228 and 229 on
than a slogan. Through the brooding centuries, it has become a battlecry grounds inter alia of separation of powers, due process, equal protection and
dramatizing the increasingly urgent demand of the dispossessed among us the constitutional limitation that no private property shall be taken for public
for a plot of earth as their place in the sun. use without just compensation.

Recognizing this need, the Constitution in 1935 mandated the policy of social They contend that President Aquino usurped legislative power when she
justice to "insure the well-being and economic security of all the people," 1 promulgated E.O. No. 228. The said measure is invalid also for violation of
especially the less privileged. In 1973, the new Constitution affirmed this Article XIII, Section 4, of the Constitution, for failure to provide for retention
goal, adding specifically that "the State shall regulate the acquisition, limits for small landowners. Moreover, it does not conform to Article VI,
ownership, use, enjoyment and disposition of private property and equitably Section 25(4) and the other requisites of a valid appropriation.
diffuse property ownership and profits.' 2 Significantly, there was also the
specific injunction to "formulate and implement an agrarian reform program In connection with the determination of just compensation, the petitioners
aimed at emancipating the tenant from the bondage of the soil." 3 argue that the same may be made only by a court of justice and not by the
President of the Philippines. They invoke the recent cases of EPZA v. Dulay
The Constitution of 1987 was not to be outdone. Besides echoing these 5 and Manotok v. National Food Authority. 6 Moreover, the just
sentiments, it also adopted one whole and separate Article XIII on Social compensation contemplated by the Bill of Rights is payable in money or in
Justice and Human Rights, containing grandiose but undoubtedly sincere cash and not in the form of bonds or other things of value.
provisions for the uplift of the common people. These include a call in the
following words for the adoption by the State of an agrarian reform program: In considering the rentals as advance payment on the land, the executive
order also deprives the petitioners of their property rights as protected by due
SEC. 4. The State shall, by law, undertake an agrarian reform program process. The equal protection clause is also violated because the order places
founded on the right of farmers and regular farmworkers, who are landless, the burden of solving the agrarian problems on the owners only of
to own directly or collectively the lands they till or, in the case of other agricultural lands. No similar obligation is imposed on the owners of other
farmworkers, to receive a just share of the fruits thereof. To this end, the State properties.
shall encourage and undertake the just distribution of all agricultural lands,
subject to such priorities and reasonable retention limits as the Congress may The petitioners also maintain that in declaring the beneficiaries under P.D.
prescribe, taking into account ecological, developmental, or equity No. 27 to be the owners of the lands occupied by them, E.O. No. 228 ignored
considerations and subject to the payment of just compensation. In judicial prerogatives and so violated due process. Worse, the measure would
determining retention limits, the State shall respect the right of small not solve the agrarian problem because even the small farmers are deprived
landowners. The State shall further provide incentives for voluntary land- of their lands and the retention rights guaranteed by the Constitution.
sharing.
In his Comment, the Solicitor General stresses that P.D. No. 27 has already
Earlier, in fact, R.A. No. 3844, otherwise known as the Agricultural Land been upheld in the earlier cases of Chavez v. Zobel, 7 Gonzales v. Estrella, 8
Reform Code, had already been enacted by the Congress of the Philippines and Association of Rice and Corn Producers of the Philippines, Inc. v. the
on August 8, 1963, in line with the above-stated principles. This was National Land Reform council 9 The determination of just compensation by
substantially superseded almost a decade later by P.D. No. 27, which was the executive authorities conformably to the formula prescribed under the
promulgated on October 21, 1972, along with martial law, to provide for the questioned order is at best initial or preliminary only. It does not foreclose
compulsory acquisition of private lands for distribution among tenant- judicial intervention whenever sought or warranted. At any rate, the
farmers and to specify maximum retention limits for landowners. challenge to the order is premature because no valuation of their property has
as yet been made by the Department of Agrarian Reform. The petitioners are
The people power revolution of 1986 did not change and indeed even also not proper parties because the lands owned by them do not exceed the
energized the thrust for agrarian reform. Thus, on July 17, 1987, President maximum retention limit of 7 hectares.
Corazon C. Aquino issued E.O. No. 228, declaring full land ownership in
favor of the beneficiaries of P.D. No. 27 and providing for the valuation of Replying, the petitioners insist they are proper parties because P.D. No. 27
still unvalued lands covered by the decree as well as the manner of their does not provide for retention limits on tenanted lands and that in any event
payment. This was followed on July 22, 1987 by Presidential Proclamation their petition is a class suit brought in behalf of landowners with landholdings
No. 131, instituting a comprehensive agrarian reform program (CARP), and below 24 hectares. They maintain that the determination of just
E.O. No. 229, providing the mechanics for its implementation. compensation by the administrative authorities is a final ascertainment. As
for the cases invoked by the public respondent, the constitutionality of P.D.
Subsequently, with its formal organization, the revived Congress of the No. 27 was merely assumed in Chavez, while what was decided in Gonzales
Philippines took over legislative power from the President and started its own was the validity of the imposition of martial law.
deliberations, including extensive public hearings, on the improvement of the
interests of farmers. The result, after almost a year of spirited debate, was the
In the amended petition dated November 22, 1988, it is contended that P.D. NASP alleges that President Aquino had no authority to fund the Agrarian
No. 27, E.O. Nos. 228 and 229 (except Sections 20 and 21) have been Reform Program and that, in any event, the appropriation is invalid because
impliedly repealed by R.A. No. 6657. Nevertheless, this statute should itself of uncertainty in the amount appropriated. Section 2 of Proc. No. 131 and
also be declared unconstitutional because it suffers from substantially the Sections 20 and 21 of E.O. No. 229 provide for an initial appropriation of
same infirmities as the earlier measures. fifty billion pesos and thus specifies the minimum rather than the maximum
authorized amount. This is not allowed. Furthermore, the stated initial
A petition for intervention was filed with leave of court on June 1, 1988 by amount has not been certified to by the National Treasurer as actually
Vicente Cruz, owner of a 1.83-hectare land, who complained that the DAR available.
was insisting on the implementation of P.D. No. 27 and E.O. No. 228 despite
a compromise agreement he had reached with his tenant on the payment of Two additional arguments are made by Barcelona, to wit, the failure to
rentals. In a subsequent motion dated April 10, 1989, he adopted the establish by clear and convincing evidence the necessity for the exercise of
allegations in the basic amended petition that the above-mentioned the powers of eminent domain, and the violation of the fundamental right to
enactments have been impliedly repealed by R.A. No. 6657. own property.

G.R. No. 79310 The petitioners also decry the penalty for non-registration of the lands, which
is the expropriation of the said land for an amount equal to the government
The petitioners herein are landowners and sugar planters in the Victorias Mill assessor's valuation of the land for tax purposes. On the other hand, if the
District, Victorias, Negros Occidental. Co-petitioner Planters' Committee, landowner declares his own valuation, he is unjustly required to immediately
Inc. is an organization composed of 1,400 planter-members. This petition pay the corresponding taxes on the land, in violation of the uniformity rule.
seeks to prohibit the implementation of Proc. No. 131 and E.O. No. 229.
In his consolidated Comment, the Solicitor General first invokes the
The petitioners claim that the power to provide for a Comprehensive presumption of constitutionality in favor of Proc. No. 131 and E.O. No. 229.
Agrarian Reform Program as decreed by the Constitution belongs to He also justifies the necessity for the expropriation as explained in the
Congress and not the President. Although they agree that the President could "whereas" clauses of the Proclamation and submits that, contrary to the
exercise legislative power until the Congress was convened, she could do so petitioner's contention, a pilot project to determine the feasibility of CARP
only to enact emergency measures during the transition period. At that, even and a general survey on the people's opinion thereon are not indispensable
assuming that the interim legislative power of the President was properly prerequisites to its promulgation.
exercised, Proc. No. 131 and E.O. No. 229 would still have to be annulled
for violating the constitutional provisions on just compensation, due process, On the alleged violation of the equal protection clause, the sugar planters
and equal protection. have failed to show that they belong to a different class and should be
differently treated. The Comment also suggests the possibility of Congress
They also argue that under Section 2 of Proc. No. 131 which provides: first distributing public agricultural lands and scheduling the expropriation
of private agricultural lands later. From this viewpoint, the petition for
Agrarian Reform Fund. - There is hereby created a special fund, to be known prohibition would be premature.
as the Agrarian Reform Fund, an initial amount of FIFTY BILLION PESOS
(P50,000,000,000.00) to cover the estimated cost of the Comprehensive The public respondent also points out that the constitutional prohibition is
Agrarian Reform Program from 1987 to 1992 which shall be sourced from against the payment of public money without the corresponding
the receipts of the sale of the assets of the Asset Privatization Trust and appropriation. There is no rule that only money already in existence can be
Receipts of sale of ill-gotten wealth received through the Presidential the subject of an appropriation law. Finally, the earmarking of fifty billion
Commission on Good Government and such other sources as government pesos as Agrarian Reform Fund, although denominated as an initial amount,
may deem appropriate. The amounts collected and accruing to this special is actually the maximum sum appropriated. The word "initial" simply means
fund shall be considered automatically appropriated for the purpose that additional amounts may be appropriated later when necessary.
authorized in this Proclamation.
On April 11, 1988, Prudencio Serrano, a coconut planter, filed a petition on
the amount appropriated is in futuro, not in esse. The money needed to cover his own behalf, assailing the constitutionality of E.O. No. 229. In addition to
the cost of the contemplated expropriation has yet to be raised and cannot be the arguments already raised, Serrano contends that the measure is
appropriated at this time. unconstitutional because:

Furthermore, they contend that taking must be simultaneous with payment of (1) Only public lands should be included in the CARP;
just compensation as it is traditionally understood, i.e., with money and in
full, but no such payment is contemplated in Section 5 of the E.O. No. 229. (2) E.O. No. 229 embraces more than one subject which is not expressed in
On the contrary, Section 6, thereof provides that the Land Bank of the the title;
Philippines "shall compensate the landowner in an amount to be established
by the government, which shall be based on the owner's declaration of current (3) The power of the President to legislate was terminated on July 2, 1987;
fair market value as provided in Section 4 hereof, but subject to certain and
controls to be defined and promulgated by the Presidential Agrarian Reform
Council." This compensation may not be paid fully in money but in any of (4) The appropriation of a P50 billion special fund from the National
several modes that may consist of part cash and part bond, with interest, Treasury did not originate from the House of Representatives.
maturing periodically, or direct payment in cash or bond as may be mutually
agreed upon by the beneficiary and the landowner or as may be prescribed or G.R. No. 79744
approved by the PARC.
The petitioner alleges that the then Secretary of Department of Agrarian
The petitioners also argue that in the issuance of the two measures, no effort Reform, in violation of due process and the requirement for just
was made to make a careful study of the sugar planters' situation. There is no compensation, placed his landholding under the coverage of Operation Land
tenancy problem in the sugar areas that can justify the application of the Transfer. Certificates of Land Transfer were subsequently issued to the
CARP to them. To the extent that the sugar planters have been lumped in the private respondents, who then refused payment of lease rentals to him.
same legislation with other farmers, although they are a separate group with
problems exclusively their own, their right to equal protection has been On September 3, 1986, the petitioner protested the erroneous inclusion of his
violated. small landholding under Operation Land Transfer and asked for the recall
and cancellation of the Certificates of Land Transfer in the name of the
A motion for intervention was filed on August 27, 1987 by the National private respondents. He claims that on December 24, 1986, his petition was
Federation of Sugarcane Planters (NASP) which claims a membership of at denied without hearing. On February 17, 1987, he filed a motion for
least 20,000 individual sugar planters all over the country. On September 10, reconsideration, which had not been acted upon when E.O. Nos. 228 and 229
1987, another motion for intervention was filed, this time by Manuel were issued. These orders rendered his motion moot and academic because
Barcelona, et al., representing coconut and riceland owners. Both motions they directly effected the transfer of his land to the private respondents.
were granted by the Court.
The petitioner now argues that:
they derive adequate income for their family. And even assuming that the
(1) E.O. Nos. 228 and 229 were invalidly issued by the President of the petitioners do not fall under its terms, the regulations implementing P.D. No.
Philippines. 27 have already been issued, to wit, the Memorandum dated July 10, 1975
(Interim Guidelines on Retention by Small Landowners, with an
(2) The said executive orders are violative of the constitutional provision that accompanying Retention Guide Table), Memorandum Circular No. 11 dated
no private property shall be taken without due process or just compensation. April 21, 1978, (Implementation Guidelines of LOI No. 474), Memorandum
Circular No. 18-81 dated December 29, 1981 (Clarificatory Guidelines on
(3) The petitioner is denied the right of maximum retention provided for Coverage of P.D. No. 27 and Retention by Small Landowners), and DAR
under the 1987 Constitution. Administrative Order No. 1, series of 1985 (Providing for a Cut-off Date for
Landowners to Apply for Retention and/or to Protest the Coverage of their
The petitioner contends that the issuance of E.O Nos. 228 and 229 shortly Landholdings under Operation Land Transfer pursuant to P.D. No. 27). For
before Congress convened is anomalous and arbitrary, besides violating the failure to file the corresponding applications for retention under these
doctrine of separation of powers. The legislative power granted to the measures, the petitioners are now barred from invoking this right.
President under the Transitory Provisions refers only to emergency measures
that may be promulgated in the proper exercise of the police power. The public respondent also stresses that the petitioners have prematurely
initiated this case notwithstanding the pendency of their appeal to the
The petitioner also invokes his rights not to be deprived of his property President of the Philippines. Moreover, the issuance of the implementing
without due process of law and to the retention of his small parcels of rules, assuming this has not yet been done, involves the exercise of discretion
riceholding as guaranteed under Article XIII, Section 4 of the Constitution. which cannot be controlled through the writ of mandamus. This is especially
He likewise argues that, besides denying him just compensation for his land, true if this function is entrusted, as in this case, to a separate department of
the provisions of E.O. No. 228 declaring that: the government.

Lease rentals paid to the landowner by the farmer-beneficiary after October In their Reply, the petitioners insist that the above-cited measures are not
21, 1972 shall be considered as advance payment for the land. applicable to them because they do not own more than seven hectares of
agricultural land. Moreover, assuming arguendo that the rules were intended
is an unconstitutional taking of a vested property right. It is also his to cover them also, the said measures are nevertheless not in force because
contention that the inclusion of even small landowners in the program along they have not been published as required by law and the ruling of this Court
with other landowners with lands consisting of seven hectares or more is in Tañada v. Tuvera. 10 As for LOI 474, the same is ineffective for the
undemocratic. additional reason that a mere letter of instruction could not have repealed the
presidential decree.
In his Comment, the Solicitor General submits that the petition is premature
because the motion for reconsideration filed with the Minister of Agrarian I
Reform is still unresolved. As for the validity of the issuance of E.O. Nos.
228 and 229, he argues that they were enacted pursuant to Section 6, Article Although holding neither purse nor sword and so regarded as the weakest of
XVIII of the Transitory Provisions of the 1987 Constitution which reads: the three departments of the government, the judiciary is nonetheless vested
with the power to annul the acts of either the legislative or the executive or
The incumbent president shall continue to exercise legislative powers until of both when not conformable to the fundamental law. This is the reason for
the first Congress is convened. what some quarters call the doctrine of judicial supremacy. Even so, this
power is not lightly assumed or readily exercised. The doctrine of separation
On the issue of just compensation, his position is that when P.D. No. 27 was of powers imposes upon the courts a proper restraint, born of the nature of
promulgated on October 21, 1972, the tenant-farmer of agricultural land was their functions and of their respect for the other departments, in striking down
deemed the owner of the land he was tilling. The leasehold rentals paid after the acts of the legislative and the executive as unconstitutional. The policy,
that date should therefore be considered amortization payments. indeed, is a blend of courtesy and caution. To doubt is to sustain. The theory
is that before the act was done or the law was enacted, earnest studies were
In his Reply to the public respondents, the petitioner maintains that the made by Congress or the President, or both, to insure that the Constitution
motion he filed was resolved on December 14, 1987. An appeal to the Office would not be breached.
of the President would be useless with the promulgation of E.O. Nos. 228
and 229, which in effect sanctioned the validity of the public respondent's In addition, the Constitution itself lays down stringent conditions for a
acts. declaration of unconstitutionality, requiring therefor the concurrence of a
majority of the members of the Supreme Court who took part in the
G.R. No. 78742 deliberations and voted on the issue during their session en banc. 11 And as
established by judge-made doctrine, the Court will assume jurisdiction over
The petitioners in this case invoke the right of retention granted by P.D. No. a constitutional question only if it is shown that the essential requisites of a
27 to owners of rice and corn lands not exceeding seven hectares as long as judicial inquiry into such a question are first satisfied. Thus, there must be an
they are cultivating or intend to cultivate the same. Their respective lands do actual case or controversy involving a conflict of legal rights susceptible of
not exceed the statutory limit but are occupied by tenants who are actually judicial determination, the constitutional question must have been
cultivating such lands. opportunely raised by the proper party, and the resolution of the question is
unavoidably necessary to the decision of the case itself. 12
According to P.D. No. 316, which was promulgated in implementation of
P.D. No. 27: With particular regard to the requirement of proper party as applied in the
cases before us, we hold that the same is satisfied by the petitioners and
No tenant-farmer in agricultural lands primarily devoted to rice and corn intervenors because each of them has sustained or is in danger of sustaining
shall be ejected or removed from his farmholding until such time as the an immediate injury as a result of the acts or measures complained of. 13
respective rights of the tenant-farmers and the landowner shall have been And even if, strictly speaking, they are not covered by the definition, it is still
determined in accordance with the rules and regulations implementing P.D. within the wide discretion of the Court to waive the requirement and so
No. 27. remove the impediment to its addressing and resolving the serious
constitutional questions raised.
The petitioners claim they cannot eject their tenants and so are unable to
enjoy their right of retention because the Department of Agrarian Reform has In the first Emergency Powers Cases, 14 ordinary citizens and taxpayers were
so far not issued the implementing rules required under the above-quoted allowed to question the constitutionality of several executive orders issued
decree. They therefore ask the Court for a writ of mandamus to compel the by President Quirino although they were invoking only an indirect and
respondent to issue the said rules. general interest shared in common with the public. The Court dismissed the
objection that they were not proper parties and ruled that "the transcendental
In his Comment, the public respondent argues that P.D. No. 27 has been importance to the public of these cases demands that they be settled promptly
amended by LOI 474 removing any right of retention from persons who own and definitely, brushing aside, if we must, technicalities of procedure." We
other agricultural lands of more than 7 hectares in aggregate area or lands have since then applied this exception in many other cases. 15
used for residential, commercial, industrial or other purposes from which
The other above-mentioned requisites have also been met in the present It should follow that the specific constitutional provisions invoked, to wit,
petitions. Section 24 and Section 25(4) of Article VI, are not applicable. With particular
reference to Section 24, this obviously could not have been complied with
In must be stressed that despite the inhibitions pressing upon the Court when for the simple reason that the House of Representatives, which now has the
confronted with constitutional issues like the ones now before it, it will not exclusive power to initiate appropriation measures, had not yet been
hesitate to declare a law or act invalid when it is convinced that this must be convened when the proclamation was issued. The legislative power was then
done. In arriving at this conclusion, its only criterion will be the Constitution solely vested in the President of the Philippines, who embodied, as it were,
as God and its conscience give it the light to probe its meaning and discover both houses of Congress.
its purpose. Personal motives and political considerations are irrelevancies
that cannot influence its decision. Blandishment is as ineffectual as The argument of some of the petitioners that Proc. No. 131 and E.O. No. 229
intimidation. should be invalidated because they do not provide for retention limits as
required by Article XIII, Section 4 of the Constitution is no longer tenable.
For all the awesome power of the Congress and the Executive, the Court will R.A. No. 6657 does provide for such limits now in Section 6 of the law,
not hesitate to "make the hammer fall, and heavily," to use Justice Laurel's which in fact is one of its most controversial provisions. This section
pithy language, where the acts of these departments, or of any public official, declares:
betray the people's will as expressed in the Constitution.
Retention Limits. - Except as otherwise provided in this Act, no person may
It need only be added, to borrow again the words of Justice Laurel, that - own or retain, directly or indirectly, any public or private agricultural land,
the size of which shall vary according to factors governing a viable family-
. . . when the judiciary mediates to allocate constitutional boundaries, it does sized farm, such as commodity produced, terrain, infrastructure, and soil
not assert any superiority over the other departments; it does not in reality fertility as determined by the Presidential Agrarian Reform Council (PARC)
nullify or invalidate an act of the Legislature, but only asserts the solemn and created hereunder, but in no case shall retention by the landowner exceed five
sacred obligation assigned to it by the Constitution to determine conflicting (5) hectares. Three (3) hectares may be awarded to each child of the
claims of authority under the Constitution and to establish for the parties in landowner, subject to the following qualifications: (1) that he is at least
an actual controversy the rights which that instrument secures and guarantees fifteen (15) years of age; and (2) that he is actually tilling the land or directly
to them. This is in truth all that is involved in what is termed "judicial managing the farm; Provided, That landowners whose lands have been
supremacy" which properly is the power of judicial review under the covered by Presidential Decree No. 27 shall be allowed to keep the area
Constitution. 16 originally retained by them thereunder, further, That original homestead
grantees or direct compulsory heirs who still own the original homestead at
The cases before us categorically raise constitutional questions that this Court the time of the approval of this Act shall retain the same areas as long as they
must categorically resolve. And so we shall. continue to cultivate said homestead.

II The argument that E.O. No. 229 violates the constitutional requirement that
a bill shall have only one subject, to be expressed in its title, deserves only
We proceed first to the examination of the preliminary issues before short attention. It is settled that the title of the bill does not have to be a
resolving the more serious challenges to the constitutionality of the several catalogue of its contents and will suffice if the matters embodied in the text
measures involved in these petitions. are relevant to each other and may be inferred from the title. 20

The promulgation of P.D. No. 27 by President Marcos in the exercise of his The Court wryly observes that during the past dictatorship, every presidential
powers under martial law has already been sustained in Gonzales v. Estrella issuance, by whatever name it was called, had the force and effect of law
and we find no reason to modify or reverse it on that issue. As for the power because it came from President Marcos. Such are the ways of despots. Hence,
of President Aquino to promulgate Proc. No. 131 and E.O. Nos. 228 and 229, it is futile to argue, as the petitioners do in G.R. No. 79744, that LOI 474
the same was authorized under Section 6 of the Transitory Provisions of the could not have repealed P.D. No. 27 because the former was only a letter of
1987 Constitution, quoted above. instruction. The important thing is that it was issued by President Marcos,
whose word was law during that time.
The said measures were issued by President Aquino before July 27, 1987,
when the Congress of the Philippines was formally convened and took over But for all their peremptoriness, these issuances from the President Marcos
legislative power from her. They are not "midnight" enactments intended to still had to comply with the requirement for publication as this Court held in
pre-empt the legislature because E.O. No. 228 was issued on July 17, 1987, Tañada v. Tuvera. 21 Hence, unless published in the Official Gazette in
and the other measures, i.e., Proc. No. 131 and E.O. No. 229, were both accordance with Article 2 of the Civil Code, they could not have any force
issued on July 22, 1987. Neither is it correct to say that these measures ceased and effect if they were among those enactments successfully challenged in
to be valid when she lost her legislative power for, like any statute, they that case. (LOI 474 was published, though, in the Official Gazette dated
continue to be in force unless modified or repealed by subsequent law or November 29, 1976.)
declared invalid by the courts. A statute does not ipso facto become
inoperative simply because of the dissolution of the legislature that enacted Finally, there is the contention of the public respondent in G.R. No. 78742
it. By the same token, President Aquino's loss of legislative power did not that the writ of mandamus cannot issue to compel the performance of a
have the effect of invalidating all the measures enacted by her when and as discretionary act, especially by a specific department of the government. That
long as she possessed it. is true as a general proposition but is subject to one important qualification.
Correctly and categorically stated, the rule is that mandamus will lie to
Significantly, the Congress she is alleged to have undercut has not rejected compel the discharge of the discretionary duty itself but not to control the
but in fact substantially affirmed the challenged measures and has discretion to be exercised. In other words, mandamus can issue to require
specifically provided that they shall be suppletory to R.A. No. 6657 action only but not specific action.
whenever not inconsistent with its provisions. 17 Indeed, some portions of
the said measures, like the creation of the P50 billion fund in Section 2 of Whenever a duty is imposed upon a public official and an unnecessary and
Proc. No. 131, and Sections 20 and 21 of E.O. No. 229, have been unreasonable delay in the exercise of such duty occurs, if it is a clear duty
incorporated by reference in the CARP Law. 18 imposed by law, the courts will intervene by the extraordinary legal remedy
of mandamus to compel action. If the duty is purely ministerial, the courts
That fund, as earlier noted, is itself being questioned on the ground that it will require specific action. If the duty is purely discretionary, the courts by
does not conform to the requirements of a valid appropriation as specified in mandamus will require action only. For example, if an inferior court, public
the Constitution. Clearly, however, Proc. No. 131 is not an appropriation official, or board should, for an unreasonable length of time, fail to decide a
measure even if it does provide for the creation of said fund, for that is not particular question to the great detriment of all parties concerned, or a court
its principal purpose. An appropriation law is one the primary and specific should refuse to take jurisdiction of a cause when the law clearly gave it
purpose of which is to authorize the release of public funds from the treasury. jurisdiction, mandamus will issue, in the first case to require a decision, and
19 The creation of the fund is only incidental to the main objective of the in the second to require that jurisdiction be taken of the cause. 22
proclamation, which is agrarian reform.
And while it is true that as a rule the writ will not be proper as long as there
is still a plain, speedy and adequate remedy available from the administrative
authorities, resort to the courts may still be permitted if the issue raised is a If those who govern the District of Columbia decide that the Nation's Capital
question of law. 23 should be beautiful as well as sanitary, there is nothing in the Fifth
Amendment that stands in the way.
III
Once the object is within the authority of Congress, the right to realize it
There are traditional distinctions between the police power and the power of through the exercise of eminent domain is clear.
eminent domain that logically preclude the application of both powers at the
same time on the same subject. In the case of City of Baguio v. NAWASA, For the power of eminent domain is merely the means to the end. 28
24 for example, where a law required the transfer of all municipal waterworks
systems to the NAWASA in exchange for its assets of equivalent value, the In Penn Central Transportation Co. v. New York City, 29 decided by a 6-3
Court held that the power being exercised was eminent domain because the vote in 1978, the U.S. Supreme Court sustained the respondent's Landmarks
property involved was wholesome and intended for a public use. Property Preservation Law under which the owners of the Grand Central Terminal had
condemned under the police power is noxious or intended for a noxious not been allowed to construct a multi-story office building over the Terminal,
purpose, such as a building on the verge of collapse, which should be which had been designated a historic landmark. Preservation of the landmark
demolished for the public safety, or obscene materials, which should be was held to be a valid objective of the police power. The problem, however,
destroyed in the interest of public morals. The confiscation of such property was that the owners of the Terminal would be deprived of the right to use the
is not compensable, unlike the taking of property under the power of airspace above it although other landowners in the area could do so over their
expropriation, which requires the payment of just compensation to the owner. respective properties. While insisting that there was here no taking, the Court
nonetheless recognized certain compensatory rights accruing to Grand
In the case of Pennsylvania Coal Co. v. Mahon, 25 Justice Holmes laid down Central Terminal which it said would "undoubtedly mitigate" the loss caused
the limits of the police power in a famous aphorism: "The general rule at least by the regulation. This "fair compensation," as he called it, was explained by
is that while property may be regulated to a certain extent, if regulation goes Prof. Costonis in this wise:
too far it will be recognized as a taking." The regulation that went "too far"
was a law prohibiting mining which might cause the subsidence of structures In return for retaining the Terminal site in its pristine landmark status, Penn
for human habitation constructed on the land surface. This was resisted by a Central was authorized to transfer to neighboring properties the authorized
coal company which had earlier granted a deed to the land over its mine but but unused rights accruing to the site prior to the Terminal's designation as a
reserved all mining rights thereunder, with the grantee assuming all risks and landmark - the rights which would have been exhausted by the 59-story
waiving any damage claim. The Court held the law could not be sustained building that the city refused to countenance atop the Terminal. Prevailing
without compensating the grantor. Justice Brandeis filed a lone dissent in bulk restrictions on neighboring sites were proportionately relaxed,
which he argued that there was a valid exercise of the police power. He said: theoretically enabling Penn Central to recoup its losses at the Terminal site
by constructing or selling to others the right to construct larger, hence more
Every restriction upon the use of property imposed in the exercise of the profitable buildings on the transferee sites. 30
police power deprives the owner of some right theretofore enjoyed, and is, in
that sense, an abridgment by the State of rights in property without making The cases before us present no knotty complication insofar as the question of
compensation. But restriction imposed to protect the public health, safety or compensable taking is concerned. To the extent that the measures under
morals from dangers threatened is not a taking. The restriction here in challenge merely prescribe retention limits for landowners, there is an
question is merely the prohibition of a noxious use. The property so restricted exercise of the police power for the regulation of private property in
remains in the possession of its owner. The state does not appropriate it or accordance with the Constitution. But where, to carry out such regulation, it
make any use of it. The state merely prevents the owner from making a use becomes necessary to deprive such owners of whatever lands they may own
which interferes with paramount rights of the public. Whenever the use in excess of the maximum area allowed, there is definitely a taking under the
prohibited ceases to be noxious - as it may because of further changes in local power of eminent domain for which payment of just compensation is
or social conditions - the restriction will have to be removed and the owner imperative. The taking contemplated is not a mere limitation of the use of the
will again be free to enjoy his property as heretofore. land. What is required is the surrender of the title to and the physical
possession of the said excess and all beneficial rights accruing to the owner
Recent trends, however, would indicate not a polarization but a mingling of in favor of the farmer-beneficiary. This is definitely an exercise not of the
the police power and the power of eminent domain, with the latter being used police power but of the power of eminent domain.
as an implement of the former like the power of taxation. The employment
of the taxing power to achieve a police purpose has long been accepted. 26 Whether as an exercise of the police power or of the power of eminent
As for the power of expropriation, Prof. John J. Costonis of the University of domain, the several measures before us are challenged as violative of the due
Illinois College of Law (referring to the earlier case of Euclid v. Ambler process and equal protection clauses.
Realty Co., 272 US 365, which sustained a zoning law under the police
power) makes the following significant remarks: The challenge to Proc. No. 131 and E.O. Nos. 228 and 299 on the ground
that no retention limits are prescribed has already been discussed and
Euclid, moreover, was decided in an era when judges located the police and dismissed. It is noted that although they excited many bitter exchanges during
eminent domain powers on different planets. Generally speaking, they the deliberation of the CARP Law in Congress, the retention limits finally
viewed eminent domain as encompassing public acquisition of private agreed upon are, curiously enough, not being questioned in these petitions.
property for improvements that would be available for "public use," literally We therefore do not discuss them here. The Court will come to the other
construed. To the police power, on the other hand, they assigned the less claimed violations of due process in connection with our examination of the
intrusive task of preventing harmful externalities, a point reflected in the adequacy of just compensation as required under the power of expropriation.
Euclid opinion's reliance on an analogy to nuisance law to bolster its support
of zoning. So long as suppression of a privately authored harm bore a The argument of the small farmers that they have been denied equal
plausible relation to some legitimate "public purpose," the pertinent measure protection because of the absence of retention limits has also become
need have afforded no compensation whatever. With the progressive growth academic under Section 6 of R.A. No. 6657. Significantly, they too have not
of government's involvement in land use, the distance between the two questioned the area of such limits. There is also the complaint that they
powers has contracted considerably. Today government often employs should not be made to share the burden of agrarian reform, an objection also
eminent domain interchangeably with or as a useful complement to the police made by the sugar planters on the ground that they belong to a particular class
power - a trend expressly approved in the Supreme Court's 1954 decision in with particular interests of their own. However, no evidence has been
Berman v. Parker, which broadened the reach of eminent domain's "public submitted to the Court that the requisites of a valid classification have been
use" test to match that of the police power's standard of "public purpose." 27 violated.

The Berman case sustained a redevelopment project and the improvement of Classification has been defined as the grouping of persons or things similar
blighted areas in the District of Columbia as a proper exercise of the police to each other in certain particulars and different from each other in these same
power. On the role of eminent domain in the attainment of this purpose, particulars. 31 To be valid, it must conform to the following requirements:
Justice Douglas declared: (1) it must be based on substantial distinctions; (2) it must be germane to the
purposes of the law; (3) it must not be limited to existing conditions only;
and (4) it must apply equally to all the members of the class. 32 The Court
finds that all these requisites have been met by the measures here challenged departments in the exercise of their discretion. We are not justified in
as arbitrary and discriminatory. reviewing that discretion in the absence of a clear showing that it has been
abused.
Equal protection simply means that all persons or things similarly situated
must be treated alike both as to the rights conferred and the liabilities A becoming courtesy admonishes us to respect the decisions of the political
imposed. 33 The petitioners have not shown that they belong to a different departments when they decide what is known as the political question. As
class and entitled to a different treatment. The argument that not only explained by Chief Justice Concepcion in the case of Tañada v. Cuenco: 36
landowners but also owners of other properties must be made to share the
burden of implementing land reform must be rejected. There is a substantial The term "political question" connotes what it means in ordinary parlance,
distinction between these two classes of owners that is clearly visible except namely, a question of policy. It refers to "those questions which, under the
to those who will not see. There is no need to elaborate on this matter. In any Constitution, are to be decided by the people in their sovereign capacity; or
event, the Congress is allowed a wide leeway in providing for a valid in regard to which full discretionary authority has been delegated to the
classification. Its decision is accorded recognition and respect by the courts legislative or executive branch of the government." It is concerned with
of justice except only where its discretion is abused to the detriment of the issues dependent upon the wisdom, not legality, of a particular measure.
Bill of Rights.
It is true that the concept of the political question has been constricted with
It is worth remarking at this juncture that a statute may be sustained under the enlargement of judicial power, which now includes the authority of the
the police power only if there is a concurrence of the lawful subject and the courts "to determine whether or not there has been a grave abuse of discretion
lawful method. Put otherwise, the interests of the public generally as amounting to lack or excess of jurisdiction on the part of any branch or
distinguished from those of a particular class require the interference of the instrumentality of the Government." 37 Even so, this should not be construed
State and, no less important, the means employed are reasonably necessary as a license for us to reverse the other departments simply because their views
for the attainment of the purpose sought to be achieved and not unduly may not coincide with ours.
oppressive upon individuals. 34 As the subject and purpose of agrarian
reform have been laid down by the Constitution itself, we may say that the The legislature and the executive have been seen fit, in their wisdom, to
first requirement has been satisfied. What remains to be examined is the include in the CARP the redistribution of private landholdings (even as the
validity of the method employed to achieve the constitutional goal. distribution of public agricultural lands is first provided for, while also
continuing space under the Public Land Act and other cognate laws). The
One of the basic principles of the democratic system is that where the rights Court sees no justification to interpose its authority, which we may assert
of the individual are concerned, the end does not justify the means. It is not only if we believe that the political decision is not unwise, but illegal. We do
enough that there be a valid objective; it is also necessary that the means not find it to be so.
employed to pursue it be in keeping with the Constitution. Mere expediency
will not excuse constitutional shortcuts. There is no question that not even In U.S. v. Chandler-Dunbar Water Power Company, 38 it was held:
the strongest moral conviction or the most urgent public need, subject only
to a few notable exceptions, will excuse the bypassing of an individual's Congress having determined, as it did by the Act of March 3, 1909 that the
rights. It is no exaggeration to say that a person invoking a right guaranteed entire St. Mary's river between the American bank and the international line,
under Article III of the Constitution is a majority of one even as against the as well as all of the upland north of the present ship canal, throughout its
rest of the nation who would deny him that right. entire length, was "necessary for the purpose of navigation of said waters,
and the waters connected therewith," that determination is conclusive in
That right covers the person's life, his liberty and his property under Section condemnation proceedings instituted by the United States under that Act, and
1 of Article III of the Constitution. With regard to his property, the owner there is no room for judicial review of the judgment of Congress . . .
enjoys the added protection of Section 9, which reaffirms the familiar rule
that private property shall not be taken for public use without just As earlier observed, the requirement for public use has already been settled
compensation. for us by the Constitution itself. No less than the 1987 Charter calls for
agrarian reform, which is the reason why private agricultural lands are to be
This brings us now to the power of eminent domain. taken from their owners, subject to the prescribed maximum retention limits.
The purposes specified in P.D. No. 27, Proc. No. 131 and R.A. No. 6657 are
IV only an elaboration of the constitutional injunction that the State adopt the
necessary measures "to encourage and undertake the just distribution of all
Eminent domain is an inherent power of the State that enables it to forcibly agricultural lands to enable farmers who are landless to own directly or
acquire private lands intended for public use upon payment of just collectively the lands they till." That public use, as pronounced by the
compensation to the owner. Obviously, there is no need to expropriate where fundamental law itself, must be binding on us.
the owner is willing to sell under terms also acceptable to the purchaser, in
which case an ordinary deed of sale may be agreed upon by the parties. 35 It The second requirement, i.e., the payment of just compensation, needs a
is only where the owner is unwilling to sell, or cannot accept the price or longer and more thoughtful examination.
other conditions offered by the vendee, that the power of eminent domain
will come into play to assert the paramount authority of the State over the Just compensation is defined as the full and fair equivalent of the property
interests of the property owner. Private rights must then yield to the taken from its owner by the expropriator. 39 It has been repeatedly stressed
irresistible demands of the public interest on the time-honored justification, by this Court that the measure is not the taker's gain but the owner's loss. 40
as in the case of the police power, that the welfare of the people is the The word "just" is used to intensify the meaning of the word "compensation"
supreme law. to convey the idea that the equivalent to be rendered for the property to be
taken shall be real, substantial, full, ample. 41
But for all its primacy and urgency, the power of expropriation is by no
means absolute (as indeed no power is absolute). The limitation is found in It bears repeating that the measures challenged in these petitions contemplate
the constitutional injunction that "private property shall not be taken for more than a mere regulation of the use of private lands under the police
public use without just compensation" and in the abundant jurisprudence that power. We deal here with an actual taking of private agricultural lands that
has evolved from the interpretation of this principle. Basically, the has dispossessed the owners of their property and deprived them of all its
requirements for a proper exercise of the power are: (1) public use and (2) beneficial use and enjoyment, to entitle them to the just compensation
just compensation. mandated by the Constitution.

Let us dispose first of the argument raised by the petitioners in G.R. No. As held in Republic of the Philippines v. Castellvi, 42 there is compensable
79310 that the State should first distribute public agricultural lands in the taking when the following conditions concur: (1) the expropriator must enter
pursuit of agrarian reform instead of immediately disturbing property rights a private property; (2) the entry must be for more than a momentary period;
by forcibly acquiring private agricultural lands. Parenthetically, it is not (3) the entry must be under warrant or color of legal authority; (4) the
correct to say that only public agricultural lands may be covered by the CARP property must be devoted to public use or otherwise informally appropriated
as the Constitution calls for "the just distribution of all agricultural lands." In or injuriously affected; and (5) the utilization of the property for public use
any event, the decision to redistribute private agricultural lands in the manner must be in such a way as to oust the owner and deprive him of beneficial
prescribed by the CARP was made by the legislative and executive
enjoyment of the property. All these requisites are envisioned in the measures minor bureaucrat or clerk to absolutely prevail over the judgment of a court
before us. promulgated only after expert commissioners have actually viewed the
property, after evidence and arguments pro and con have been presented, and
Where the State itself is the expropriator, it is not necessary for it to make a after all factors and considerations essential to a fair and just determination
deposit upon its taking possession of the condemned property, as "the have been judiciously evaluated.
compensation is a public charge, the good faith of the public is pledged for
its payment, and all the resources of taxation may be employed in raising the A reading of the aforecited Section 16(d) will readily show that it does not
amount." 43 Nevertheless, Section 16(e) of the CARP Law provides that: suffer from the arbitrariness that rendered the challenged decrees
constitutionally objectionable. Although the proceedings are described as
Upon receipt by the landowner of the corresponding payment or, in case of summary, the landowner and other interested parties are nevertheless allowed
rejection or no response from the landowner, upon the deposit with an an opportunity to submit evidence on the real value of the property. But more
accessible bank designated by the DAR of the compensation in cash or in importantly, the determination of the just compensation by the DAR is not
LBP bonds in accordance with this Act, the DAR shall take immediate by any means final and conclusive upon the landowner or any other interested
possession of the land and shall request the proper Register of Deeds to issue party, for Section 16(f) clearly provides:
a Transfer Certificate of Title (TCT) in the name of the Republic of the
Philippines. The DAR shall thereafter proceed with the redistribution of the Any party who disagrees with the decision may bring the matter to the court
land to the qualified beneficiaries. of proper jurisdiction for final determination of just compensation.

Objection is raised, however, to the manner of fixing the just compensation, The determination made by the DAR is only preliminary unless accepted by
which it is claimed is entrusted to the administrative authorities in violation all parties concerned. Otherwise, the courts of justice will still have the right
of judicial prerogatives. Specific reference is made to Section 16(d), which to review with finality the said determination in the exercise of what is
provides that in case of the rejection or disregard by the owner of the offer of admittedly a judicial function.
the government to buy his land -
The second and more serious objection to the provisions on just
. . . the DAR shall conduct summary administrative proceedings to determine compensation is not as easily resolved.
the compensation for the land by requiring the landowner, the LBP and other
interested parties to submit evidence as to the just compensation for the land, This refers to Section 18 of the CARP Law providing in full as follows:
within fifteen (15) days from the receipt of the notice. After the expiration of
the above period, the matter is deemed submitted for decision. The DAR shall SEC. 18. Valuation and Mode of Compensation. - The LBP shall compensate
decide the case within thirty (30) days after it is submitted for decision. the landowner in such amount as may be agreed upon by the landowner and
the DAR and the LBP, in accordance with the criteria provided for in
To be sure, the determination of just compensation is a function addressed to Sections 16 and 17, and other pertinent provisions hereof, or as may be finally
the courts of justice and may not be usurped by any other branch or official determined by the court, as the just compensation for the land.
of the government. EPZA v. Dulay 44 resolved a challenge to several decrees
promulgated by President Marcos providing that the just compensation for The compensation shall be paid in one of the following modes, at the option
property under expropriation should be either the assessment of the property of the landowner:
by the government or the sworn valuation thereof by the owner, whichever
was lower. In declaring these decrees unconstitutional, the Court held (1) Cash payment, under the following terms and conditions:
through Mr. Justice Hugo E. Gutierrez, Jr.:
(a) For lands above fifty (50) hectares, insofar as the excess hectarage is
The method of ascertaining just compensation under the aforecited decrees concerned - Twenty-five percent (25%) cash, the balance to be paid in
constitutes impermissible encroachment on judicial prerogatives. It tends to government financial instruments negotiable at any time.
render this Court inutile in a matter which under this Constitution is reserved
to it for final determination. (b) For lands above twenty-four (24) hectares and up to fifty (50) hectares -
Thirty percent (30%) cash, the balance to be paid in government financial
Thus, although in an expropriation proceeding the court technically would instruments negotiable at any time.
still have the power to determine the just compensation for the property,
following the applicable decrees, its task would be relegated to simply stating (c) For lands twenty-four (24) hectares and below - Thirty-five percent (35%)
the lower value of the property as declared either by the owner or the cash, the balance to be paid in government financial instruments negotiable
assessor. As a necessary consequence, it would be useless for the court to at any time.
appoint commissioners under Rule 67 of the Rules of Court. Moreover, the
need to satisfy the due process clause in the taking of private property is (2) Shares of stock in government-owned or controlled corporations, LBP
seemingly fulfilled since it cannot be said that a judicial proceeding was not preferred shares, physical assets or other qualified investments in accordance
had before the actual taking. However, the strict application of the decrees with guidelines set by the PARC;
during the proceedings would be nothing short of a mere formality or charade
as the court has only to choose between the valuation of the owner and that (3) Tax credits which can be used against any tax liability;
of the assessor, and its choice is always limited to the lower of the two. The
court cannot exercise its discretion or independence in determining what is (4) LBP bonds, which shall have the following features:
just or fair. Even a grade school pupil could substitute for the judge insofar
as the determination of constitutional just compensation is concerned. (a) Market interest rates aligned with 91-day treasury bill rates. Ten percent
(10%) of the face value of the bonds shall mature every year from the date of
xxx xxx xxx issuance until the tenth (10th) year: Provided, That should the landowner
choose to forego the cash portion, whether in full or in part, he shall be paid
In the present petition, we are once again confronted with the same question correspondingly in LBP bonds;
of whether the courts under P.D. No. 1533, which contains the same
provision on just compensation as its predecessor decrees, still have the (b) Transferability and negotiability. Such LBP bonds may be used by the
power and authority to determine just compensation, independent of what is landowner, his successors-in-interest or his assigns, up to the amount of their
stated by the decree and to this effect, to appoint commissioners for such face value, for any of the following:
purpose.
(i) Acquisition of land or other real properties of the government, including
This time, we answer in the affirmative. assets under the Asset Privatization Program and other assets foreclosed by
government financial institutions in the same province or region where the
xxx xxx xxx lands for which the bonds were paid are situated;

It is violative of due process to deny the owner the opportunity to prove that (ii) Acquisition of shares of stock of government owned or controlled
the valuation in the tax documents is unfair or wrong. And it is repulsive to corporations or shares of stock owned by the government in private
the basic concepts of justice and fairness to allow the haphazard work of a corporations;
domain. This is not an ordinary expropriation where only a specific property
(iii) Substitution for surety or bail bonds for the provisional release of of relatively limited area is sought to be taken by the State from its owner for
accused persons, or for performance bonds; a specific and perhaps local purpose. What we deal with here is a
revolutionary kind of expropriation.
(iv) Security for loans with any government financial institution, provided
the proceeds of the loans shall be invested in an economic enterprise, The expropriation before us affects all private agricultural lands whenever
preferably in a small and medium-scale industry, in the same province or found and of whatever kind as long as they are in excess of the maximum
region as the land for which the bonds are paid; retention limits allowed their owners. This kind of expropriation is intended
for the benefit not only of a particular community or of a small segment of
(v) Payment for various taxes and fees to government: Provided, That the use the population but of the entire Filipino nation, from all levels of our society,
of these bonds for these purposes will be limited to a certain percentage of from the impoverished farmer to the land-glutted owner. Its purpose does not
the outstanding balance of the financial instruments; Provided, further, That cover only the whole territory of this country but goes beyond in time to the
the PARC shall determine the percentages mentioned above; foreseeable future, which it hopes to secure and edify with the vision and the
sacrifice of the present generation of Filipinos. Generations yet to come are
(vi) Payment for tuition fees of the immediate family of the original as involved in this program as we are today, although hopefully only as
bondholder in government universities, colleges, trade schools, and other beneficiaries of a richer and more fulfilling life we will guarantee to them
institutions; tomorrow through our thoughtfulness today. And, finally, let it not be
forgotten that it is no less than the Constitution itself that has ordained this
(vii) Payment for fees of the immediate family of the original bondholder in revolution in the farms, calling for "a just distribution" among the farmers of
government hospital; and lands that have heretofore been the prison of their dreams but can now
become the key at least to their deliverance.
(viii) Such other uses as the PARC may from time to time allow.
Such a program will involve not mere millions of pesos. The cost will be
The contention of the petitioners in G.R. No. 79777 is that the above tremendous. Considering the vast areas of land subject to expropriation under
provision is unconstitutional insofar as it requires the owners of the the laws before us, we estimate that hundreds of billions of pesos will be
expropriated properties to accept just compensation therefor in less than needed, far more indeed than the amount of P50 billion initially appropriated,
money, which is the only medium of payment allowed. In support of this which is already staggering as it is by our present standards. Such amount is
contention, they cite jurisprudence holding that: in fact not even fully available at this time.

The fundamental rule in expropriation matters is that the owner of the We assume that the framers of the Constitution were aware of this difficulty
property expropriated is entitled to a just compensation, which should be when they called for agrarian reform as a top priority project of the
neither more nor less, whenever it is possible to make the assessment, than government. It is a part of this assumption that when they envisioned the
the money equivalent of said property. Just compensation has always been expropriation that would be needed, they also intended that the just
understood to be the just and complete equivalent of the loss which the owner compensation would have to be paid not in the orthodox way but a less
of the thing expropriated has to suffer by reason of the expropriation. 45 conventional if more practical method. There can be no doubt that they were
(Emphasis supplied.) aware of the financial limitations of the government and had no illusions that
there would be enough money to pay in cash and in full for the lands they
In J.M. Tuazon Co. v. Land Tenure Administration, 46 this Court held: wanted to be distributed among the farmers. We may therefore assume that
their intention was to allow such manner of payment as is now provided for
It is well-settled that just compensation means the equivalent for the value of by the CARP Law, particularly the payment of the balance (if the owner
the property at the time of its taking. Anything beyond that is more, and cannot be paid fully with money), or indeed of the entire amount of the just
anything short of that is less, than just compensation. It means a fair and full compensation, with other things of value. We may also suppose that what
equivalent for the loss sustained, which is the measure of the indemnity, not they had in mind was a similar scheme of payment as that prescribed in P.D.
whatever gain would accrue to the expropriating entity. The market value of No. 27, which was the law in force at the time they deliberated on the new
the land taken is the just compensation to which the owner of condemned Charter and with which they presumably agreed in principle.
property is entitled, the market value being that sum of money which a person
desirous, but not compelled to buy, and an owner, willing, but not compelled The Court has not found in the records of the Constitutional Commission any
to sell, would agree on as a price to be given and received for such property. categorial agreement among the members regarding the meaning to be given
(Emphasis supplied.) the concept of just compensation as applied to the comprehensive agrarian
reform program being contemplated. There was the suggestion to "fine tune"
In the United States, where much of our jurisprudence on the subject has been the requirement to suit the demands of the project even as it was also felt that
derived, the weight of authority is also to the effect that just compensation they should "leave it to Congress" to determine how payment should be made
for property expropriated is payable only in money and not otherwise. Thus to the landowner and reimbursement required from the farmer-beneficiaries.
- Such innovations as "progressive compensation" and "State-subsidized
compensation" were also proposed. In the end, however, no special definition
The medium of payment of compensation is ready money or cash. The of the just compensation for the lands to be expropriated was reached by the
condemnor cannot compel the owner to accept anything but money, nor can Commission. 50
the owner compel or require the condemnor to pay him on any other basis
than the value of the property in money at the time and in the manner On the other hand, there is nothing in the records either that militates against
prescribed by the Constitution and the statutes. When the power of eminent the assumptions we are making of the general sentiments and intention of the
domain is resorted to, there must be a standard medium of payment, binding members on the content and manner of the payment to be made to the
upon both parties, and the law has fixed that standard as money in cash. 47 landowner in the light of the magnitude of the expenditure and the limitations
(Emphasis supplied.) of the expropriator.

Part cash and deferred payments are not and cannot, in the nature of things, With these assumptions, the Court hereby declares that the content and
be regarded as a reliable and constant standard of compensation. 48 manner of the just compensation provided for in the afore-quoted Section 18
of the CARP Law is not violative of the Constitution. We do not mind
"Just compensation" for property taken by condemnation means a fair admitting that a certain degree of pragmatism has influenced our decision on
equivalent in money, which must be paid at least within a reasonable time this issue, but after all this Court is not a cloistered institution removed from
after the taking, and it is not within the power of the Legislature to substitute the realities and demands of society or oblivious to the need for its
for such payment future obligations, bonds, or other valuable advantage. 49 enhancement. The Court is as acutely anxious as the rest of our people to see
(Emphasis supplied.) the goal of agrarian reform achieved at last after the frustrations and
deprivations of our peasant masses during all these disappointing decades.
It cannot be denied from these cases that the traditional medium for the We are aware that invalidation of the said section will result in the
payment of just compensation is money and no other. And so, conformably, nullification of the entire program, killing the farmer's hopes even as they
has just compensation been paid in the past solely in that medium. However, approach realization and resurrecting the spectre of discontent and dissent in
we do not deal here with the traditional exercise of the power of eminent
the restless countryside. That is not in our view the intention of the piece of land can be finally and irrevocably taken from an unwilling owner
Constitution, and that is not what we shall decree today. until compensation is paid . . . (Emphasis supplied.)

Accepting the theory that payment of the just compensation is not always It is true that P.D. No. 27 expressly ordered the emancipation of tenant-
required to be made fully in money, we find further that the proportion of farmer as October 21, 1972 and declared that he shall "be deemed the owner"
cash payment to the other things of value constituting the total payment, as of a portion of land consisting of a family-sized farm except that "no title to
determined on the basis of the areas of the lands expropriated, is not unduly the land owned by him was to be actually issued to him unless and until he
oppressive upon the landowner. It is noted that the smaller the land, the had become a full-fledged member of a duly recognized farmers'
bigger the payment in money, primarily because the small landowner will be cooperative." It was understood, however, that full payment of the just
needing it more than the big landowners, who can afford a bigger balance in compensation also had to be made first, conformably to the constitutional
bonds and other things of value. No less importantly, the government requirement.
financial instruments making up the balance of the payment are "negotiable
at any time." The other modes, which are likewise available to the landowner When E.O. No. 228, categorically stated in its Section 1 that:
at his option, are also not unreasonable because payment is made in shares
of stock, LBP bonds, other properties or assets, tax credits, and other things All qualified farmer-beneficiaries are now deemed full owners as of October
of value equivalent to the amount of just compensation. 21, 1972 of the land they acquired by virtue of Presidential Decree No. 27.
(Emphasis supplied.)
Admittedly, the compensation contemplated in the law will cause the
landowners, big and small, not a little inconvenience. As already remarked, it was obviously referring to lands already validly acquired under the said
this cannot be avoided. Nevertheless, it is devoutly hoped that these decree, after proof of full-fledged membership in the farmers' cooperatives
countrymen of ours, conscious as we know they are of the need for their and full payment of just compensation. Hence, it was also perfectly proper
forebearance and even sacrifice, will not begrudge us their indispensable for the Order to also provide in its Section 2 that the "lease rentals paid to the
share in the attainment of the ideal of agrarian reform. Otherwise, our pursuit landowner by the farmer-beneficiary after October 21, 1972 (pending
of this elusive goal will be like the quest for the Holy Grail. transfer of ownership after full payment of just compensation), shall be
considered as advance payment for the land."
The complaint against the effects of non-registration of the land under E.O.
No. 229 does not seem to be viable any more as it appears that Section 4 of The CARP Law, for its part, conditions the transfer of possession and
the said Order has been superseded by Section 14 of the CARP Law. This ownership of the land to the government on receipt by the landowner of the
repeats the requisites of registration as embodied in the earlier measure but corresponding payment or the deposit by the DAR of the compensation in
does not provide, as the latter did, that in case of failure or refusal to register cash or LBP bonds with an accessible bank. Until then, title also remains with
the land, the valuation thereof shall be that given by the provincial or city the landowner. 57 No outright change of ownership is contemplated either.
assessor for tax purposes. On the contrary, the CARP Law says that the just
compensation shall be ascertained on the basis of the factors mentioned in its Hence, the argument that the assailed measures violate due process by
Section 17 and in the manner provided for in Section 16. arbitrarily transferring title before the land is fully paid for must also be
rejected.
The last major challenge to CARP is that the landowner is divested of his
property even before actual payment to him in full of just compensation, in It is worth stressing at this point that all rights acquired by the tenant-farmer
contravention of a well-accepted principle of eminent domain. under P.D. No. 27, as recognized under E.O. No. 228, are retained by him
even now under R.A. No. 6657. This should counterbalance the express
The recognized rule, indeed, is that title to the property expropriated shall provision in Section 6 of the said law that "the landowners whose lands have
pass from the owner to the expropriator only upon full payment of the just been covered by Presidential Decree No. 27 shall be allowed to keep the area
compensation. Jurisprudence on this settled principle is consistent both here originally retained by them thereunder, further, That original homestead
and in other democratic jurisdictions. Thus: grantees or direct compulsory heirs who still own the original homestead at
the time of the approval of this Act shall retain the same areas as long as they
Title to property which is the subject of condemnation proceedings does not continue to cultivate said homestead."
vest the condemnor until the judgment fixing just compensation is entered
and paid, but the condemnor's title relates back to the date on which the In connection with these retained rights, it does not appear in G.R. No. 78742
petition under the Eminent Domain Act, or the commissioner's report under that the appeal filed by the petitioners with the Office of the President has
the Local Improvement Act, is filed. 51 already been resolved. Although we have said that the doctrine of exhaustion
of administrative remedies need not preclude immediate resort to judicial
. . . although the right to appropriate and use land taken for a canal is complete action, there are factual issues that have yet to be examined on the
at the time of entry, title to the property taken remains in the owner until administrative level, especially the claim that the petitioners are not covered
payment is actually made. 52 (Emphasis supplied.) by LOI 474 because they do not own other agricultural lands than the subjects
of their petition.
In Kennedy v. Indianapolis, 53 the US Supreme Court cited several cases
holding that title to property does not pass to the condemnor until just Obviously, the Court cannot resolve these issues. In any event, assuming that
compensation had actually been made. In fact, the decisions appear to be the petitioners have not yet exercised their retention rights, if any, under P.D.
uniformly to this effect. As early as 1838, in Rubottom v. McLure, 54 it was No. 27, the Court holds that they are entitled to the new retention rights
held that "actual payment to the owner of the condemned property was a provided for by R.A. No. 6657, which in fact are on the whole more liberal
condition precedent to the investment of the title to the property in the State" than those granted by the decree.
albeit "not to the appropriation of it to public use." In Rexford v. Knight, 55
the Court of Appeals of New York said that the construction upon the statutes V
was that the fee did not vest in the State until the payment of the
compensation although the authority to enter upon and appropriate the land The CARP Law and the other enactments also involved in these cases have
was complete prior to the payment. Kennedy further said that "both on been the subject of bitter attack from those who point to the shortcomings of
principle and authority the rule is . . . that the right to enter on and use the these measures and ask that they be scrapped entirely. To be sure, these
property is complete, as soon as the property is actually appropriated under enactments are less than perfect; indeed, they should be continuously re-
the authority of law for a public use, but that the title does not pass from the examined and rehoned, that they may be sharper instruments for the better
owner without his consent, until just compensation has been made to him." protection of the farmer's rights. But we have to start somewhere. In the
pursuit of agrarian reform, we do not tread on familiar ground but grope on
Our own Supreme Court has held in Visayan Refining Co. v. Camus and terrain fraught with pitfalls and expected difficulties. This is inevitable. The
Paredes, 56 that: CARP Law is not a tried and tested project. On the contrary, to use Justice
Holmes's words, "it is an experiment, as all life is an experiment," and so we
If the laws which we have exhibited or cited in the preceding discussion are learn as we venture forward, and, if necessary, by our own mistakes. We
attentively examined it will be apparent that the method of expropriation cannot expect perfection although we should strive for it by all means.
adopted in this jurisdiction is such as to afford absolute reassurance that no Meantime, we struggle as best we can in freeing the farmer from the iron
shackles that have unconscionably, and for so long, fettered his soul to the
soil.

By the decision we reach today, all major legal obstacles to the


comprehensive agrarian reform program are removed, to clear the way for
the true freedom of the farmer. We may now glimpse the day he will be
released not only from want but also from the exploitation and disdain of the
past and from his own feelings of inadequacy and helplessness. At last his
servitude will be ended forever. At last the farm on which he toils will be his
farm. It will be his portion of the Mother Earth that will give him not only
the staff of life but also the joy of living. And where once it bred for him only
deep despair, now can he see in it the fruition of his hopes for a more fulfilling
future. Now at last can he banish from his small plot of earth his insecurities
and dark resentments and "rebuild in it the music and the dream."

WHEREFORE, the Court holds as follows:

1. R.A. No. 6657, P.D. No. 27, Proc. No. 131, and E.O. Nos. 228 and 229 are
SUSTAINED against all the constitutional objections raised in the herein
petitions.

2. Title to all expropriated properties shall be transferred to the State only


upon full payment of compensation to their respective owners.

3. All rights previously acquired by the tenant-farmers under P.D. No. 27 are
retained and recognized.

4. Landowners who were unable to exercise their rights of retention under


P.D. No. 27 shall enjoy the retention rights granted by R.A. No. 6657 under
the conditions therein prescribed.

5. Subject to the above-mentioned rulings, all the petitions are DISMISSED,


without pronouncement as to costs.

SO ORDERED.
ICHONG vs. HERNANDEZ
G.R. No. L-7995 | 1957-05-31 There is no question that the Act was approved in the exercise of the police
LABRADOR, J.: power, but petitioner claims that its exercise in this instance is attended by a
violation of the constitutional requirements of due process and equal
I. The case and issue, in general protection of the laws. But before proceeding to the consideration and
resolution of the ultimate issue involved, it would be well to bear in mind
This Court has before it the delicate task of passing upon the validity and certain basic and fundamental, albeit preliminary, considerations in the
constitutionality of a legislative enactment, fundamental and far-reaching in determination of the ever recurrent conflict between police power and the
significance. The enactment poses questions of due process, police power guarantees of due process and equal protection of the laws. What is the scope
and equal protection of the laws. It also poses an important issue of fact, that of police power, and how are the due process and equal protection clauses
is whether the conditions which the disputed law purports to remedy really related to it? What is the province and power of the legislature, and what is
or actually exist. Admittedly springing from a deep, militant, and positive the function and duty of the courts? These consideration must be clearly and
nationalistic impulse, the law purports to protect citizen and country from the correctly understood that their application to the facts of the case may be
alien retailer. Through it, and within the field of economy it regulates, brought forth with clarity and the issue accordingly resolved.
Congress attempts to translate national aspirations for economic
independence and national security, rooted in the drive and urge for national It has been said the police power is so far - reaching in scope, that it has
survival and welfare, into a concrete and tangible measures designed to free become almost impossible to limit its sweep. As it derives its existence from
the national retailer from the competing dominance of the alien, so that the the very existence of the State itself, it does not need to be expressed or
country and the nation may be free from a supposed economic dependence defined in its scope; it is said to be co-extensive with self-protection and
and bondage. Do the facts and circumstances justify the enactment? survival, and as such it is the most positive and active of all governmental
processes, the most essential, insistent and illimitable. Especially is it so
II. Pertinent provisions of Republic Act No. 1180 under a modern democratic framework where the demands of society and of
nations have multiplied to almost unimaginable proportions; the field and
Republic Act No. 1180 is entitled "An Act to Regulate the Retail Business." scope of police power has become almost boundless, just as the fields of
In effect it nationalizes the retail trade business. The main provisions of the public interest and public welfare have become almost all-embracing and
Act are: (1) a prohibition against persons, not citizens of the Philippines, and have transcended human foresight. Otherwise stated, as we cannot foresee
against associations, partnerships, or corporations the capital of which are the needs and demands of public interest and welfare in this constantly
not wholly owned by citizens of the Philippines, from engaging directly or changing and progressive world, so we cannot delimit beforehand the extent
indirectly in the retail trade; (2) an exception from the above prohibition in or scope of police power by which and through which the State seeks to attain
favor of aliens actually engaged in said business on May 15, 1954, who are or achieve interest or welfare. So it is that Constitutions do not define the
allowed to continue to engaged therein, unless their licenses are forfeited in scope or extent of the police power of the State; what they do is to set forth
accordance with the law, until their death or voluntary retirement in case of the limitations thereof. The most important of these are the due process clause
natural persons, and for ten years after the approval of the Act or until the and the equal protection clause.
expiration of term in case of juridical persons; (3) an exception therefrom in
favor of citizens and juridical entities of the United States; (4) a provision for b. Limitations on police power. —
the forfeiture of licenses (to engage in the retail business) for violation of the
laws on nationalization, control weights and measures and labor and other The basic limitations of due process and equal protection are found in the
laws relating to trade, commerce and industry; (5) a prohibition against the following provisions of our Constitution:
establishment or opening by aliens actually engaged in the retail business of
additional stores or branches of retail business, (6) a provision requiring SECTION 1.(1) No person shall be deprived of life, liberty or property
aliens actually engaged in the retail business to present for registration with without due process of law, nor any person be denied the equal protection of
the proper authorities a verified statement concerning their businesses, the laws. (Article III, Phil. Constitution)
giving, among other matters, the nature of the business, their assets and
liabilities and their offices and principal offices of judicial entities; and (7) a These constitutional guarantees which embody the essence of individual
provision allowing the heirs of aliens now engaged in the retail business who liberty and freedom in democracies, are not limited to citizens alone but are
die, to continue such business for a period of six months for purposes of admittedly universal in their application, without regard to any differences of
liquidation. race, of color, or of nationality. (Yick Wo vs. Hopkins, 30, L. ed. 220, 226.)

III. Grounds upon which petition is based-Answer thereto c. The, equal protection clause. —

Petitioner, for and in his own behalf and on behalf of other alien residents The equal protection of the law clause is against undue favor and individual
corporations and partnerships adversely affected by the provisions of or class privilege, as well as hostile discrimination or the oppression of
Republic Act. No. 1180, brought this action to obtain a judicial declaration inequality. It is not intended to prohibit legislation, which is limited either in
that said Act is unconstitutional, and to enjoin the Secretary of Finance and the object to which it is directed or by territory within which is to operate. It
all other persons acting under him, particularly city and municipal treasurers, does not demand absolute equality among residents; it merely requires that
from enforcing its provisions. Petitioner attacks the constitutionality of the all persons shall be treated alike, under like circumstances and conditions
Act, contending that: (1) it denies to alien residents the equal protection of both as to privileges conferred and liabilities enforced. The equal protection
the laws and deprives of their liberty and property without due process of law clause is not infringed by legislation which applies only to those persons
; (2) the subject of the Act is not expressed or comprehended in the title falling within a specified class, if it applies alike to all persons within such
thereof; (3) the Act violates international and treaty obligations of the class, and reasonable grounds exists for making a distinction between those
Republic of the Philippines; (4) the provisions of the Act against the who fall within such class and those who do not. (2 Cooley, Constitutional
transmission by aliens of their retail business thru hereditary succession, and Limitations, 824-825.)
those requiring 100% Filipino capitalization for a corporation or entity to
entitle it to engage in the retail business, violate the spirit of Sections 1 and d. The due process clause. —
5, Article XIII and Section 8 of Article XIV of the Constitution.
The due process clause has to do with the reasonableness of legislation
In answer, the Solicitor-General and the Fiscal of the City of Manila contend enacted in pursuance of the police power. Is there public interest, a public
that: (1) the Act was passed in the valid exercise of the police power of the purpose; is public welfare involved? Is the Act reasonably necessary for the
State, which exercise is authorized in the Constitution in the interest of accomplishment of the legislature's purpose; is it not unreasonable, arbitrary
national economic survival; (2) the Act has only one subject embraced in the or oppressive? Is there sufficient foundation or reason in connection with the
title; (3) no treaty or international obligations are infringed; (4) as regards matter involved; or has there not been a capricious use of the legislative
hereditary succession, only the form is affected but the value of the property power? Can the aims conceived be achieved by the means used, or is it not
is not impaired, and the institution of inheritance is only of statutory origin. merely an unjustified interference with private interest? These are the
questions that we ask when the due process test is applied.
IV. Preliminary consideration of legal principles involved
The conflict, therefore, between police power and the guarantees of due
a. The police power. — process and equal protection of the laws is more apparent than real. Properly
related, the power and the guarantees are supposed to coexist. The balancing Derogatory epithets are hurled at him, but he laughs these off without
is the essence or, shall it be said, the indispensable means for the attainment murmur; insults of ill-bred and insolent neighbors and customers are made in
of legitimate aspirations of any democratic society. There can be no absolute his face, but he heeds them not, and he forgets and forgives. The community
power, whoever exercise it, for that would be tyranny. Yet there can neither takes note of him, as he appears to be harmless and extremely useful.
be absolute liberty, for that would mean license and anarchy. So the State can
deprive persons of life, liberty and property, provided there is due process of c. Alleged alien control and dominance. —
law; and persons may be classified into classes and groups, provided
everyone is given the equal protection of the law. The test or standard, as There is a general feeling on the part of the public, which appears to be true
always, is reason. The police power legislation must be firmly grounded on to fact, about the controlling and dominant position that the alien retailer
public interest and welfare, and a reasonable relation must exist between holds in the nation's economy. Food and other essentials, clothing, almost all
purposes and means. And if distinction and classification has been made, articles of daily life reach the residents mostly through him. In big cities and
there must be a reasonable basis for said distinction. centers of population he has acquired not only predominance, but apparent
control over distribution of almost all kinds of goods, such as lumber,
e. Legislative discretion not subject to judicial review. — hardware, textiles, groceries, drugs, sugar, flour, garlic, and scores of other
goods and articles. And were it not for some national corporations like the
Now, in this matter of equitable balancing, what is the proper place and role Naric, the Namarco, the Facomas and the Acefa, his control over principal
of the courts? It must not be overlooked, in the first place, that the legislature, foods and products would easily become full and complete.
which is the constitutional repository of police power and exercises the
prerogative of determining the policy of the State, is by force of Petitioner denies that there is alien predominance and control in the retail
circumstances primarily the judge of necessity, adequacy or reasonableness trade. In one breath it is said that the fear is unfounded and the threat is
and wisdom, of any law promulgated in the exercise of the police power, or imagined; in another, it is charged that the law is merely the result of
of the measures adopted to implement the public policy or to achieve public radicalism and pure and unabashed nationalism. Alienage, it is said, is not an
interest. On the other hand, courts, although zealous guardians of individual element of control; also so many unmanageable factors in the retail business
liberty and right, have nevertheless evinced a reluctance to interfere with the make control virtually impossible. The first argument which brings up an
exercise of the legislative prerogative. They have done so early where there issue of fact merits serious consideration. The others are matters of opinion
has been a clear, patent or palpable arbitrary and unreasonable abuse of the within the exclusive competence of the legislature and beyond our
legislative prerogative. Moreover, courts are not supposed to override prerogative to pass upon and decide.
legitimate policy, and courts never inquire into the wisdom of the law.
The best evidence are the statistics on the retail trade, which put down the
V. Economic problems sought to be remedied figures in black and white. Between the constitutional convention year
(1935), when the fear of alien domination and control of the retail trade
With the above considerations in mind, we will now proceed to delve directly already filled the minds of our leaders with fears and misgivings, and the year
into the issue involved. If the disputed legislation were merely a regulation, of the enactment of the nationalization of the retail trade act (1954), official
as its title indicates, there would be no question that it falls within the statistics unmistakably point out to the ever-increasing dominance and
legitimate scope of legislative power. But it goes further and prohibits a control by the alien of the retail trade, as witness the following tables:
group of residents, the aliens, from engaging therein. The problem becomes
more complex because its subject is a common, trade or occupation, as old (Estimated Assets and Gross Sales of Retail Establishments, By Year and
as society itself, which from the immemorial has always been open to Nationality of Owners, Benchmark: 1948 Census, issued by the Bureau of
residents, irrespective of race, color or citizenship. Census and Statistics, Department of Commerce and Industry; pp. 18-19 of
Answer.)
a. Importance of retail trade in the economy of the nation. —
In a primitive economy where families produce all that they consume and The above statistics do not include corporations and partnerships, while the
consume all that they produce, the dealer, of course, is unknown. But as figures on Filipino establishments already include mere market vendors,
group life develops and families begin to live in communities producing whose capital is necessarily small..
more than what they consume and needing an infinite number of things they
do not produce, the dealer comes into existence. As villages develop into big The above figures reveal that in percentage distribution of assests and gross
communities and specialization in production begins, the dealer's importance sales, alien participation has steadily increased during the years. It is true, of
is enhanced. Under modern conditions and standards of living, in which course, that Filipinos have the edge in the number of retailers, but aliens more
man's needs have multiplied and diversified to unlimited extents and than make up for the numerical gap through their assests and gross sales
proportions, the retailer comes as essential as the producer, because thru him which average between six and seven times those of the very many Filipino
the infinite variety of articles, goods and needed for daily life are placed retailers. Numbers in retailers, here, do not imply superiority; the alien
within the easy reach of consumers. Retail dealers perform the functions of invests more capital, buys and sells six to seven times more, and gains much
capillaries in the human body, thru which all the needed food and supplies more. The same official report, pointing out to the known predominance of
are ministered to members of the communities comprising the nation. foreign elements in the retail trade, remarks that the Filipino retailers were
largely engaged in minor retailer enterprises. As observed by respondents,
There cannot be any question about the importance of the retailer in the life the native investment is thinly spread, and the Filipino retailer is practically
of the community. He ministers to the resident's daily needs, food in all its helpless in matters of capital, credit, price and supply.
increasing forms, and the various little gadgets and things needed for home
and daily life. He provides his customers around his store with the rice or d. Alien control and threat, subject of apprehension in Constitutional
corn, the fish, the salt, the vinegar, the spices needed for the daily cooking. convention. —
He has cloths to sell, even the needle and the thread to sew them or darn the
clothes that wear out. The retailer, therefore, from the lowly peddler, the It is this domination and control, which we believe has been sufficiently
owner of a small sari-sari store, to the operator of a department store or, a shown to exist, that is the legislature's target in the enactment of the disputed
supermarket is so much a part of day-to-day existence. nationalization would never have been adopted. The framers of our
Constitution also believed in the existence of this alien dominance and
b. The alien retailer's trait. — control when they approved a resolution categorically declaring among other
things, that "it is the sense of the Convention that the public interest requires
The alien retailer must have started plying his trades in this country in the the nationalization of the retail trade; . . . ." (II Aruego, The Framing of the
bigger centers of population (Time there was when he was unknown in Philippine Constitution, 662-663, quoted on page 67 of Petitioner.) That was
provincial towns and villages). Slowly but gradually be invaded towns and twenty-two years ago; and the events since then have not been either pleasant
villages; now he predominates in the cities and big centers of population. He or comforting. Dean Sinco of the University of the Philippines College of
even pioneers, in far away nooks where the beginnings of community life Law, commenting on the patrimony clause of the Preamble opines that the
appear, ministering to the daily needs of the residents and purchasing their fathers of our Constitution were merely translating the general preoccupation
agricultural produce for sale in the towns. It is an undeniable fact that in many of Filipinos "of the dangers from alien interests that had already brought
communities the alien has replaced the native retailer. He has shown in this under their control the commercial and other economic activities of the
trade, industry without limit, and the patience and forbearance of a slave. country" (Sinco, Phil. Political Law, 10th ed., p. 114); and analyzing the
concern of the members of the constitutional convention for the economic
life of the citizens, in connection with the nationalistic provisions of the engaged in corrupting public officials with fabulous bribes, indirectly
Constitution, he says: causing the prevalence of graft and corruption in the Government. As a
matter of fact appeals to unscrupulous aliens have been made both by the
But there has been a general feeling that alien dominance over the economic Government and by their own lawful diplomatic representatives, action
life of the country is not desirable and that if such a situation should remain, which impliedly admits a prevailing feeling about the existence of many of
political independence alone is no guarantee to national stability and the above practices.
strength. Filipino private capital is not big enough to wrest from alien hands
the control of the national economy. Moreover, it is but of recent formation The circumstances above set forth create well founded fears that worse things
and hence, largely inexperienced, timid and hesitant. Under such conditions, may come in the future. The present dominance of the alien retailer,
the government as the instrumentality of the national will, has to step in and especially in the big centers of population, therefore, becomes a potential
assume the initiative, if not the leadership, in the struggle for the economic source of danger on occasions of war or other calamity. We do not have here
freedom of the nation in somewhat the same way that it did in the crusade for in this country isolated groups of harmless aliens retailing goods among
political freedom. Thus . . . it (the Constitution) envisages an organized nationals; what we have are well organized and powerful groups that
movement for the protection of the nation not only against the possibilities dominate the distribution of goods and commodities in the communities and
of armed invasion but also against its economic subjugation by alien interests big centers of population. They owe no allegiance or loyalty to the State, and
in the economic field. (Phil. Political Law by Sinco, 10th ed., p. 476.) the State cannot rely upon them in times of crisis or emergency. While the
national holds his life, his person and his property subject to the needs of his
Belief in the existence of alien control and predominance is felt in other country, the alien may even become the potential enemy of the State.
quarters. Filipino businessmen, manufacturers and producers believe so; they
fear the dangers coming from alien control, and they express sentiments of f. Law enacted in interest of national economic survival and security. —
economic independence. Witness thereto is Resolution No. 1, approved on
July 18, 1953, of the Fifth National convention of Filipino Businessmen, and We are fully satisfied upon a consideration of all the facts and circumstances
a similar resolution, approved on March 20, 1954, of the Second National that the disputed law is not the product of racial hostility, prejudice or
Convention of Manufacturers and Producers. The man in the street also discrimination, but the expression of the legitimate desire and determination
believes, and fears, alien predominance and control; so our newspapers, of the people, thru their authorized representatives, to free the nation from
which have editorially pointed out not only to control but to alien the economic situation that has unfortunately been saddled upon it rightly or
stranglehold. We, therefore, find alien domination and control to be a fact, a wrongly, to its disadvantage. The law is clearly in the interest of the public,
reality proved by official statistics, and felt by all the sections and groups that nay of the national security itself, and indisputably falls within the scope of
compose the Filipino community. police power, thru which and by which the State insures its existence and
security and the supreme welfare of its citizens.
e. Dangers of alien control and dominance in retail. —
VI. The Equal Protection Limitation
But the dangers arising from alien participation in the retail trade does not
seem to lie in the predominance alone; there is a prevailing feeling that such a. Objections to alien participation in retail trade. —
predominance may truly endanger the national interest. With ample capital,
unity of purpose and action and thorough organization, alien retailers and The next question that now poses solution is, Does the law deny the equal
merchants can act in such complete unison and concert on such vital matters protection of the laws? As pointed out above, the mere fact of alienage is the
as the fixing of prices, the determination of the amount of goods or articles root and cause of the distinction between the alien and the national as a trader.
to be made available in the market, and even the choice of the goods or The alien resident owes allegiance to the country of his birth or his adopted
articles they would or would not patronize or distribute, that fears of country; his stay here is for personal convenience; he is attracted by the lure
dislocation of the national economy and of the complete subservience of of gain and profit. His aim or purpose of stay, we admit, is neither illegitimate
national economy and of the consuming public are not entirely unfounded. nor immoral, but he is naturally lacking in that spirit of loyalty and
Nationals, producers and consumers alike can be placed completely at their enthusiasm for this country where he temporarily stays and makes his living,
mercy. This is easily illustrated. Suppose an article of daily use is desired to or of that spirit of regard, sympathy and consideration for his Filipino
be prescribed by the aliens, because the producer or importer does not offer customers as would prevent him from taking advantage of their weakness
them sufficient profits, or because a new competing article offers bigger and exploiting them. The faster he makes his pile, the earlier can the alien go
profits for its introduction. All that aliens would do is to agree to refuse to back to his beloved country and his beloved kin and countrymen. The
sell the first article, eliminating it from their stocks, offering the new one as experience of the country is that the alien retailer has shown such utter
a substitute. Hence, the producers or importers of the prescribed article, or its disregard for his customers and the people on whom he makes his profit, that
consumers, find the article suddenly out of the prescribed article, or its it has been found necessary to adopt the legislation, radical as it may seem.
consumers, find the article suddenly out of circulation. Freedom of trade is
thus curtailed and free enterprise correspondingly suppressed. Another objection to the alien retailer in this country is that he never really
makes a genuine contribution to national income and wealth. He undoubtedly
We can even go farther than theoretical illustrations to show the pernicious contributes to general distribution, but the gains and profits he makes are not
influences of alien domination. Grave abuses have characterized the exercise invested in industries that would help the country's economy and increase
of the retail trade by aliens. It is a fact within judicial notice, which courts of national wealth. The alien's interest in this country being merely transient and
justice may not properly overlook or ignore in the interests of truth and temporary, it would indeed be ill-advised to continue entrusting the very
justice, that there exists a general feeling on the part of the public that alien important function of retail distribution to his hands.
participation in the retail trade has been attended by a pernicious and
intolerable practices, the mention of a few of which would suffice for our The practices resorted to by aliens in the control of distribution, as already
purposes; that at some time or other they have cornered the market of pointed out above, their secret manipulations of stocks of commodities and
essential commodities, like corn and rice, creating artificial scarcities to prices, their utter disregard of the welfare of their customers and of the
justify and enhance profits to unreasonable proportions; that they have ultimate happiness of the people of the nation of which they are mere guests,
hoarded essential foods to the inconvenience and prejudice of the consuming which practices, manipulations and disregard do not attend the exercise of
public, so much so that the Government has had to establish the National the trade by the nationals, show the existence of real and actual, positive and
Rice and Corn Corporation to save the public from their continuous hoarding fundamental differences between an alien and a national which fully justify
practices and tendencies; that they have violated price control laws, the legislative classification adopted in the retail trade measure. These
especially on foods and essential commodities, such that the legislature had differences are certainly a valid reason for the State to prefer the national
to enact a law (Sec. 9, Republic Act No. 1168), authorizing their immediate over the alien in the retail trade. We would be doing violence to fact and
and automatic deportation for price control convictions; that they have secret reality were we to hold that no reason or ground for a legitimate distinction
combinations among themselves to control prices, cheating the operation of can be found between one and the other.
the law of supply and demand; that they have connived to boycott honest
merchants and traders who would not cater or yield to their demands, in b. Difference in alien aims and purposes sufficient basis for distinction. —
unlawful restraint of freedom of trade and enterprise. They are believed by
the public to have evaded tax laws, smuggled goods and money into and out The above objectionable characteristics of the exercise of the retail trade by
of the land, violated import and export prohibitions, control laws and the like, the aliens, which are actual and real, furnish sufficient grounds for legislative
in derision and contempt of lawful authority. It is also believed that they have classification of retail traders into nationals and aliens. Some may disagree
with the wisdom of the legislature's classification. To this we answer, that In Commonwealth vs. Hana, 81 N. E. 149 (Massachusetts, 1907), a statute
this is the prerogative of the law-making power. Since the Court finds that on the licensing of hawkers and peddlers, which provided that no one can
the classification is actual, real and reasonable, and all persons of one class obtain a license unless he is, or has declared his intention, to become a citizen
are treated alike, and as it cannot be said that the classification is patently of the United States, was held valid, for the following reason: It may seem
unreasonable and unfounded, it is in duty bound to declare that the legislature wise to the legislature to limit the business of those who are supposed to have
acted within its legitimate prerogative and it can not declare that the act regard for the welfare, good order and happiness of the community, and the
transcends the limit of equal protection established by the Constitution. court cannot question this judgment and conclusion. In Bloomfield vs. State,
99 N. E. 309 (Ohio, 1912), a statute which prevented certain persons, among
Broadly speaking, the power of the legislature to make distinctions and them aliens, from engaging in the traffic of liquors, was found not to be the
classifications among persons is not curtailed or denied by the equal result of race hatred, or in hospitality, or a deliberate purpose to discriminate,
protection of the laws clause. The legislative power admits of a wide scope but was based on the belief that an alien cannot be sufficiently acquainted
of discretion, and a law can be violative of the constitutional limitation only with "our institutions and our life as to enable him to appreciate the relation
when the classification is without reasonable basis. In addition to the of this particular business to our entire social fabric", and was not, therefore,
authorities we have earlier cited, we can also refer to the case of Linsey vs. invalid. In Ohio ex rel. Clarke vs. Deckebach, 274 U. S. 392, 71 L. ed. 115
Natural Carbonic Fas Co. (1911), 55 L. ed., 369, which clearly and succinctly (1926), the U.S. Supreme Court had under consideration an ordinance of the
defined the application of equal protection clause to a law sought to be voided city of Cincinnati prohibiting the issuance of licenses (pools and billiard
as contrary thereto: rooms) to aliens. It held that plainly irrational discrimination against aliens
is prohibited, but it does not follow that alien race and allegiance may not
. . . . "1. The equal protection clause of the Fourteenth Amendment does not bear in some instances such a relation to a legitimate object of legislation as
take from the state the power to classify in the adoption of police laws, but to be made the basis of permitted classification, and that it could not state
admits of the exercise of the wide scope of discretion in that regard, and that the legislation is clearly wrong; and that latitude must be allowed for the
avoids what is done only when it is without any reasonable basis, and legislative appraisement of local conditions and for the legislative choice of
therefore is purely arbitrary. 2. A classification having some reasonable basis methods for controlling an apprehended evil. The case of State vs. Carrol,
does not offend against that clause merely because it is not made with 124 N. E. 129 (Ohio, 1919) is a parallel case to the one at bar. In Asakura vs.
mathematical nicety, or because in practice it results in some inequality. 3. City of Seattle, 210 P. 30 (Washington, 1922), the business of pawn brooking
When the classification in such a law is called in question, if any state of facts was considered as having tendencies injuring public interest, and limiting it
reasonably can be conceived that would sustain it, the existence of that state to citizens is within the scope of police power. A similar statute denying
of facts at the time the law was enacted must be assumed. 4. One who assails aliens the right to engage in auctioneering was also sustained in Wright vs.
the classification in such a law must carry the burden of showing that it does May, L.R.A., 1915 P. 151 (Minnesota, 1914). So also in Anton vs. Van
not rest upon any reasonable basis but is essentially arbitrary." Winkle, 297 F. 340 (Oregon, 1924), the court said that aliens are judicially
known to have different interests, knowledge, attitude, psychology and
c. Authorities recognizing citizenship as basis for classification. — loyalty, hence the prohibitions of issuance of licenses to them for the business
of pawnbroker, pool, billiard, card room, dance hall, is not an infringement
The question as to whether or not citizenship is a legal and valid ground for of constitutional rights. In Templar vs. Michigan State Board of Examiners,
classification has already been affirmatively decided in this jurisdiction as 90 N.W. 1058 (Michigan, 1902), a law prohibiting the licensing of aliens as
well as in various courts in the United States. In the case of Smith Bell & Co. barbers was held void, but the reason for the decision was the court's findings
vs. Natividad, 40 Phil. 136, where the validity of Act No. 2761 of the that the exercise of the business by the aliens does not in any way affect the
Philippine Legislature was in issue, because of a condition therein limiting morals, the health, or even the convenience of the community. In Takahashi
the ownership of vessels engaged in coastwise trade to corporations formed vs. Fish and Game Commission, 92 L. ed. 1479 (1947), a California statute
by citizens of the Philippine Islands or the United States, thus denying the banning the issuance of commercial fishing licenses to person ineligible to
right to aliens, it was held that the Philippine Legislature did not violate the citizenship was held void, because the law conflicts with Federal power over
equal protection clause of the Philippine Bill of Rights. The legislature in immigration, and because there is no public interest in the mere claim of
enacting the law had as ultimate purpose the encouragement of Philippine ownership of the waters and the fish in them, so there was no adequate
shipbuilding and the safety for these Islands from foreign interlopers. We justification for the discrimination. It further added that the law was the
held that this was a valid exercise of the police power, and all presumptions outgrowth of antagonism toward the persons of Japanese ancestry. However,
are in favor of its constitutionality. In substance, we held that the limitation two Justices dissented on the theory that fishing rights have been treated
of domestic ownership of vessels engaged in coastwise trade to citizens of traditionally as natural resources. In Fraser vs. McConway & Tarley Co., 82
the Philippines does not violate the equal protection of the law and due Fed. 257 (Pennsylvania, 1897), a state law which imposed a tax on every
process or law clauses of the Philippine Bill of Rights. In rendering said employer of foreign-born unnaturalized male persons over 21 years of age,
decision we quoted with approval the concurring opinion of Justice Johnson was declared void because the court found that there was no reason for the
in the case of Gibbons vs. Ogden, 9 Wheat., I, as follows: classification and the tax was an arbitrary deduction from the daily wage of
an employee.
"Licensing acts, in fact, in legislation, are universally restraining acts; as, for
example, acts licensing gaming houses, retailers of spirituous liquors, etc. d. Authorities contra explained. —
The act, in this instance, is distinctly of that character, and forms part of an
extensive system, the object of which is to encourage American shipping, It is true that some decisions of the Federal court and of the State courts in
and place them on an equal footing with the shipping of other nations. Almost the United States hold that the distinction between aliens and citizens is not
every commercial nation reserves to its own subjects a monopoly of its a valid ground for classification. But in this decision the laws declared invalid
coasting trade; and a countervailing privilege in favor of American shipping were found to be either arbitrary, unreasonable or capricious, or were the
is contemplated, in the whole legislation of the United States on this subject. result or product of racial antagonism and hostility, and there was no question
It is not to give the vessel an American character, that the license is granted; of public interest involved or pursued. In Yu Cong Eng vs. Trinidad, 70 L.
that effect has been correctly attributed to the act of her enrollment. But it is ed. 1059 (1925), the United States Supreme Court declared invalid a
to confer on her American privileges, as contra distinguished from foreign; Philippine law making unlawful the keeping of books of account in any
and to preserve the Government from fraud by foreigners; in surreptitiously language other than English, Spanish or any other local dialect, but the main
intruding themselves into the American commercial marine, as well as frauds reasons for the decisions are: (1) that if Chinese were driven out of business
upon the revenue in the trade coastwise, that this whole system is projected." there would be no other system of distribution, and (2) that the Chinese would
fall prey to all kinds of fraud, because they would be deprived of their right
The rule in general is as follows: to be advised of their business and to direct its conduct. The real reason for
the decision, therefore, is the court's belief that no public benefit would be
Aliens are under no special constitutional protection which forbids a derived from the operations of the law and on the other hand it would deprive
classification otherwise justified simply because the limitation of the class Chinese of something indispensable for carrying on their business. In Yick
falls along the lines of nationality. That would be requiring a higher degree Wo vs. Hopkins, 30 L. ed 220 (1885) an ordinance conferring powers on
of protection for aliens as a class than for similar classes than for similar officials to withhold consent in the operation of laundries both as to persons
classes of American citizens. Broadly speaking, the difference in status and place, was declared invalid, but the court said that the power granted was
between citizens and aliens constitutes a basis for reasonable classification arbitrary, that there was no reason for the discrimination which attended the
in the exercise of police power. (2 Am., Jur. 468-469.) administration and implementation of the law, and that the motive thereof
was mere racial hostility. In State vs. Montgomery, 47 A. 165 (Maine, 1900),
a law prohibiting aliens to engage as hawkers and peddlers was declared void, second, that the means are reasonably necessary for the accomplishment of
because the discrimination bore no reasonable and just relation to the act in the purpose, and not unduly oppressive upon individuals. . . .
respect to which the classification was proposed.
Prata Undertaking Co. vs. State Board of Embalming, 104 ALR, 389, 395,
The case at bar is radically different, and the facts make them so. As we fixes this test of constitutionality:
already have said, aliens do not naturally possess the sympathetic
consideration and regard for the customers with whom they come in daily In determining whether a given act of the Legislature, passed in the exercise
contact, nor the patriotic desire to help bolster the nation's economy, except of the police power to regulate the operation of a business, is or is not
in so far as it enhances their profit, nor the loyalty and allegiance which the constitutional, one of the first questions to be considered by the court is
national owes to the land. These limitations on the qualifications of the aliens whether the power as exercised has a sufficient foundation in reason in
have been shown on many occasions and instances, especially in times of connection with the matter involved, or is an arbitrary, oppressive, and
crisis and emergency. We can do no better than borrow the language of Anton capricious use of that power, without substantial relation to the health, safety,
vs. Van Winkle, 297 F. 340, 342, to drive home the reality and significance morals, comfort, and general welfare of the public.
of the distinction between the alien and the national, thus:
b. Petitioner's argument considered. —
. . . . It may be judicially known, however, that alien coming into this country
are without the intimate knowledge of our laws, customs, and usages that our Petitioner's main argument is that retail is a common, ordinary occupation,
own people have. So it is likewise known that certain classes of aliens are of one of those privileges long ago recognized as essential to the orderly
different psychology from our fellow countrymen. Furthermore, it is natural pursuant of happiness by free men; that it is a gainful and honest occupation
and reasonable to suppose that the foreign born, whose allegiance is first to and therefore beyond the power of the legislature to prohibit and penalized.
their own country, and whose ideals of governmental environment and This arguments overlooks fact and reality and rests on an incorrect
control have been engendered and formed under entirely different regimes assumption and premise, i.e., that in this country where the occupation is
and political systems, have not the same inspiration for the public weal, nor engaged in by petitioner, it has been so engaged by him, by the alien in an
are they as well disposed toward the United States, as those who by honest creditable and unimpeachable manner, without harm or injury to the
citizenship, are a part of the government itself. Further enlargement, is citizens and without ultimate danger to their economic peace, tranquility and
unnecessary. I have said enough so that obviously it cannot be affirmed with welfare. But the Legislature has found, as we have also found and indicated,
absolute confidence that the Legislature was without plausible reason for that the privilege has been so grossly abused by the alien, thru the illegitimate
making the classification, and therefore appropriate discriminations against use of pernicious designs and practices, that he now enjoys a monopolistic
aliens as it relates to the subject of legislation. . . . . control of the occupation and threatens a deadly stranglehold on the nation's
economy endangering the national security in times of crisis and emergency.
VII. The Due Process of Law Limitation.
The real question at issue, therefore, is not that posed by petitioner, which
a. Reasonability, the test of the limitation; determination by legislature overlooks and ignores the facts and circumstances, but this, Is the exclusion
decisive. — in the future of aliens from the retail trade unreasonable. Arbitrary capricious,
taking into account the illegitimate and pernicious form and manner in which
We now come to due process as a limitation on the exercise of the police the aliens have heretofore engaged therein? As thus correctly stated the
power. It has been stated by the highest authority in the United States that: answer is clear. The law in question is deemed absolutely necessary to bring
about the desired legislative objective, i.e., to free national economy from
. . . . And the guaranty of due process, as has often been held, demands only alien control and dominance. It is not necessarily unreasonable because it
that the law shall not be unreasonable, arbitrary or capricious, and that the affects private rights and privileges (11 Am. Jur. pp. 1080-1081.) The test of
means selected shall have a real and substantial relation to the subject sought reasonableness of a law is the appropriateness or adequacy under all
to be attained. . . . . circumstances of the means adopted to carry out its purpose into effect (Id.)
Judged by this test, disputed legislation, which is not merely reasonable but
xxx xxx xxx actually necessary, must be considered not to have infringed the
constitutional limitation of reasonableness.
So far as the requirement of due process is concerned and in the absence of
other constitutional restriction a state is free to adopt whatever economic The necessity of the law in question is explained in the explanatory note that
policy may reasonably be deemed to promote public welfare, and to enforce accompanied the bill, which later was enacted into law:
that policy by legislation adapted to its purpose. The courts are without
authority either to declare such policy, or, when it is declared by the This bill proposes to regulate the retail business. Its purpose is to prevent
legislature, to override it. If the laws passed are seen to have a reasonable persons who are not citizens of the Philippines from having a strangle hold
relation to a proper legislative purpose, and are neither arbitrary nor upon our economic life. If the persons who control this vital artery of our
discriminatory, the requirements of due process are satisfied, and judicial economic life are the ones who owe no allegiance to this Republic, who have
determination to that effect renders a court functus officio. . . . (Nebbia vs. no profound devotion to our free institutions, and who have no permanent
New York, 78 L. ed. 940, 950, 957.) stake in our people's welfare, we are not really the masters of our destiny. All
aspects of our life, even our national security, will be at the mercy of other
Another authority states the principle thus: people.

. . . . Too much significance cannot be given to the word "reasonable" in In seeking to accomplish the foregoing purpose, we do not propose to deprive
considering the scope of the police power in a constitutional sense, for the persons who are not citizens of the Philippines of their means of livelihood.
test used to determine the constitutionality of the means employed by the While this bill seeks to take away from the hands of persons who are not
legislature is to inquire whether the restriction it imposes on rights secured citizens of the Philippines a power that can be wielded to paralyze all aspects
to individuals by the Bill of Rights are unreasonable, and not whether it of our national life and endanger our national security it respects existing
imposes any restrictions on such rights. . . . rights.
The approval of this bill is necessary for our national survival.
xxx xxx xxx
If political independence is a legitimate aspiration of a people, then economic
. . . . A statute to be within this power must also be reasonable in its operation independence is none the less legitimate. Freedom and liberty are not real
upon the persons whom it affects, must not be for the annoyance of a and positive if the people are subject to the economic control and domination
particular class, and must not be unduly oppressive. (11 Am. Jur. Sec. 302., of others, especially if not of their own race or country. The removal and
1:1)- 1074-1075.) eradication of the shackles of foreign economic control and domination, is
one of the noblest motives that a national legislature may pursue. It is
In the case of Lawton vs. Steele, 38 L. ed. 385, 388. it was also held: impossible to conceive that legislation that seeks to bring it about can infringe
the constitutional limitation of due process. The attainment of a legitimate
. . . . To justify the state in thus interposing its authority in behalf of the aspiration of a people can never be beyond the limits of legislative authority.
public, it must appear, first, that the interests of the public generally, as
distinguished from those of a particular class, require such interference; and
c. Law expressly held by Constitutional Convention to be within the sphere
of legislative action. — A subordinate ground or reason for the alleged invalidity of the law is the
claim that the title thereof is misleading or deceptive, as it conceals the real
The framers of the Constitution could not have intended to impose the purpose of the bill which is to nationalize the retail business and prohibit
constitutional restrictions of due process on the attainment of such a noble aliens from engaging therein. The constitutional provision which is claimed
motive as freedom from economic control and domination, thru the exercise to be violated in Section 21 (1) of Article VI, which reads:
of the police power. The fathers of the Constitution must have given to the
legislature full authority and power to enact legislation that would promote No bill which may be enacted in the law shall embrace more than one subject
the supreme happiness of the people, their freedom and liberty. On the which shall be expressed in the title of the bill.
precise issue now before us, they expressly made their voice clear; they
adopted a resolution expressing their belief that the legislation in question is What the above provision prohibits is duplicity, that is, if its title completely
within the scope of the legislative power. Thus they declared the their fails to appraise the legislators or the public of the nature, scope and
Resolution: consequences of the law or its operation (I Sutherland, Statutory
Construction, Sec. 1707, p. 297.) A cursory consideration of the title and the
That it is the sense of the Convention that the public interest requires the provisions of the bill fails to show the presence of duplicity. It is true that the
nationalization of retail trade; but it abstain from approving the amendment term "regulate" does not and may not readily and at first glance convey the
introduced by the Delegate for Manila, Mr. Araneta, and others on this matter idea of "nationalization" and "prohibition", which terms express the two main
because it is convinced that the National Assembly is authorized to purposes and objectives of the law. But "regulate" is a broader term than
promulgate a law which limits to Filipino and American citizens the privilege either prohibition or nationalization. Both of these have always been included
to engage in the retail trade. (11 Aruego, The Framing of the Philippine within the term regulation.
Constitution, quoted on pages 66 and 67 of the Memorandum for the
Petitioner.) Under the title of an act to "regulate", the sale of intoxicating liquors, the
Legislature may prohibit the sale of intoxicating liquors. (Sweet vs. City of
It would do well to refer to the nationalistic tendency manifested in various Wabash, 41 Ind., 7; quoted in page 41 of Answer.)
provisions of the Constitution. Thus in the preamble, a principle objective is
the conservation of the patrimony of the nation and as corollary the provision Within the meaning of the Constitution requiring that the subject of every act
limiting to citizens of the Philippines the exploitation, development and of the Legislature shall be stated in the tale, the title to regulate the sale of
utilization of its natural resources. And in Section 8 of Article XIV, it is intoxicating liquors, etc." sufficiently expresses the subject of an act
provided that "no franchise, certificate, or any other form of authorization for prohibiting the sale of such liquors to minors and to persons in the habit of
the operation of the public utility shall be granted except to citizens of the getting intoxicated; such matters being properly included within the subject
Philippines." The nationalization of the retail trade is only a continuance of of regulating the sale. (Williams vs. State, 48 Ind. 306, 308, quoted in p. 42
the nationalistic protective policy laid down as a primary objective of the of Answer.)
Constitution. Can it be said that a law imbued with the same purpose and
spirit underlying many of the provisions of the Constitution is unreasonable, The word "regulate" is of broad import, and necessarily implies some degree
invalid and unconstitutional? of restraint and prohibition of acts usually done in connection with the thing
to be regulated. While word regulate does not ordinarily convey meaning of
The seriousness of the Legislature's concern for the plight of the nationals as prohibit, there is no absolute reason why it should not have such meaning
manifested in the approval of the radical measures is, therefore, fully when used in delegating police power in connection with a thing the best or
justified. It would have been recreant to its duties towards the country and its only efficacious regulation of which involves suppression. (State vs. Morton,
people would it view the sorry plight of the nationals with the complacency 162 So. 718, 182 La. 887, quoted in p. 42 of Answer.)
and refuse or neglect to adopt a remedy commensurate with the demands of
public interest and national survival. As the repository of the sovereign power The general rule is for the use of general terms in the title of a bill; it has also
of legislation, the Legislature was in duty bound to face the problem and been said that the title need not be an index to the entire contents of the law
meet, through adequate measures, the danger and threat that alien domination (I Sutherland, Statutory Construction, See. 4803, p. 345.) The above rule was
of retail trade poses to national economy. followed the title of the Act in question adopted the more general term
"regulate" instead of "nationalize" or "prohibit". Furthermore, the law also
d. Provisions of law not unreasonable. — contains other rules for the regulation of the retail trade which may not be
included in the terms "nationalization" or "prohibition"; so were the title
A cursory study of the provisions of the law immediately reveals how changed from "regulate" to "nationalize" or "prohibit", there would have
tolerant, how reasonable the Legislature has been. The law is made been many provisions not falling within the scope of the title which would
prospective and recognizes the right and privilege of those already engaged have made the Act invalid. The use of the term "regulate", therefore, is in
in the occupation to continue therein during the rest of their lives; and similar accord with the principle governing the drafting of statutes, under which a
recognition of the right to continue is accorded associations of aliens. The simple or general term should be adopted in the title, which would include
right or privilege is denied to those only upon conviction of certain offenses. all other provisions found in the body of the Act.
In the deliberations of the Court on this case, attention was called to the fact
that the privilege should not have been denied to children and heirs of aliens One purpose of the constitutional directive that the subject of a bill should be
now engaged in the retail trade. Such provision would defeat the law itself, embraced in its title is to apprise the legislators of the purposes, the nature
its aims and purposes. Beside, the exercise of legislative discretion is not and scope of its provisions, and prevent the enactment into law of matters
subject to judicial review. It is well settled that the Court will not inquire into which have received the notice, action and study of the legislators or of the
the motives of the Legislature, nor pass upon general matters of legislative public. In the case at bar it cannot be claimed that the legislators have been
judgment. The Legislature is primarily the judge of the necessity of an appraised of the nature of the law, especially the nationalization and the
enactment or of any of its provisions, and every presumption is in favor of its prohibition provisions. The legislators took active interest in the discussion
validity, and though the Court may hold views inconsistent with the wisdom of the law, and a great many of the persons affected by the prohibitions in the
of the law, it may not annul the legislation if not palpably in excess of the law conducted a campaign against its approval. It cannot be claimed,
legislative power. Furthermore, the test of the validity of a law attacked as a therefore, that the reasons for declaring the law invalid ever existed. The
violation of due process, is not its reasonableness, but its unreasonableness, objection must therefore, be overruled.
and we find the provisions are not unreasonable. These principles also answer
various other arguments raised against the law, some of which are: that the IX. Alleged violation of international treaties and obligations
law does not promote general welfare; that thousands of aliens would be
thrown out of employment; that prices will increase because of the Another subordinate argument against the validity of the law is the supposed
elimination of competition; that there is no need for the legislation; that violation thereby of the Charter of the United Nations and of the Declaration
adequate replacement is problematical; that there may be general breakdown; of the Human Rights adopted by the United Nations General Assembly. We
that there would be repercussions from foreigners; etc. Many of these find no merit in the Nations Charter imposes no strict or legal obligations
arguments are directed against the supposed wisdom of the law which lies regarding the rights and freedom of their subjects (Hans Kelsen, The Law of
solely within the legislative prerogative; they do not import invalidity. the United Nations, 1951 ed. pp. 29-32), and the Declaration of Human
Rights contains nothing more than a mere recommendation or a common
VIII. Alleged defect in the title of the law standard of achievement for all peoples and all nations (Id. p. 39.) That such
is the import of the United Nations Charter aid of the Declaration of Human
Rights can be inferred the fact that members of the United Nations
Organizations, such as Norway and Denmark, prohibit foreigners from
engaging in retail trade, and in most nations of the world laws against
foreigners engaged in domestic trade are adopted.

The Treaty of Amity between the Republic of the Philippines and the
Republic of China of April 18, 1947 is also claimed to be violated by the law
in question. All that the treaty guarantees is equality of treatment to the
Chinese nationals "upon the same terms as the nationals of any other
country." But the nationals of China are not discriminating against because
nationals of all other countries, except those of the United States, who are
granted special rights by the Constitution, are all prohibited from engaging
in the retail trade. But even supposing that the law infringes upon the said
treaty, the treaty is always subject to qualification or amendment by a
subsequent law (U. S. vs. Thompson, 258, Fed. 257, 260), and the same may
never curtail or restrict the scope of the police power of the State (plaston vs.
Pennsylvania, 58 L. ed. 539.)

X. Conclusion

Resuming what we have set forth above we hold that the disputed law was
enacted to remedy a real actual threat and danger to national economy posed
by alien dominance and control of the retail business and free citizens and
country from dominance and control; that the enactment clearly falls within
the scope of the police power of the State, thru which and by which it protects
its own personality and insures its security and future; that the law does not
violate the equal protection clause of the Constitution because sufficient
grounds exist for the distinction between alien and citizen in the exercise of
the occupation regulated, nor the due process of law clause, because the law
is prospective in operation and recognizes the privilege of aliens already
engaged in the occupation and reasonably protects their privilege; that the
wisdom and efficacy of the law to carry out its objectives appear to us to be
plainly evident — as a matter of fact it seems not only appropriate but
actually necessary — and that in any case such matter falls within the
prerogative of the Legislature, with whose power and discretion the Judicial
department of the Government may not interfere; that the provisions of the
law are clearly embraced in the title, and this suffers from no duplicity and
has not misled the legislators or the segment of the population affected; and
that it cannot be said to be void for supposed conflict with treaty obligations
because no treaty has actually been entered into on the subject and the police
power may not be curtailed or surrendered by any treaty or any other
conventional agreement.

Some members of the Court are of the opinion that the radical effects of the
law could have been made less harsh in its impact on the aliens. Thus it is
stated that the more time should have been given in the law for the liquidation
of existing businesses when the time comes for them to close. Our legal duty,
however, is merely to determine if the law falls within the scope of legislative
authority and does not transcend the limitations of due process and equal
protection guaranteed in the Constitution. Remedies against the harshness of
the law should be addressed to the Legislature; they are beyond our power
and jurisdiction.

The petition is hereby denied, with costs against petitioner.


MOSTERO vs. THE CHIEF OF POLICE such warrant of commitment, the detention of the person arrested for more
G.R. No. L-2128 | 1948-05-12 than six hours would be illegal and in violation of our Constitution.
FERIA, J:
Our conclusion is confirmed by section 17, Rule 109 of the Rules of Court,
Upon complaint of one Bernardino Malinao, charging the petitioners with which, referring to the duty of an officer after arrest without warrant,
having committed the crime of robbery, Benjamin Dumlao, a policeman of provides that "a person making arrest for legal ground shall, without
the City of Manila, arrested the petitioners on April 2, 1948, and presented a unnecessary delay, and within the time prescribed in the Revised Penal Code,
complaint against them with the fiscal's office of Manila. Until April 7, 1948, take the person arrested to the proper court or judge for such action as they
when the petition for habeas corpus filed with this Court was heard, the may deem proper to take ;" and by section 11 of Rule 108, which reads that
petitioners were still detained or under arrest, and the city fiscal had not yet "after the arrest by the defendant and his delivery to the Court, he shall be
released or filed against them an information with the proper courts of justice. informed of the complaint or information filed against him. He shall also be
informed of the substance of the testimony and evidence presented against
This case has not been decided before this time because there was not a him, and, if he desires to testify or to present witnesses or evidence in his
sufficient number of Justices to form a quorum in Manila, and it had to be favor, he may be allowed to do so. The testimony of the witnesses need not
transferred to the Supreme Court acting in division here in Baguio for be reduced to writing but that of the defendant shall be taken in writing and
deliberation and decision. We have not until now an official information as subscribed by him."
to the action taken by the office of the city fiscal on the complaint filed by
Dumlao against the petitioners. But whatever might have been the action And it is further corroborated by the provisions of sections 1 and 4, Rule 102
taken by said office, if there was any, we have to decide this case in order to of the Rules of Court. According to the provisions of said section, "a writ of
lay down a ruling on the question involved herein for the information and habeas corpus shall extend to all cases of illegal confinement or detention by
guidance in the future of the officers concerned. which any person is illegally deprived of his liberty"; and "if it appears that
the person alleged to be restrained of his liberty is in the custody of an officer
The principal question to be determined in the present case in order to decide under process issued by a court or judge, or by virtue of a judgment or order
whether or not the petitioners are being illegally restrained of their liberty, is of a court of record, and that the court or judge had jurisdiction to issue the
the following: Is the city fiscal of Manila a judicial authority within the process, render judgment, or make the order, the writ shall not be allowed."
meaning of the provisions of article 125 of the Revised Penal Code? Which a contrario sensu means that, otherwise, the writ shall be allowed and
the person detained shall be released.
Article 125 of the Revised Penal Code provides that "the penalties provided
in the next preceding article shall be imposed upon the public officer or The judicial authority mentioned in section 125 of the Revised Penal Code
employee who shall detain any person for some legal ground and shall fail to can not be construed to include the fiscal of the City of Manila or any other
deliver such person to the proper judicial authorities within the period of six city, because they cannot issue a warrant of arrest or of commitment or
hours." temporary confinement of a person surrendered to legalize the detention of a
person arrested without warrant. (Section 7, Rule 108; Hashin vs. Boncan,
Taking into consideration the history of the provisions of the above quoted 40 Off. Gaz. 13th Suppl., p. 13; Lino vs. Fugoso, L-1159, promulgated on
article, the precept of our Constitution guaranteeing individual liberty, and January 30, 1947, 43 Off. Gaz., 1214). The investigation which the city fiscal
the provisions of Rules of Court regarding arrest and habeas corpus, we are of Manila makes is not the preliminary investigation proper provided for in
of the opinion that the words "judicial authority", as used in said article, mean section 11, Rule 108, above quoted, to which all persons charged with
the courts of justices or judges of said courts vested with judicial power to offenses cognizable by the Court of First Instance in provinces are entitled,
order the temporary detention or confinement of a person charged with but it is a mere investigation made by the city fiscal for the purpose of filing
having committed a public offense, that is, "the Supreme Court and such the corresponding information against the defendant with the proper
inferior courts as may be established by law". (Section 1, Article VIII of the municipal court or Court of First Instance of Manila if the result of the
Constitution.). investigation so warrants, in order to obtain or secure from the court a warrant
of arrest of the defendant. It is provided by law as a substitute, in a certain
Article 125 of the Revised Penal Code was substantially taken from article sense, of the preliminary investigation proper to avoid or prevent a hasty or
202 of the old Penal Code formerly in force in these Islands, which penalized malicious prosecution, since defendants charged with offenses triable by the
a public officer other than a judicial officer who, without warrant, "shall courts in the City of Manila are not entitled to a proper preliminary
arrest a person upon a charge of crime and shall fail to deliver such person to investigation.
the judicial authority within twenty four hours after his arrest." There was no
doubt that the judicial authority therein referred to was the judge of a court The only executive officers authorized by law to make a proper preliminary
of justice empowered by law, after a proper investigation, to order the investigation in case of temporary absence of both the justice of the peace
temporary commitment or detention of the person arrested; and not the city and the auxiliary justice of the peace from the municipality, town or place,
fiscals or any other officers, who are not authorized by law to do so. Because are the municipal mayors who are empowered in such case to issue a warrant
article 204, which complements said section 202, of the same Code provided of arrest of the accused. (Section 3, Rule 108, in connection with section 6,
that "the penalty of suspension in its minimum and medium degrees shall be Rule 108, and section 2 of Rule 109.) The preliminary investigation which a
imposed upon the following persons: 1. Any judicial officer who, within the city fiscal may conduct under section 2, Rule 108, is the investigation
period prescribed by the provisions of the law of criminal procedure in force, referred to in the preceding paragraph.
shall fail to release any prisoner under arrest or to commit such prisoner
formally by written order containing a statement of the grounds upon which Under the law, a complaint charging a person with the commission of an
the same is based." offense cognizable by the courts of Manila is not filed with the municipal
court or the Court of First Instance of Manila, because as above stated, the
Although the above quoted provision of article 204 of the old Penal Code has latter do not make or conduct a preliminary investigation proper. The
not been incorporated in the Revised Penal Code the import of said words complaint must be made or filed with the city fiscal of Manila who,
judicial authority or officer can not be construed as having been modified by personally or through one of his assistants, makes the investigation, not for
the mere omission of said provision in the Revised Penal Code. the purpose of ordering the arrest of the accused, but of filing with the proper
court the necessary information against the accused if the result of the
Besides, section 1(3), Article III, of our Constitution provides that "the right investigation so warrants, and obtaining from the court a warrant of arrest or
of the people to be secure in their persons . . . against unreasonable seizure commitment of the accused.
shall not be violated, and no warrant [of arrest, detention or confinement]
shall issue but upon probable cause, to be determined by the judge after When a person is arrested without warrant in cases permitted by law, the
examination under oath or affirmation of the complainant and the witness he officer or person making the arrest should, as above stated, without
may produce." Under this constitutional precept no person may be deprived unnecessary delay take or surrender the person arrested, within the period of
of his liberty, except by warrant of arrest or commitment issued upon time prescribed in the Revised Penal Code, to the court or judge having
probable cause by a judge after examination of the complainant and his jurisdiction to try or make a preliminary investigation of the offense (section
witness. And the judicial authority to whom a person arrested by a public 17, Rule 109); and the court or judge shall try and decide the case if the court
officer must be surrendered can not be any other but a court or judge who has original jurisdiction over the offense charged, or make the preliminary
alone is authorized to issue a warrant of commitment or provisional detention investigation if it is a justice of the peace court having no original
of the person arrested pending the trial of the case against the latter. Without
jurisdiction, and then transfer the case to the proper Court of First Instance
in accordance with the provisions of section 13, Rule 108.

In the City of Manila, where complaints are not filed directly with the
municipal court or the Court of First Instance, the officer or person making
the arrest without warrant shall surrender or take the person arrested to the
city fiscal, and the latter shall make the investigation above mentioned and
file, if proper, the corresponding information within the time prescribed by
section 125 of the Revised Penal Code, so that the court may issue a warrant
of commitment for the temporary detention of the accused. And the city fiscal
or his assistants shall make the investigation forthwith, unless it is materially
impossible for them to do so, because the testimony of the person or officer
making the arrest without warrant is in such cases ready and available, and
shall, immediately after the investigation, either release the person arrested
or file the corresponding information. If the city fiscal has any doubt as to
the probability of the defendant having committed the offense charged, or is
not ready to file the information on the strength of the testimony or evidence
presented, he should release and not detain the person arrested for a longer
period than that prescribed in the Penal Code, without prejudice to making
or continuing the investigation and filing afterwards the proper information
against him with the court, in order to obtain or secure a warrant of his arrest.
Of course, for the purpose of determining the criminal liability of an officer
detaining a person for more than six hours prescribed by the Revised Penal
Code, the means of communication as well as the hour of arrest and other
circumstances, such as the time of surrender and the material possibility for
the fiscal to make the investigation and file in time the necessary information,
must be taken into consideration.

To consider the city fiscal as the judicial authority referred to in article 125
of the Revised Penal Code, would be to authorize the detention of a person
arrested without warrant for a period longer than that permitted by law
without any process issued by a court of competent jurisdiction. The city
fiscal, may not, after due investigation, find sufficient ground for filing an
information or prosecuting the person arrested and release him, after the latter
had been illegally detained for days or weeks without any process issued by
a court or judge.

A peace officer has no power or authority to arrest a person without a warrant


upon complaint of the offended party or any other person, except in those
cases expressly authorized by law. What he or the complainant may do in
such case is to file a complaint with the city fiscal of Manila, or directly with
the justice of the peace courts in municipalities and other political
subdivisions. If the City Fiscal has no authority, and he has not, to order the
arrest of a person charged with having committed a public offense even if he
finds, after due investigation, that there is a probability that a crime has been
committed and the accused is guilty thereof, a fortiori a police officer has no
authority to arrest and detain a person charged with an offense upon
complaint of the offended party or other persons even though, after
investigation, he becomes convinced that the accused is guilty of the offense
charged.

In view of all the foregoing, without making any pronouncement as to the


responsibility of the officers who intervened in the detention of the
petitioners, for the policeman Dumlao may have acted in good faith, in the
absence of a clear cut ruling on the matter, in believing that he had complied
with the mandate of article 125 by delivering the petitioners within six hours
to the office of the city fiscal, and the latter might have ignored the fact that
the petitioners were being actually detained when the said policeman filed a
complaint against them with the city fiscal, we hold that the petitioners are
being illegally restrained of their liberty, and their release is hereby ordered
unless they are now detained by virtue of a process issued by a competent
court of justice. So ordered.
PEOPLE OF THE PHILIPPINES vs. CUIZON preparatory to their boarding a car. While there, accused Cuizon, together
G.R. No. 109287 | 1996-04-18 with his wife, handed four (4) traveling bags to accused Steve Pua y Clofas
PANGANIBAN, J.: and accused Paul Lee y Wong, who were at the vicinity of the Arrival Area.
Accused Pua and Lee loaded the bags in a taxicab which they boarded in
In deciding the case at bench, the Court reiterates doctrines on illegal leaving the airport. Accused Cuizon and his wife took another vehicle (tsn,
searches and seizures, and the requirements for a valid warrantless search May 19, 1992, pp. 4-5, 8-9).
incident to a valid warrantless arrest. While the Court appreciates and
encourages pro-active law enforcement, it nonetheless upholds the At this juncture, Diño, who was observing the activities of the accused,
sacredness of constitutional rights and repeats the familiar maxim, "the end radioed the group of Yap at the parking area, describing the vehicle boarded
never justifies the means." by accused Pua and Lee so that Yap and his companions could apprehend the
two. However, the message of Diño was not completely received by his
This is an appeal from the Decision 1 dated January 5, 1993 (Criminal Case teammates as the radio he was using ran short of battery power (tsn, May 13,
No. 92-0230) of the Regional Trial Court, Branch 116, 2 Pasay City finding 1992, pp. 25-26).
appellants guilty of violating Section 15 of R.A. 6425, otherwise known as
the Dangerous Drugs Act of 1972. Immediately after the vehicle boarded by Pua and Lee had left, Diño
proceeded to the place where his companions were stationed for the purpose
On March 10, 1992, an Information 3 was filed against the appellants of giving assistance to them, believing that they were already in the process
charging them as follows: of apprehending accused Pua and Lee. When he realized that the two accused
were not apprehended, Diño told the group of Yap to follow him as he was
That on or about February 21, 1992 in Pasay City, Philippines and within the following the vehicle taken by Pua and Lee which, according to an earlier tip
jurisdiction of this Honorable Court, the above-named accused, conspiring, he learned, was proceeding to the Manila Peninsula Hotel in Makati, Metro
confederating and mutually helping one another, did then and there, willfully, Manila (tan, May 19, 1992 pp. 25-26; tsn, May 21, 1992, pp. 6, 15).
unlawfully and feloniously carry and transport into the country, without
lawful authority, 16 kilograms, more or less, of METHAMPHETAMINE Upon arriving at about 2:00 p.m. of the same date of February 21, 1992, in
HYDROCHLORIDE, also popularly known as "SHABU", a regulated drug. the Manila Peninsula Hotel, in whose premises the taxicab boarded by
accused Pua and Lee entered, Diño and the other members of the team
CONTRARY TO LAW. coordinated with Col. Regino Arellano, Chief Security Officer of the hotel,
for the purpose of apprehending the two accused. A verification made by the
Upon arraignment, appellant Antolin Cuizon, assisted by counsel de parte, Chief Security Officer showed that accused Pua and Lee occupied Room 340
pleaded not guilty. During the arraignment of appellants Paul Lee and Steve of the hotel. The two accused allowed Diño and Yap, together with Col.
Pua, the latter translated the information into Chinese-Cantonese for the Arellano, to enter their room. Found inside Room 340 were four (4) traveling
understanding of appellant Lee, who does not speak nor understand English, bags, which were similar to the ones handed by accused Cuizon to accused
Pilipino or any other Philippine dialect. Both of them, duly assisted by their Pua and Lee at the Arrival Area of the NAIA. After having introduced
counsel, also pleaded not guilty. 4 Trial ensued and on January 5, 1993, the themselves as NBI agents, Diño and Yap were permitted by accused Pua and
court a quo found appellants guilty as charged and rendered the following Lee to search their bags in the presence of Col. Arellano. The permission was
disposition: 5 made in writing. (Exh. I). Three (3) of the four (4) bags each yielded a plastic
package containing a considerable quantity of white crystalline substance
WHEREFORE, accused Antolin Cuizon y Ortega, Steve Pua y Clofas alias suspected to be methamphethamine hydrochloride or "shabu". Each package
Stephen Po y Uy or Tommy Sy, and Paul Lee y Wong, alias Paul Leung, are was sandwiched between two (2) pieces of board which appear to be
found guilty beyond reasonable doubt of transporting, without legal "lawanit" placed at the bottom of each of the three (3) bags. The suspected
authority, methamphetamine hydrochloride, or "shabu", a regulated drug, as "shabu" contained in one bag weighed 2.571 kilos, that found in the other
charged in the aforequoted Information; and they are each sentenced to suffer had a weight of 2.768 kilos, and the suspected "shabu" retrieved from the
the penalty of life imprisonment and to pay a fine of P20,000.00. third bag weighed 2.970 kilos. Pua and Lee were then apprehended by Diño
and his companions (tsn, May 20, 1992, pp. 9-13; tsn, May 7, 1992, p. 9,
The methamphetamine hydrochloride or "shabu" involved in this case is Exh. "F-2", p. 75, Records).
declared forfeited in favor of the government and is ordered turned over to
the Dangerous Drug Board for proper disposal. Immediately thereafter, Diño and the other members of the team proceeded
to the house of accused Cuizon in Caloocan City, taking with them accused
The Facts Pua and Lee and the bags with their contents of suspected dangerous drugs.
According to the Prosecution They reached the place at about 5:50 in the afternoon of the same date of
February 21, 1992. Retrieved from accused Cuizon in his residence was
The facts as summarized by the trial court and adopted by the Solicitor another bag also containing a white crystalline substance weighing 2.695
General, who added the page references to the transcript of stenographic kilos, likewise believed to be methamphetamine hydrochloride or "shabu".
notes as indicated in brackets, are as follows: 6 In addition, a .38 Cal. firearm was taken from accused Cuizon (tsn, May 19,
1992, pp. 10-11).
In January 1992, the Reaction Group of the National Bureau of Investigation
(NBI) gathered an information regarding the drug, activities of accused Pua, Lee, Cuizon and his wife were then brought by the arresting officers to
Antolin Cuizon y Ortega and his wife, Susan Cuizon. A surveillance was the NBI headquarters at Taft Avenue, Manila, for further investigation. They
conducted on them. The residence of the spouses was traced to Caloocan City were subsequently referred to the Prosecution Division of the Department of
(tsn, May 19, 1992, pp. 17-18, 21). Justice for inquest. However, only the present three accused were charged in
court (tsn, May 19, 1992, pp. 12-13, 16-17).
In the morning of February 21, 1992, the Reaction Group received a report
from its informant in Hong Kong that accused Cuizon, together with his wife, In the meantime, at about 5:30 p.m. of the same date of February 21, 1992,
was arriving on the same day at the Ninoy Aquino International Airport Joselito Soriano, roomboy of the Manila Peninsula Hotel, while cleaning
(NAIA) in Pasay City, Metro Manila, from the British crown colony, Room 340, observed that a portion of the ceiling was misaligned. While
carrying with him a big quantity of "shabu". A team was organized to fixing it, he discovered in the ceiling a laundry bag containing suspected
intercept the suspects. Heading the team was Jose Yap, with Ernesto Diño, "shabu" of more than five (5) kilos (Exh. "X," p. 110). Informed of the
Marcelino Amurao, Jose Bataller and Alfredo Jacinto, as members. Some discovery while they were already in their office in the NBI, Yap and some
belonged to the Narcotics Division and the others to the Reaction Group of companions returned to the hotel. The suspected "shabu" was turned over to
the NBI (tsn, May 19, 1992, pp. 4, 18). them (tsn, May 20, 1992, pp. 19-22).

Arriving at the NAIA shortly before 12:00 noon of February 21, 1992, Diño When examined in the Forensic Chemistry Section of the NBI, the white
positioned himself at the Arrival Area, while Yap and the other members of crystalline substance taken from the three (3) travelling bags found in the
the team posted themselves at the parking area of the airport. At about 12:45 room of accused Pua and Lee in the Manila Peninsula hotel, the white
in the afternoon of the same date, accused Cuizon and his wife, who had just crystalline substance retrieved from the bag confiscated from accused Cuizon
returned from Hong Kong, after passing through the Immigration and in his house in Caloocan City, and the white crystalline substance hidden in
Customs Areas at the NAIA, proceeded to the Arrival Area of the airport the ceiling of Room 340 of the hotel were confirmed to be methamphetamine
hydrochloride or "shabu", a regulated drug. (Board Regulation No. 6, dated II. The trial court erred in giving credence to the testimonies of prosecution
December 11, 1972, of the Dangerous Drugs Board) (tsn, May 7, 1992, p. witnesses Marcelino Amurao, Jose Yap and Ernesto Diño despite
12). contradictions made on material points.

The Defense's Version(s) III. The trial court erred in not giving accused Paul Lee the opportunity to
present his evidence in his defense in violation of his constitutional right to
Appellant Pua, on his part, interposed the defense of alibi. On direct due process.
examination, he testified that at the time of the alleged commission of the
offense, he and his co-appellant Lee were in their room at the Manila Appellant Cuizon, in a separate brief, essentially reiterates the first two
Peninsula Hotel. 7 His version of what happened on February 21, 1992 can assignments of errors above-quoted, and in addition challenges the legality
be summarized as follows: and validity of his warrantless arrest and the search and seizure incidental
thereto. 15
At around 9:30 in the morning, he accompanied appellant Paul Lee to check-
in at the Manila Peninsula Hotel for and in behalf of the latter's personal As this Court sees it, the resolution of this case hinges on the pivotal question
friend named Leong Chong Chong or Paul Leung, who was expected to of the legality of the arrest and search of herein appellants effected by the
arrive that evening because of a delayed flight. Appellant Pua was engaged NBI operatives. Put differently, were the warrantless arrests and the
by appellant Lee to act as interpreter as Lee does not know how to speak warrantless searches conducted by the NBI legal and constitutional?
English and the local language. 8
The answer to this threshold question determines whether the judgment of
While in Room 340, past 1:00 in the afternoon, they received a call from the the court a quo will stand or fall. Consequently, there is a need to resolve first
lobby informing them of the arrival of Paul Leung's luggage. At Pua's this issue before endeavoring to consider the other issues raised by
instructions, the said luggage were brought to the room by a bellboy. appellants.
Thereafter, two persons knocked on their door, accompanied by a "tomboy"
and a thin man with curly hair. The two men identified themselves as NBI A necessary side issue to be considered is, assuming the searches and arrests
agents and asked appellant Pua to let them in. He declined since he did not to have been illegal, whether failure by appellants Pua and Lee to explicitly
know who they were. However, when Col. Arellano, the Chief Security assign the same as errors before this Court amounted to a waiver of their
Officer of the hotel, arrived and identified the two NBI agents, he and Lee constitutional rights against such illegal searches and arrests.
relented and permitted them to enter. Thereafter, he and Lee were told by the
agents to sign a piece of paper. Made to understand that they were merely The Court's Ruling
giving their consent for the agents to enter their room, Pua and Lee signed
the same. Whereupon, the agents told them that they will open Paul Leung's General Rule on Warrantless Arrests, Searches & Seizures
bags. Again appellant Pua refused, saying that the bags did not belong to
them. Just the same, the agents, without appellants Pua and Lee's consent, Well entrenched in this country is the rule that no arrest, search and seizure
opened the bags and found the shabu. Pua and Lee were then apprehended can be made without a valid warrant issued by a competent judicial authority.
and brought to the NBI headquarters. 9 So sacred is this right that no less than the fundamental law of theland 16
ordains it:
Appellant Cuizon, on the other hand, flatly rejected the prosecution's version
of the incident. While admitting that on February 21, 1992, he and his wife The right of the people to be secure in their persons, houses, papers and
Susan did arrive from Hong Kong with several pieces of luggage, he denied effects against unreasonable searches and seizures of whatever nature and for
that he met Pua and Lee at the arrival area of the airport, much less passed to any purpose, shall be inviolable, and no search warrant or warrant of arrest
them the four pieces of luggage. According to him, only his two-year old son, shall issue except upon probable cause to be determined personally by the
accompanied by his cousin, Ronald Allan Ong, met them outside the airport. judge after examination under oath or affirmation of the complainant and the
Ong fetched them from the airport and brought them to their home in witnesses he may "produce, and particularly describing the place to be
Caloocan city. They arrived at their house around 3:00 in the afternoon. 10 Searched, and the persons or things to be seized."

About two hours later, while he was resting together with his wife and son It further decrees that any evidence obtained in violation of said right shall
on his bed, two NBI agents suddenly barged in and poked a gun at him. They be inadmissible for any purpose in any proceeding. 17
manhandled him in front of his wife and son. His hands were tied with a
necktie and he was forcibly brought out of their house while the NBI agents However, the right against warrantless arrest and search and seizure is not
ransacked the place without any warrant. He, his wife Susan, and his cousin absolute. Thus, under Section 5 of Rule 113 of the Revised Rules of Court,
Ronald Allan Ong, were afterwards brought to the NBI Headquarters in an arrest without a warrant may be lawfully made by a peace officer or a
Manila and there the NBI agents continued mauling him. 11 private person:

Appellant Cuizon's wife Susan, his cousin Ronald Allan Ong, and his nephew (a) When, in his presence, the person to be arrested has committed, is actually
Nestor Dalde, testified in his favor basically reiterating or confirming his committing, or is attempting to commit an offense;
testimony. 12
(b) When an offense has in fact just been committed, and he has personal
Unfortunately, appellant Paul Lee, who does not speak or understand a word knowledge of facts indicating that the person to be arrested has committed it;
of English or Pilipino and only knows Chinese-Cantonese, was not able to and
take the witness stand for lack of an interpreter who would translate his
testimony to English. In the hearing set on October 28, 1992, the last trial (c) When the person to be arrested is a prisoner who has escaped from a penal
date allotted to the defense for the reception of Lee's testimony, his counsel, establishment or place where he is serving final judgment or temporarily
although notified of the proceedings, did not appear. Thus, the trial court confined while his case is pending, or has escaped while being transferred
deemed him and Pua to have waived their right to present additional from one confinement to another.
evidence, 13 and the case was considered submitted for decision after the
filing of memoranda. The counsel for Pua and Lee did not ask for the On the occasion of any of the aforementioned instances of legitimate arrest
reconsideration of such ruling; neither did he submit any memorandum. Only without warrant, the person arrested may be subjected to a search of his body
accused Cuizon, who was assisted by another counsel, was able to submit his and of his personal effects or belongings, "for dangerous weapons or
memorandum. anything which may be used as proof of the commission of an offense,"
likewise without need of a search warrant. 18
The Issues
However, where a person is searched without a warrant, and under
In their brief, appellants Pua and Lee made the following assignments of circumstances other than those justifying a warrantless arrest, as discussed
errors: 14 above, upon a mere suspicion that he has embarked on some criminal
activity, and/or for the purpose of discovering if indeed a crime has been
I. The trial court erred in finding conspiracy among the accused. committed by him, then the search made of such person as well as his arrest
are deemed illegal. 19 Consequently, any evidence which may have been
obtained during such search, even if tending to confirm or actually personal knowledge of that fact. The offense must also be committed in his
confirming such initial suspicion, is absolutely inadmissible for any purpose presence or within his view. (Sayo v. Chief of Police, 80 Phil. 859).
and in any proceeding, 20 the same being "the fruit of the poisonous tree".
21 Emphasis is to be laid on the fact that the law requires that the search be The same decision is highly instructive as it goes on to state:
incident to a lawful arrest, in order that the search itself may likewise be
considered legal. Therefore, it is beyond cavil that a lawful arrest must The Solicitor General is of the persuasion that the arrest may still be
precede the search of a person and his belongings. Were a search first considered lawful under Section 6 (b) using the test of reasonableness. He
undertaken, then an arrest effected based on evidence produced by the search, submits that the information given by Cesar Masamlok was sufficient to
both such search and arrest would be unlawful, for being contrary to law. induce a reasonable ground (for belief) that a crime has been committed and
that the accused is probably guilty thereof.
The Instant Case Does Not Fall Under
The Exceptions for Warrantless Searches, etc. In arrests without a warrant under Section 6(b), however, it is not enough that
there is reasonable ground to believe that the person to be arrested has
Re-assessing the factual backdrop of the case at bench, this Court cannot committed a crime. A crime must in fact or actually have been committed
agree with and accept the conclusion of the trial court that the appellants were first. That a crime has actually been committed is an essential precondition.
caught in flagrante delicto which would justify the search without a warrant. It is not enough to suspect that a crime may have been committed. The fact
The shaky reasoning of the court a quo gives away the baselessness of its of the commission of the offense must be undisputed. The test of reasonable
findings and conclusion: ground applies only to the identity of the perpetrator.

. . . the search conducted on their bags in the hotel room could still be In this case, the accused was arrested on the sole basis of Masamlok's verbal
regarded as valid for being incidential to a lawful arrest. . . . The arrest of report. Masamlok led the authorities to suspect that the accused had
accused Pua and Lee without a warrant of arrest was lawful, as they could be committed a crime. They were still fishing for evidence of a crime not yet
considered to have committed the crime of transporting "shabu" in the ascertained. The subsequent recovery of the subject firearm on the basis of
presence of the arresting officers from the time they received the bags information from the lips of a frightened wife cannot make the arrest lawful.
containing the regulated drug in the airport up to the time they brought the ...
bags to the hotel. Or their arrest without a warrant was legal as falling under
the situation where an offense had in fact just been committed, and the The foregoing doctrine was affirmed in the case of Alih vs. Castro, 27 where
arresting officers had personal knowledge of facts indicating that the said this Court ruled that ". . . under the Revised Rule 113, Section 5(b), the officer
accused were the ones who committed it. . . . 22 making the arrest must have personal knowledge of the ground therefor as
stressed in the recent case of People v. Burgos."
Scrutinizing the provisions of Sec. 5 of Rule 113 of the Rules of Court on
lawful arrests without warrant, we note that par. (c) of said section is In the case at bench, not only did the NBI agents rely merely on hearsay
obviously inapplicable, the appellants not being escapees from a penal information ("tips"), but they were completely uncertain that anything was
institution at the time of arrest. Par. (a) an the other hand requires that the really "going down" that day. That much is undisputed, from a reading of the
person be arrested (i) after he has committed or while he is actually testimony of Agent Diño:
committing or is at least attempting to commit an offense, (ii) in the presence
of the arresting officer(s). These requirements are not present in the case at Q Now, but you were informed by the personnel of the airport that the
bench, for at the time of their arrest, appellants Pua and Lee were merely spouses Cuizon were going to bring in or transport into the country shabu on
resting in their hotel room, and appellant Cuizon for his part was in bed February 21, 1992?
resting with his wife and child inside his home. No offense had just been A Yes, sir.
committed, or was being actually committed or being attempted by any of
the accused in the presence of the lawmen. 23 Q Now, you were not sure or your group was not sure that they indeed would
bring in shabu, is it not? That was only the information relayed to your group?
Par. (b) of the same provision is likewise inapplicable since its equally A Yes, sir.
exacting requirements have also not been met. The prosecution failed to
establish that at the time of the arrest, an offense had in fact just been Xxx xxx xxx
committed and the arresting officers had personal knowledge of facts
indicating that the accused-appellants had committed it. Appellant Cuizon Q But then you were jumping ahead. You were not sure is it not that they
could not, by the mere act of handing over four pieces of luggage to the other were bringing in shabu?
two appellants, be considered to have committed the offense of "carrying and A Yes, sir. (TSN, May 19, 1992, pp. 37-38.)
transporting" prohibited drugs. Under the circumstances of the case, there
was no sufficient probable cause for the arresting officers to believe that the In his testimony, NBI Investigator Jose Justo Yap, who was with Agent Diño
accused were then and there committing a crime. The act per se of handing during the operation, likewise admitted in substantially the same tenor their
over the baggage, assuming the prosecution's version to be true; cannot in uncertainty regarding the commission of the offense (cf. TSN, May 20, 1992,
any way be considered a criminal act. It was not even an act performed under pp. 29 & 34.).
suspicious circumstances as indeed, it took place in broad daylight,
practically at high noon, and out in the open, in full view of the public. 24 We therefore hold that under the circumstances obtaining, the prosecution
Furthermore, it can hardly be considered unusual, in an airport setting, for failed to establish that there was sufficient and reasonable ground for the NBI
travellers and/or their welcomers to be passing, handing over and delivering agents to believe that appellants had committed a crime at the point when the
pieces of baggage, especially considering the somewhat obsessive penchant search and arrest of Pua and Lee were made; hence, said search and arrest do
of our fellow countrymen for sending along ("pakikipadala") things and gifts not come under the exception in par. (b) of Sec. 5 of Rule 113, and therefore
through friends and relatives. Moreover, one cannot determine from the should be deemed illegal. We eight add that the search conducted on Pua and
external appearance of the luggage that they contained "shabu" hidden Lee was not incident to a lawful warrantless arrest, having preceded the same
beneath some secret panel or false bottom. The only reason why such act of and produced the justification therefor. On the other hand, the search on
parting with luggage took on the color and dimensions of a felonious deed, Cuizon's residence, without the benefit of a search warrant, was clearly
at least as far as the lawmen were concerned, was the alleged tip that the NBI illegal and the "shabu" seized thereat cannot but be considered inadmissible
agents purportedly received that morning, to the effect that appellant Cuizon in evidence. More an these points later.
would be arriving that same day with a shipment of shabu. To quote from
another decision of like import, "(A)11 they had was hearsay information Comparison Between The Present Case and Earlier Decisions of This Court
(from the telephone caller), and about a crime that had yet to be committed."
25 For clarity's sake, it is imperative to compare the foregoing holding with
previous decisions by this Court in various drug cases, in which apparently
In the leading case of People vs. Burgos, 26 this Court laid down clear different conclusions were reached, in order to distinguish them from the
guidelines, as follows: instant case and avoid any potential misunderstanding of the foregoing
holding as well as the constitutional and legal principles on which it is based.
Under Section 6(a) of Rule 113, the officer arresting a person who has just
committed, is committing, or is about to commit an offense must have
1. In People vs. Claudio, 28 the accused, a passenger on a bus bound for papers when asked to do so. The warrantless arrest and search were thus
Baguio city, was arrested by a policeman on the same bus because of the justified.
distinctive odor of marijuana emanating from the plastic bag she was
carrying. The Court held the warrantless arrest under the circumstances to be In all the cases discussed hereinabove, there were facts which were found by
lawful, the search justified and the evidence thus discovered admissible in the Court to provide probable cause justifying warrantless arrests and
evidence. searches, i.e., distinct odor of marijuana, reports about drug transporting or
positive identification by informers, suspicious behaviour, attempt to flee,
2. In People vs. Tangliben, 29 the accused, carrying a travelling bag at a bus failure to produce identification papers, and so on. Too, urgency attended the
terminal, was noticed by lawmen to be acting suspiciously, and was also arrests and searches because each of the above-mentioned cases involved the
positively fingered by an informer as carrying marijuana, and so he was use of motor vehicles and the great likelihood that the accused would get
accosted by policemen who happened to be on a surveillance mission; the away long before a warrant can be procured. And, lest it be overlooked,
lawmen asked him to open the bag, in which was found a package of unlike in the case before us now, the law enforcers in the aforementioned
marijuana leaves. It was held that there was a valid warrantless arrest and cases acted immediately on the information received, suspicions raised, and
search incident thereto. The Court in effect considered the evidence on hand probable causes established, and effected the arrests and searches without
sufficient to have enabled the law enforcers to secure a search warrant had any delay.
there been time, but as the case "presented urgency," and there was actually
no time to obtain a warrant since the accused was about to board a bus, and Unexplained Matters in the Instant Case
inasmuch as an informer had given information "on the spot" that the accused
was carrying marijuana, the search of his person and effects was thus In the case before us, the NBI agents testified that they purportedly decided
considered valid. against arresting the accused-appellants inside the airport as they allegedly
wanted to discover the identities of the airport immigration, security or
3. In Posadas vs. Court of Appeals, 30 the accused was seen acting customs personnel who might be protecting the accused or otherwise
suspiciously, and when accosted by two members of the Davao INP who involved in the drug smuggling activities, and also in order to avoid the
identified themselves as lawmen, he suddenly fled, but was pursued, subdued possibility of an armed encounter with such protectors, which might result in
and placed in custody. The buri bag he was carrying yielded an unlicensed injuries to innocent bystanders. These excuses are simply unacceptable. They
revolver, live ammunition and a tear gas grenade. This Court upheld his are obviously after-thoughts concocted to justify their rank failure to effect
conviction for illegal possession of firearms, holding that there was under the the arrest within constitutional limits. Indeed, the NBI men failed to explain
circumstances sufficient probable cause for a warrantless search. how come they did not apprehend the appellants at the moment Cuizon
handed over the baggage to Pua and Lee, or even afterwards, in relative
4. In People vs. Moises Maspil, Jr., et al., 31 agents of the Narcotics safety. Such arrest would have been consistent with the settled constitutional,
Command set up a checkpoint on a highway in Atok, Benguet, to screen legal and jurisprudential precedents earlier cited.
vehicular traffic on the way to Baguio City due to confidential reports from
informers that Maspil and a certain Bagking would be transporting a large The spouses Cuizon had already passed through the airport security checks
quantity of marijuana. At about 2 a.m. of November 1, 1986, the two allegedly with their contraband cargo undetected in their luggage.
suspects, riding a jeepney, pulled up to the checkpoint and were made to stop. Apparently, the NBI agents did not see (as indeed they did not testify that
The officers noticed that the vehicle was loaded with some sacks and tin cans, they saw) anyone from the airport immigration, security or customs who
which, when opened, were seen to contain marijuana leaves. The Court could have escorted the spouses Cuizon, and therefore, there was no danger
upheld the search thus conducted as being incidental to a valid warrantless of any "live ammo encounter" with such group(s). The alleged drug couriers
arrest. had already made their way outside the NAIA, had allegedly made contact
with the accused Pua and Lee, and were in the very act of handing over the
5. In People vs. Lo Ho Wing, et al., 32 the Court ruled that the search of the luggage to the latter. Why the NBI men did not move in and pounce on them
appellants' moving vehicles and the seizure of "shabu" therefrom was legal, at that very instant has not been satisfactorily explained. Instead, one of the
in view of the intelligence information, including notably, clandestine reports agents, Diño, merely watched as Pua and Lee loaded the luggage into a cab
by a planted deep penetration agent or spy who was even participating in the and took off for Makati. Furthermore, it taxes the imagination too much to
drug smuggling activities of the syndicate, to the effect that appellants were think that at the most critical and climactic moment, when agent Diño radioed
bringing in prohibited drugs into the country. The Court also held that it is his companions for help to close in on the suspects, the most amazing and
not practicable to secure a search warrant in cases of smuggling with the use stupendous thing actually happened: Murphy's Law kicked in whatever could
of a moving vehicle to transport contraband, because the vehicle can be go wrong, did, and at the worst possible time the batteries in Agent Diño's
quickly moved out of the locality or jurisdiction in which the warrant must hand-held radio supposedly went dead and his message was not transmitted.
be sought. Thus the departing Pua and Lee proceeded merrily and unimpeded to the
Peninsula Hotel, while the spouses Cuizon simultaneously sped off to their
6. In People vs. Malmstedt, 33 NARCOM agents stationed at Camp Dangwa, residence in Caloocan City, leaving the lawmen empty-handed and
Mountain Province, set up a temporary checkpoint to check vehicles coming scampering madly to catch up. Such absolutely astounding and incredible
from the Cordillera Region, due to persistent reports that vehicles from happenstance might find a place in a fourth-rate movie script, but expecting
Sagada were transporting marijuana and other drugs, and because of the courts to swallow it hook, line and sinker is infinite naivete, if not
particular information to the effect that a Caucasian would be travelling from downright malevolence.
Sagada that day with prohibited drugs. The bus in which accused was riding
was stopped at the checkpoint. While conducting an inspection, one of the Even granting arguendo that the radio really went dead, nevertheless, the
NARCOM men noticed that accused, the only foreigner on board, had a agents were not thereby rendered helpless or without recourse. The NBI
bulge at the waist area. Thinking it might be a gun, the officer sought agents, numbering five in all, not counting their so-called informant, claimed
accused's passport or other identification papers. When the latter failed to to have piled into three cars (TSN, May 19, 1992) and tailed the suspects Pua
comply, the lawman directed him to bring out whatever it was that was and Lee into Makati, keeping a safe two-car distance behind (TSN, May 20,
bulging at his waist. It was a pouch bag which, when opened by the accused, 1992). The lawmen and the prosecutors failed to explain why the agents did
was found to contain packages of hashish, a derivative of marijuana. Invited not intercept the vehicle in which Pua and Lee were riding, along the way,
for questioning, the accused disembarked from the bus and brought along pull them over, arrest them and search the luggage. And since the agents were
with him two pieces of luggage; found inside were two teddy bears stuffed in three (3) cars, they also could have easily arranged to have agents in one
with more hashish. The Court held that there was sufficient probable cause vehicle follow, intercept and apprehend the Cuizons while the others went
in the premises for the lawmen to believe that the accused was then and there after Pua and Lee. All or any of these possible moves are mere ordinary,
committing a crime and/or trying to hide something illegal from the common-sense steps, not requiring a great deal of intelligence. The NBI men
authorities. Said probable cause arose not only from the persistent reports of who testified claimed to have conducted or participated in previous drug
the transport of prohibited drugs from Sagada, and the "tip" received by the busts or similar operations and therefore must have been familiar with
NARCOM that same day that a Caucasian coming from Sagada would be contingency planning, or at least should have known what to do in this
bringing prohibited drugs, but also from the failure of the accused to present situation where their alleged original plan fell through. At any rate, what the
his passport or other identification papers when confronted by the lawmen, lawmen opted to do, i.e., allow Pua and Lee to freely leave the airport,
which only triggered suspicion on the part of the law enforcers that accused allegedly bringing the drug cache to the hotel, and Cuizon to leave
was trying to hide his identity, it being the normal thing expected of an unimpededly the airport and reach his residence with one of the luggage,
innocent man with nothing to hide, that he readily present identification increased significantly the risk of the suspects (and/or the drugs) slipping
through the lawmen's fingers, and puts into question the regularity of reasonable doubt; and (iii) appellant Cuizon had timely raised before this
performance of their official functions. The agents' alleged actions in this Court the issue of the illegality of his own arrest and the search and seizure
case compare poorly with the forthright and decisive steps taken by lawmen conducted at his residence, and questioned the admission of the seized shabu
in the cases earlier cited where this Court held the arrests and seizures to be in evidence.
valid.
Re: Appellant Steve Pua @ "Tommy Sy"
Had the arrests and searches been made in transitu, i.e., had the agents
intercepted and collared the suspects on the way to Makati and Caloocan, or What has been said for Cuizon cannot, alas, be said for appellant Pua. While
better yet, at the very moment of the hand-over, then there would not have the search and arrest carried out on him and Lee may have been illegal for
been any question at all as to the legality of their arrest and search, as they not being incident to a lawful warrantless arrest, the unfortunate fact is that
would presumably have been caught red-handed with the evidence, and appellant Pua failed to challenge the validity of his arrest and search, as well
consequently for that reason and by the very nature and manner of as the admission of the evidence obtained thereby; he did not raise the issue
commission of the offense charged, there would have been no doubt also as or assign the same as an error before this Court. Accordingly, any possible
to the existence of conspiracy among the appellants to transport the drugs. challenge thereto based on constitutional grounds is deemed waived. This
However, because of the way the operation actually turned out, there is no Court has upheld and recognized waivers of constitutional rights, including,
sufficient proof of conspiracy between Pua and Lee on the one hand, and particularly, the right against unreasonable searches and seizures, in cases
Cuizon on the other, inasmuch as there is no clear and convincing evidence such as People vs. Malasugui 35 and De Garcia vs. Locsin. 36
that the four (4) bags handed by Cuizon to Pua and Lee at the airport were
the very same ones found in the possession of the latter in Room 340 of the Additionally, the prosecution had argued and the trial court agreed that by
Peninsula Hotel. Not one of the NBI agents when testifying could definitely virtue of the handwritten consent (Exhibit "I") secured by the arresting
and positively state that the bags seized from Room 340 were the very same officers from appellants Pua and Lee, the latter freely gave their consent to
ones passed by Cuizon at the airport; at best, they could only say that they the search of their baggage, and thus, the drugs discovered as a result of the
"looked like" the ones they saw at the airport. And even assuming them to be consented search is admissible in evidence. The said written permission is in
the same bags, there remains doubt and uncertainty as to the actual ownership English, and states plainly that they (Pua and Lee) freely consent to the search
of the said bags as at the alleged turnover vis-a-vis the time they were seized of their luggage to be conducted by NBI agents to determine if Pua and Lee
by the agents. For these reasons, we cannot sustain the finding of conspiracy are carrying shabu. It appears that appellant Pua understands both English
as between Cuizon on the one hand and Pua and Lee on the other. Well- and Tagalog; he is born of a Filipino mother, had resided in Vito Cruz,
settled is the rule that conspiracy must be proved independently and beyond Manila, and gave his occupation as that of salesman. He admitted that he was
reasonable doubt. 34 asked to sign the written consent, and that he did in fact sign it (TSN, May
28, 1992, pp. 33-34). His barefaced claim made during his direct and cross
Additionally, in light of the foregoing discussion, we find it extremely examinations to the effect that he did not really read the consent but signed
difficult to subscribe to the trial court's finding as to the existence and it right away, and that by signing it he only meant to give permission for the
sufficiency of probable cause in this case, one major component of which NBI agents to enter the room (and not to search) is hardly worthy of belief,
would have been the alleged information or "tip" purportedly received by the considering that prior to the search, he seemed to have been extra careful
agents as to the expected arrival of the spouses Cuizon that fateful day with about who to let into the hotel room.
a large cache of "shabu". The question that defies resolution in our minds is
why, if indeed the information or "tip" was genuine and from a highly reliable Thus, the full weight of the prosecution's testimonial evidence plus the large
source as claimed by the government agents, did they not act on it? Throw in amount of prohibited drugs found, must be given full force vis-a-vis Pua's
the alleged month-long surveillance supposedly conducted by some of the claim of innocent presence in the hotel room, which is weak and not worthy
NBI people on the Cuizon couple, and the mystery only deepens. Even with of credence.
the so-called tip and the results of surveillance, the government officers were
still seemingly hesitant, reluctant, uncertain, or perhaps afraid, to arrest and Re: Appellant Paul Lee @ "Paul Leung"
search the accused appellants, so much so that the NBI agents who went after
Pua and Lee at the Peninsula Hotel, instead of outrightly cuffing and Appellant Lee's situation is different from that of Pua. We agree with the
searching them, as they were supposed to, opted instead to play it safe and Solicitor General when he noted that the trial judge did not exert sufficient
meekly beseeched the two to sign a written consent for the agents to search effort to make available compulsory process and to see to it that accused-
their personal effects! Indeed, this is one for the books. If this is how appellant Lee was given his day in court. It is clear that appellant Lee was
confident the agents were about their "hot tips", reliable informers and effectively denied his right to counsel, for although he was provided with
undercover surveillance, then we cannot be blamed for failing to appreciate one, he could not understand and communicate with him concerning his
the existence/sufficiency of probable cause to justify a warrantless arrest and defense such that, among other things, no memorandum was filed on his
search in this case. behalf; further, he was denied his right to have compulsory process to
guarantee the availability of witnesses and the production of evidence on his
There is a whole lot more that can be said on this score, but we shall leave it behalf, including the services of a qualified and competent interpreter to
at that for now. We shall now dispose of the appeals of the accused-appellants enable him to present his testimony. 37 In sum, he was denied due process.
individually. For this reason, we hold that the case as against Lee must be remanded to the
court of origin for a re-trial.
Re: Appellant Antolin Cuizon
Epilogue
The search of the house of appellant Cuizon, having been conducted without
any warrant, and not on the occasion or as an incident of a valid warrantless It is evident and clear to us that the NBI agents gravely mishandled the drug
arrest, was indubitably illegal, and the shabu seized thereat could not be bust operation and in the process violated the constitutional guarantees
admissible in evidence. That is why even the trial judge did not make an against unlawful arrests and illegal searches and seizures. Because of the
effort to hold him liable under such seizure. He lamely argued: "(A)t any rate, large haul of illegal drugs that the government officers claimed to have
accused Cuizon is not held criminally liable in this case in connection with recovered, this Court agonized over the case before us and struggled to apply
the bag containing "shabu" confiscated from his residence. His responsibility the law with an even hand. In the final analysis, we in the administration of
is based on the bags containing "shabu" which he handed to Pua and Lee at justice would have no right to expect ordinary people to be law-abiding if we
the NAIA. Consequently, even if the bag and its contents of "shabu" taken do not insist on the full protection of their rights.
from his house were not admitted in evidence, the remaining proofs of the
prosecution would still be sufficient to establish the charge against him." Some lawmen, prosecutors and judges may still tend to gloss over an illegal
However, contrary to the trial judge's conclusion, we hold that insofar as search and seizure as long as the law enforcers show the alleged evidence of
Cuizon is concerned, all the evidence seized are considered fruit of the the crime regardless of the methods by which they were obtained. This kind
poisonous tree and are inadmissible as against him, and thus, he should be of attitude condones law-breaking in the name of law enforcement.
acquitted, since, as shown hereinabove, (i) the warrantless search conducted Ironically, it only fosters the more rapid breakdown of our system of justice,
on Pua and Lee was clearly illegal per se, not being incident to a valid and the eventual denigration of society. While this Court appreciates and
warrantless arrest either; (ii) and even if the search on Pua and Lee were not encourages the efforts of law enforcers to uphold the law and to preserve the
illegal, conspiracy as between Cuizon on the one hand and appellants Pua peace and security of society, we nevertheless admonish them to act with
and Lee on the other had not been established by sufficient proof beyond
deliberate care and within the parameters set by the Constitution and the law.
Truly, the end never justifies the means.

WHEREFORE, in view of the foregoing considerations, accused-appellant


Antolin Cuizon y Ortega is hereby ACQUITTED on constitutional grounds.
His immediate release is ordered unless he is detained for other valid causes.
Accused-appellant Steve Pua y Clofas is hereby found GUILTY of the crime
of Illegal Transport of Regulated Drugs, penalized under Section 15, R.A.
No. 6425, as amended, and is hereby sentenced to suffer the penalty of
reclusion perpetua; the Decision appealed from, as herein modified, is hereby
affirmed as to appellant Pua. Finally, the case as to appellant Lee is hereby
ordered REMANDED to the trial court in order that said accused may be
given his day in court. The Decision appealed from is also AFFIRMED with
respect to the disposition of the prohibited drugs involved in the case.

SO ORDERED.
VALMONTE vs. DE VILLA
G.R. No. 83988 September 29, 1989 Where, for example, the officer merely draws aside the curtain of a vacant
PADILLA, J.: vehicle which is parked on the public fair grounds, 7 or simply looks into a
vehicle, 8 or flashes a light therein, 9 these do not constitute unreasonable
This is a petition for prohibition with preliminary injunction and/or search.
temporary restraining order, seeking the declaration of checkpoints in
Valenzuela, Metro Manila or elsewhere, as unconstitutional and the The setting up of the questioned checkpoints in Valenzuela (and probably in
dismantling and banning of the same or, in the alternative, to direct the other areas) may be considered as a security measure to enable the NCRDC
respondents to formulate guidelines in the implementation of checkpoints, to pursue its mission of establishing effective territorial defense and
for the protection of the people. maintaining peace and order for the benefit of the public. Checkpoints may
also be regarded as measures to thwart plots to destabilize the government,
Petitioner Ricardo C. Valmonte sues in his capacity as citizen of the in the interest of public security. In this connection, the Court may take
Republic, taxpayer, member of the Integrated Bar of the Philippines (IBP), judicial notice of the shift to urban centers and their suburbs of the insurgency
and resident of Valenzuela, Metro Manila; while petitioner Union of Lawyers movement, so clearly reflected in the increased killings in cities of police and
and Advocates for People's Rights (ULAP) sues in its capacity as an military men by NPA "sparrow units," not to mention the abundance of
association whose members are all members of the IBP. unlicensed firearms and the alarming rise in lawlessness and violence in such
urban centers, not all of which are reported in media, most likely brought
The factual background of the case is as follows: about by deteriorating economic conditions — which all sum up to what one
can rightly consider, at the very least, as abnormal times. Between the
On 20 January 1987, the National Capital Region District Command inherent right of the state to protect its existence and promote public welfare
(NCRDC) was activated pursuant to Letter of Instruction 02/87 of the and an individual's right against a warrantless search which is however
Philippine General Headquarters, AFP, with the mission of conducting reasonably conducted, the former should prevail.
security operations within its area of responsibility and peripheral areas, for
the purpose of establishing an effective territorial defense, maintaining peace True, the manning of checkpoints by the military is susceptible of abuse by
and order, and providing an atmosphere conducive to the social, economic the men in uniform, in the same manner that all governmental power is
and political development of the National Capital Region.1 As part of its duty susceptible of abuse. But, at the cost of occasional inconvenience, discomfort
to maintain peace and order, the NCRDC installed checkpoints in various and even irritation to the citizen, the checkpoints during these abnormal
parts of Valenzuela, Metro Manila. times, when conducted within reasonable limits, are part of the price we pay
for an orderly society and a peaceful community.
Petitioners aver that, because of the installation of said checkpoints, the
residents of Valenzuela are worried of being harassed and of their safety Finally, on 17 July 1988, military and police checkpoints in Metro Manila
being placed at the arbitrary, capricious and whimsical disposition of the were temporarily lifted and a review and refinement of the rules in the
military manning the checkpoints, considering that their cars and vehicles are conduct of the police and military manning the checkpoints was ordered by
being subjected to regular searches and check-ups, especially at night or at the National Capital Regional Command Chief and the Metropolitan Police
dawn, without the benefit of a search warrant and/or court order. Their Director. 10
alleged fear for their safety increased when, at dawn of 9 July 1988, Benjamin
Parpon, a supply officer of the Municipality of Valenzuela, Bulacan, was WHEREFORE, the petition is DISMISSED.
gunned down allegedly in cold blood by the members of the NCRDC
manning the checkpoint along McArthur Highway at Malinta, Valenzuela, SO ORDERED.
for ignoring and/or refusing to submit himself to the checkpoint and for
continuing to speed off inspire of warning shots fired in the air. Petitioner
Valmonte also claims that, on several occasions, he had gone thru these
checkpoints where he was stopped and his car subjected to search/check-up
without a court order or search warrant.

Petitioners further contend that the said checkpoints give the respondents a
blanket authority to make searches and/or seizures without search warrant or
court order in violation of the Constitution; 2 and, instances have occurred
where a citizen, while not killed, had been harassed.

Petitioners' concern for their safety and apprehension at being harassed by


the military manning the checkpoints are not sufficient grounds to declare
the checkpoints as per se illegal. No proof has been presented before the
Court to show that, in the course of their routine checks, the military indeed
committed specific violations of petitioners' right against unlawful search
and seizure or other rights.

In a case filed by the same petitioner organization, Union of Lawyers and


Advocates for People's Right (ULAP) vs. Integrated National Police, 3 it was
held that individual petitioners who do not allege that any of their rights were
violated are not qualified to bring the action, as real parties in interest.

The constitutional right against unreasonable searches and seizures is a


personal right invocable only by those whose rights have been infringed, 4
or threatened to be infringed. What constitutes a reasonable or unreasonable
search and seizure in any particular case is purely a judicial question,
determinable from a consideration of the circumstances involved. 5

Petitioner Valmonte's general allegation to the effect that he had been


stopped and searched without a search warrant by the military manning the
checkpoints, without more, i.e., without stating the details of the incidents
which amount to a violation of his right against unlawful search and seizure,
is not sufficient to enable the Court to determine whether there was a
violation of Valmonte's right against unlawful search and seizure. Not all
searches and seizures are prohibited. Those which are reasonable are not
forbidden. A reasonable search is not to be determined by any fixed formula
but is to be resolved according to the facts of each case. 6
PEOPLE OF THE PHILIPPINES vs. BOLANOS "(1) Any person under investigation for the commission of an offense shall
G.R. No. 101808 | 1992-07-03 have the right to remain silent and to have competent and independent
PARAS, J.: preferably of his own choice. If the person cannot afford the service of
counsel, he must be provided with one. These rights cannot be waived except
This is a review of the decision of the Regional Trial Court of Malolos, in writing and in the presence of counsel.
Bulacan, Branch 14, under Criminal Case No. 1831-M-90, for "Murder",
wherein the accused-appellant, Ramon Bolanos was convicted, as follows: "(2) No torture, force, violence, threat, intimidation, or any other means
which vitiate the free will shall be used against him. Secret detention places,
"WHEREFORE, judgment is rendered finding the accused guilty beyond solitary, incommunicado, or other similar forms of detention are prohibited.
reasonable doubt of the Crime of Murder and the Court hereby imposed upon
the accused Ramon Bolanos the penalty of Reclusion Perpetua (life "(3) Any confession or admission obtained in violation of this or the
imprisonment) and to pay the heirs of the victim P50,000.00. With Costs. preceding section shall be inadmissible in evidence against him.

"SO ORDERED." (Judgment, p. 6) "(4) The law shall provide for penal and civil sanctions for violation of this
section as well as compensation and rehabilitation of victims of torture or
The antecedent facts and circumstances, follow: similar practices and their families." (Underscoring supplied)

The evidence for the prosecution consisted of the testimonies of Patrolmen Considering the clear requirements of the Constitution with respect to the
Marcelo J. Fidelino and Francisco Dayao of the Integrated National Police manner by which confession can be admissible in evidence, and the glaring
(INP), Balagtas, Bulacan, Calixto Guinsaya, and Dr. Benito Caballero, fact that the alleged confession obtained while on board the police vehicle
Medico-Legal Officer of Bocaue, Bulacan and documentary exhibits. The was the only reason for the conviction, besides appellant's conviction was not
testimonial evidence were after the fat narration of events based on the report proved beyond reasonable doubt, this Court has no recourse but to reverse
regarding the death of the victim, Oscar Pagdalian which was communicated the subject judgment under review.
to the Police Station where the two (2) policemen who responded to the
incident are assigned and subsequently became witnesses for the prosecution. WHEREFORE, finding that the Constitutional rights of the accused-
(Appellant's Brief, p. 2) appellant have been violated, the appellant is ACQUITTED, with costs de
oficio.
Patrolmen Rolando Alcantara and Francisco Dayao testified that they
proceeded to the scene of the crime of Marble Supply, Balagtas, Bulacan and SO ORDERED.
upon arrival they saw the deceased Oscar Pagdalian lying on an improvised
bed full of blood with stab wounds. They then inquired about the
circumstances of the incident and were informed that the deceased was with
two (2) companions, on the previous night, one of whom was the accused
who had a drinking spree with the deceased and another companion (Claudio
Magtibay) till the wee hours of the following morning, June 23, 1990. (Ibid.,
p. 3)

The corroborating testimony of Patrolmen Francisco Dayao, further


indicated that when they apprehended the accused-appellant, they found the
firearm of the deceased on the chair where the accused was allegedly seated;
that they boarded Ramon Bolanos and Claudio Magtibay on the police
vehicle and brought them to the police station. In the vehicle where the
suspect was riding, "Ramon Bolanos accordingly admitted that he killed the
deceased Oscar Pagdalian because he was abusive." (Ibid., p. 4)

During the trial, it was clearly established that the alleged oral admission of
the appellant was given without the assistance of counsel as it was made
while on board the police vehicle on their way to the police station. The
specific portion of the decision of the court a quo reads as follows:

". . . the police boarded the two, the accused Ramon Bolanos and Claudio
Magtibay in their jeep and proceeded to the police station of Balagtas,
Bulacan to be investigated, on the way the accused told the police, after he
was asked by the police if he killed the victim, that he killed the victim
because the victim was abusive; this statement of the accused was considered
admissible in evidence against him by the Court because it was given freely
and before the investigation.

"The foregoing circumstances clearly lead to a fair and reasonable conclusion


that the accused Ramon Bolanos is guilty of having killed the victim Oscar
Pagdalian." (Judgment, p. 6)

A Manifestation (in lieu of Appellee's Brief), was filed by the Solicitor


General's Office, dated April 2, 1992, with the position that the lower court
erred in admitting in evidence the extra-judicial confession of appellant while
on board the police patrol jeep. Said office even postulated that: "(A)ssuming
that it was given, it was done in violation of appellant's Constitutional right
to be informed, to remain silent and to have a counsel of his choice, while
already under police custody." (Manifestation, p. 4)

Being already under custodial investigation while on board the police patrol
jeep on the way to the Police Station where formal investigation may have
been conducted, appellant should have been informed of his Constitutional
rights under Article III, Section 12 of the 1987 Constitution which explicitly
provides:
PEOPLE OF THE PHILIPPINES vs. BASAY After both accused entered a not guilty plea during their arraignment on 23
G.R. No. 86941 | 1993-03-03 February 1987, 7 trial on the merits ensued. The prosecution presented Dr.
DAVIDE, JR., J p: Edgardo Barredo, MCTC Judge Teopisto Calumpang, Jaime Saguban, Sgt.
Reynaldo Tabanao, Dr. Edgar Gantalao and Dr. Lucio Togonon as its
Teodoro Basay and Jaime Ramirez were charged with Multiple Murder with witnesses for the evidence in chief, and Judge Calumpang and Elpedio
Arson in a criminal complaint 1 filed on 24 March 1986 with the Municipal Catacutan in rebuttal; for its surrebuttal, Pfc. Urbano Cavallida was
Circuit Trial Court (MCTC) of Pamplona-Amlan-San Jose in the Province of presented. On the other hand, the accused testified for the defense together
Negros Oriental for having allegedly killed the spouses Zosimo and Beatrice with witnesses Joven Lopez and Maxima Basay. Accused Ramirez took the
Toting and their six-year old daughter, Bombie, and for having burned the witness stand again in surrebuttal.
said spouses' house to conceal the crime; as a consequence of such fire, the
spouses' other daughter, Manolita, was burned to death. On 15 December 1988, the trial court promulgated its Decision, dated 14
December 1988, acquitting accused Teodoro Basay but convicting accused
On 31 March 1986, the MCTC issued a warrant for the arrest of the accused; Jaime Ramirez. 8 Its dispositive portion reads:
no bail was recommended. 2 It appears, however, that the accused had earlier
been apprehended on 6 March 1986 by elements of the Philippine WHEREFORE, the prosecution having failed to prove the guilt of the
Constabulary (PC) and Civilian Home Defense Forces (CHDF) and were accused beyond reasonable doubt for the crime of Multiple Murder,
detained at the Pamplona municipal jail. Frustrated Murder With Arson against the accused Teodoro Basay, this Court
hereby finds said accused Teodoro Basay NOT GUILTY and orders his
On 15 April 1986, the accused filed a Waiver of Preliminary Investigation 3 immediate release from detention.
which prompted the MCTC, the following day, to order the clerk of court to
forward the records of the case to the Office of the Provincial Fiscal. 4 The prosecution has proven the guilt of the accused beyond reasonable doubt
for the crime of Multiple Murder, Frustrated Murder With Arson against
Meanwhile, on 14 August 1986, the Integrated National Police (INP) Station accused Jaime Ramirez (sic), this Court finds him GUILTY to (sic) said
Commander of Pamplona amended the complaint by including therein the crime and hereby sentences him to suffer the penalty of life imprisonment
name of another victim, Manolo Toting, who suffered second and third and to indemnify the heirs of the victims in the sum of Thirty Thousand
degree burns because of the burning of the house. 5 (P30,000.00) Pesos as his civil indemnity.

On 11 December 1986, the Second Assistant Provincial Fiscal of Negros SO ORDERED." 9


Oriental filed with the Regional Trial Court (RTC) of Negros Oriental an
Information for Multiple Murder and Frustrated Murder with Arson 6 against The evidence for the prosecution upon which the decision is based is
the accused. The accusatory portion of the Information reads: summarized in detail in the trial court's decision and is further condensed in
the Appellee's Brief 10 as follows:
xxx xxx xxx
"On March 6, 1986, Zosimo Toting Jr. reported to the Nabalabag Philippine
"That on or about March 4, 1986, at sitio Tigbao, Barangay Banawe, Constabulary Patrol at Pamplona, Negros Oriental, that his parents had been
Pamplona, Negros Oriental, Philippines, and within the jurisdiction of this killed and their house at Tigbaw, [Pamplona] Negros Oriental, burned. This
Honorable Court, the above-named accused, conspiring and confederating prompted PC Sgt. Reynaldo Tabanao, Sgt. Nestorio Rubia, Jaime Saguban
together and acting in common accord, with intent to kill, evident and three members of the Civilian Home Defense Force to go to Tigbaw,
premeditation and treachery, did then and there willfully, unlawfully and [Pamplona] Negros Oriental, to investigate the incident (TSN, January 20,
feloniously assault, attack, stab and hack with the use of a bolo and sickle, 1988, p. 5).
with which the accused were then respectively armed and provided, one
ZOSIMO TOTING, SR., thereby inflicting upon the victim hack wound, Upon arriving at Tigbaw, they found a burned house and several dead bodies.
neck posterior area 5" long, 3" depth, hack wounds, left upper back 3" long, The trial court identified the four (4) fatalities and their injuries as follows:
4" depth, stab wound, thru and thru, lower abdomen, 4" width , exit lower
back 1" width, 90% 2nd and 3rd degree burns of the body, and which wounds (1) Zosimo Toting, Sr., with hack wound neck, posterior area, . . . hack
caused the death of said Zosimo Toting, Sr., immediately thereafter; one wound, left upper back . . . stab wound, through and through, lower abdomen,
BEATRICE TOTING, thereby inflicting upon the victim hacking (sic) . . . 90% second and third degree burns of the body;
wound, neck posterior area, 5" long, 6" depth, incised wound, epigastric area
11" long, 4" depth, exposing vital organs, lower abdomen, 11" long, 4" depth (2) Beatrice Toting, hack wound, neck posterior are . . . incised wound,
exposing intestines, 90% 2nd and 3rd degree burns of the body, and which epigastric area . . . exposing vital organs, lower abdomen . . . exist (sic) lower
wounds caused the death of said Beatrice Toting immediately thereafter; one back, 90% second and third degree burns of the body;
BOMBIE TOTING, thereby inflicting upon the victim infected hack wound
from the right anterior lumbar area transecting mid-abdomen, inguial area (3) Bombie Toting, inflicted hack wound from the anterior lumbar area
left to the medial thigh left, through and through, with necrotic transected transecting mid-abdomen, inguial area left to the medial thigh left, through
muscle, and which wounds caused the death of said Bombie Toting shortly and through, with necrotic transected muscle;
thereafter; and in order to cover-up the heinous crime committed, the above-
named accused, conspiring and confederating together and acting in common (4) Manolita Toting, third degree burns, all burned body, head, extrimities
accord, did then and there willfully, unlawfully and feloniously set to fire the (sic) or 100% burns;
house of the aforesaid victim (sic) spouses Zosimo Toting, Sr. and Beatrice
Toting, thereby razing it to the ground, and as a consequence thereto Manolo Toting did not die but suffered 20% second and third degree burns
MANOLITA TOTING suffered Third degree burns, all burn (sic) body, on the upper extremity bilateral, posterior shoulder, left and back (Records,
head, extremities or 100% burns, and which wounds caused the death of said p. 213).
Manolita Toting immediately thereafter and also causing injuries to
MANOLO TOTING, to wit: 20% 2nd and 3rd degree burns on the upper Zosimo Toting, Sr, Beatrice Toting, Manolita Toting and Manolo Toting
extremity bilateral, posterior shoulder, left and back, and which wounds were found near the vicinity of the burned house. About forty (40) meters
would have caused the death of victim Manolo Toting, thus performing all away, the investigating officers found six year old Bombie Toting suffering
the overt acts of execution which would have produced the crime of Murder from serious hack wounds (TSN, January 20, 1988, p. 18). The young girl
as a consequence, but nevertheless did not produce it by reason of causes said that she had been in this condition for one and a half days already.
independent of the will of the perpetrator, that is, the timely medical
assistance extended to said Manolo Toting which prevented his death. Bombie Toting related to Sgt. Tabanao that on March 4, 1986 at 7:00 o'clock
in the evening, appellant and Teodoro Basay killed her parents and burned
Contrary to Article 248 in relation to Articles 6, 48 and 50 of the Revised their house (TSN, January 20, 1988, p. 18, Records, p. 9).
Penal Code."
On the same day the investigating officers went to the appellant's house. They
The case was docketed as Criminal Case No. 7411 and was raffled off to saw appellant fixing the roof of his house and when appellant saw them, he
Branch 40 of the said court. went down and tried to ran (sic) away (TSN, January, 20, 1988, p. 22).
Appellant was turned over to the Pamplona Police Station (TSN, January 20, identifying Ramirez and Basay as the perpetrators of the crime and
1988, p. 25). considered as flight - which is indicative of guilt - Ramirez's running away
when he saw the law enforcers on 6 March 1986. It further ruled that the
Bombie Toting was brought to the hospital but due to the gravity of her latter signed the extra-judicial confession voluntarily and in the presence of
injuries she died on March 7, 1986 at 1:40 P.M. (Records, p. 12, Exhibit I). Elpedio Catacutan, the COMELEC registrar of Pamplona - "a barister (sic)
who appeared as counsel for accused Jaime Ramirez;" hence it is admissible
Appellant was brought into the chamber of Judge Teopisto Calumpang, the against the latter. 19
municipal circuit trial judge of Pamplona, Amlan, and San Jose, on March
14, 1986. He was accompanied by Mr. Elpedio Catacutan who acted as On the other hand, the trial court did not admit the statement of Bombie
appellant's counsel (TSN, June 6, 1988, p. 6). They brought with them an Toting as a dying declaration but merely as part of the res gestae because the
affidavit previously typed by a police investigating officer. The Judge then prosecution failed to prove two (2) of the requisites for the admissibility of a
made the court interpreter translate the allegations of the sworn statement dying declaration, viz., that the statement was given under consciousness of
into the local dialect for appellant (TSN, June 6, 1988). Thereafter, in the an impending death and that Bombie Toting is a competent witness. 20
presence of the Judge, appellant and Mr. Catacutan signed the affidavit.
(TSN, January 20, 1988, p. 14). Appellant and counsel also signed the Accused Jaime Ramirez neither filed a notice of appeal nor orally manifested
vernacular translation of Exhibit F (Records, p . 12)." his intention to appeal. However, on 31 January 1983, the trial court handed
down an order directing the clerk of court to transmit to this Court the entire
Upon the other hand, the evidence for accused Jaime Ramirez is substantially records of the case because in view of the penalty imposed - life
summarized in the Appellant's Brief 11 in this wise: imprisonment - "such Decision is subject for automatic review by the
Supreme Court." 21 This of course is erroneous as, pursuant to Section 10,
"Evidence for the Defense: Rule 122 of the Rules of Court, the automatic review of a criminal case is
applicable only where the penalty of death has been imposed which,
xxx xxx xxx nevertheless, is now banned under Section 19(1), Article III of the 1987
Constitution.
Accused Jaime Ramirez testified that he was cooking food for the pig when
the armed uniformed men arrested him on March 5, 1986 and was brought In the interest of justice, however, We accepted the appeal in the Resolution
(sic) to the Nabalabag PC Detachment where he was maltreated. Later, he of 8 May 1989. 22
was brought to Municipal (sic) Jail where he stayed for one month and 23
days. In his Appellant's Brief, 23 Jaime Ramirez, hereinafter referred to as the
Appellant, imputes upon the trial court the commission of this lone error:
Queried on the 'Joint Waiver', this witness said he did not read it because he
did not know how to read. When it was read to him, he did not understand it "THE TRIAL COURT ERRED IN FINDING THE APPELLANT GUILTY
because it was read in English. Elpedio Catacutan was not his lawyer and he AS CHARGED ON THE BASIS OF EXHIBIT "F" (AFFIDAVIT) WHICH
did not know him (TSN, March 5, pp. 3, 5-6, 9-10). WAS EXECUTED IN VIOLATION OF HIS CONSTITUTIONAL
RIGHTS AND ON THE BASIS OF HEARSAY EVIDENCE AND ON THE
On cross-examination, this witness said he reached Grade II and knows how PRESUMPTION OF GUILT."
to write his name. He was alone at the time he was arrested. He was arrested
ahead of Teodoro Basay and those who arrested him where (sic) not the same Appellant contends that his so-called extra-judicial confession, Exhibit "F",
persons who arrested Teodoro Basay. was executed in blatant disregard of his constitutional right to counsel and to
remain silent during custodial investigation. It is therefore inadmissible in
He first saw Elpedio Catacutan in the Pamplona Municipal Hall when evidence. 24 Without the said confession, the only piece of evidence which
Elpedio was going upstairs. When he signed Exhibit "F", Catacutan was in seems to point to his guilt is the alleged statement of Bombie Toting.
front of him. They did not converse with each others (sic). He did not engage Appellant asserts, however, that the said statement was "very doubtful and .
Catacutan to assist him, nor solicit his services. He does know (sic) any one . . no reasonable mind would conclude that she was candidly truthful;" hence,
who solicited Catacutan's services for him. He did not ask the Judge her statement, besides being hearsay as it came from a person who was not
(Calumpang) that a lawyer be designated to help him in connection with the presented in court to testify, should not have been taken at "face value against
affidavit. The Pamplona Judge did not offer to give him a lawyer to assist any of the accused, much less against the appellant." 25 Besides, the
him in the execution of the affidavit (TSN, October 4, 1988, p.4)." 12 appellant asserts that the same statement was not used against his co-accused
Basay who was, unlike him, acquitted by the trial court. As to his having run
Jaime Ramirez is a farmer and at the time he testified on 8 March 1988, was away upon seeing the armed law enforcers, appellant claims that he did so
nineteen (19) years old and single. 13 The prosecution did not rebut his claim out of fear as the latter were armed. 26
that he had only finished Grade II and that he does not know how to read.
He, however, understands the Cebuano dialect. 14 On the other hand, it is maintained by the People, in the Appellee's Brief 27
submitted by the Office of the Solicitor General, that the appellant executed
The Exhibit "F" referred to above is the Sworn Statement, 15 in English, of the extra-judicial confession voluntarily and without duress; in signing such
accused Jaime Ramirez taken in the Pamplona police station on 7 March confession, he was accompanied by a certain Mr. Catacutan, a non-lawyer,
1986 and subscribed and sworn to only on 14 March 1986 before Judge inside the chambers of Judge Calumpang - "an environment . . . other than
Teopisto L. Calumpang of the MCTC of Pamplona-Amlan-San Jose. The vindictive and oppressive which the courts desired to guard against in
trial court described this document as the Extra-Judicial Confession 16 of Miranda vs. Arizona, 384 US 436." 28 As to Bombie's statement, it is claimed
Ramirez. that the same should be considered as a dying declaration.

The Joint Waiver (Exhibit "G") mentioned the testimony of Jaime Ramirez We find merit in the appeal.
is in the Cebuano dialect and was signed by accused Basay and Ramirez on
7 March 1986. Both accused state therein that for their safety and security, 1. Jaime Ramirez's sworn statement or extra-judicial confession was
they voluntarily decided to be detained and that they killed the spouses prepared on 7 March 1986 at about 11:00 o'clock in the morning in the
Zosimo Toting and Betty Toting and thereafter burned the spouses' house; Pamplona police station. Pertinent portions thereof read as follows:
this fire resulted in the death of one and the hospitalization of two Toting
children. 17 xxx xxx xxx

The trial court disregarded this Joint Waiver insofar as it tended to "PRELIMINARY - MR. JAIME RAMIREZ, you are now under
incriminate the accused "because when they signed said Joint Waiver, they investigation in connection with the death of the couple and the burning of
were not represented by counsel;" thus, the same was prepared in violation their house, ZOSIMO TOTING and BEATRICE TOTING alias BETTY
of "Section 12, Article 3 of the Bill of Rights of the 1987 Constitution." 18 TOTING on March 4, 1986 at about 7:00 o'clock in the evening at sitio
There being no other evidence against Basay, the trial court acquitted him. Togbao, Barangay Banawe, Pamplona, Negros Oriental. You are also
However, it admitted in evidence the so-called extra-judicial confession of informed that under our new constitution you have the right to remain silent
Jaime Ramirez, considered as part of the res gestae the alleged statement and not to answer questions which will incriminate you and to have a counsel
given by Bombie Toting to PC Sgt. Reynaldo Tabanao and Jaime Saguban
of your own choice to assist you in this investigation, do (sic) you aware of
this? The source of this provision is Miranda vs. Arizona, 30 in connection
therewith, this Court stated in People vs. Caguioa 31 that:
ANSWER - Yes.
" . . . The landmark opinion of Miranda vs. Arizona, decided in 1966, as noted
Q - You are also informed that whatever statement you may offer in this above, the source of this constitutional provision, emphasized that statements
investigation it (sic) might be used as evidence in your favor or against you made during the period of custodial interrogation to be admissible require a
in the future, do (sic) you aware of this this (sic)? clear intelligent waiver of constitutional rights, the suspect being warned
prior to questioning that he has a right to remain silent, that any utterance
A - Yes. may be used against him, and that he has the right to the presence of a
counsel, either retained or appointed. In the language of Chief Justice
Q - After you have informed (sic) of your rights are you willing to proceed Warren: 'Our holding will be spelled out with some specificity in the pages
with this investigation of yours even if you have no counsel of your own which follow, but briefly stated, it is this: the prosecution may not use
choice that will assist you in this investigation? statements, whether exculpatory or inculpatory, stemming from custodial
interrogation of the defendant unless it demonstrates the use of procedural
A - Yes. I don't need any counsel in this investigation because I will just tell safeguards effective to secure the privilege against self-incrimination. By
the truth. custodial interrogation, we mean questioning initiated by law enforcement
officers after a person has been taken into custody or otherwise deprived of
1. Question - If so, please state your name, age and other personal his freedom of action in any significant way. As for the procedural safeguards
circumstances? to be employed, unless other fully effective means are devised to inform
accused persons of their right of silence and to assure a continuous
Answer - Jaime Ramirez y Tano, 19 years old, single, Filipino, farmer and a opportunity to exercise it, the following measures are required. Prior to any
resident of sitio Palale, Barangay San Isidro, Pamplona, Negros Oriental. questioning, the person must be warned that he has a right to remain silent,
that any statement he does not make (sic) may be used as evidence against
xxx xxx xxx him, and that he has a right to the presence of an attorney, either retained or
appointed. The defendant may waive effectuation of those rights, provided
11. Q - What more can you say? the waiver is made voluntarily, knowingly and intelligently. If, however, he
indicates in any manner and at any stage of the process that he wishes to
A - No more. I proved that my statement is correct I signed this 7 March 1986 consult with an attorney before speaking, there can be no questioning.
(sic), at Pamplona, Negros Oriental. Likewise, if the individual is alone and indicates in any manner that he does
not wish to be interrogated, the police may not question him. The mere fact
(Sgd.) JAIME T. RAMIREZ that he may have answered some questions or volunteered some statements
on his own does not deprive him of the right to refrain from answering any
(TYP) JAIME T. RAMIREZ further inquiries until he has consulted with an attorney and thereafter
consents to be questioned.'" (citations omitted).
Affiant
Then, in Morales vs. Enrile, 32 in the light of the said Section 20, prescribed
NOTE: ASSISTED BY: the procedure to be followed by peace officers when making an arrest and
when conducting a custodial investigation. Thus:
(Sgd.) ELPEDIO B. CATACUTAN
"7. At the time a person is arrested, it shall be the duty of the arresting officer
(TYP) ELPEDIO B. CATACUTAN to inform him of the reason for the arrest and he must be shown the warrant
of arrest, if any. He shall be informed of his constitutional rights to remain
Counsel of the accused silent and to counsel, and that any statement he might make could be used
against him. The person arrested shall have the right to communicate with
SUBSCRIBED AND SWORN to before me this 14th day of March 1986, at his lawyer, a relative, or anyone he chooses by the most expedient means -
Pamplona, Negros Oriental, Philippines. by telephone if possible - or by letter or messenger. It shall be the
responsibility of the arresting officer to see to it that this is accomplished. No
(Sgd.) TEOPISTO L. CALUMPANG custodial investigation shall be conducted unless it be in the presence of
counsel engaged by the reason arrested, by any person on his behalf, or
(TYP) TEOPISTO L. GALUMPANG appointed by the court upon petition either of the detainee himself or by
anyone on his behalf. The right to counsel may be waived but the waiver shall
Mun Trial Circuit Judge not be valid unless made with the assistance of counsel. Any statement
obtained in violation of the procedure herein laid down, whether exculpatory
CERTIFICATION or inculpatory, in whole or in part, shall be inadmissible in evidence."

I HEREBY CERTIFY that I have personally examined the affiant and that I This was reiterated in People vs. Galit. 33
am satisfied that he voluntarily executed and understood his affidavit.
In People vs. Nicandro, 34 this Court declared that one's right to be informed
(Sgd.) TEOPISTO L. CALUMPANG of the right to remain silent and to counsel contemplates "the transmission of
meaningful information rather than just the ceremonial and perfunctory
(TYP) TEOPISTO L. CALUMPANG recitation of an abstract constitutional principle." Thus, is not enough for the
interrogator to merely repeat to the person under investigation the provisions
Mun Trial Circuit Judge" 29 of section 20, Article IV of the 1973 Constitution, now Section 12, Article
III of the 1987 Constitution; the former must also explain the effects of such
We do not hesitate to rule that this purported extra-judicial confession provision in practical terms - e.g., what the person under interrogation may
belonging to appellant Jaime Ramirez and obtained during custodial or may not do - and in a language the subject fairly understands. The right
interrogation was taken in blatant disregard of his right to counsel, to remain "to be informed" carries with it a correlative obligation on the part of the
silent and to be informed of such rights, guaranteed by Section 20, Article IV police investigator to explain, and contemplates effective communication
of the 1973 Constitution - the governing law at that time. Said section reads: which results in the subject's understanding of what is conveyed. Since it is
comprehension that is sought to be attained, the degree of explanation
"SEC. 20. No person shall be compelled to be a witness against himself. Any required will necessarily vary and depend on the education, intelligence and
person under investigation for the commission of an offense shall have the other relevant personal circumstances of the person undergoing
right to remain silent and to counsel, and to be informed of such right. No investigation. In further ensuring the right to counsel, it is not enough that
force, violence, threat, intimidation, or any other means which vitiates the the subject is informed of such right; he should also be asked if he wants to
free will shall be used against him. Any confession obtained in violation of avail of the same and should be told that he could ask for counsel if he so
this section shall be inadmissible in evidence." desired or that one could be provided him at his request. 35 If he decides not
to retain counsel of his choice or avail of one to be provided for him and, Office was too late and had no palliative effect; it did not cure the absence of
therefore, chooses to waive his right to counsel, such waiver, to be valid and counsel at the time of the custodial investigation when the extra-judicial
effective, must still be made with the assistance of counsel. 36 That counsel statement was being taken.
must be a lawyer. 37
(f). Furthermore, Elpedio Calumpang is not a lawyer; according to the trial
The foregoing pronouncements are now synthesized in paragraphs 1 and 3, court, he is "a barister (sic)." In fact, he candidly admitted that he is not a
Section 12, Article III of the 1987 Constitution, to wit: lawyer but that he obtained a law degree from the Siliman University in 1959.
Unfortunately, however, he failed in three Bar Examinations. 42
"SEC 12(1). Any person under investigation for the commission of an offense
shall have the right to be informed of his right to remain silent and to have (g). There is no showing that the so-called extra-judicial confession, which
competent and independent counsel preferably of his own choice. If the is in English, was correctly explained and translated to the appellant by Judge
person cannot afford the services of counsel, he must be provided with one. Calumpang. Although the latter claimed in his testimony on direct
These rights cannot be waived except in writing and in the presence of examination that he translated the same in the local dialect to the appellant
counsel. before the latter affixed his signature thereto, 43 Elpedio Catacutan
categorically declared that it was the interpreter, one Pedro Rodriguez, who
xxx xxx xxx translated it to the appellant. Thus:

(3) Any confession or admission obtained in violation of this or Section 17 "Q. Who is the interpreter who made the translation?
hereof shall be inadmissible in evidence against him."
A. Pedro Rodriguez.
The adjectives competent and independent, which qualify the kind of counsel
an accused is entitled to during investigation, were not found in the previous Q. Were you there when the translation was made?
Constitution. Their incorporation in the 1987 Constitution was thus meant to
stress the primacy of this right to counsel. A. Sure.

A close scrutiny of the questioned extra-judicial confession in the case at bar Q. So it was not the Judge who made the translation, is that what you mean?
reveals all possible violations of the appellant's right to remain silent, to
counsel and to be informed of such rights, and of the safeguards prescribed A. The translation was course (sic) through the interpreter." 44
by this Court for the holding of custodial interrogations:
(h). Finally, the kind of "advice" proffered by the unidentified interrogator
(a) The interrogation was the conducted and the confession was written in belongs to that stereotyped class - a long question by the investigator
English a language the appellant, a farmer in a remote barangay of Pamplona, informing the appellant of his right followed by a monosyllabic answer -
cannot speak and does not understand; he only finished Grade II. There is no which this Court has condemned for being unsatisfactory. 45 The
evidence to show that the interrogator, who was not even presented as a investigator gave his advice perfunctorily or in a pro-forma manner,
witness and remains unidentified, translated the questions and the answers obviously to pay mere lip service to the prescribed norms. As this Court
into a dialect known and fairly understood by the appellant. observed in People vs. Newman, 46 this stereotyped "advice":

(b) Appellant was not told that he could retain a counsel of choice and that if " . . . has assumed the nature of a 'legal form' or model. Its tired, punctilious,
he cannot afford to do so, he could be provided with one. fixed and artificially stately style does not create an impression of
voluntariness or even understanding on the part of the accused. The showing
(c) He did not sign any waiver of his right to remain silent and to counsel. of a spontaneous, free and unconstrained giving up of a right is missing."

(d) He was not assisted by any counsel during the investigation. Instead, a Consequently, Exhibit "F", which is indisputably an uncounselled confession
certain Elpedio Catacutan, who claimed to have appeared for him as a or admission, is inadmissible in evidence. The trial court, therefore,
"friend-counsel," 38 was present only at the time that appellant was brought committed a fatal error in admitting it.
to the office of Judge Catacutan for the preparation of the jurat. It was
precisely for this reason that the following notations were inserted above the 2. We harbor very serious doubts about the alleged statement given by
jurat of the so-called extra-judicial confession: Bombie Toting to Sgt. Tabanao and Jaime Saguban identifying the appellant
and Teodoro Basay as the perpetrators of the heinous crime. In the first place,
"NOTE: ASSISTED BY: the trial court itself ruled that Bombie was not a competent witness. We agree
with such a conclusion, not necessarily because she was only six (6) years
(Sgd.) ELPEDIO B. CATACUTAN old, but because her condition at the time she supposedly gave her statement
made it impossible for her to have communicated effectively. She suffered
(TYP) ELPEDIO B. CATACUTAN" the following injuries:

In reality, Catacutan signed not as counsel, but only as a witness. Thus: "Infected hack wound from the right anterior lumbar area transecting mid
abdomen, inguinal area left to the medial thigh left through and through, with
"Q. Do you recall having signed as a witness of an affidavit of one Jaime necrotic transected muscle." 47
(sic) T. Ramirez which affidavit is now marked as Exhibit "F"?
She was taken from the crime scene only on 6 March 1986, or two (2) days
A. Yes. after the commission of the crime, and died in the hospital on 7 March 1986.
The doctor who first attended to her when she arrived at the Provincial
Q. Can you tell the court where did you sign that Exhibit "F"? Hospital, a certain Dr. Sy, was not presented as a witness. On the other hand,
the doctor who attended to her before she died, Dr. Edgar Cantalao, testified
A. I signed this affidavit in the office of the Municipal Judge of Pamplona." that when he last saw Bombie alive, she could not talk. 48 It was this inability
39 to talk which led the trial court to express its doubts on the veracity of the
latter's supposed statement:
Moreover, it is to be observed that the appellant does not even know the said
Elpedio Catacutan. 40 " . . . Although persons of tender age are prone to tell the truth, however, the
Court must be cautious in appreciating said testimony where the person had
(e). Assuming arguendo that Elpedio Catacutan may have been summoned a serious wound and had not eaten for one day and one night. There is no
to act as appellant's counsel, he was, nevertheless, not present during the evidence to show that Bombie Toting told the doctor as to who were the
custodial interrogation which, by the way, was conducted exactly a week perpetrators of the crime; neither did she tell her own brother, Zosimo Toting,
before he appeared -or more correctly, was made to appear - before Judge Jr. that it was the accused, Teodoro Basay and Jaime Ramirez who killed her
Calumpang. His presence before the latter did not change the situation. As parents and her brother and sisters and burned their house. . . . The Court
this Court stated in People vs. Burgos, 41 the securing of counsel to help the cannot understand why P.C. Sgt. Tabano did not ask Bombie Toting
accused when the latter subscribed under oath to his statement at the Fiscal's questions concerning the commission of the crime by the accused. Neither
did the P.C. or (sic) the police take any statement from her on her way to the
hospital or at the hospital. Surprisingly, Bombie Toting did not even tell her
own brother, Zosimo Toting, Jr. that it was the accused who committed the
crime. Had the statement of Bombie Toting been made to the doctor or to the
barangay captain or to any reputable member of the community where the
incident happened, the Court will have to put weight and consider her
statement as a dying declaration. Our experience has shown that persons in
authority are prone to fabricate or misrepresent the facts to serve their own
purpose. Innocent people had been charged in Court simply by the false
statements of peace officers. The Court therefore has to be cautious when
these peace officers testify in Court." 49

In the second place, as a result of the foregoing observations, the trial court
completely disregarded Bombie Toting's so-called statement as against
Teodoro Basay. We therefore see neither rhyme nor reason for the trial
court's admission of the same as against the appellant.

3. While it may be true that the appellant ran away when he first saw the
armed law officers, he did so merely out of fear of them. This act should not
be considered as the flight which is indicative of guilt. The appellant had not
left his house or barangay since 4 March 1986, the day the crime was
committed. If he were indeed one of the perpetrators and had the intention to
flee in order to avoid arrest, he should have vanished sooner and should not
have remained in his house. Besides, if indeed his running away could be
construed as flight, it could only be considered as circumstantial evidence.
Such evidence would still be insufficient for a conviction. Under Section 4,
Rule 133 of the Rules of Court, in order that circumstantial evidence may
sustain a conviction, there must, inter alia, be more than one (1)
circumstance. No other circumstance was established in this case.

Hence, the appellant's guilt was not established with moral certainty. He
should be acquitted.

We cannot, however, close this case without making some observations


about the legal conclusions of the trial court anent the crimes committed and
the penalty imposed. The facts indisputably establish that Zosimo Toting, Sr.,
Beatrice Toting and Bombie Toting were stabbed and hacked before their
house was burned. Zosimo and Beatrice died immediately while Bombie
lived for a few days. As a matter of fact, the thesis of the prosecution is that
the house was burned to conceal the stabbing and hacking. As a result of this
fire, Manolita Toting and Manolo Toting suffered burns which caused the
death of the former; the latter, however, survived due to timely medical
attention. Four (4) crimes were therefore committed, viz.: three (3) separate
murders under Article 248 of the Revised Penal Code 50 for the deaths of
Zosimo, Beatrice and Bombie, and arson as punished under Section 5 of P.D.
No. 1613 51 for the death of Manolita and the injuries sustained by Manolo
as a consequence of the burning of the house. The aforementioned Section 5
reads:

"SEC. 5. Where Death Results from Arson. - If by reason of or on the


occasion of the arson death results, the penalty of Reclusion Perpetua to death
shall be imposed."

Also, the information that was filed is clearly duplicitous and thus vulnerable
to a motion to quash under Section 3(e), Rule 117 of the Rules of Court. No
such motion having been filed, appellant is deemed to have waived the
defect.

Finally, We have time and again said that life imprisonment is not a penalty
provided for in the Revised Penal Code and is not the same as reclusion
perpetua. 52 Unfortunately, the trial court still disregarded this
pronouncement. It is hoped that it will not happen again.

WHEREFORE, the challenged Decision in Criminal Case No. 7411 of


Branch 40 of the Regional Trial Court of Negros Oriental is REVERSED and
appellant JAIME RAMIREZ alias "NEBOY" is hereby ACQUITTED with
costs de oficio. His immediate release from detention is hereby ordered.

SO ORDERED.
JAVIER vs. THE COMMISSION ON ELECTIONS Antique in 1984 hewed to the line and equaled if it did not surpass the
G.R. Nos. L-68379-81 | 1986-09-22 viciousness of elections in other provinces dominated by the KBL. Terrorism
CRUZ, J: was a special feature, as demonstrated by the killings previously mentioned,
which victimized no less than one of the main protagonists and implicated
The new Solicitor General has moved to dismiss this petition on the ground his rival as a principal perpetrator. Opposition leaders were in constant peril
that as a result of supervening events it has become moot and academic. It is of their lives even as their supporters were gripped with fear of violence at
not as simple as that, Several lives have been lost in connection with this the hands of the party in power.
case, including that of the petitioner himself. The private respondent is now
in hiding. The purity of suffrage has been defiled and the popular will scorned What made the situation especially deplorable was the apparently indifferent
through a confabulation of those in authority. This Court cannot keep silent attitude of the Commission on Elections toward the anomalies being
in the face of these terrible facts. The motion is denied. committed. It is a matter of record that the petitioner complained against the
terroristic acts of his opponents. All the electoral body did was refer the
The petitioner and the private respondent were candidates in Antique for the matter to the Armed Forces without taking a more active step as befitted its
Batasang Pambansa in the May 1984 elections. The former appeared to enjoy constitutional role as the guardian of free, orderly and honest elections. A
more popular support but the latter had the advantage of being the nominee more assertive stance could have averted the Sibalom election eve massacre
of the KBL with all its perquisites of power. On May 13, 1984, the eve of the and saved the lives of the nine victims of the tragedy.
elections, the bitter contest between the two came to a head when several
followers of the petitioner were ambushed and killed, allegedly by the latter's Public confidence in the Commission on Elections was practically nil
men. Seven suspects, including respondent Pacificador, are now facing trial because of its transparent bias in favor of the administration. This prejudice
for these murders. The incident naturally heightened tension in the province left many opposition candidates without recourse except only to this Court.
and sharpened the climate of fear among the electorate. Conceivably, it
intimidated voters against supporting the Opposition candidate or into Alleging serious anomalies in the conduct of the elections and the canvass of
supporting the candidate of the ruling party. the election returns, the petitioner went to the Commission on Elections to
prevent the impending proclamation of his rival, the private respondent
It was in this atmosphere that the voting was held, and the post-election herein. 1 Specifically, the petitioner charged that the elections were marred
developments were to run true to form. Owing to what he claimed were by "massive terrorism, intimidation, duress, vote-buying, fraud, tampering
attempts to railroad the private respondent's proclamation, the petitioner went and falsification of election returns under duress, threat and intimidation,
to the Commission on Elections to question the canvass of the election snatching of ballot boxes perpetrated by the armed men of respondent
returns. His complaints were dismissed and the private respondent was Pacificador." 2 Particular mention was made of the municipalities of Caluya,
proclaimed winner by the Second Division of the said body. The petitioner Cabate, Tibiao, Barbaza, Laua-an, and also of San Remigio, where the
thereupon came to this Court, arguing that the proclamation was void because petitioner claimed the election returns were not placed in the ballot boxes but
made only by a division and not by the Commission on Elections en banc as merely wrapped in cement bags or manila paper.
required by the Constitution. Meanwhile, on the strength of his proclamation,
the private respondent took his oath as a member of the Batasang Pambansa. On May 18, 1984, the Second Division of the Commission on Elections
directed the provincial board of canvassers of Antique to proceed with the
The case was still being considered by this Court when on February 11, 1986, canvass but to suspend the proclamation of the winning candidate until
the petitioner was gunned down in cold blood and in broad daylight. The further orders. 3 On June 7, 1984, the same Second Division ordered the
nation, already indignant over the obvious manipulation of the presidential board to immediately convene and to proclaim the winner without prejudice
elections in favor of Marcos, was revolted by the killing, which flaunted a to the outcome of the case before the Commission. 4 On certiorari before this
scornful disregard for the law by the assailants who apparently believed they Court, the proclamation made by the board of canvassers was set aside as
were above the law. This ruthless murder was possibly one of the factors that premature, having been made before the lapse of the 5-day period of appeal,
strengthened the cause of the Opposition in the February revolution that which the petitioner had seasonably made. 5 Finally, on July 23, 1984, the
toppled the Marcos regime and installed the present government under Second Division promulgated the decision now subject of this petition which
President Corazon C. Aquino. inter alia proclaimed Arturo F. Pacificador the elected assemblyman of the
province of Antique. 6
The abolition of the Batasang Pambansa and the disappearance of the office
in dispute between the petitioner and the private respondent ---- both of This decision was signed by Chairman Victoriano Savellano and
whom have gone their separate ways ---- could be a convenient justification Commissioners Jaime Opinion and Froilan M. Bacungan. Previously asked
for dismissing this case. But there are larger issues involved that must be to inhibit himself on the ground that he was a former law partner of private
resolved now, once and for all, not only to dispel the legal ambiguities here respondent Pacificador, Opinion had refused. 7
raised. The more important purpose is to manifest in the clearest possible
terms that this Court will not disregard and in effect condone wrong on the The petitioner then came to this Court, asking us to annul the said decision.
simplistic and tolerant pretext that the case has become moot and academic.
The core question in this case is one of jurisdiction, to wit: Was the Second
The Supreme Court is not only the highest arbiter of legal questions but also Division of the Commission on Elections authorized to promulgate its
the conscience of the government. The citizen comes to us in quest of law decision of July 23, 1984, proclaiming the private respondent the winner in
but we must also give him justice. The two are not always the same. There the election?
are times when we cannot grant the latter because the issue has been settled The applicable provisions are found in Article XII-C, Sections 2 and 3, of the
and decision is no longer possible according to the law. But there are also 1973 Constitution.
times when although the dispute has disappeared, as in this case, it
nevertheless cries out to be resolved. Justice demands that we act then, not Section 2 confers on the Commission on Elections the power to:
only for the vindication of the outraged right, though gone, but also for the
guidance of and as a restraint upon the future. "(2) Be the sole judge of all contests relating to the election, returns and
qualifications of all member of the Batasang Pambansa and elective
It is a notorious fact decried by many people and even by the foreign press provincial and city officials."
that elections during the period of the Marcos dictatorship were in the main
a desecration of the right of suffrage. Vote-buying, intimidation and violence, Section 3 provides:
illegal listing of voters, falsified returns, and other elections anomalies
misrepresented and vitiated the popular will and led to the induction in office "The Commission on Elections may sit en banc or in three divisions. All
of persons who did not enjoy the confidence of the sovereign electorate. election cases may be heard and decided by divisions except contests
Genuine elections were a rarity. The price at times was human lives. The rule involving members of the Batasang Pambansa, which shall be heard and
was chicanery and irregularity, and on all levels of the polls, from the decided en banc. Unless otherwise provided by law, all election cases shall
barangay to the presidential. This included the rigged plebiscites and be decided within ninety days from the date of their submission for decision."
referenda that also elicited the decision and provoked the resentments of the
people. While both invoking the above provisions, the petitioner and the respondents
have arrived at opposite conclusions. The records are voluminous and some
of the pleadings are exhaustive and in part even erudite. And well they might
be, for the noble profession of the law ---- despite all the canards that have It is worth observing that the special procedure for the settlement of what are
been flung against it ---- exerts all efforts and considers all possible now called "pre-proclamation controversies" is a relatively recent innovation
viewpoints in its earnest search of the truth. in our laws, having been introduced only in 1978, through P.D. No. 1296,
otherwise known as the 1978 Election Code. Section 175 thereof provided:
The petitioner complains that the proclamation made by the Second Division
is invalid because all contests involving the members of the Batasang "Sec. 175. Suspension and annulment of proclamation. The Commission
Pambansa come under the jurisdiction of the Commission on Elections en shall be the sole judge of all pre-proclamation controversies and any of its
banc. This is as it should be, he says, to insure a more careful decision, decisions, orders or rulings shall he final and executory. It may, motu proprio
considering the importance of the offices involved. The respondents, for their or upon written petition, and after due notice and hearing order the
part, argue that only contests need to be heard and decided en banc and all suspension of the proclamation of a candidate-elect or annul any
other cases can be ---- in fact, should be ---- filed with and decided only by proclamation, if one has been made, on any of the grounds mentioned in
any of the three divisions. Sections 172, 173 and 174 thereof."

The former Solicitor General makes much of this argument and lays a Before that time all proceedings affecting the election, returns and
plausible distinction between the terms "contests" and "cases" to prove his qualifications of public officers came under the complete jurisdiction of the
point. 8 Simply put, his contention is that the pre-proclamation controversy competent court or tribunal from beginning to end and in the exercise of
between the petitioner and the private respondent was not yet a contest at that judicial power only. It therefore could not have been the intention of the
time and therefore could be validly heard by a mere division of the framers in 1935, when the Commonwealth Charter was adopted, and even in
Commission on Elections, consonant with Section 3. The issue was at this 1973, when the past Constitution was imposed, to divide the electoral process
stage still administrative and so was resoluble by the Commission under its into the pre-proclamation stage and the post-proclamation stage and to
power to administer all laws relative to the conduct of elections, 9 not its provide for a separate jurisdiction for each stage, considering the first
authority as sole judge of the election contest. administrative and the second judicial.

A contest, according to him, should involve a contention between the parties Besides, the term "contest" as it was understood at the time Article XII-C,
for the same office "in which the contestant seeks not only to oust the intruder Section 2(2) was incorporated in the 1973 Constitution did not follow the
but also to have himself inducted into the office." 10 No proclamation had as strict definition of a contention between the parties for the same office. Under
yet been made when the petition was filed and later decided. Hence, since the Election Code of 1971, which presumably was taken into consideration
neither the petitioner nor the private respondent had at that time assumed when the 1973 Constitution was being drafted, election contests included the
office, there was no Member of the Batasang Pambansa from Antique whose quo warranto petition that could be filed by any voter on the ground of
election, returns or qualifications could be examined by the Commission on disloyalty or ineligibility of the contestee although such voter was himself
Elections en banc. not claiming the office involved. 12

In providing that the Commission on Elections could act in division when The word "contests" should not be given a restrictive meaning; on the
deciding election cases, according to this theory, the Constitution was laying contrary, it should receive the widest possible scope conformably to the rule
down the general rule. The exception was the election contest involving the that the words used in the Constitution should be interpreted liberally. As
members of the Batasang Pambansa, which had to be heard and decided en employed in the 1973 Constitution, the term should be understood as
banc. 11 The en banc requirement would apply only from the time a referring to any matter involving the title or claim of title to an elective office,
candidate for the Batasang Pambansa was proclaimed as winner, for it was made before or after proclamation of the winner, whether or not the
only then that a contest could be permitted under the law. All matters arising contestant is claiming the office in dispute. Needless to stress, the term
before such time were, necessarily, subject to decision only by division of should be given a consistent meaning and understood in the same sense under
the Commission as these would come under the general heading of "election both Section 2(2) and Section 3 of Article XII-C of the Constitution.
cases."
The phrase "election, returns and qualifications" should be interpreted in its
As the Court sees it, the effect of this interpretation would be to divide the totality as referring to all matters affecting the validity of the contestee's title.
jurisdiction of the Commission on Elections into two, viz.: (1) over matters But if it is necessary to specify, we can say that "election" referred to the
arising before the proclamation, which should be heard and decided by conduct of the polls, including the listing of voters, the holding of the
division in the exercise of its administrative power; and (2) over matters electoral campaign, and the casting and counting of the votes; "returns" to
arising after the proclamation, which could be heard and decided only en the canvass of the returns and the proclamation of the winners, including
banc in the exercise of its judicial power. Stated otherwise, the Commission questions concerning the composition of the board of canvassers and the
as a whole could not act as sole judge as long as one of its divisions was authenticity of the election returns; and "qualifications" to matters that could
hearing a pre-proclamation matter affecting the candidates for the Batasang be raised in a quo warranto proceeding against the proclaimed winner, such
Pambansa because there was as yet no contest; or to put it still another way, as his disloyalty or ineligibility or the inadequacy of his certificate of
the Commission en banc could not do what one of its divisions was candidacy.
competent to do, i.e., decide a pre-proclamation controversy. Moreover, a
mere division of the Commission on Elections could hear and decide, save All these came under the exclusive jurisdiction of the Commission on
only those involving the election, returns and qualifications of the members Elections insofar as they applied to the members of the defunct Batasang
of the Batasang Pambansa, all cases involving elective provincial and city Pambansa and, under Article XII-C, Section 3, of the 1973 Constitution,
officials from start to finish, including pre-proclamation controversies and up could be heard and decided by it only en banc.
to the election protest, In doing so, it would exercise first administrative and
then judicial powers. But in the case of the Commission en banc, its We interpret "cases" as the generic term denoting the actions that might be
jurisdiction would begin only after the proclamation was made and a contest heard and decided by the Commission on Elections, only by division as a
was filed and not at any time and on any matter before that, and always in general rule except where the case was a "contest" involving members of the
the exercise only of judicial power. Batasang Pambansa, which had to be heard and decided en banc.

This interpretation would give to the part more powers than were enjoyed by As correctly observed by the petitioner, the purpose of Section 3 in requiring
the whole, granting to the division while denying to the banc. We do not think that cases involving members of the Batasang Pambansa be heard and
this was the intention of the Constitution. The framers could not have decided by the Commission en banc was to insure the most careful
intended such an irrational rule. consideration of such cases. Obviously, that objective could not be achieved
if the Commission could act en banc only after the proclamation had been
We believe that in making the Commission on Elections the sole judge of all made, for it might then be too late already. We are all-too-familiar with the
contests involving the election, returns and qualifications of the members of grab-the-proclamation-and-delay-the-protest strategy of many unscrupulous
the Batasang Pambansa and elective provincial and city officials, the candidates which has resulted in the frustration of the popular will and the
Constitution intended to give it full authority to hear and decide these cases virtual defeat of the real winners in the election. The respondent's theory
from beginning to end and on all matters related thereto, including those would make this gambit possible for the pre-proclamation proceedings, being
arising before the proclamation of the winners. summary in nature, could be hastily decided by only three members in
division, without the care and deliberation that would have otherwise been
observed by the Commission en banc.
Since this case began in 1984, many significant developments have taken
After that, the delay. The Commission en banc might then no longer be able place, not the least significant of which was the February revolution of
to rectify in time the proclamation summarily and not very judiciously made "people power" that dislodged the past regime and ended well nigh twenty
by the division. While in the end the protestant might be sustained, he might years of travail for this captive nation. The petitioner is gone, felled by a hail
find himself with only a Phyrric victory because the term of his office would of bullets sprayed with deadly purpose by assassins whose motive is yet to
have already expired. be disclosed. The private respondent has disappeared with the "pomp of
power" he had before enjoyed. Even the Batasang Pambansa itself has been
It may be argued that in conferring the initial power to decide the pre- abolished, "an iniquitous vestige of the previous regime" discontinued by the
proclamation question upon the division, the Constitution did not intend to Freedom Constitution. It is so easy now, as has been suggested not without
prevent the Commission en banc from exercising the power directly, on the reason, to send the records of this case to the archives and say the case is
theory that the greater power embraces the lesser. It could if it wanted to but finished and the book is closed.
then it could also allow the division to act for it. That argument would militate
against the purpose of the provision, which precisely limited all questions But not yet.
affecting the election contest, as distinguished from election cases in general,
to the jurisdiction of the Commission en banc as sole judge thereof. "Sole Let us first say these meager words in tribute to a fallen hero who was struck
judge" excluded not only all other tribunals but also and even the division of down in the vigor of his youth because he dared to speak against tyranny.
the Commission. A decision made on the contest by less than the Where many kept a meekly silence for fear of retaliation, and still others
Commission en banc would not meet the exacting standard of care and feigned and fawned in hopes of safety and even reward, he chose to fight. He
deliberation ordained by the Constitution. was not afraid. Money did not tempt him. Threats did not daunt him. Power
did not awe him. His was a singular and allexacting obsession: the return of
Incidentally, in making the Commission the "sole judge" of pre-proclamation freedom to his country. And though he fought not in the barricades of war
controversies in Section 175, supra, the law was obviously referring to the amid the sound and smoke of shot and shell, he was a soldier nonetheless,
body sitting en banc. In fact, the pre-proclamation controversies involved in fighting valiantly for the liberties of his people against the enemies of his
Aratuc vs. Commission on Elections, 13 where the said provision was race, unfortunately of his race too, who would impose upon the land a
applied, were heard and decided en banc. perpetual night of dark enslavement. He did not see the breaking of the dawn,
sad to say, but in a very real sense Evelio B. Javier made that dawn draw
Another matter deserving the highest consideration of this Court but nearer because he was, like Saul and Jonathan, "swifter than eagles and
accorded cavalier attention by the respondent Commission on Elections is stronger than lions."
due process of law, that ancient guaranty of justice and fair play which is the
hallmark of the free society. Commissioner Opinion ignored it. Asked to A year ago this Court received a letter which began: "I am the sister of the
inhibit himself on the ground that he was formerly a law partner of the private late Justice Calixto Zaldivar. I am the mother of Rhium Z. Sanchez, the
respondent, he obstinately insisted on participating in the case, denying he grandmother of Plaridel Sanchez IV and Aldrich Sanchez, the aunt of
was biased. 14 Mamerta Zaldivar. I lost all four of them in the election eve ambush in
Antique last year." She pleaded, as so did hundreds of others of her
Given the general attitude of the Commission on Elections toward the party provincemates in separate signed petitions sent us, for the early resolution of
in power at the time, and the particular relationship between Commissioner that horrible crime, saying "I am 82 years old now. I am sick. May I convey
Opinion and MP Pacificador, one could not be at least apprehensive, if not to you my prayer in church and my plea to you, 'Before I die, I would like to
certain, that the decision of the body would be adverse to the petitioner. As see justice to my son and grandsons,' May I also add that the people of
in fact it was. Commissioner Opinion's refusal to inhibit himself and his Antique have not stopped praying that the true winner of the last elections
objection to the transfer of the case to another division cannot be justified by will be decided upon by the Supreme Court soon."
any criterion of propriety. His conduct on this matter belied his wounded
protestations of innocence and proved the motives of the Second Division That was a year ago and since then a new government has taken over in the
when it rendered its decision. wake of the February revolution. The despot has escaped, and with him, let
us pray, all the oppressions and repressions of the past have also been
This Court has repeatedly and consistently demanded "the cold neutrality of banished forever. A new spirit is now upon our land. A new vision limns the
an impartial judge" as the indispensable imperative of due process. 15 To horizon. Now we can look forward with new hope that under the Constitution
bolster that requirement, we have held that the judge must not only be of the future every Filipino shall be truly sovereign in his own country, able
impartial but must also appear to be impartial as an added assurance to the to express his will through the pristine ballow with only his conscience as his
parties that his decision will be just. 16 The litigants are entitled to no less counsel.
than that. They should be sure that when their rights are violated they can go
to a judge who shall give them justice. They must trust the judge, otherwise This is not an impossible dream. Indeed, it is an approachable goal. It can
they will not go to him at all. They must believe in his sense of fairness, and will be won if we are able at last, after our long ordeal, to say never again
otherwise they will not seek his judgment. Without such confidence, there to tyranny. If we can do this with courage and conviction, then and only then,
would be no point in invoking his action for the justice they expect. and not until then, can we truly say that the case is finished and the book is
closed.
Due process is intended to insure that confidence by requiring compliance
with what Justice Frankfurter calls the rudiments of fair play. Fair play calls WHEREFORE, let it be spread in the records of this case that were it not for
for equal justice. There cannot be equal justice where a suitor approaches a the supervening events that have legally rendered it moot and academic, this
court already committed to the other party and with a judgment already made petition would have been granted and the decision of the Commission on
and waiting only to be formalized after the litigants shall have undergone the Elections dated July 23, 1984, set aside as violative of the Constitution.
charade of a formal hearing. Judicial (and also extrajudicial) proceedings are
not orchestrated plays in which the parties are supposed to make the motions SO ORDERED.
and reach the denouement according to a prepared script. There is no writer
to foreordain the ending. The judge will reach his conclusions only after all
the evidence is in and all the arguments are filed, on the basis of the
established facts and the pertinent law.

The relationship of the judge with one of the parties may color the facts and
distort the law to the prejudice of a just decision. Where this is probable or
even only possible, due process demands that the judge inhibit himself, if
only out of a sense of delicadeza. For like Caesar's wife, he must be above
suspicion. Commissioner Opinion, being a lawyer, should have recognized
his duty and abided by this well-known rule of judicial conduct. For refusing
to do so, he divested the Second Division of the necessary vote for the
questioned decision, assuming it could act, and rendered the proceeding null
and void. 17
PEOPLE OF THE PHILIPPINES vs. RAMOS Exhibit "C-1" The bracketed portions of Exhibit "C" stating among others
G.R. No. L-59318 | 1983-05-16 that it was Rogelio Ramos herein accused who furnished Malcon Olevere the
GUERRERO, J.: marijuana leaves;

This is an automatic review of the decision of the Court of First Instance of Exhibit "D-1" marijuana leaves examined;
Manila finding the accused Rogelio Ramos y Gaerlan in Criminal Case No.
61029 guilty beyond reasonable doubt of violation of Section 4, Article II, in "Exhibit "E" the envelope containing the marijuana leaves which was
relation to Section 2(i), Article I of the Republic Act No. 6425, as amended confiscated from Malcon Olevere.
by P.D. No. 44 and further amended by P.D. No. 1675, and imposing upon
him the penalty to reclusion perpetua. After the trial, the Court of First Instance of Manila (now the Regional Trial
Court) found the accused-appellant Ramos guilty beyond reasonable doubt
There is no dispute about the facts of this case. At about 10:00 o'clock in the of the crime charged in view of the verbal admission given by the appellant
evening of May 3, 1981, while P/Lt. E. Mediavillo and P/Sgt. A. Linga were himself and the evidence offered and admitted in court. The dispositive
on routine patrol along Taft Avenue, they had seen and observed one portion of its judgment reads:
MALCON OLEVERE y NAPA, acting suspiciously near the corner of
Estrada Street. 1 The police officers, after identifying themselves, stopped "WHEREFORE, accused ROGELIO RAMOS y GAERLAN is hereby found
and frisked the suspect and found in his possession dried marijuana leaves. 2 guilty beyond reasonable doubt of a violation of Section 4, Article II in
The police officers thereafter placed Malcon Olevere under arrest. Upon relation to Section 2(i), Article I Republic Act No. 6425, as amended by PD
investigation, suspect Olevere declared that he bought the recovered 44 and further amended by PD 1675 as charged in the present information,
marijuana leaves from one ROGELIO RAMOS y GAERLAN, alias for selling subject prohibited drugs (marijuana leaves) without any lawful
"Balanchoy". 3 authority and is hereby sentenced to suffer the penalty of reclusion perpetua
(life imprisonment); to pay a fine of Twenty Thousand (P20,000.00) pesos,
The following day, May 4, 1981, at about 12:00 o'clock noon, a police team without any subsidiary imprisonment in case of insolvency; and to pay the
with suspect Malcon Olevere y Napa proceeded to the residence of appellant costs. Let the accused be given full credit of the entire period of his
Rogelio Ramos y Gaerlan in 2366 Singalong, Malate, Manila and arrested preventive imprisonment.
him. The police operatives immediately brought appellant to the Drugs
Enforcement Section Western Police Department Headquarters for Subject marijuana leaves (Exhibit E) are confiscated, to be destroyed by the
investigation. Dangerous Drugs Board pursuant to law.

During the custodial investigation, suspect Malcon Olevere executed a SO ORDERED." 11


written sworn statement implicating the accused-appellant Rogelio Ramos as
the source of the marijuana leaves. 4 The accused, after having been duly The case is now before Us for automatic review. Accused-appellant submits
apprised of his constitutional rights, verbally admitted before Lt. E. before this Honorable Court the following errors: 12
Mediavillo and Sgt. A. Linga the commission of the offense charged. He
likewise admitted that he sold to Malcon Olevere the marijuana leaves for I
P10.00. 5 That the court erred in finding the accused guilty of violation of Section 4
Article II of Republic Act No. 6425 otherwise known as the Dangerous Drugs
On May 22, 1981, upon arraignment, the accused-appellant Ramos entered a Act of 1972, as amended (Selling-Pushing).
plea of not guilty to the information filed by assistant fiscal Antonio J.
Ballena which states: II
That the court erred in its findings both in question of law and fact in
"That on or about May 4, 1981, in the City of Manila, Philippines, the said convicting the accused notwithstanding the failure of the prosecution to
accused, not being authorized by law to sell, deliver, give away to another or adduce the quantum of evidence necessary to establish the guilt of the
distribute any prohibited drug, did then and there willfully and unlawfully accused beyond reasonable doubt by failing to present Malcon Olevere y
sell or offer for sale and deliver dried marijuana leaves, which is a prohibited Napa, the person who claimed that it was the therein accused who allegedly
drug. sold the marijuana leaves.

Contrary to law." 6 III


That the constitutional rights of the accused, more particularly the right to
At the trial, the prosecution presented three witnesses to wit: Patrolman Jaime meet the witness against him face to face and to cross-examine him has been
Cruz, a police investigator, Patrolman Agapito Linga, a police agent, and violated.
Felisa Vequilla, an NBI forensic chemist.
IV
Patrolman Cruz testified that on May 5, 1981, he investigated and took down That the court has acted with grave abuse of discretion amounting to a denial
the sworn statement of one Malcon Olevere who disclosed that the accused- of due process of law.
appellant Ramos was the source of the marijuana leaves. Patrolman Cruz also
testified that he prepared the Booking Sheet and Arrest Report of the The principal issue in this case is whether there is competent and/or
appellant Ramos and the corresponding Crime Report. 7 Patrolman Agapito admissible evidence in the record to justify the conviction of the accused-
Linga declared on the witness stand that Lt. Mediavilla arrested appellant appellant Ramos.
Ramos because Malcon Olevere declared that the appellant sold to him the
confiscated marijuana leaves. 8 The third witness, Felisa Vequilla, a forensic We find petitioner's case meritorious. The lower court erred in admitting as
chemist, affirmed that after conducting a dangerous drug test, the leaves evidence the written sworn affidavit of Malcon Olevere. It can be gleaned
confiscated from Malcon Olevere are positive for marijuana. 9 from the records that Malcon Olevere executed the written sworn statement
declaring that appellant Ramos sold to him the marijuana leaves for P10.00.
The prosecution offered the following as documentary evidence. 10 This piece of evidence is a mere scrap of paper because Malcon Olevere was
not produced in court for cross-examination. An affidavit being taken ex-
Exhibit "A" The Booking Sheet and Arrest Report of accused Rogelio Ramos parte is often incomplete and inaccurate. 13 Such kind of evidence is
prepared by witness Patrolman Cruz which was offered as part of his considered hearsay. 14 The constitutional right to meet witnesses face to face
testimony; 15 in order not to deprive persons of their lives and properties without due
process of law is well-protected in our jurisprudence. Thus, in People vs.
Exhibit "B" Crime Report dated May 6, 1981 also prepared by the witness Toledo, 16 We elucidated:
Patrolman Cruz;
"Testimony in open court in actual trial cannot be equated with any out-of-
Exhibit "B-1" second page of Exhibit "B' court declaration, even when the witness has in fact been confronted already
by the defendant. The direct relevance of the trial to the ultimate judgment
Exhibit "C" Sworn Statement of Malcon Olevere y Napa; as to the guilt or innocence of the accused is not present in any other
proceeding and is thus a factor that can influence materially the conduct and
demeanor of the witness as well as the respective efforts of the counsels of "At the time a person is arrested, it shall be the duty of the arresting officer
the parties." to inform him of the reason for the arrest and he must be shown the warrant
of arrest, if any. He shall be informed of his constitutional rights to remain
For the court to admit the sworn statement of Malcon Olevere without giving silent and to counsel, and that any statement he might make could be used
the adverse party the right to cross-examine him would easily facilitate the against him. The person arrested shall have the right to communicate with
fabrication of evidence and the perpetration of fraud. The inadmissibility of his lawyer, a relative, or anyone he chooses by the most expedient means -
this sort of evidence is based, not only on the lack of opportunity on the part by telephone if possible - or by letter or messenger. It shall be the
of the adverse party to cross-examine the affiant, 17 but also on the responsibility of the arresting officer to see to it that this is accomplished. No
commonly known fact that, generally, an affidavit is not prepared by the custodial investigation shall be conducted unless it be in the presence of
affiant himself but by another who uses his own language in writing the counsel engaged by the person arrested, by any person on his behalf, or
affiant's statements which may either be omitted or misunderstood by the one appointed by the court upon petition either of the detainee himself or by
writing them. 18 anyone on his behalf. The right to counsel may be waived but the waiver shall
not be valid unless made with the assistance of counsel. Any statement
The Booking Sheet and the Dangerous Drug Report of chemist Felisa obtained in violation of the procedure herein laid down, whether exculpatory
Vequilla which were presented as evidence by the prosecution, established or inculpatory, in whole or in part, shall be inadmissible in evidence."
nothing to support the conviction of the appellant herein. For the same
reason, that Malcon Olevere was not presented as a witness and insofar as In the case at bar, appellant has only finished Grade VI, 22 which means that
they impute to appellant the commission of the crime charged, the adduced he is not adequately educated to understand fairly and fully the significance
evidence are nothing but hearsay evidence. They cannot be regarded as of his constitutional rights to silence and to counsel. As mandated, it is not
competent evidence as to the veracity of the contents therein. enough that the police investigator merely informs him of his constitutional
rights to silence and to counsel, and then taking his statements down, the
It is not disputed that the marijuana leaves recovered and tested by witness interrogating officer must have patience in explaining these rights to him.
Vequilla came from Malcon Olevere and not from appellant. It would be The records do not reveal that these requirements have been fully complied
absurd and manifestly unjust to conclude that appellant had been selling with, nor was there any showing that appellant has been represented by
marijuana stuff just because what were recovered from Olevere were real counsel during custodial investigation. In consonance with Section 20 of the
marijuana. Proof of one does not necessarily prove another. Nowhere can it Bill of Rights which states that "any confession obtained in violation of this
be found on the record that appellant was caught in possession or in the act section shall be inadmissible in evidence," We hold that the verbal
of selling the prohibited marijuana leaves. admissions of appellant during custodial investigation may not be taken in
evidence against him.
The oral testimonies given by the witnesses for the prosecution prove nothing
material and culpable against the accused. As correctly pointed out by the We hold and rule that the guilt of the accused has not been established beyond
Solicitor General, not anyone of the three witnesses presented testified on the reasonable doubt and he is, therefore, entitled to acquittal.
basis of their personal knowledge that the appellant sold the marijuana leaves
to Malcon Olevere. Under Rule 130, Sec. 30 of the Revised Rules of Court, WHEREFORE, IN VIEW OF THE FOREGOING, the decision of the Court
"a witness can testify only to those facts which he knows of his own of First Instance of Manila is REVERSED, and appellant is hereby
knowledge, that is, which are derived from his own perception. . . " ACQUITTED of the crime charged in the information. No costs.

A witness, therefore, may not testify as to what he merely learned from SO ORDERED.
others, either because he was told or having read or heard the same. Such
testimony is considered hearsay and may not be received as proof of the truth
of what he has learned. Since Malcon Olevere was not presented as a witness,
the testimonies offered by the witnesses for the prosecution are regarded as
hearsay, insofar as they impute to the appellant the commission of the offense
charged.

The lower court, in convicting appellant of the crime charged, partly relief
on the verbal admission made by appellant himself before Lt. Mediavillo and
Sgt. Linga during the custodial investigation. Although the records prove that
the appellant has been duly apprised of his constitutional rights to silence and
to counsel, 19 We are not fully convinced that this appriasal was sufficiently
manifested and intelligently understood and accepted by the appellant. This
is fatal to the admissibility of appellant's verbal admission. We have
repeatedly emphasized that care should be taken in accepting extrajudicial
admissions, especially when taken during custodial investigation. In People
vs. Caquioa, 20 We ruled:

"As for the procedural safeguards to be employed, unless other fully effective
means are devised to inform accused persons of their right to silence and
assure a continuous opportunity to exercise it, the following measures are
required. Prior to questioning, the person must be warned that he has a right
to remain silent, that any statement he does may be used as evidence against
him, and that he has a right to the presence of an attorney, either retained or
appointed. The defendant may waive effectuation of those rights provided
the waiver is made voluntarily, knowingly and intelligently. If however, he
indicates in any manner and at any stage of the prosecution that he wishes to
consult with an attorney before speaking, there can be no questioning.
Likewise, if the individual is alone and indicates in any manner that he does
not wish to be interrogated, the police may not question him. The mere fact
that he may have answered some questions or volunteered some statements
on his own does not deprive him of the right to refrain from answering any
further inquiries until he has consulted with an attorney and thereafter
consents to be questioned."

Again, the constitutional rights of the accused to silence and to counsel is


fortified in the very recent case of Morales and Moncupa vs. Enrile 21 where
this Court said:
CHAVEZ vs. THE HONORABLE COURT OF APPEALS COURT:
G.R. No. L-29169 August 19, 1968
SANCHEZ, J.: The Court will give counsel for Roger Chavez fifteen minutes within which
to confer and explain to his client about the giving of his testimony.
The thrust of petitioner's case presented in his original and supplementary
petitions invoking jurisdiction of this Court is that he is entitled, on habeas xxx xxx xxx
corpus, to be freed from imprisonment upon the ground that in the trial which
resulted in his conviction1 he was denied his constitutional right not to be COURT: [after the recess]
compelled to testify against himself. There is his prayer, too, that, should he
fail in this, he be granted the alternative remedies of certiorari to strike down Are the parties ready? .
the two resolutions of the Court of Appeals dismissing his appeal for failure
to file brief, and of mandamus to direct the said court to forward his appeal FISCAL:
to this Court for the reason that he was raising purely questions of law.
We are ready to call on our first witness, Roger Chavez.
The indictment in the court below — the third amended information — upon
which the judgment of conviction herein challenged was rendered, was for ATTY. CARBON:
qualified theft of a motor vehicle, one (1) Thunderbird car, Motor No. H9YH-
143003, with Plate No. H-16648 Pasay City '62 together with its accessories As per understanding, the proceeding was suspended in order to enable me
worth P22,200.00. Accused were the following: Petitioner herein, Roger to confer with my client.
Chavez, Ricardo Sumilang alias "Romeo Vasquez", Edgardo P. Pascual alias
"Ging" Pascual, Pedro Rebullo alias "Pita", Luis Asistio alias "Baby" Asistio, I conferred with my client and he assured me that he will not testify for the
Lorenzo Meneses alias "Lory" Meneses, Peter Doe, Charlie Doe and Paul prosecution this morning after I have explained to him the consequences of
Doe.2 what will transpire.

Averred in the aforesaid information was that on or about the 14th day of COURT:
November, 1962, in Quezon City, the accused conspired, with intent of gain,
abuse of confidence and without the consent of the owner thereof, Dy Sun What he will testify to does not necessarily incriminate him, counsel.
Hiok y Lim, in asporting the motor vehicle above-described.
And there is the right of the prosecution to ask anybody to act as witness on
Upon arraignment, all the accused, except the three Does who have not been the witness-stand including the accused.
identified nor apprehended, pleaded not guilty.1äwphï1.ñët
If there should be any question that is incriminating then that is the time for
On July 23, 1963, trial commenced before the judge presiding Branch IX of counsel to interpose his objection and the court will sustain him if and when
the Court of First Instance of Rizal in Quezon City. the court feels that the answer of this witness to the question would
incriminate him.
The trial opened with the following dialogue, which for the great bearing it
has on this case, is here reproduced:. Counsel has all the assurance that the court will not require the witness to
answer questions which would incriminate him.
COURT:
But surely, counsel could not object to have the accused called on the
The parties may proceed. witnessstand.

FISCAL GRECIA: ATTY. CARBON:

Our first witness is Roger Chavez [one of the accused]. I submit.

ATTY. CARBON [Counsel for petitioner Chavez]: xxx xxx xxx

I am quite taken by surprise, as counsel for the accused Roger Chavez, with ATTY. CRUZ [Counsel for defendants Pascual and Meneses]: .
this move of the Fiscal in presenting him as his witness. I object.
MAY IT PLEASE THE COURT:
COURT:
This incident of the accused Roger Chavez being called to testify for the
On what ground, counsel? . prosecution is something so sudden that has come to the knowledge of this
counsel.
ATTY. CARBON:
This representation has been apprised of the witnesses embraced in the
On the ground that I have to confer with my client. It is really surprising that information.
at this stage, without my being notified by the Fiscal, my client is being
presented as witness for the prosecution. I want to say in passing that it is For which reason I pray this court that I be given at least some days to meet
only at this very moment that I come to know about this strategy of the whatever testimony this witness will bring about. I therefore move for
prosecution. postponement of today's hearing.

COURT (To the Fiscal): COURT:

You are not withdrawing the information against the accused Roger Chavez The court will give counsel time within which to prepare his cross-
by making [him a] state witness?. examination of this witness.

FISCAL GRECIA: ATTY. CRUZ:

I am not making him as state witness, Your Honor. I labored under the impression that the witnesses for the prosecution in this
I am only presenting him as an ordinary witness. criminal case are those only listed in the information.

ATTY. CARBON: I did not know until this morning that one of the accused will testify as
witness for the prosecution.
As a matter of right, because it will incriminate my client, I object.
COURT:
As payment was to be made at Eugene's restaurant in Quezon City, all of
That's the reason why the court will go along with counsels for the accused them then drove in the Thunderbird car to that place. The deed of sale and
and will give them time within which to prepare for their cross-examination other papers remained in the pockets of Johnson Lee.
of this witness.
At Eugene's, a man approached Sumilang with a note which stated that the
The court will not defer the taking of the direct examination of the witness. money was ready at the Dalisay Theater. Sumilang then wrote on the same
note that the money should be brought to the restaurant. At the same time he
Call the witness to the witness stand. requested Lee to exhibit the deed of sale of the car to the note bearer.4

EVIDENCE FOR THE PROSECUTION Then, the two Chinese were left alone in the restaurant. For Sumilang, who
had left the table to pose for pictures with some fans and come back, again
ROGER CHAVEZ, 31 years old, single, buy and sell merchant, presently left never to return. So did Chavez, who disappeared after he left on the
detained at the Manila Police Department headquarters, after being duly pretext of buying cigarettes. The two Chinese could not locate Sumilang and
sworn according to law, declared as follows: Chavez. They went out to the place where the Thunderbird was parked, found
that it was gone. They then immediately reported its loss to the police. Much
ATTY. IBASCO [Counsel for defendant Luis Asistio]: later, the NBI recovered the already repainted car and impounded it.

WITH THE LEAVE OF THE COURT: Right after the meeting at Eugene's, Chavez, Sumilang and Asistio converged
that same day at Barrio Fiesta, a restaurant at Highway 54 near the
This witness, Roger Chavez is one of the accused in this case No. Q-5311. Balintawak monument in Caloocan. There, Asistio handed to Sumilang
P1,000.00 cash and a golf set worth P800.00 as the latter's share in the
The information alleges conspiracy. Under Rule 123, Section 12, it states: transaction. On the 14th of November, the registration of the car was
transferred in the name of Sumilang in Cavite City, and three days later, in
'The act or declaration of a conspirator relating to the conspiracy and during the name of Asistio in Caloocan.
its existence, may be given in evidence against the co-conspirator after the
conspiracy is shown by evidence other than such act or declaration.' From the court's decision, Ricardo Sumilang's version, corroborated in part
by Asistio, may be condensed as follows:
COURT:
In the last week of September, 1962, Sumilang saw Roger Chavez at a gas
That is premature, counsel. Neither the court nor counsels for the accused station. The latter informed him that there was a Thunderbird from Clark
know what the prosecution events to establish by calling this witness to the Field for sale for a price between P20,000.00 and P22,000.00. Chavez said
witness stand. that it could be held for him with a down payment of P10,000.00.

ATTY. IBASCO: To raise this sum, Sumilang and Chavez, on October 1, went to the house of
a certain Nena Hernaez de los Reyes who wrote out a check for P5,000.00 as
I submit. a loan to Sumilang. That check was exhibited in court. Sumilang and Chavez
then went to Pasay City to see a certain Mario Baltazar, an agent of the Pasay
COURT: The Fiscal may proceed.3 City Mayor, and Narsing Cailles, Chief of the Fire Department. Sumilang
asked the two for a P10,000-loan backed up by the P5,000.00-check aforesaid
And so did the trial proceed. It began with the "direct examination" of Roger on condition that it should not be cashed immediately as there were not
Chavez by "Fiscal Grecia". enough funds therefor. Baltazar and Cailles agreed to give the money the
nextday as long as the check would be left with them and Sumilang would
Came the judgment of February 1, 1965. The version of the prosecution as sign a promissory note for P10,000.00. Baltazar later informed Sumilang that
found by the court below may be briefly narrated as follows: Chavez picked up the money the next day. Four or five days afterwards,
Chavez returned P4,000.00 to Sumilang because P6,000.00 was enough for
A few days before November 12, 1962, Roger Chavez saw Johnson Lee, a the deposit. And so, Sumilang gave back the P4,000.00 to Baltazar.
Chinese, driving a Thunderbird car. With Ricardo Sumilang (movie actor
Romeo Vasquez) in mind, whom he knew was in the market for such a car, About the end of October or at the beginning of November, Chavez asked
Chavez asked Lee whether his car was for sale. Lee answered affirmatively Sumilang for another P3,000.00. Sumilang sent Chavez to Baltazar and
and left his address with Chavez. Then, on November 12, Chavez met Cailles, with a note requesting that they accommodate him once more. He
Sumilang at a barbershop informed him about the Thunderbird. But also sent a check, again without funds. Baltazar gave the money after
Sumilang said that he had changed his mind about buying a new car. Instead, verifying the authenticity of the note.
he told Chavez that he wanted to mortgage his Buick car for P10,000.00 to
cover an indebtedness in Pasay City. Upon the suggestion of Chavez, they On November 14, Chavez appeared at Sumilang's house with the news that
went to see Luis Asistio, who he knew was lending money on car mortgages the car was ready if Sumilang was ready with the rest of the money. So
and who, on one occasion, already lent Romeo Vasquez P3,000.00 on the Sumilang got P9,000.00 from his mother and another P4,000.00 from his
same Buick car. Asistio however told the two that he had a better idea on aparador. He immediately gave P6,000.00 to Chavez, intending to pay out
how to raise the money. His plan was to capitalize on Romeo Vasquez' the balance upon the car's delivery. It was then that Chavez told Sumilang
reputation as a wealthy movie star, introduce him as a buyer to someone who that the car was already bought by a Chinese who would be the vendor.
was selling a car and, after the deed of sale is signed, by trickery to run away
with the car. Asistio would then register it, sell it to a third person for a profit. The purchase price finally agreed upon between Sumilang and Johnson Lee
Chavez known to be a car agent was included in the plan. He furnished the was P21,000.00, plus P500.00 agents commission at the expense of the
name of Johnson Lee who was selling his Thunderbird. 1äwphï1.ñët buyer. Sumilang told Lee that he already paid part of the price to Chavez.

In the morning of November 14, Chavez telephoned Johnson Lee and At Eugene's, Chavez asked Sumilang for the balance. Sumilang
arranged for an appointment. Sometime in the afternoon. Chavez and accommodated. There, Sumilang, also saw a friend, "Ging" Pascual. In the
Sumilang met Lee in his Thunderbird on Highway 54. Sumilang was course of their conversation at the bar, Sumilang mentioned the proposed
introduced as the interested buyer. Sumilang's driver inspected the car, took transaction thru Chavez. Pascual warned that Chavez was a "smart" agent
the wheel for a while. After Sumilang and Lee agreed on the purchase price and advised that Sumilang should have a receipt for his money. A certain
(P21.000.00), they went to Binondo to Johnson Lee's cousin, Dy Sun Hiok, Bimbo, a friend of Pascual, offered to make out a receipt for Chavez to sign.
in whose name the car was registered. Thereafter, they went to see a lawyer
notary public in Quezon City, known to Chavez for the drafting of the deed After Sumilang returned from posing for some photographs with some of his
of sale. After the deed of sale was drawn up, it was signed by Sumilang as fans, Bimbo showed him the receipt already signed by Chavez. Sumilang
the vendee, Dy Sun Hiok the vendor, and Sumilang's driver and Johnson Lee requested Pascual and Bimbo to sign the receipt as witnesses. And they did.
the witnesses thereto. This receipt was offered as an exhibit by the prosecution and by Sumilang.
When Sumilang was ready to leave Eugene's, Johnson Lee turned over to Bilibid Prisons pending execution of the judgment below, and ordered
him the deed of sale, the registration papers and the keys to the car. After remand of the case to the Quezon City court for execution of judgment.
shaking hands with Lee, Sumilang drove away in the car with his driver at
the wheel. It was at this stage that the present proceedings were commenced in this
Court.
Two or three days afterwards, Sumilang dropped by the Barrio Fiesta on his
way to a film shooting at Bulacan. He saw Asistio with many companions. Upon the petitions, the return, and the reply, and after hearing on oral
Asistio liked his Thunderbird parked outside. Asistio offered to buy it from arguments, we now come to grips with the main problem presented.
him for P22,500.00. As the offer was good, and knowing Asistio's and his
friends' reputation for always getting what they wanted, Sumilang consented We concentrate attention on that phase of the issues which relates petitioner's
to the sale. Asistio tendered a down payment of P1,000.00; the balance he assertion that he was compelled to testify against himself. For indeed if this
promised to pay the next day after negotiating with some financing company. one question is resolved in the affirmative, we need not reach the others; in
Before said balance could be paid, the car was impounded. which case, these should not be pursued here.

The trial court gave evidence to Sumilang's averment, strengthened by 1. Petitioner's plea on this score rests upon his averment, with proof, of
Baltazar's and Cailles' corroborations, that he paid good money for the car. violation of his right — constitutionally entrenched — against self-
Sumilang was thus cleared. So was Asistio whom the trial court believed to incrimination. He asks that the hand of this Court be made to bear down upon
be a mere buyer of the car. And so, the prosecution's theory of conspiracy his conviction; that he be relieved of the effects thereof. He asks us to
was discounted. consider the constitutional injunction that "No person shall be compelled to
be a witness against himself,"9 fully echoed in Section 1, Rule 115, Rules of
As to the other accused, the court found no case against Pedro Rebullo alias Court where, in all criminal prosecutions, the defendant shall be entitled: "(e)
"Pita" and Lorenzo Meneses alias "Lory". The accused "Ging" Pascual was To be exempt from being a witness against himself." .
also acquitted for in the first place he was not identified by Johnson Lee in
court. It has been said that forcing a man to be a witness against himself is at war
with "the fundamentals of a republican government"; 10 that [i]t may suit the
As to Roger Chavez, however, the court had this to say: "Roger Chavez does purposes of despotic power but it can not abide the pure atmosphere of
not offer any defense. As a matter of fact, his testimony as witness for the political liberty and personal freedom."11 Mr. Justice Abad Santos recounts
prosecution establishes his guilt beyond reasonable doubt."5 The trial court the historical background of this constitutional inhibition, thus: " "The maxim
branded him "a self-confessed culprit".6 The court further continued: Nemo tenetur seipsum accusare had its origin in a protest against the
inquisitorial and manifestly unjust methods of interrogating accused persons,
It is not improbable that true to the saying that misery loves company Roger which has long obtained in the continental system, and, until the expulsion
Chavez tried to drag his co-accused down with him by coloring his story with of the Stuarts from the British throne in 1688, and the erection of additional
fabrications which he expected would easily stick together what with the barriers for the protection of the people against the exercise of arbitrary
newspaper notoriety of one and the sensationalism caused by the other. But power, was not uncommon even in England. While the admissions of
Roger Chavez' accusations of Asistio's participation is utterly confessions of the prisoner, when voluntarily and freely made, have always
uncorroborated. And coming, as it does, from a man who has had at least two ranked high in the scale of incriminating evidence, if an accused person be
convictions for acts not very different from those charged in this information, asked to explain his apparent connection with a crime under investigation,
the Court would be too gullible if it were to give full credence to his words the ease with which the questions put to him may assume an inquisitorial
even if they concerned a man no less notorious than himself.7 character, the temptation to press, the witness unduly, to browbeat him if he
be timid or reluctant, to push him into a corner, and to entrap him into fatal
The trial court then came to the conclusion that if Johnson Lee was not paid contradictions, which is so painfully evident in many of the earlier state trials,
for his car, he had no one but Roger Chavez to blame. notably in those of Sir Nicholas Throckmorton, and Udal, the Puritan
minister, made the system so odious as to give rise to a demand for its total
The sum of all these is that the trial court freed all the accused except Roger abolition. The change in the English criminal procedure in that particular
Chavez who was found guilty beyond reasonable doubt of the crime of seems to be founded upon no statute and no judicial opinion, but upon a
qualified theft. He was accordingly sentenced to suffer an indeterminate general and silent acquiescence of the courts in a popular demand. But,
penalty of not less than ten (10) years, one (1) day, as minimum and not more however adopted, it has become firmly embedded in English, as well as in
than fourteen (14) years, eight (8) months and one (1) day as maximum, to American jurisprudence. So deeply did the iniquities of the ancient system
indemnify Dy Sun Hiok and/or Johnson Lee in the sum of P21,000.00 impress themselves upon the minds of the American colonists that the states,
without subsidiary imprisonment in case of insolvency, to undergo the with one accord, made a denial of the right to question an accused person a
accessory penalties prescribed by law, and to pay the costs. The Thunderbird part of their fundamental law, so that a maxim which in England was a mere
car then in the custody of the NBI was ordered to be turned over to Ricardo rule of evidence, became clothed in this country with the impregnability of a
Sumilang, who was directed to return to Asistio the sum of P1,000.00 unless constitutional enactment." (Brown vs. Walker, 161 U.S., 591, 597; 40 Law.
the latter chose to pay P21,500.00, representing the balance of the contract ed., 819, 821)." 12 Mr. Justice Malcolm, in expressive language, tells us that
price for the car. this maxim was recognized in England in the early days "in a revolt against
the thumbscrew and the rack." 13 An old Philippine case [1904] 14 speaks
The foregoing sentence was promulgated on March 8, 1965. Roger Chavez of this constitutional injunction as "older than the Government of the United
appealed to the Court of Appeals. States"; as having "its origin in a protest against the inquisitorial methods of
interrogating the accused person"; and as having been adopted in the
On April 18, 1968, the Court of Appeals required Atty. Natividad Marquez, Philippines "to wipe out such practices as formerly prevailed in these Islands
counsel for Roger Chavez, to show cause within ten days from notice why of requiring accused persons to submit to judicial examinations, and to give
Chavez' appeal should not be considered abandoned and dismissed. Reason testimony regarding the offenses with which they were charged."
for this is that said lawyer received notice to file brief on December 28, 1967
and the period for the filing thereof lapsed on January 27, 1968 without any So it is then that this right is "not merely a formal technical rule the
brief having been filed. enforcement of which is left to the discretion of the court"; it is mandatory;
it secures to a defendant a valuable and substantive right; 15 it is fundamental
On May 13, 1968, Atty. Marquez registered a detailed written explanation. to our scheme of justice. Just a few months ago, the Supreme Court of the
She also stated that if she were allowed to file appellant's brief she would go United States (January 29, 1968), speaking thru Mr. Justice Harlan warned
along with the factual findings of the court below but will show however that that "[t]he constitutional privilege was intended to shield the guilty and
its conclusion is erroneous.8 imprudent as well as the innocent and foresighted." 16

On May 14, 1968, the Court of Appeals, despite the foregoing explanation, It is in this context that we say that the constitutional guarantee may not be
resolved to dismiss the appeal. A move to reconsider was unavailing. For, on treated with unconcern. To repeat, it is mandatory; it secures to every
June 21, 1968, the Court of Appeals, through a per curiam resolution, defendant a valuable and substantive right. Tañada and Fernando
disposed to maintain its May 14 resolution dismissing the appeal, directed (Constitution of the Philippines, 4th ed., vol. I, pp. 583-584) take note of U.S.
the City Warden of Manila where Chavez is confined by virtue of the warrant vs. Navarro, supra, which reaffirms the rule that the constitutional
of arrest issued by the Court of Appeals, to turn him over to Muntinlupa proscription was established on broad grounds of public policy and
humanity; of policy because it would place the witness against the strongest the truth, no genuine consent underlay submission to take the witness stand.
temptation to commit perjury, and of humanity because it would be to extort Constitutionally sound consent was absent.
a confession of truth by a kind of duress every species and degree of which
the law abhors. 17 3. Prejudice to the accused for having been compelled over his objections to
be a witness for the People is at once apparent. The record discloses that by
Therefore, the court may not extract from a defendant's own lips and against leading questions Chavez, the accused, was made to affirm his statement
his will an admission of his guilt. Nor may a court as much as resort to given to the NBI agents on July 17, 1963 at 5:00 o'clock in the afternoon. 26
compulsory disclosure, directly or indirectly, of facts usable against him as a And this statement detailed the plan and execution thereof by Sumilang
confession of the crime or the tendency of which is to prove the commission (Vasquez), Asistio and himself to deprive the Chinese of his Thunderbird car.
of a crime. Because, it is his right to forego testimony, to remain silent, unless And he himself proceeded to narrate the same anew in open court. He
he chooses to take the witness stand — with undiluted, unfettered exercise of identified the Thunderbird car involved in the case. 27
his own free, genuine will.
The decision convicting Roger Chavez was clearly of the view that the case
Compulsion as it is understood here does not necessarily connote the use of for the People was built primarily around the admissions of Chavez himself.
violence; it may be the product of unintentional statements. Pressure which The trial court described Chavez as the "star witness for the prosecution".
operates to overbear his will, disable him from making a free and rational Indeed, the damaging facts forged in the decision were drawn directly from
choice, or impair his capacity for rational judgment would in our opinion be the lips of Chavez as a prosecution witness and of course Ricardo Sumilang
sufficient. So is moral coercion "tending to force testimony from the for the defense. There are the unequivocal statements in the decision that
unwilling lips of the defendant." 18 "even accused Chavez" identified "the very same Thunderbird that Johnson
Lee had offered for sale"; that Chavez "testimony as witness for the
2. With the foregoing as guideposts, we now turn to the facts. Petitioner is a prosecution establishes his guilt beyond reasonable doubt and that Chavez is
defendant in a criminal case. He was called by the prosecution as the first "a self-confessed culprit". 1äwphï1.ñët
witness in that case to testify for the People during the first day of trial
thereof. Petitioner objected and invoked the privilege of self-incrimination. 4. With all these, we have no hesitancy in saying that petitioner was forced
This he broadened by the clear cut statement that he will not testify. But to testify to incriminate himself, in full breach of his constitutional right to
petitioner's protestations were met with the judge's emphatic statement that remain silent. It cannot be said now that he has waived his right. He did not
it "is the right of the prosecution to ask anybody to act as witness on the volunteer to take the stand and in his own defense; he did not offer himself
witness stand including the accused," and that defense counsel "could not as a witness; on the contrary, he claimed the right upon being called to testify.
object to have the accused called on the witness stand." The cumulative If petitioner nevertheless answered the questions inspite of his fear of being
impact of all these is that accused-petitioner had to take the stand. He was accused of perjury or being put under contempt, this circumstance cannot be
thus peremptorily asked to create evidence against himself. The foregoing counted against him. His testimony is not of his own choice. To him it was a
situation molds a solid case for petitioner, backed by the Constitution, the case of compelled submission. He was a cowed participant in proceedings
law, and jurisprudence. before a judge who possessed the power to put him under contempt had he
chosen to remain silent. Nor could he escape testifying. The court made it
Petitioner, as accused, occupies a different tier of protection from an ordinary abundantly clear that his testimony at least on direct examination would be
witness. Whereas an ordinary witness may be compelled to take the witness taken right then and thereon the first day of the trial.
stand and claim the privilege as each question requiring an incriminating
answer is shot at him, 19 and accused may altogether refuse to take the It matters not that, after all efforts to stave off petitioner's taking the stand
witness stand and refuse to answer any and all questions. 20 For, in reality, became fruitless, no objections to questions propounded to him were made.
the purpose of calling an accused as a witness for the People would be to Here involve is not a mere question of self-incrimination. It is a defendant's
incriminate him. 21 The rule positively intends to avoid and prohibit the constitutional immunity from being called to testify against himself. And the
certainly inhuman procedure of compelling a person "to furnish the missing objection made at the beginning is a continuing one. 1äwphï1.ñët
evidence necessary for his conviction." 22 This rule may apply even to a co-
defendant in a joint trial.23 There is therefore no waiver of the privilege. "To be effective, a waiver must
be certain and unequivocal, and intelligently, understandably, and willingly
And the guide in the interpretation of the constitutional precept that the made; such waiver following only where liberty of choice has been fully
accused shall not be compelled to furnish evidence against himself "is not the accorded. After a claim a witness cannot properly be held to have waived his
probability of the evidence but it is the capability of abuse." 24 Thus it is, privilege on vague and uncertain evidence." 28 The teaching in Johnson vs.
that it was undoubtedly erroneous for the trial judge to placate petitioner with Zerbst 29 is this: "It has been pointed out that "courts indulge every
these words:. reasonable presumption against waiver" of fundamental constitutional rights
and that we "do not presume acquiescence in the loss of fundamental rights."
What he will testify to does not necessarily incriminate him, counsel. A waiver is ordinarily an intentional relinquishment or abandonment of a
known right or privilege." Renuntiatio non praesumitur.
And there is the right of the prosecution to ask anybody to act as witness on
the witness-stand including the accused. The foregoing guidelines, juxtaposed with the circumstances of the case
heretofore adverted to, make waiver a shaky defense. It cannot stand. If, by
If there should be any question that is incriminating then that is the time for his own admission, defendant proved his guilt, still, his original claim
counsel to interpose his objection and the court will sustain him if and when remains valid. For the privilege, we say again, is a rampart that gives
the court feels that the answer of this witness to the question would protection - even to the guilty. 30
incriminate him.
5. The course which petitioner takes is correct. Habeas corpus is a high
Counsel has all the assurance that the court will not require the witness to prerogative writ. 31 It is traditionally considered as an exceptional remedy to
answer questions which would incriminate him. release a person whose liberty is illegally restrained such as when the
accused's constitutional rights are disregarded. 32 Such defect results in the
But surely, counsel could not object to have the accused called on the witness absence or loss of jurisdiction 33 and therefore invalidates the trial and the
stand. consequent conviction of the accused whose fundamental right was violated.
34 That void judgment of conviction may be challenged by collateral attack,
Paraphrasing Chief Justice Marshall in Aaron Burr's Trial, Robertsons Rep. which precisely is the function of habeas corpus. 35 This writ may issue even
I, 208, 244, quoted in VIII Wigmore, p. 355, 25 While a defendant's if another remedy which is less effective may be availed of by the defendant.
knowledge of the facts remains concealed within his bosom, he is safe; but 36 Thus, failure by the accused to perfect his appeal before the Court of
draw it from thence, and he is exposed" — to conviction. Appeals does not preclude a recourse to the writ. 37 The writ may be granted
upon a judgment already final. 38 For, as explained in Johnson vs. Zerbst, 39
The judge's words heretofore quoted — "But surely counsel could not object the writ of habeas corpus as an extraordinary remedy must be liberally given
to have the accused called on the witness stand" — wielded authority. By effect 40 so as to protect well a person whose liberty is at stake. The propriety
those words, petitioner was enveloped by a coercive force; they deprived him of the writ was given the nod in that case, involving a violation of another
of his will to resist; they foreclosed choice; the realities of human nature tell constitutional right, in this wise:
us that as he took his oath to tell the truth, the whole truth and nothing but
Since the Sixth Amendment constitutionally entitles one charged with crime
to the assistance of Counsel, compliance with this constitutional mandate is
an essential jurisdictional prerequisite to a Federal Court's authority. When
this right is properly waived, the assistance of Counsel is no longer a
necessary element of the Court's jurisdiction to proceed to conviction and
sentence. If the accused, however, is not represented by Counsel and has not
competently and intelligently waived his constitutional right, the Sixth
Amendment stands as a jurisdictional bar to a valid conviction and sentence
depriving him of his liberty. A court's jurisdiction at the beginning of trial
may be lost "in the course of the proceedings" due to failure to complete the
court — as the Sixth Amendment requires — by providing Counsel for an
accused who is unable to obtain Counsel, who has not intelligently waived
this constitutional guaranty, and whose life or liberty is at stake. If this
requirement of the Sixth Amendment is not complied with, the court no
longer has jurisdiction to proceed. The judgment of conviction pronounced
by a court without jurisdiction is void, and one imprisoned thereunder may
obtain release of habeas corpus. 41

Under our own Rules of Court, to grant the remedy to the accused Roger
Chavez whose case presents a clear picture of disregard of a constitutional
right is absolutely proper. Section 1 of Rule 102 extends the writ, unless
otherwise expressly provided by law, "to all cases of illegal confinement or
detention by which any person is deprived of his liberty, or by which the
rightful custody of any person is withheld from the person entitled thereto.

Just as we are about to write finis to our task, we are prompted to restate that:
"A void judgment is in legal effect no judgment. By it no rights are divested.
From it no rights can be obtained. Being worthless in itself, all proceedings
founded upon it are equally worthless. It neither binds nor bars any one. All
acts performed under it and all claims flowing out of it are void. The parties
attempting to enforce it may be responsible as trespassers. ... " 42

6. Respondents' return 43 shows that petitioner is still serving under a final


and valid judgment of conviction for another offense. We should guard
against the improvident issuance of an order discharging a petitioner from
confinement. The position we take here is that petitioner herein is entitled to
liberty thru habeas corpus only with respect to Criminal Case Q-5311 of the
Court of First Instance of Rizal, Quezon City Branch, under which he was
prosecuted and convicted.

Upon the view we take of this case, judgment is hereby rendered directing
the respondent Warden of the City Jail of Manila or the Director of Prisons
or any other officer or person in custody of petitioner Roger Chavez by
reason of the judgment of the Court of First Instance of Rizal, Quezon City
Branch, in Criminal Case Q-5311, entitled "People of the Philippines,
plaintiff, vs. Ricardo Sumilang, et al., accused," to discharge said Roger
Chavez from custody, unless he is held, kept in custody or detained for any
cause or reason other than the said judgment in said Criminal Case Q-5311
of the Court of First Instance of Rizal, Quezon City Branch, in which event
the discharge herein directed shall be effected when such other cause or
reason ceases to exist.

No costs. So ordered.
BELTRAN vs. SAMSON The question, then, is reduced to a determination of whether the writing from
G.R. No. 32025 September 23, 1929 the fiscal's dictation by the petitioner for the purpose of comparing the latter's
ROMUALDEZ, J.: handwriting and determining whether he wrote certain documents supposed
to be falsified, constitutes evidence against himself within the scope and
This is a petition for a writ of prohibition, wherein the petitioner complains meaning of the constitutional provision under examination.
that the respondent judge ordered him to appear before the provincial fiscal
to take dictation in his own handwriting from the latter. Whenever the defendant, at the trial of his case, testifying in his own behalf,
denies that a certain writing or signature is in his own hand, he may on cross-
The order was given upon petition of said fiscal for the purpose of comparing examination be compelled to write in open court in order that the jury maybe
the petitioner's handwriting and determining whether or not it is he who wrote able to compare his handwriting with the one in question.
certain documents supposed to be falsified.
It was so held in the case of Bradford vs. People (43 Pacific Reporter, 1013)
There is no question as to the facts alleged in the complaint filed in these inasmuch as the defendant, in offering himself as witness in his own behalf,
proceedings; but the respondents contend that the petitioner is not entitled to waived his personal privileges.
the remedy applied for, inasmuch as the order prayed for by the provincial
fiscal and later granted by the court below, and again which the instant action Of like character is the case of Sprouse vs. Com. (81 Va., 374,378), where
was brought, is based on the provisions of section 1687 of the Administrative the judge asked the defendant to write his name during the hearing, and the
Code and on the doctrine laid down in the cases of People vs. Badilla (48 latter did so voluntarily.
Phil., 718); United States vs. Tan Teng (23 Phil., 145); United States vs. Ong
Siu Hong (36 Phil., 735), cited by counsel for the respondents, and in the case But the cases so resolved cannot be compared to the one now before us. We
of Villaflor vs. Summers (41 Phil., 62) cited by the judge in the order in are not concerned here with the defendant, for it does not appear that any
question. information was filed against the petitioner for the supposed falsification, and
still less as it a question of the defendant on trial testifying and under cross-
Of course, the fiscal under section 1687 of the Administrative Code, and the examination. This is only an investigation prior to the information and with
proper judge, upon motion of the fiscal, may compel witnesses to be present a view to filing it. And let it further be noted that in the case of Sprouse vs.
at the investigation of any crime or misdemeanor. But this power must be Com., the defendant performed the act voluntarily.
exercised without prejudice to the constitutional rights of persons cited to
appear. We have also come upon a case wherein the handwriting or the form of
writing of the defendant was obtained before the criminal action was
And the petitioner, in refusing to perform what the fiscal demanded, seeks instituted against him. We refer to the case of People vs. Molineux (61
refuge in the constitutional provision contained in the Jones Law and Northeastern Reporter, 286).
incorporated in General Orders, No. 58.
Neither may it be applied to the instant case, because there, as in the aforesaid
Therefore, the question raised is to be decided by examining whether the case of Sprouse vs. Com., the defendant voluntarily offered to write, to
constitutional provision invoked by the petitioner prohibits compulsion to furnish a specimen of his handwriting.
execute what is enjoined upon him by the order against which these
proceedings were taken. We cite this case particularly because the court there gives prominence to the
defendant's right to decline to write, and to the fact that he voluntarily wrote.
Said provision is found in paragraph 3, section 3 of the Jones Law which (in The following appears in the body of said decision referred to (page 307 of
Spanish) reads: "Ni se le obligara a declarar en contra suya en ningun proceso the volume cited):
criminal" and has been incorporated in our Criminal Procedure (General
Orders, No. 58) in section 15 (No. 4 ) and section 56. The defendant had the legal right to refuse to write for Kinsley. He preferred
to accede to the latter's request, and we can discover no ground upon which
As to the extent of the privilege, it should be noted first of all, that the English the writings thus produced can be excluded from the case. (Emphasis ours.)
text of the Jones Law, which is the original one, reads as follows: "Nor shall
be compelled in any criminal case to be a witness against himself." For the reason it was held in the case of First National Bank vs. Robert (41
Mich., 709; 3 N. W., 199), that the defendant could not be compelled to write
This text is not limited to declaracion but says "to be a witness." Moreover, his name, the doctrine being stated as follows:
as we are concerned with a principle contained both in the Federal
constitution and in the constitutions of several states of the United States, but The defendant being sworn in his own behalf denied the endorsement.
expressed differently, we should take it that these various phrasings have a
common conception. He was then cross-examined the question in regard to his having signed
papers not in the case, and was asked in particular whether he would not
In the interpretation of the principle, nothing turns upon the variations of produce signatures made prior to the note in suit, and whether he would not
wording in the constitutional clauses; this much is conceded (ante, par. 2252). write his name there in the court. The judge excluded all these inquiries, on
It is therefore immaterial that the witness is protected by one constitution objection, and it is of these rulings that complaint is made. The object of the
from 'testifying', or by another from 'furnishing evidence', or by another from questions was to bring into the case extrinsic signatures, for the purpose of
'giving evidence,' or by still another from 'being a witness.' These various comparison by the jury, and we think that the judge was correct in ruling
phrasings have a common conception, in respect to the form of the protected against it.
disclosure. What is that conception? (4 Wigmore on Evidence, p. 863, 1923
ed.) It is true that the eminent Professor Wigmore, in his work cited (volume 4,
page 878), says:
As to its scope, this privilege is not limited precisely to testimony, but extends
to all giving or furnishing of evidence. Measuring or photographing the party is not within the privilege. Nor it is the
removal or replacement of his garments or shoes. Nor is the requirement that
The rights intended to be protected by the constitutional provision that no the party move his body to enable the foregoing things to be done. Requiring
man accused of crime shall be compelled to be a witness against himself is him to make specimens of handwriting is no more than requiring him to move
so sacred, and the pressure toward their relaxation so great when the his body . . ." but he cites no case in support of his last assertion on specimens
suspicion of guilt is strong and the evidence obscure, that is the duty of courts of handwriting. We note that in the same paragraph 2265, where said authors
liberally to construe the prohibition in favor of personal rights, and to refuse treats of "Bodily Exhibition." and under preposition "1. A great variety of
to permit any steps tending toward their invasion. Hence, there is the well- concrete illustrations have been ruled upon," he cites many cases, among
established doctrine that the constitutional inhibition is directed not merely them that of People vs. Molineux (61 N. E., 286) which, as we have seen, has
to giving of oral testimony, but embraces as well the furnishing of evidence no application to the case at bar because there the defendant voluntary gave
by other means than by word of mouth, the divulging, in short, of any fact specimens of his handwriting, while here the petitioner refuses to do so and
which the accused has a right to hold secret. (28 R. C. L., paragraph 20, page has even instituted these prohibition proceedings that he may not be
434 and notes.) (Emphasis ours.) compelled to do so.
Furthermore, in the case before us, writing is something more than moving testimonial act, to write and give a specimen of his handwriting for the
the body, or the hands, or the fingers; writing is not a purely mechanical act, purpose of comparison. Besides, in the case of Villamor vs. Summers, it was
because it requires the application of intelligence and attention; and in the sought to exhibit something already in existence, while in the case at bar, the
case at bar writing means that the petitioner herein is to furnish a means to question deals with something not yet in existence, and it is precisely sought
determine whether or not he is the falsifier, as the petition of the respondent to compel the petitioner to make, prepare, or produce by this means, evidence
fiscal clearly states. Except that it is more serious, we believe the present case not yet in existence; in short, to create this evidence which may seriously
is similar to that of producing documents or chattels in one's possession. And incriminate him.
as to such production of documents or chattels. which to our mind is not so
serious as the case now before us, the same eminent Professor Wigmore, in Similar considerations suggest themselves to us with regard to the case of
his work cited, says (volume 4, page 864): United States vs. Ong Siu Hong (36 Phil., 735), wherein the defendant was
not compelled to perform any testimonial act, but to take out of his mouth
. . . 2264. Production or Inspection of Documents and Chattels. — 1. It the morphine he had there. It was not compelling him to testify or to be a
follows that the production of documents or chattels by a person (whether witness or to furnish, much less make, prepare, or create through a
ordinary witness or party-witness) in response to a subpoena, or to a motion testimonial act, evidence for his own condemnation.
to order production, or to other form of process treating him as a witness (
i.e. as a person appearing before a tribunal to furnish testimony on his moral Wherefore, we find the present action well taken, and it is ordered that the
responsibility for truthtelling), may be refused under the protection of the respondents and those under their orders desist and abstain absolutely and
privilege; and this is universally conceded. (And he cites the case of People forever from compelling the petitioner to take down dictation in his
vs. Gardner, 144 N. Y., 119; 38 N.E., 1003) handwriting for the purpose of submitting the latter for comparison.

We say that, for the purposes of the constitutional privilege, there is a Without express pronouncement as to costs. So ordered.
similarity between one who is compelled to produce a document, and one
who is compelled to furnish a specimen of his handwriting, for in both cases,
the witness is required to furnish evidence against himself.

And we say that the present case is more serious than that of compelling the
production of documents or chattels, because here the witness is compelled
to write and create, by means of the act of writing, evidence which does not
exist, and which may identify him as the falsifier. And for this reason the
same eminent author, Professor Wigmore, explaining the matter of the
production of documents and chattels, in the passage cited, adds:

For though the disclosure thus sought be not oral in form, and though the
documents or chattels be already in existence and not desired to be first
written and created by testimonial act or utterance of the person in response
to the process, still no line can be drawn short of any process which treats
him as a witness; because in virtue it would be at any time liable to make
oath to the identity or authenticity or origin of the articles produced. (Ibid.,
pp. 864-865.) (Emphasis ours.)

It cannot be contended in the present case that if permission to obtain a


specimen of the petitioner's handwriting is not granted, the crime would go
unpunished. Considering the circumstance that the petitioner is a municipal
treasurer, according to Exhibit A, it should not be a difficult matter for the
fiscal to obtained genuine specimens of his handwriting. But even supposing
it is impossible to obtain specimen or specimens without resorting to the
means complained herein, that is no reason for trampling upon a personal
right guaranteed by the constitution. It might be true that in some cases
criminals may succeed in evading the hand of justice, but such cases are
accidental and do not constitute the raison d' etre of the privilege. This
constitutional privilege exists for the protection of innocent persons.

With respect to the judgments rendered by this court and cited on behalf of
the respondents, it should be remembered that in the case of People vs.
Badilla (48 Phil., 718), it does not appear that the defendants and other
witnesses were questioned by the fiscal against their will, and if they did not
refuse to answer, they must be understood to have waived their constitutional
privilege, as they could certainly do.

The privilege not to give self-incriminating evidence, while absolute when


claimed, maybe waived by any one entitled to invoke it. (28 R. C. L.,
paragraph 29, page 442, and cases noted.)

The same holds good in the case of United States vs. Tan Teng (23 Phil.,
145), were the defendant did not oppose the extraction from his body of the
substance later used as evidence against him.

In the case of Villaflor vs. Summers (41 Phil., 62), it was plainly stated that
the court preferred to rest its decision on the reason of the case rather than on
blind adherence to tradition. The said reason of the case there consisted in
that it was the case of the examination of the body by physicians, which could
be and doubtless was interpreted by this court, as being no compulsion of the
petitioner therein to furnish evidence by means of testimonial act. In reality
she was not compelled to execute any positive act, much less a testimonial
act; she was only enjoined from something preventing the examination; all
of which is very different from what is required of the petitioner of the
present case, where it is sought to compel him to perform a positive,
PEOPLE OF THE PHILIPPINES vs. AUSTRIA
G.R. No. 55109 | 1991-04-08 The prosecution then presented the testimonies of the following witnesses:
BIDIN, J.:
Rodolfo Labajas, cashier of the RLC Management Corp., testified that in the
This is an appeal from the decision * dated March 18, 1980 of the then Circuit morning of August 9, 1975, Tomas Azuelo, as overseer of Hda. Austria,
Criminal Court, 12th Judicial District, Bacolod City, finding accused- received from him the sum of P771.40 representing the wages of laborers of
appellants Eduardo Austria, Pablo Austria and Jaime dela Torre guilty of the said hacienda. Tomas Azuelo left his office at 12:00 o'clock noon and that
crime of robbery with homicide and sentencing them to suffer the penalty of was the last time he saw Tomas Azuelo alive (TSN, January 27, 1978, pp.
death, to jointly and severally indemnify the heirs of deceased Tomas Azuelo 315-333).
in the amount of P12,000.00 and to pay the costs Rollo, pp. 12-13)
Nestor Colegado, tricycle driver, testified that at about 3:30 in the afternoon
It appears from the records that in the morning of August 10, 1975, the police of August 9, 1975, the deceased and accused Pablo Austria boarded his
dug out of the ground in a sugarcane field in sitio Palanas, Sagay, Negros tricycle together with three women passengers at the poblacion of Sagay.
Occidental, the lifeless body of Tomas Azuelo. Found near his grave were Both Pablo Austria and Tomas Azuelo, who were neighbors, alighted at
the traces of blood and a bloodstained piece of wood. Post mortem crossing Tupas. (TSN, January 30, 1978, pp. 334-352).
examination showed that Tomas Azuelo's skull was fractured and his body
sustained eighteen (18) stab wounds, fifteen (15) of which were fatal. The Andres Caro, a farmer/resident of Hda. Palanas, Sagay, Negros Occidental,
payroll, together with the sum of P771.40 intended for the wages of laborers testified that at about 1:00 o'clock in the afternoon of August 9, 1975, he saw
of Hacienda Austria, of which Azuelo was the overseer, was missing. accused Eduardo Austria standing at the side of the latter's farm along the
road going to Hda. Austria. (TSN, February 23, 1978, pp. 368-372).
Four suspects for the death of Tomas Azuelo were picked up by the police
working together with the Philippine Constabulary. One of the suspects, Diego de Ocampo, Rural Health Physician of Sagay, Negros Occidental,
Pablo Austria, was the last person who was seen with Tomas Azuelo. They declared that in the morning of August 10, 1975, he, together with policeman
boarded a tricycle together, on August 9, 1975 at about 3:30 p.m., from the Voltaire Yee, recovered the body of Tomas Azuelo which was buried in the
poblacion of Sagay and alighted at about 4:00 p.m. at crossing Tupas. sugarcane field. His post mortem examination showed that Tomas Azuelo
had a fractured skull and his body sustained several stab wounds, fifteen (15)
The other suspects were implicated based on the sworn statement dated of which were fatal. Shock secondary to severe hemmorhage due to multiple
September 17, 1975 of Pablo Austria, who imputed to his son Eduardo, his stab wound was the cause of Tomas Azuelo's death (TSN, February 23, 1978,
brother-in-law Jaime de la Torre and Leopoldo Abanilla, the commission of pp. 356-368).
the crime. In his sworn statement, Pablo Austria stated that while he and
Tomas Azuelo were on their way home on August 9, 1975, Eduardo Austria, Voltaire Yee, a police investigator, testified that in the morning of August
Jaime de la Torre and Leopoldo Abanilla suddenly appeared and pulled 10, 1975, Eugenio Azuelo reported to the desk officer the disappearance of
Tomas Azuelo toward the sugarcane field. Thereafter, Jaime de la Torre his father Tomas Azuelo. After receiving the report, they proceeded to the
struck the victims on the head with a piece of wood while Leopoldo Abanilla scene of the crime together with other policemen. There, they saw the payroll
stabbed Tomas Azuelo in different parts of his body. Jaime de la Torre got and the protruding toe of a human being buried in the sugarcane field which
the money inside Tomas Azuelo's pocket and together, they buried the victim the people in the area discovered. A wooden bat with bloodstains was found
(Original Records, pp. 7-9). six (6) meters from the protruding toe. Coins, slippers, fish, and bloodstains
were also discovered in the area. The lifeless body of Tomas Azuelo,
On September 17, 1975, Gregorio Eras, Deputy Chief of Police, Sagay, identified by his daughter Nenita, was exhumed in the presence of Dr.
Negros Occidental, filed a complaint for robbery with homicide against Ocampo. Joint investigation by the police and the Philippine Constabulary
Eduardo Austria, Pablo Austria, Jaime de la Torre and Leopoldo Abanilla. ensued, after which Pablo Austria and his son Eduardo were picked up by the
police for investigation. Jaime de la Torre was also picked up and
After preliminary examination, Judge Emilio Ignalaga, Acting Municipal investigated on account of a bloodstained hoe found in his home. On
Judge, Sagay, Negros Occidental, issued a warrant of arrest against herein September 17, 1975, Pablo Austria executed a sworn statement implicating
appellants. No bail was recommended (Original Records, p. 9). his son Eduardo in the commission of the offense (TSN, August 17, 1978 to
September 20, 1978, pp. 373-391; 1-47).
On December 11, 1975, an information was filed against accused/appellants,
which reads: Vicente Aquino a PC soldier, testified that on September 16, 1975, he
investigated Jaime dela Torre as one of the suspects in the killing of Tomas
"The undersigned provincial Fiscal accuses Eduardo Austria y Montano, Azuelo. He declared that de la Torre was informed of his constitutional rights
Jaime de la Torre y Pahiluna, Pablo Austria y Camelloso, and Leopoldo to remain silent and to counsel but he voluntarily waived said rights (TSN,
Abanilla y Bantalagon of the crime of Robbery with Homicide, committed January 13, 1976, pp. 253-283).
as follows:
Myrna Areola, a forensic chemist of the Third Regional PC Crime
"That on or about the 9th day of August 1975, in the municipality of Sagay, Laboratory testified that upon the request of Capt. Natino she conducted a
province of Negros Occidental, Philippines, and within the jurisdiction of laboratory examination of the short pants of Jaime de la Torre, a hoe, a piece
this Honorable Court, the abovenamed accused, armed with a piece of wood of wood, soil scraped from the surface of the hoe and sample of the soil where
and sharp instrument, by means of force, violence and intimidation of person the victim was buried. Her findings showed that the blood stain on the handle
and with intent of gain, conspiring, confederating and helping one another, of the hoe and that the piece of wood submitted for examination were positive
did then and there, wilfully, unlawfully and feloniously take, steal and carry for human blood belonging to the same blood group. The pants, however,
away the money in the total sum of Seven Hundred Seventy-one Pesos and were negative for human blood. Analysis of the soil also revealed that the
40/100 (P771.40), Philippine Currency, belonging to Tomas Azuelo y soil scraped from the surface of the hoe and the soil from the place where the
Binigay, without the consent of the latter, to the damage and prejudice of the victim was buried were of the same origin (TSN, November 17, 1977, pp.
said Tomas Azuelo y Binigay the aforestated amount. 283-295).

"That on the occasion of the said robbery and for the purpose of enabling Emilio Ignalaga, Municipal Judge of Escalante, Toboso and Sagay, Negros
them to take, steal and carry away the money to be robbed, the said accused, Occidental, testified that on September 17, 1975 the accused Jaime de la
in furtherance of their conspiracy, with intent to kill, did then and there, Torre and Pablo Austria subscribed before him their written extra-judicial
wilfully, unlawfully and feloniously attack, assault strike and stab the said statements. Before they affixed their thumbmarks, he read and explained to
Tomas Azuelo y Binigay, thereby inflicting multiple injuries on the different them the contents of their extrajudicial statements and warned them that the
part of the body of the latter which caused the death of the said victim. same can be used against them. They affixed their thumbmarks on said
document voluntarily (TSN, November 17, 1977, pp. 296-307).
"Contrary to law."
Accused/appellants testified as follows:
(Original Records, pp. 51-52).
Pablo Austria testified that Tomas Azuelo was his second degree cousin. On
All the accused pleaded not guilty to the charge. August 9, 1975, he was summoned by the wife of Tomas Azuelo to fetch
water. He was in Azuelo's house from 11:00 o'clock in the morning, took his In its sur-rebuttal, the defense presented Paquito Tolentin. Tolentin testified
lunch there at upon invitation of Azuelo's wife and waited for Tomas Azuelo that from 2:00 o'clock in the afternoon up to 5:45 or 6:00 o'clock in the
up to 7:00 o'clock in the evening for his compensation in plowing their farm. evening of August 9, 1975, he, together with Pablo Austria, Diosdado Alova,
On August 14, 1975, he was picked up without a warrant of arrest by a Ricardo Alova and Eugenio Azuelo were playing a card game known in the
policeman and a PC soldier while working on the concrete cover of the locality as "41" in the house of Tomas Azuelo. During the time they were
deceased's tomb. playing, Pablo Austria never left the place and was in fact there when he left
the house of Tomas Azuelo (November 13, 1979, TSN, pp. 222-232).
He was investigated without informing him of his right to remain silent and
to counsel. After six (6) days in detention he was released upon the On March 18, 1980, the trial court rendered its decision convicting appellants
intercession of Iluminada, wife of the deceased. On September 17, 1975 he of the imputed crime, the dispositive portion of which reads as follows:
was again picked up by Voltaire Yee at about 7:00 o'clock in the evening.
Jaime de la Torre, who was with him in the PC stockade, struck him in the "WHEREFORE, in view of all the foregoing, the Court finds all accused
head with a revolver on orders of a certain Torenas, a PC soldier. Later, he Eduardo Austria y Montaño, Jaime de la Torre y Pahiluna and Pablo Austria
was brought out of the PC stockade and Torenas kicked and boxed him. The y Camelloso, GUILTY beyond a reasonable doubt of the offense of robbery
following day, Alberto Olario, the commanding officer, again maltreated him with homicide, as charged, . . . and pursuant to paragraph 1, Articles 294, in
as he refused to admit participation in the killing of Tomas Azuelo. On orders relation to paragraphs 13 and 15, Article 14, of the Revised Penal Code, the
of the commanding officer, Voltaire Yee prepared an affidavit. He did not Court sentences all accused. Eduardo Austria y Montaño, Jaime de la Torre
read the affidavit, as he does not know how to read, nor was it read to him. y Pahiluna and Pablo Austria y Camelloso, each to suffer the maximum
Voltaire Yee forced him to affix his thumbmark in the affidavit inside the penalty provided by law which is DEATH, to jointly and severally indemnify
office of Judge Ignalaga. (TSN, December 4, 1978, pp. 171-200). the heirs of the late Tomas Azuelo the amount of P12,000.00. as indemnity
for his death, and to pay the costs."
In his testimony, Jaime de la Torre disowned the statements attributed to him
during the investigation conducted by Sgt. Vicente Aquino and instead During the pendency of the appeal, Pablo Austria and Jaime de la Torre died
declared that it was not Eduardo Austria but Carlos Capitle, Jr. who borrowed of undetermined cause and hypertension, respectively, at the New Bilibid
his hoe in the afternoon of August 9, 1975. He also contradicted the Prisons Hospital. Hence, the resolution of the instant appeal should be based
statements contained in the affidavit that he saw Pablo Austria, Eduardo on the evidence against the surviving appellant, Eduardo Austria.
Austria and another person standing near the body of the deceased. Instead,
he testified that in the sugarcane field that day (August 14, 1975), he saw After a review of the records, the Court finds that the evidence presented by
Carlos Capitle, Jr. and Celestino Capitle with another person looking at the the prosecution failed to establish the guilt of appellant Eduardo Austria
dead body of Azuelo. He helped cover the dead with grass on orders of Carlos beyond reasonable doubt.
Capitle with warning not to tell his family or anybody, otherwise his family
will be killed. He admitted ownership of the hoe, but denied any participation In the first place, as stated in the appealed decision, the evidence of the
in the killing. He also claimed that he was arrested without warrant and prosecution against appellant Eduardo Austria is merely circumstantial.
detained for more than a month in the PC headquarters, at Sagay, Negros Aside from the extra-judicial confessions of the deceased appellants, there is
Occidental. neither direct evidence nor actual witness to the commission of the crime.
This is not to say, however, that conviction cannot be had simply because the
During said detention he was investigated and allegedly maltreated by evidence is circumstantial. To sustain a conviction based on circumstantial
Captain Olario (TSN, January 22, 1980, pp. 149-170, 210-221, 393-404). evidence under Sec. 5, Rule 133, there must be (a) more than one
circumstance, (b) the facts from which the inferences are derived are proven
On rebuttal, the prosecution presented four (4) witnesses, as follows: and (c) the combination of all the circumstances is such as to produce a
conviction beyond reasonable doubt.
Capt. Olario denied that he manhandled Pablo Austria and Jaime de la Torre
and claimed then he merely interviewed the suspects; it was Sgt. Aquino who The series of circumstances proved must be consistent with each other and
conducted the formal investigation after Jaime de la Torre narrated the that each and every circumstance must be consistent with the guilt of the
commission of the crime and implicated Pablo Austria and Eduardo Austria. accused and inconsistent with his innocence. To warrant conviction in
Both Jaime de la Torre and Pablo Austria admitted to him the commission of criminal cases based upon circumstantial evidence, it must constitute an
the crime. unbroken chain of events so as to lead to a conviction that the accused is
guilty beyond reasonable doubt. In the case at bar, the circumstantial
On cross-examination, however, Capt. Olario admitted that the suspects were evidence do not prove an unbroken link of events that could give rise to a
not apprised of their constitutional rights to remain silent and to counsel as reasonable and fair conclusion that appellant committed the imputed offense.
he was merely conducting an informal interview (TSN, August 23, 1979, pp.
86-148). As regards appellant Eduardo Austria, the only evidence against him is that
he was seen at about 1:00 o'clock in the afternoon of August 9, 1975 along
Celedonio Capitle, a watchman denied the imputation of accused Jaime de la the road going to Hda. Austria. This evidence even if tied up with the
Torre that he participated in the killing of Tomas Azuelo. He was not even testimony of Iluminada Azuelo that Austria harbored ill-feelings against the
investigated by the police for the death of the latter and learned of his deceased because he was dismissed from the hacienda by the deceased does
implication only through Roger Azuelo, son of the deceased (TSN, August not establish or support an inference, much less a conclusion, that he
24, 1979, pp. 426-446). participated in the commission of the offense charged. The conviction of
appellant Eduardo Austria on an inference based on another inference cannot
Carlos Capitle, Jr., also denied that he borrowed a hoe from Jaime de la Torre be maintained.
on August 19, 1975; he, his brother, Celedonio, and another person were not
standing and looking at the dead body of Tomas Azuelo on the day he was It is axiomatic that conviction should be made on the basis of a strong, clear
killed; nor did they warn and threaten de la Torre not to tell anybody about and compelling evidence (People v. Tulagan, 143 SCRA 107 [1986]. Thus,
the incident; he has no participation in the killing of Tomas Azuelo (TSN, "if the inculpatory facts and circumstances are capable of two or more
October 15, 1979, pp. 405-425). explanations, one of which is consistent with the innocence of the accused
and the other consistent with his guilt, then the evidence does not fulfill the
Iluminada Azuelo, wife of the deceased, declared that: before the death of tests of moral certainty and is not sufficient to support a conviction" (People
Tomas, only Pablo Austria was working in the hacienda as her late husband v. Ale, 145 SCRA 64 [1986]; People v. Modesto, 25 SCRA 36 [1968].
dismissed Jaime de la Torre and Eduardo Austria; Pablo Austria worked in
her house in the morning but did not return in the afternoon of August 9, To overcome the presumption of innocence, proof beyond reasonable doubt
1975; she interceded for the release of Pablo Austria upon the request of the is needed. Thus, in People v. Dramayo, 42 SCRA 60 [1971], this Court held:
latter's wife, Leonora, since there was no strong evidence against him; the
second time Pablo Austria was picked up, she turned down the request of "Accusation is not, according to the fundamental law, synonymous with
Leonora for her to intercede in behalf of Leonora's husband (October 15, guilt; the prosecution must overthrow the presumption of innocence with
1975, TSN, pp. 405-417). proof of guilt beyond reasonable doubt. To meet this standard, there is need
far the most careful scrutiny of the testimony of the state, both oral and
documentary, independently of whatever defense is offered by the accused.
Only if the judge below and the appellate tribunal could arrive at a conclusion
that the crime had been committed precisely by the person on trial under such
an exacting test should the sentence be one of conviction. It is thus required
that every circumstance favoring his innocence be duly taken into account.
The proof against him must survive the test of reason; the strongest suspicion
must not be permitted to sway judgment."

WHEREFORE, the appealed judgment is hereby REVERSED and the


accused/appellant Eduardo Austria is ACQUITTED on the ground of
reasonable doubt.

SO ORDERED.
DE LA CAMARA vs. ENAGE fact that this case is moot and academic should not preclude thisTribunal
G.R. Nos. L-32951-2 September 17, 1971 from setting forth in language clear and unmistakable, the obligationof
FERNANDO, J.: fidelity on the part of lower court judges to the unequivocal command of
theConstitution that excessive bail shall not be required.
An order of respondent Judge Manuel Lopez Enage, fixing the bail of
petitioner, Ricardo de la Camara, in the sum of P1,195,200.00 is assailed in 1. Before conviction, every person is bailable except if charged with capital
this petition for certiorari as repugnant to the constitutional mandate offenses when the evidence of guilt is strong.5 Such a right flows from the
prohibiting excessive bail.1 The merit of the petition on its face is thus presumption of innocence in favor of every accused who should not be
apparent. Nonetheless, relief sought setting aside the above order by reducing subjected to the loss of freedom as thereafter he would be entitled to acquittal,
the amount of bail to P40,000.00 cannot be granted, as in the meanwhile, unless his guilt be proved beyond reasonable doubt. Thereby a regimeof
petitioner had escaped from the provincial jail, thus rendering this case moot liberty is honored in the observance and not in the breach. It is not beyondthe
and academic. It is deemed advisable, however, for the guidance of lower realm of probability, however, that a person charged with a crime, especially
court judges, to set forth anew the controlling and authoritative doctrines that so where his defense is weak, would just simply make himself scarceand thus
should be observed in fixing the amount of the bail sought in order that full frustrate the hearing of his case. A bail is intended as a guarantee that such
respect be accorded to such a constitutional right. an intent would be thwarted. It is, in the language of Cooley, a "mode short
of confinement which would, with reasonable certainty, insure the attendance
The facts are not in dispute. Petitioner, Ricardo, de la Camara, Municipal of the accused" for the subsequent trial.6 Nor is there, anything unreasonable
Mayor of Magsaysay, Misamis Oriental, was arrested on November 7, 1968 in denying this right to one charged with a capital offense when evidence of
and detained at the Provincial Jail of Agusan, for his alleged participation in guilt is strong, as the likelihood is, rather than await the outcome of the
the killing of fourteen and the wounding of twelve other laborers of the proceeding against him with a death sentence, an ever-present threat,
Tirador Logging Co., at Nato, Esperanza, Agusan del Sur, on August 21, temptation to flee the jurisdiction would be too great to be resisted.
1968. Thereafter, on November 25, 1968, the Provincial Fiscal of Agusan
filed with the Court of First Instance a case for multiple frustrated murder2 2. Where, however, the right to bail exists, it should not be rendered nugatory
and another for multiple murder3 against petitioner, his co-accused by requiring a sum that is excessive. So the Constitution commands. It is
Nambinalot Tagunan and Fortunato Galgo, resulting from the aforesaid understandable why. If there were no such prohibition, the right to bail
occurrence. Then on January 14, 1969, came an application for bail filed by becomes meaningless. It would have been more forthright if no mention of
petitioner with the lower court, premised on the assertion that there was no such a guarantee were found in the fundamental law. It is not to be lost sight
evidence to link him with such fatal incident of August 21, 1968. He likewise of that the United States Constitution limits itself to a prohibition against
mantained his innocence. Respondent Judge started the trial of petitioner on excessive bail.7 As construed in the latest American decision, "the sole
February 24, 1969, the prosecution resting its case on July 10, 1969. As of permissible function of money bail is to assure the accused's presence at trial,
the time of the filing ofthe petition, the defense had not presented its and declared that "bail set at a higher figure than an amount
evidence. reasonablycalculated to fulfill thus purpose is "excessive" under the Eighth
Amendment."8
Respondent Judge, on August 10, 1970, issued an order granting petitioner's
application for bail, admitting that there was a failure on the part of the Nothing can be clearer, therefore, than that the challenged order of August
prosecution to prove that petitioner would flee even if he had the 10, 1970 fixing the amount of P1,195,200.00 as the bail that should be posted
opportunity,but fixed the amount of the bail bond at the excessive amount of by petitioner, the sum of P840,000.00 for the information charging multiple
P1,195,200.00,the sum of P840,000.00 for the information charging multiple murder, there being fourteen victim, and the sum of P355,200 for the other
murder and P355,200.00 for the offense of multiple frustrated murder. Then offense of multiple frustrated murder, there being twelve victims, is clearly
came the allegation that on August 12, 1970, the Secretary of Justice, Vicente violative of constitutional provision. Under the circumstances, there being
Abad Santos, upon being informed of such order, sent a telegram to only two offenses charged, the amount required as bail could not possibly
respondent Judgestating that the bond required "is excessive" and suggesting exceed P50,000.00 for the information for murder and P25,000.00 for the
that a P40,000.00bond, either in cash or property, would be reasonable. There other information for frustrated murder. Nor should it be ignored in this case
was likewise a motion for reconsideration to reduce the amount. Respondent that the Department of Justice did recomend the total sum of P40,000.00 for
Judge however remained adamant. Hence this petition. the twooffenses.

The answer filed by respondent Judge on March 5, 1971 set forth the 3. There is an attempt on the part of respondent Judge to justify what, on its
circumstances concerning the issuance of the above order and the other face, appears to be indefensible by the alleged reliance on Villaseñor v.
incidents of the case, which, to his mind would disprove any charge that he Abano.9 The guidelines in the fixing of bail was there summarized, in the
wasguilty of grave abuse of discretion. It stressed, moreover, that the opinion of Justice Sanchez, as follows: "(1) ability of the accused to give bail;
challengedorder would find support in circulars of the Department of Justice (2) nature of the offense; (3) penalty for the offense charged; (4) character
given sanction by this Court. He sought the dismissal of the petition for lack and reputation of the accused; (5) health of the accused; (6) character and
of merit. strength of the evidence; (7) probability of the accused appearing in trial; (8)
forfeiture of other bonds; (9) whether the accused wasa fugitive from justice
In the hearing of the case set for March 31, 1971, there was no appearance when arrested; and (10) if the accused is under bond for appearance at trial
for both the petitioner and respondents with the former, upon written motion, in other cases." 10 Respondent Judge, however, did ignore this decisive
being given thirty days within which to submit a memorandum in lieu of oral consideration appearing at the end of the above opinion: "Discretion, indeed,
argument, respondent Judge in turn having the same period from receipt is with the court called upon to rule on the question of bail. We must stress,
thereofto file his reply. Such a memorandum as duly submitted by petitioner however, that where conditions imposed upon a defendant seeking bail
on April 6, 1971. would amount to a refusal thereof and render nugatory the constitutional right
to bail, we will not hesitate to exercise our supervisorypowers to provide the
Instead of a reply, respondent Judge submitted, on May 26, 1971, a required remedy." 11
supplemental answer wherein he alleged that petitioner escaped from the
provincial jail on April 28, 1971 and had since then remained at large. There No attempt at rationalization can therefore give a color of validity to the
was a reiteration then of the dismissal of this petition for lack of merit, challenged order. There is grim irony in an accused being told that he has a
towhich petitioner countered in a pleading dated June 7, 1971, and filed with right to bail but at the same time being required to post such an exorbitant
this Court the next day with this plea: "The undersigned counsel, therefore, sum. What aggravates the situation is that the lower court judge would
vehemently interpose opposition, on behalf of petitioner, to respondent's apparently yield to the command of the fundamental law. In reality, such a
prayer for dismissal of the present petition for lack of merit. For, the issue in sanctimonious avowal of respect for a mandate of the Constitution was on a
this case is not alone the fate of petitioner Ricardo de la Camara. The issue purely verbal level. There is reason to believe that any person in the position
in the present petition that calls for the resolution of this Honorable Tribunal of petitioner would under the circumstances be unable to resists thoughts of
is the fate of countless other Ricardo de la Camaras who maybe awaiting the escaping from confinement, reduced as he must have been to a stateof
clear-cut definition and declaration of the power of trial courts in regard to desperation. In the same breath that he was told he could be bailed out, the
the fixing of bail."4 excessive amount required could only mean that provisional liberty would
bebeyond his reach. It would have been more forthright if he were informed
While under the circumstances a ruling on the merits of the petition for categorically that such a right could not be availed of. There would have
certiorari is not warranted, still, as set forth at the opening of this opinion, the beenno disappointment of expectations then. It does call to mind these words
of Justice Jackson, "a promise to the ear to be broken to the hope, a teasing
illusion like a munificent bequest in a pauper's will."12 It is no wonder that
the resulting frustration left resentment and bitterness in its wake.Petitioner's
subsequent escape cannot be condoned. That is why he is not entitled to the
relief prayed for. What respondent Judge did, however, does call for
repudiation from this Court.

Nor is there any justification then for imputing his inability to fix a lesser
amount by virtue of an alleged reliance on a decision of this Tribunal. Even
if one were charitably inclined, the mildest characterization of such a result
is that there was a clear reading of the Abano opinion when such a meaning
was ascribed to it. No doctrine refinement may elicit approval if to doso
would be to reduce the right to bail to a barren form of words. Not only isthe
order complained of absolutely bereft of support in law, but it flies in the face
of common sense. It is not too much to say that it is at war with thecommand
of reason.

With petitioner, however, having escaped from the provincial jail, no ruling
can be had on his plea to nullify the above order.

WHEREFORE, this case is dismissed for being moot and academic. Without
pronouncement as to costs.
OPLE vs. TORRES SEC. 7. Submission of Regular Reports. The NSO, GSIS and SSS shall
G.R. No. 127685 | 1998-07-23 submit regular reports to the Office of the President, through the IACC, on
PUNO, J: the status of implementation of this undertaking.

The petition at bar is a commendable effort on the part of Senator Blas F. SEC. 8. Effectivity. This Administrative Order shall take effect immediately.
Ople to prevent the shrinking of the right to privacy, which the revered Mr.
Justice Brandeis considered as "the most comprehensive of rights and the DONE in the City of Manila, this 12th day of December in the year of Our
right most valued by civilized men.'' 1 Petitioner Ople prays that we Lord, Nineteen Hundred and Ninety-Six.
invalidate Administrative Order No. 308 entitled "Adoption of a National
Computerized Identification Reference System" on two important (SGD.) FIDEL V. RAMOS"
constitutional grounds, viz: one, it is a usurpation of the power of Congress
to legislate, and two, it impermissibly intrudes on our citizenry's protected A.O. No. 308 was published in four newspapers of general circulation on
zone of privacy. We grant the petition for the rights sought to be vindicated January 22, 1997 and January 23, 1997. On January 24, 1997, petitioner filed
by the petitioner need stronger barriers against further erosion. the instant petition against respondents, then Executive Secretary Ruben
Torres and the heads of the government agencies, who as members of the
A.O. No. 308 was issued by President Fidel V. Ramos on December 12, 1996 Inter-Agency Coordinating Committee, are charged with the implementation
and reads as follows: of A.O. No. 308. On April 8, 1997, we issued a temporary restraining order
enjoining its implementation.
"ADOPTION OF A NATIONAL COMPUTERIZED IDENTIFICATION
REFERENCE SYSTEM Petitioner contends:

WHEREAS, there is a need to provide Filipino citizens and foreign residents "A. THE ESTABLISHMENT OF A NATIONAL COMPUTERIZED
with the facility to conveniently transact business with basic service and IDENTIFICATION REFERENCE SYSTEM REQUIRES A
social security providers and other government instrumentalities; LEGISLATIVE ACT. THE ISSUANCE OF A.O. NO. 308 BY THE
PRESIDENT OF THE REPUBLIC OF THE PHILIPPINES IS,
WHEREAS, this will require a computerized system to properly and THEREFORE, AN UNCONSTITUTIONAL USURPATION OF THE
efficiently identify persons seeking basic services on social security and LEGISLATIVE POWERS OF THE CONGRESS OF THE REPUBLIC OF
reduce, if not totally eradicate, fraudulent transactions and THE PHILIPPINES.
misrepresentations;
B. THE APPROPRIATION OF PUBLIC FUNDS BY THE PRESIDENT
WHEREAS, a concerted and collaborative effort among the various basic FOR THE IMPLEMENTATION OF A.O. NO. 308 IS AN
services and social security providing agencies and other government UNCONSTITUTIONAL USURPATION OF THE EXCLUSIVE RIGHT
instrumentalities is required to achieve such a system; OF CONGRESS TO APPROPRIATE PUBLIC FUNDS FOR
EXPENDITURE.
NOW, THEREFORE, I, FIDEL V. RAMOS, President of the Republic of
the Philippines, by virtue of the powers vested in me by law, do hereby direct C. THE IMPLEMENTATION OF A.O. NO. 308 INSIDIOUSLY LAYS
the following: THE GROUNDWORK FOR A SYSTEM WHICH WILL VIOLATE THE
BILL OF RIGHTS ENSHRINED IN THE CONSTITUTION." 2
SEC. 1. Establishment of a National Computerized Identification Reference
System. A decentralized Identification Reference System among the key Respondents counter-argue:
basic services and social security providers is hereby established.
A. THE INSTANT PETITION IS NOT A JUSTICIABLE CASE AS
SEC. 2. Inter-Agency Coordinating Committee. An Inter-Agency WOULD WARRANT A JUDICIAL REVIEW;
Coordinating Committee (IACC) to draw-up the implementing guidelines
and oversee the implementation of the System is hereby created, chaired by B. A.O. NO. 308 [1996] WAS ISSUED WITHIN THE EXECUTIVE AND
the Executive Secretary, with the following as members: ADMINISTRATIVE POWERS OF THE PRESIDENT WITHOUT
ENCROACHING ON THE LEGISLATIVE POWERS OF CONGRESS;
Head, Presidential Management Staff
Secretary, National Economic Development Authority C. THE FUNDS NECESSARY FOR THE IMPLEMENTATION OF THE
Secretary, Department of the Interior and Local Government IDENTIFICATION REFERENCE SYSTEM MAY BE SOURCED FROM
Secretary, Department of Health THE BUDGETS OF THE CONCERNED AGENCIES;
Administrator, Government Service Insurance System,
Administrator, Social Security System, D. A.O. NO. 308 [1996] PROTECTS AN INDIVIDUAL'S INTEREST IN
Administrator, National Statistics Office PRIVACY. 3
Managing Director, National Computer Center.
We now resolve.
SEC. 3. Secretariat. The National Computer Center (NCC) is hereby
designated as secretariat to the IACC and as such shall provide administrative I
and technical support to the IACC.
As is usual in constitutional litigation, respondents raise the threshold issues
SEC. 4. Linkage Among Agencies. The Population Reference Number relating to the standing to sue of the petitioner and the justiciability of the
(PRN) generated by the NSO shall serve as the common reference number to case at bar. More specifically, respondents aver that petitioner has no legal
establish a linkage among concerned agencies. The IACC Secretariat shall interest to uphold and that the implementing rules of A.O. No. 308 have yet
coordinate with the different Social Security and Services Agencies to to be promulgated.
establish the standards in the use of Biometrics Technology and in computer
application designs of their respective systems. These submissions do not deserve our sympathetic ear. Petitioner Ople is a
distinguished member of our Senate. As a Senator, petitioner is possessed of
SEC. 5. Conduct of Information Dissemination Campaign. The Office of the the requisite standing to bring suit raising the issue that the issuance of A.O.
Press Secretary, in coordination with the National Statistics Office, the GSIS No. 308 is a usurpation of legislative power. 4 As taxpayer and member of
and SSS as lead agencies and other concerned agencies shall undertake a the Government Service Insurance System (GSIS), petitioner can also
massive tri-media information dissemination campaign to educate and raise impugn the legality of the misalignment of public funds and the misuse of
public awareness on the importance and use of the PRN and the Social GSIS funds to implement A.O. No. 308. 5
Security Identification Reference.
The ripeness for adjudication of the petition at bar is not affected by the fact
SEC. 6. Funding. The funds necessary for the implementation of the system that the implementing rules of A.O. No. 308 have yet to be promulgated.
shall be sourced from the respective budgets of the concerned agencies. Petitioner Ople assails A.O. No. 308 as invalid per se and as infirmed on its
face. His action is not premature for the rules yet to be promulgated cannot
cure its fatal defects. Moreover, the respondents themselves have started the that A.O. No. 308 implements the legislative policy of the Administrative
implementation of A.O. No. 308 without waiting for the rules. As early as Code of 1987. The Code is a general law and "incorporates in a unified
January 19, 1997, respondent Social Security System (SSS) caused the document the major structural, functional and procedural principles of
publication of a notice to bid for the manufacture of the National governance" 25 and "embodies changes in administrative structures and
Identification (ID) card. 6 Respondent Executive Secretary Torres has procedures designed to serve the people." 26 The Code is divided into seven
publicly announced that representatives from the GSIS and the SSS have (7) Books: Book I deals with Sovereignty and General Administration, Book
completed the guidelines for the national identification system. 7 All signals II with the Distribution of Powers of the three branches of Government, Book
from the respondents show their unswerving will to implement A.O. No. 308 III on the Office of the President, Book IV on the Executive Branch, Book V
and we need not wait for the formality of the rules to pass judgment on its on the Constitutional Commissions, Book VI on National Government
constitutionality. In this light, the dissenters insistence that we tighten the Budgeting, and Book VII on Administrative Procedure. These Books contain
rule on standing is not a commendable stance as its result would be to throttle provisions on the organization, powers and general administration of the
an important constitutional principle and a fundamental right. executive, legislative and judicial branches of government, the organization
and administration of departments, bureaus and offices under the executive
II branch, the organization and functions of the Constitutional Commissions
and other constitutional bodies, the rules on the national government budget,
We now come to the core issues. Petitioner claims that A.O. No. 308 is not a as well as guidelines for the exercise by administrative agencies of quasi-
mere administrative order but a law and hence, beyond the power of the legislative and quasi-judicial powers. The Code covers both the internal
President to issue. He alleges that A.O. No. 308 establishes a system of administration of government, i.e, internal organization, personnel and
identification that is all-encompassing in scope, affects the life and liberty of recruitment, supervision and discipline, and the effects of the functions
every Filipino citizen and foreign resident, and more particularly, violates performed by administrative officials on private individuals or parties outside
their right to privacy. government. 27

Petitioner's sedulous concern for the Executive not to trespass on the It cannot be simplistically argued that A.O. No. 308 merely implements the
lawmaking domain of Congress is understandable. The blurring of the Administrative Code of 1987. It establishes for the first time a National
demarcation line between the power of the Legislature to make laws and the Computerized Identification Reference System. Such a System requires a
power of the Executive to execute laws will disturb their delicate balance of delicate adjustment of various contending state policies - the primacy of
power and cannot be allowed. Hence, the exercise by one branch of national security, the extent of privacy interest against dossier-gathering by
government of power belonging to another will be given a stricter scrutiny government, the choice of policies, etc. Indeed, the dissent of Mr. Justice
by this Court. Mendoza states that the A.O. No. 308 involves the all-important freedom of
thought. As said administrative order redefines the parameters of some basic
The line that delineates Legislative and Executive power is not indistinct. rights of our citizenry vis-a-vis the State as well as the line that separates the
Legislative power is "the authority, under the Constitution, to make laws, and administrative power of the President to make rules and the legislative power
to alter and repeal them." 8 The Constitution, as the will of the people in their of Congress, it ought to be evident that it deals with a subject that should be
original, sovereign and unlimited capacity, has vested this power in the covered by law.
Congress of the Philippines. 9 The grant of legislative power to Congress is
broad, general and comprehensive. 10 The legislative body possesses plenary Nor is it correct to argue as the dissenters do that A.O. No. 308 is not a law
power for all purposes of civil government. 11 Any power, deemed to be because it confers no right, imposes no duty, affords no protection, and
legislative by usage and tradition, is necessarily possessed by Congress, creates no office. Under A.O. No. 308, a citizen cannot transact business with
unless the Constitution has lodged it elsewhere. 12 In fine, except as limited government agencies delivering basic services to the people without the
by the Constitution, either expressly or impliedly, legislative power embraces contemplated identification card. No citizen will refuse to get this
all subjects and extends to matters of general concern or common interest. 13 identification card for no one can avoid dealing with government. It is thus
clear as daylight that without the ID, a citizen will have difficulty exercising
While Congress is vested with the power to enact laws, the President executes his rights and enjoying his privileges. Given this reality, the contention that
the laws. 14 The executive power is vested in the President. 15 It is generally A.O. No. 308 gives no right and imposes no duty cannot stand.
defined as the power to enforce and administer the laws. 16 It is the power
of carrying the laws into practical operation and enforcing their due Again, with due respect, the dissenting opinions unduly expand the limits of
observance. 17 administrative legislation and consequently erodes the plenary power of
Congress to make laws. This is contrary to the established approach defining
As head of the Executive Department, the President is the Chief Executive. the traditional limits of administrative legislation. As well stated by Fisher:
He represents the government as a whole and sees to it that all laws are ". . . Many regulations however, bear directly on the public. It is here that
enforced by the officials and employees of his department. 18 He has control administrative legislation must be restricted in its scope and application.
over the executive department, bureaus and offices. This means that he has Regulations are not supposed to be a substitute for the general policy-making
the authority to assume directly the functions of the executive department, that Congress enacts in the form of a public law. Although administrative
bureau and office, or interfere with the discretion of its officials. 19 Corollary regulations are entitled to respect, the authority to prescribe rules and
to the power of control, the President also has the duty of supervising the regulations is not an independent source of power to make laws." 28
enforcement of laws for the maintenance of general peace and public order.
Thus, he is granted administrative power over bureaus and offices under his III
control to enable him to discharge his duties effectively. 20
Assuming, arguendo, that A.O. No. 308 need not be the subject of a law, still
Administrative power is concerned with the work of applying policies and it cannot pass constitutional muster as an administrative legislation because
enforcing orders as determined by proper governmental organs. 21 It enables facially it violates the right to privacy. The essence of privacy is the "right to
the President to fix a uniform standard of administrative efficiency and check be let alone." 29 In the 1965 case of Griswold v. Connecticut, 30 the United
the official conduct of his agents. 22 To this end, he can issue administrative States Supreme Court gave more substance to the right of privacy when it
orders, rules and regulations. ruled that the right has a constitutional foundation. It held that there is a right
of privacy which can be found within the penumbras of the First, Third,
Prescinding from these precepts, we hold that A.O. No. 308 involves a Fourth, Fifth and Ninth Amendments, 31 viz:
subject that is not appropriate to be covered by an administrative order. An
administrative order is: "Specific guarantees in the Bill of Rights have penumbras formed by
emanations from these guarantees that help give them life and substance . . .
"Sec. 3. Administrative Orders. Acts of the President which relate to Various guarantees create zones of privacy. The right of association
particular aspects of governmental operation in pursuance of his duties as contained in the penumbra of the First Amendment is one, as we have seen.
administrative head shall be promulgated in administrative orders." 23 The Third Amendment in its prohibition against the quartering of soldiers 'in
any house' in time of peace without the consent of the owner is another facet
An administrative order is an ordinance issued by the President which relates of that privacy. The Fourth Amendment explicitly affirms the 'right of the
to specific aspects in the administrative operation of government. It must be people to be secure in their persons, houses, papers, and effects, against
in harmony with the law and should be for the sole purpose of implementing unreasonable searches and seizures.' The Fifth Amendment in its Self-
the law and carrying out the legislative policy. 24 We reject the argument Incrimination Clause enables the citizen to create a zone of privacy which
government may not force him to surrender to his detriment. The Ninth as actionable torts several acts by a person of meddling and prying into the
Amendment provides: 'The enumeration in the Constitution, of certain rights, privacy of another. 35 It also holds a public officer or employee or any private
shall not be construed to deny or disparage others retained by the people.'" individual liable for damages for any violation of the rights and liberties of
another person, 36 and recognizes the privacy of letters and other private
In the 1968 case of Morfe v. Mutuc, 32 we adopted the Griswold ruling that communications. 37 The Revised Penal Code makes a crime the violation of
there is a constitutional right to privacy. Speaking thru Mr. Justice, later Chief secrets by an officer, 38 the revelation of trade and industrial secrets, 39 and
Justice, Enrique Fernando, we held: trespass to dwelling. 40 Invasion of privacy is an offense in special laws like
the Anti-Wiretapping Law, 41 the Secrecy of Bank Deposits Act 42 and the
"xxx xxx xxx Intellectual Property Code. 43 The Rules of Court on privileged
communication likewise recognize the privacy of certain information. 44
The Griswold case invalidated a Connecticut statute which made the use of
contraceptives a criminal offense on the ground of its amounting to an Unlike the dissenters, we prescind from the premise that the right to privacy
unconstitutional invasion of the right of privacy of married persons; is a fundamental right guaranteed by the Constitution, hence, it is the burden
rightfully it stressed "a relationship lying within the zone of privacy created of government to show that A.O. No. 308 is justified by some compelling
by several fundamental constitutional guarantees." It has wider implications state interest and that it is narrowly drawn. A.O. No. 308 is predicated on two
though. The constitutional right to privacy has come into its own. considerations: (1) the need to provide our citizens and foreigners with the
facility to conveniently transact business with basic service and social
So it is likewise in our jurisdiction. The right to privacy as such is accorded security providers and other government instrumentalities and (2) the need
recognition independently of its identification with liberty; in itself, it is fully to reduce, if not totally eradicate, fraudulent transactions and
deserving of constitutional protection. The language of Prof. Emerson is misrepresentations by persons seeking basic services. It is debatable whether
particularly apt: 'The concept of limited government has always included the these interests are compelling enough to warrant the issuance of A.O. No.
idea that governmental powers stop short of certain intrusions into the 308. But what is not arguable is the broadness, the vagueness, the overbreadth
personal life of the citizen. This is indeed one of the basic distinctions of A.O. No. 308 which if implemented will put our people's right to privacy
between absolute and limited government. Ultimate and pervasive control of in clear and present danger.
the individual, in all aspects of his life, is the hallmark of the absolute state.
In contrast, a system of limited government safeguards a private sector, The heart of A.O. No. 308 lies in its Section 4 which provides for a
which belongs to the individual, firmly distinguishing it from the public Population Reference Number (PRN) as a "common reference number to
sector, which the state can control. Protection of this private sector - establish a linkage among concerned agencies" through the use of
protection, in other words, of the dignity and integrity of the individual - has "Biometrics Technology" and "computer application designs."
become increasingly important as modern society has developed. All the
forces of a technological age - industrialization, urbanization, and Biometry or biometrics is "the science of the application of statistical
organization - operate to narrow the area of privacy and facilitate intrusion methods to biological facts; a mathematical analysis of biological data." 45
into it. In modern terms, the capacity to maintain and support this enclave of The term "biometrics" has now evolved into a broad category of technologies
private life marks the difference between a democratic and a totalitarian which provide precise confirmation of an individual's identity through the
society.'" use of the individual's own physiological and behavioral characteristics. 46
A physiological characteristic is a relatively stable physical characteristic
Indeed, if we extend our judicial gaze we will find that the right of privacy is such as a fingerprint, retinal scan, hand geometry or facial features. A
recognized and enshrined in several provisions of our Constitution. 33 It is behavioral characteristic is influenced by the individual's personality and
expressly recognized in Section 3(1) of the Bill of Rights: includes voice print, signature and keystroke. 47 Most biometric
identification systems use a card or personal identification number (PIN) for
"Sec. 3. (1) The privacy of communication and correspondence shall be initial identification. The biometric measurement is used to verify that the
inviolable except upon lawful order of the court, or when public safety or individual holding the card or entering the PIN is the legitimate owner of the
order requires otherwise as prescribed by law." card or PIN. 48

Other facets of the right to privacy are protected in various provisions of the A most common form of biological encoding is finger-scanning where
Bill of Rights, viz: 34 technology scans a fingertip and turns the unique pattern therein into an
individual number which is called a biocrypt. The biocrypt is stored in
"Sec. 1. No person shall be deprived of life, liberty, or property without due computer data banks 49 and becomes a means of identifying an individual
process of law, nor shall any person be denied the equal protection of the using a service. This technology requires one's fingertip to be scanned every
laws. time service or access is provided. 50 Another method is the retinal scan.
Retinal scan technology employs optical technology to map the capillary
Sec. 2. The right of the people to be secure in their persons, houses, papers, pattern of the retina of the eye. This technology produces a unique print
and effects against unreasonable searches and seizures of whatever nature similar to a finger print. 51 Another biometric method is known as the
and for any purpose shall be inviolable, and no search warrant or warrant of "artificial nose." This device chemically analyzes the unique combination of
arrest shall issue except upon probable cause to be determined personally by substances excreted from the skin of people. 52 The latest on the list of
the judge after examination under oath or affirmation of the complainant and biometric achievements is the thermogram. Scientists have found that by
the witnesses he may produce, and particularly describing the place to be taking pictures of a face using infrared cameras, a unique heat distribution
searched and the persons or things to be seized. pattern is seen. The different densities of bone, skin, fat and blood vessels all
contribute to the individual's personal "heat signature." 53
xxx xxx xxx
In the last few decades, technology has progressed at a galloping rate. Some
Sec. 6. The liberty of abode and of changing the same within the limits science fictions are now science facts. Today, biometrics is no longer limited
prescribed by law shall not be impaired except upon lawful order of the court. to the use of fingerprint to identify an individual. It is a new science that uses
Neither shall the right to travel be impaired except in the interest of national various technologies in encoding any and all biological characteristics of an
security, public safety, or public health, as may be provided by law. individual for identification. It is noteworthy that A.O. No. 308 does not state
what specific biological characteristics and what particular biometrics
xxx xxx xxx. technology shall be used to identify people who will seek its coverage.
Considering the banquet of options available to the implementors of A.O.
Sec. 8. The right of the people, including those employed in the public and No. 308, the fear that it threatens the right to privacy of our people is not
private sectors, to form unions, associations, or societies for purposes not groundless.
contrary to law shall not be abridged.
A.O. No. 308 should also raise our antennas for a further look will show that
Sec. 17. No person shall be compelled to be a witness against himself." it does not state whether encoding of data is limited to biological information
alone for identification purposes. In fact, the Solicitor General claims that the
Zones of privacy are likewise recognized and protected in our laws. The Civil adoption of the Identification Reference System will contribute to the
Code provides that "[e]very person shall respect the dignity, personality, "generation of population data for development planning." 54 This is an
privacy and peace of mind of his neighbors and other persons" and punishes admission that the PRN will not be used solely for identification but for the
generation of other data with remote relation to the avowed purposes of A.O. privacy. 70 As technology advances, the level of reasonably expected privacy
No. 308. Clearly, the indefiniteness of A.O. No. 308 can give the government decreases. 71 The measure of protection granted by the reasonable
the roving authority to store and retrieve information for a purpose other than expectation diminishes as relevant technology becomes more widely
the identification of the individual through his PRN . accepted. 72 The security of the computer data file depends not only on the
physical inaccessibility of the file but also on the advances in hardware and
The potential for misuse of the data to be gathered under A.O. No. 308 cannot software computer technology. A.O. No. 308 is so widely drawn that a
be underplayed as the dissenters do. Pursuant to said administrative order, an minimum standard for a reasonable expectation of privacy, regardless of
individual must present his PRN everytime he deals with a government technology used, cannot be inferred from its provisions.
agency to avail of basic services and security. His transactions with the
government agency will necessarily be recorded - whether it be in the The rules and regulations to be drawn by the IACC cannot remedy this fatal
computer or in the documentary file of the agency. The individual's file may defect. Rules and regulations merely implement the policy of the law or
include his transactions for loan availments, income tax returns, statement of order. On its face, A.O. No. 308 gives the IACC virtually unfettered
assets and liabilities, reimbursements for medication, hospitalization, etc. discretion to determine the metes and bounds of the ID System.
The more frequent the use of the PRN, the better the chance of building a
huge and formidable information base through the electronic linkage of the Nor do our present laws provide adequate safeguards for a reasonable
files. 55 The data may be gathered for gainful and useful government expectation of privacy. Commonwealth Act No. 591 penalizes the disclosure
purposes; but the existence of this vast reservoir of personal information by any person of data furnished by the individual to the NSO with
constitutes a covert invitation to misuse, a temptation that may be too great imprisonment and fine. 73 Republic Act No. 1161 prohibits public disclosure
for some of our authorities to resist. 56 of SSS employment records and reports. 74 These laws, however, apply to
records and data with the NSO and the SSS. It is not clear whether they may
We can even grant, arguendo, that the computer data file will be limited to be applied to data with the other government agencies forming part of the
the name, address and other basic personal information about the individual. National ID System. The need to clarify the penal aspect of A.O. No. 308 is
57 Even that hospitable assumption will not save A.O. No. 308 from another reason why its enactment should be given to Congress.
constitutional infirmity for again said order does not tell us in clear and
categorical terms how these information gathered shall be handled. It does Next, the Solicitor General urges us to validate A.O. No. 308's abridgment
not provide who shall control and access the data, under what circumstances of the right of privacy by using the rational relationship test. 75 He stressed
and for what purpose. These factors are essential to safeguard the privacy and that the purposes of A.O. No. 308 are: (1) to streamline and speed up the
guaranty the integrity of the information. 58 Well to note, the computer implementation of basic government services, (2) eradicate fraud by avoiding
linkage gives other government agencies access to the information. Yet, there duplication of services, and (3) generate population data for development
are no controls to guard against leakage of information. When the access code planning. He concludes that these purposes justify the incursions into the
of the control programs of the particular computer system is broken, an right to privacy for the means are rationally related to the end. 76
intruder, without fear of sanction or penalty, can make use of the data for
whatever purpose, or worse, manipulate the data stored within the system. 59 We are not impressed by the argument. In Morfe v. Mutuc, 77 we upheld the
constitutionality of R.A. 3019, the Anti-Graft and Corrupt Practices Act, as
It is plain and we hold that A.O. No. 308 falls short of assuring that personal a valid police power measure. We declared that the law, in compelling a
information which will be gathered about our people will only be processed public officer to make an annual report disclosing his assets and liabilities,
for unequivocally specified purposes. 60 The lack of proper safeguards in his sources of income and expenses, did not infringe on the individual's right
this regard of A.O. No. 308 may interfere with the individual's liberty of to privacy. The law was enacted to promote morality in public administration
abode and travel by enabling authorities to track down his movement; it may by curtailing and minimizing the opportunities for official corruption and
also enable unscrupulous persons to access confidential information and maintaining a standard of honesty in the public service. 78
circumvent the right against self-incrimination; it may pave the way for
"fishing expeditions" by government authorities and evade the right against The same circumstances do not obtain in the case at bar. For one, R.A. 3019
unreasonable searches and seizures. 61 The possibilities of abuse and misuse is a statute, not an administrative order. Secondly, R.A. 3019 itself is
of the PRN, biometrics and computer technology are accentuated when we sufficiently detailed. The law is clear on what practices were prohibited and
consider that the individual lacks control over what can be read or placed on penalized, and it was narrowly drawn to avoid abuses. In the case at bar, A.O.
his ID, much less verify the correctness of the data encoded. 62 They threaten No. 308 may have been impelled by a worthy purpose, but, it cannot pass
the very abuses that the Bill of Rights seeks to prevent. 63 constitutional scrutiny for it is not narrowly drawn. And we now hold that
when the integrity of a fundamental right is at stake, this court will give the
The ability of a sophisticated data center to generate a comprehensive cradle- challenged law, administrative order, rule or regulation a stricter scrutiny. It
to-grave dossier on an individual and transmit it over a national network is will not do for the authorities to invoke the presumption of regularity in the
one of the most graphic threats of the computer revolution. 64 The computer performance of official duties. Nor is it enough for the authorities to prove
is capable of producing a comprehensive dossier on individuals out of that their act is not irrational for a basic right can be diminished, if not
information given at different times and for varied purposes. 65 It can defeated, even when the government does not act irrationally. They must
continue adding to the stored data and keeping the information up to date. satisfactorily show the presence of compelling state interests and that the law,
Retrieval of stored data is simple. When information of a privileged character rule, or regulation is narrowly drawn to preclude abuses. This approach is
finds its way into the computer, it can be extracted together with other data demanded by the 1987 Constitution whose entire matrix is designed to
on the subject. 66 Once extracted, the information is putty in the hands of protect human rights and to prevent authoritarianism. In case of doubt, the
any person. The end of privacy begins. least we can do is to lean towards the stance that will not put in danger the
rights protected by the Constitution.
Though A.O. No. 308 is undoubtedly not narrowly drawn, the dissenting
opinions would dismiss its danger to the right to privacy as speculative and The case of Whalen v. Roe 79 cited by the Solicitor General is also off-line.
hypothetical. Again, we cannot countenance such a laidback posture. The In Whalen, the United States Supreme Court was presented with the question
Court will not be true to its role as the ultimate guardian of the people's liberty of whether the State of New York could keep a centralized computer record
if it would not immediately smother the sparks that endanger their rights but of the names and addresses of all persons who obtained certain drugs
would rather wait for the fire that could consume them. pursuant to a doctor's prescription. The New York State Controlled
Substances Act of 1972 required physicians to identify patients obtaining
We reject the argument of the Solicitor General that an individual has a prescription drugs enumerated in the statute, i.e., drugs with a recognized
reasonable expectation of privacy with regard to the National ID and the use medical use but with a potential for abuse, so that the names and addresses
of biometrics technology as it stands on quicksand. The reasonableness of a of the patients can be recorded in a centralized computer file of the State
person's expectation of privacy depends on a two-part test: (1) whether by Department of Health. The plaintiffs, who were patients and doctors, claimed
his conduct, the individual has exhibited an expectation of privacy; and (2) that some people might decline necessary medication because of their fear
whether this expectation is one that society recognizes as reasonable. 67 The that the computerized data may be readily available and open to public
factual circumstances of the case determines the reasonableness of the disclosure; and that once disclosed, it may stigmatize them as drug addicts.
expectation. 68 However, other factors, such as customs, physical 80 The plaintiffs alleged that the statute invaded a constitutionally protected
surroundings and practices of a particular activity, may serve to create or zone of privacy, i.e, the individual interest in avoiding disclosure of personal
diminish this expectation. 69 The use of biometrics and computer technology matters, and the interest in independence in making certain kinds of
in A.O. No. 308 does not assure the individual of a reasonable expectation of important decisions. The U.S. Supreme Court held that while an individual's
interest in avoiding disclosure of personal matters is an aspect of the right to
privacy, the statute did not pose a grievous threat to establish a constitutional IN VIEW WHEREOF, the petition is granted and Administrative Order No.
violation. The Court found that the statute was necessary to aid in the 308 entitled "Adoption of a National Computerized Identification Reference
enforcement of laws designed to minimize the misuse of dangerous drugs. System" declared null and void for being unconstitutional.
The patient-identification requirement was a product of an orderly and
rational legislative decision made upon recommendation by a specially SO ORDERED.
appointed commission which held extensive hearings on the matter.
Moreover, the statute was narrowly drawn and contained numerous
safeguards against indiscriminate disclosure. The statute laid down the
procedure and requirements for the gathering, storage and retrieval of the
information. It enumerated who were authorized to access the data. It also
prohibited public disclosure of the data by imposing penalties for its
violation. In view of these safeguards, the infringement of the patients' right
to privacy was justified by a valid exercise of police power. As we discussed
above, A.O. No. 308 lacks these vital safeguards.

Even while we strike down A.O. No. 308, we spell out in neon that the Court
is not per se against the use of computers to accumulate, store, process,
retrieve and transmit data to improve our bureaucracy. Computers work
wonders to achieve the efficiency which both government and private
industry seek. Many information systems in different countries make use of
the computer to facilitate important social objectives, such as better law
enforcement, faster delivery of public services, more efficient management
of credit and insurance programs, improvement of telecommunications and
streamlining of financial activities. 81 Used wisely, data stored in the
computer could help good administration by making accurate and
comprehensive information for those who have to frame policy and make key
decisions. 82 The benefits of the computer has revolutionized information
technology. It developed the internet, 83 introduced the concept of
cyberspace 84 and the information superhighway where the individual,
armed only with his personal computer, may surf and search all kinds and
classes of information from libraries and databases connected to the net.

In no uncertain terms, we also underscore that the right to privacy does not
bar all incursions into individual privacy. The right is not intended to stifle
scientific and technological advancements that enhance public service and
the common good. It merely requires that the law be narrowly focused 85
and a compelling interest justify such intrusions. 86 Intrusions into the right
must be accompanied by proper safeguards and well-defined standards to
prevent unconstitutional invasions. We reiterate that any law or order that
invades individual privacy will be subjected by this Court to strict scrutiny.
The reason for this stance was laid down in Morfe v. Mutuc, to wit:

"The concept of limited government has always included the idea that
governmental powers stop short of certain intrusions into the personal life of
the citizen. This is indeed one of the basic distinctions between absolute and
limited government. Ultimate and pervasive control of the individual, in all
aspects of his life, is the hallmark of the absolute state. In contrast, a system
of limited government safeguards a private sector, which belongs to the
individual, firmly distinguishing it from the public sector, which the state can
control. Protection of this private sector - protection, in other words, of the
dignity and integrity of the individual - has become increasingly important
as modern society has developed. All the forces of a technological age -
industrialization, urbanization, and organization - operate to narrow the area
of privacy and facilitate intrusion into it. In modern terms, the capacity to
maintain and support this enclave of private life marks the difference
between a democratic and a totalitarian society." 87

IV

The right to privacy is one of the most threatened rights of man living in a
mass society. The threats emanate from various sources - governments,
journalists, employers, social scientists, etc. 88 In the case at bar, the threat
comes from the executive branch of government which by issuing A.O. No.
308 pressures the people to surrender their privacy by giving information
about themselves on the pretext that it will facilitate delivery of basic
services. Given the record-keeping power of the computer, only the
indifferent will fail to perceive the danger that A.O. No. 308 gives the
government the power to compile a devastating dossier against unsuspecting
citizens. It is timely to take note of the well-worded warning of Kalvin, Jr.,
"the disturbing result could be that everyone will live burdened by an
unerasable record of his past and his limitations. In a way, the threat is that
because of its record-keeping, the society will have lost its benign capacity
to forget." 89 Oblivious to this counsel, the dissents still say we should not
be too quick in labelling the right to privacy as a fundamental right. We close
with the statement that the right to privacy was not engraved in our
Constitution for flattery.
ZACARIAS VILLAVICENCIO, ET AL., petitioners, city of Manila. According to an exhibit attached to the answer of the fiscal,
vs. the 170 women were destined to be laborers, at good salaries, on the
JUSTO LUKBAN, ET AL., respondents. haciendas of Yñigo and Governor Sales. In open court, the fiscal admitted,
G.R. No. L-14639 | 1919-03-25 in answer to question of a member of the court, that these women had been
sent out of Manila without their consent. The court awarded the writ, in an
View Summary order of November 4, that directed Justo Lukban, Mayor of the city of
Manila, Anton Hohmann, chief of police of the city of Manila, Francisco
EN BANC Sales, governor of the province of Davao, and Feliciano Yñigo, an hacendero
of Davao, to bring before the court the persons therein named, alleged to be
MALCOLM, J.: deprived of their liberty, on December 2, 1918.

The annals of juridical history fail to reveal a case quite as remarkable as the Before the date mentioned, seven of the women had returned to Manila at
one which this application for habeas corpus submits for decision. While their own expense. On motion of counsel for petitioners, their testimony was
hardly to be expected to be met with in this modern epoch of triumphant taken before the clerk of the Supreme Court sitting as commissioners. On the
democracy, yet, after all, the cause presents no great difficulty if there is kept day named in the order, December 2nd, 1918, none of the persons in whose
in the forefront of our minds the basic principles of popular government, and behalf the writ was issued were produced in court by the respondents. It has
if we give expression to the paramount purpose for which the courts, as an been shown that three of those who had been able to come back to Manila
independent power of such a government, were constituted. The primary through their own efforts, were notified by the police and the secret service
question is — Shall the judiciary permit a government of the men instead of to appear before the court. The fiscal appeared, repeated the facts more
a government of laws to be set up in the Philippine Islands? comprehensively, reiterated the stand taken by him when pleading to the
original petition copied a telegram from the Mayor of the city of Manila to
Omitting much extraneous matter, of no moment to these proceedings, but the provincial governor of Davao and the answer thereto, and telegrams that
which might prove profitable reading for other departments of the had passed between the Director of Labor and the attorney for that Bureau
government, the facts are these: The Mayor of the city of Manila, Justo then in Davao, and offered certain affidavits showing that the women were
Lukban, for the best of all reasons, to exterminate vice, ordered the contained with their life in Mindanao and did not wish to return to Manila.
segregated district for women of ill repute, which had been permitted for a Respondents Sales answered alleging that it was not possible to fulfill the
number of years in the city of Manila, closed. Between October 16 and order of the Supreme Court because the women had never been under his
October 25, 1918, the women were kept confined to their houses in the control, because they were at liberty in the Province of Davao, and because
district by the police. Presumably, during this period, the city authorities they had married or signed contracts as laborers. Respondent Yñigo
quietly perfected arrangements with the Bureau of Labor for sending the answered alleging that he did not have any of the women under his control
women to Davao, Mindanao, as laborers; with some government office for and that therefore it was impossible for him to obey the mandate. The court,
the use of the coastguard cutters Corregidor and Negros, and with the after due deliberation, on December 10, 1918, promulgated a second order,
Constabulary for a guard of soldiers. At any rate, about midnight of October which related that the respondents had not complied with the original order
25, the police, acting pursuant to orders from the chief of police, Anton to the satisfaction of the court nor explained their failure to do so, and
Hohmann and the Mayor of the city of Manila, Justo Lukban, descended therefore directed that those of the women not in Manila be brought before
upon the houses, hustled some 170 inmates into patrol wagons, and placed the court by respondents Lukban, Hohmann, Sales, and Yñigo on January 13,
them aboard the steamers that awaited their arrival. The women were given 1919, unless the women should, in written statements voluntarily made
no opportunity to collect their belongings, and apparently were under the before the judge of first instance of Davao or the clerk of that court, renounce
impression that they were being taken to a police station for an investigation. the right, or unless the respondents should demonstrate some other legal
They had no knowledge that they were destined for a life in Mindanao. They motives that made compliance impossible. It was further stated that the
had not been asked if they wished to depart from that region and had neither question of whether the respondents were in contempt of court would later
directly nor indirectly given their consent to the deportation. The involuntary be decided and the reasons for the order announced in the final decision.
guests were received on board the steamers by a representative of the Bureau
of Labor and a detachment of Constabulary soldiers. The two steamers with Before January 13, 1919, further testimony including that of a number of the
their unwilling passengers sailed for Davao during the night of October 25. women, of certain detectives and policemen, and of the provincial governor
of Davao, was taken before the clerk of the Supreme Court sitting as
The vessels reached their destination at Davao on October 29. The women commissioner and the clerk of the Court of First Instance of Davao acting in
were landed and receipted for as laborers by Francisco Sales, provincial the same capacity. On January 13, 1919, the respondents technically
governor of Davao, and by Feliciano Yñigo and Rafael Castillo. The presented before the Court the women who had returned to the city through
governor and the hacendero Yñigo, who appear as parties in the case, had no their own efforts and eight others who had been brought to Manila by the
previous notification that the women were prostitutes who had been expelled respondents. Attorneys for the respondents, by their returns, once again
from the city of Manila. The further happenings to these women and the recounted the facts and further endeavored to account for all of the persons
serious charges growing out of alleged ill-treatment are of public interest, but involved in the habeas corpus. In substance, it was stated that the
are not essential to the disposition of this case. Suffice it to say, generally, respondents, through their representatives and agents, had succeeded in
that some of the women married, others assumed more or less clandestine bringing from Davao with their consent eight women; that eighty-one women
relations with men, others went to work in different capacities, others were found in Davao who, on notice that if they desired they could return to
assumed a life unknown and disappeared, and a goodly portion found means Manila, transportation fee, renounced the right through sworn statements;
to return to Manila. that fifty-nine had already returned to Manila by other means, and that despite
all efforts to find them twenty-six could not be located. Both counsel for
To turn back in our narrative, just about the time the Corregidor and the petitioners and the city fiscal were permitted to submit memoranda. The first
Negros were putting in to Davao, the attorney for the relatives and friends of formally asked the court to find Justo Lukban, Mayor of the city of Manila,
a considerable number of the deportees presented an application for habeas Anton Hohmann, chief of police of the city of Manila, Jose Rodriguez and
corpus to a member of the Supreme Court. Subsequently, the application, Fernando Ordax, members of the police force of the city of Manila, Feliciano
through stipulation of the parties, was made to include all of the women who Yñigo, an hacendero of Davao, Modesto Joaquin, the attorney for the Bureau
were sent away from Manila to Davao and, as the same questions concerned of Labor, and Anacleto Diaz, fiscal of the city of Manila, in contempt of
them all, the application will be considered as including them. The court. The city fiscal requested that the replica al memorandum de los
application set forth the salient facts, which need not be repeated, and alleged recurridos, (reply to respondents' memorandum) dated January 25, 1919, be
that the women were illegally restrained of their liberty by Justo Lukban, struck from the record.
Mayor of the city of Manila, Anton Hohmann, chief of police of the city of
Manila, and by certain unknown parties. The writ was made returnable before In the second order, the court promised to give the reasons for granting the
the full court. The city fiscal appeared for the respondents, Lukban and writ of habeas corpus in the final decision. We will now proceed to do so.
Hohmann, admitted certain facts relative to sequestration and deportation,
and prayed that the writ should not be granted because the petitioners were One fact, and one fact only, need be recalled — these one hundred and
not proper parties, because the action should have been begun in the Court seventy women were isolated from society, and then at night, without their
of First Instance for Davao, Department of Mindanao and Sulu, because the consent and without any opportunity to consult with friends or to defend their
respondents did not have any of the women under their custody or control, rights, were forcibly hustled on board steamers for transportation to regions
and because their jurisdiction did not extend beyond the boundaries of the unknown. Despite the feeble attempt to prove that the women left voluntarily
and gladly, that such was not the case is shown by the mere fact that the What are the remedies of the unhappy victims of official oppression? The
presence of the police and the constabulary was deemed necessary and that remedies of the citizen are three:
these officers of the law chose the shades of night to cloak their secret and
stealthy acts. Indeed, this is a fact impossible to refute and practically (1) Civil action;
admitted by the respondents.
(2) criminal action, and
With this situation, a court would next expect to resolve the question — By
authority of what law did the Mayor and the Chief of Police presume to act (3) habeas corpus.
in deporting by duress these persons from Manila to another distant locality
within the Philippine Islands? We turn to the statutes and we find — The first is an optional but rather slow process by which the aggrieved party
may recoup money damages. It may still rest with the parties in interest to
Alien prostitutes can be expelled from the Philippine Islands in conformity pursue such an action, but it was never intended effectively and promptly to
with an Act of congress. The Governor-General can order the eviction of meet any such situation as that now before us.
undesirable aliens after a hearing from the Islands. Act No. 519 of the
Philippine Commission and section 733 of the Revised Ordinances of the city As to criminal responsibility, it is true that the Penal Code in force in these
of Manila provide for the conviction and punishment by a court of justice of Islands provides:
any person who is a common prostitute. Act No. 899 authorizes the return of
any citizen of the United States, who may have been convicted of vagrancy, Any public officer not thereunto authorized by law or by regulations of a
to the homeland. New York and other States have statutes providing for the general character in force in the Philippines who shall banish any person to
commitment to the House of Refuge of women convicted of being common a place more than two hundred kilometers distant from his domicile, except
prostitutes. Always a law! Even when the health authorities compel it be by virtue of the judgment of a court, shall be punished by a fine of not
vaccination, or establish a quarantine, or place a leprous person in the Culion less than three hundred and twenty-five and not more than three thousand
leper colony, it is done pursuant to some law or order. But one can search in two hundred and fifty pesetas.
vain for any law, order, or regulation, which even hints at the right of the
Mayor of the city of Manila or the chief of police of that city to force citizens Any public officer not thereunto expressly authorized by law or by regulation
of the Philippine Islands — and these women despite their being in a sense of a general character in force in the Philippines who shall compel any person
lepers of society are nevertheless not chattels but Philippine citizens to change his domicile or residence shall suffer the penalty of destierro and
protected by the same constitutional guaranties as are other citizens — to a fine of not less than six hundred and twenty-five and not more than six
change their domicile from Manila to another locality. On the contrary, thousand two hundred and fifty pesetas. (Art. 211.)
Philippine penal law specifically punishes any public officer who, not being
expressly authorized by law or regulation, compels any person to change his We entertain no doubt but that, if, after due investigation, the proper
residence. prosecuting officers find that any public officer has violated this provision of
law, these prosecutors will institute and press a criminal prosecution just as
In other countries, as in Spain and Japan, the privilege of domicile is deemed vigorously as they have defended the same official in this action.
so important as to be found in the Bill of Rights of the Constitution. Under Nevertheless, that the act may be a crime and that the persons guilty thereof
the American constitutional system, liberty of abode is a principle so deeply can be proceeded against, is no bar to the instant proceedings. To quote the
imbedded in jurisprudence and considered so elementary in nature as not words of Judge Cooley in a case which will later be referred to — "It would
even to require a constitutional sanction. Even the Governor-General of the be a monstrous anomaly in the law if to an application by one unlawfully
Philippine Islands, even the President of the United States, who has often confined, ta be restored to his liberty, it could be a sufficient answer that the
been said to exercise more power than any king or potentate, has no such confinement was a crime, and therefore might be continued indefinitely until
arbitrary prerogative, either inherent or express. Much less, therefore, has the the guilty party was tried and punished therefor by the slow process of
executive of a municipality, who acts within a sphere of delegated powers. If criminal procedure." (In the matter of Jackson [1867], 15 Mich., 416, 434.)
the mayor and the chief of police could, at their mere behest or even for the The writ of habeas corpus was devised and exists as a speedy and effectual
most praiseworthy of motives, render the liberty of the citizen so insecure, remedy to relieve persons from unlawful restraint, and as the best and only
then the presidents and chiefs of police of one thousand other municipalities sufficient defense of personal freedom. Any further rights of the parties are
of the Philippines have the same privilege. If these officials can take to left untouched by decision on the writ, whose principal purpose is to set the
themselves such power, then any other official can do the same. And if any individual at liberty.
official can exercise the power, then all persons would have just as much
right to do so. And if a prostitute could be sent against her wishes and under Granted that habeas corpus is the proper remedy, respondents have raised
no law from one locality to another within the country, then officialdom can three specific objections to its issuance in this instance. The fiscal has argued
hold the same club over the head of any citizen. (l) that there is a defect in parties petitioners, (2) that the Supreme Court
should not a assume jurisdiction, and (3) that the person in question are not
Law defines power. Centuries ago Magna Charta decreed that — "No restrained of their liberty by respondents. It was finally suggested that the
freeman shall be taken, or imprisoned, or be disseized of his freehold, or jurisdiction of the Mayor and the chief of police of the city of Manila only
liberties, or free customs, or be outlawed, or exiled, or any other wise extends to the city limits and that perforce they could not bring the women
destroyed; nor will we pass upon him nor condemn him, but by lawful from Davao.
judgment of his peers or by the law of the land. We will sell to no man, we
will not deny or defer to any man either justice or right." (Magna Charta, 9 The first defense was not presented with any vigor by counsel. The
Hen., 111, 1225, Cap. 29; 1 eng. stat. at Large, 7.) No official, no matter how petitioners were relatives and friends of the deportees. The way the expulsion
high, is above the law. The courts are the forum which functionate to was conducted by the city officials made it impossible for the women to sign
safeguard individual liberty and to punish official transgressors. "The law," a petition for habeas corpus. It was consequently proper for the writ to be
said Justice Miller, delivering the opinion of the Supreme Court of the United submitted by persons in their behalf. (Code of Criminal Procedure, sec. 78;
States, "is the only supreme power in our system of government, and every Code of Civil Procedure, sec. 527.) The law, in its zealous regard for personal
man who by accepting office participates in its functions is only the more liberty, even makes it the duty of a court or judge to grant a writ of habeas
strongly bound to submit to that supremacy, and to observe the limitations corpus if there is evidence that within the court's jurisdiction a person is
which it imposes upon the exercise of the authority which it gives." (U.S. vs. unjustly imprisoned or restrained of his liberty, though no application be
Lee [1882], 106 U.S., 196, 220.) "The very idea," said Justice Matthews of made therefor. (Code of Criminal Procedure, sec. 93.) Petitioners had
the same high tribunal in another case, "that one man may be compelled to standing in court.
hold his life, or the means of living, or any material right essential to the
enjoyment of life, at the mere will of another, seems to be intolerable in any The fiscal next contended that the writ should have been asked for in the
country where freedom prevails, as being the essence of slavery itself." (Yick Court of First Instance of Davao or should have been made returnable before
Wo vs. Hopkins [1886], 118 U.S., 356, 370.) All this explains the motive in that court. It is a general rule of good practice that, to avoid unnecessary
issuing the writ of habeas corpus, and makes clear why we said in the very expense and inconvenience, petitions for habeas corpus should be presented
beginning that the primary question was whether the courts should permit a to the nearest judge of the court of first instance. But this is not a hard and
government of men or a government of laws to be established in the fast rule. The writ of habeas corpus may be granted by the Supreme Court or
Philippine Islands. any judge thereof enforcible anywhere in the Philippine Islands. (Code of
Criminal Procedure, sec. 79; Code of Civil Procedure, sec. 526.) Whether the
writ shall be made returnable before the Supreme Court or before an inferior I have not yet seen sufficient reason to doubt the power of this court to issue
court rests in the discretion of the Supreme Court and is dependent on the the present writ on the petition which was laid before us. . . .
particular circumstances. In this instance it was not shown that the Court of
First Instance of Davao was in session, or that the women had any means by It would be strange indeed if, at this late day, after the eulogiums of six
which to advance their plea before that court. On the other hand, it was shown centuries and a half have been expended upon the Magna Charta, and rivers
that the petitioners with their attorneys, and the two original respondents with of blood shed for its establishment; after its many confirmations, until Coke
their attorney, were in Manila; it was shown that the case involved parties could declare in his speech on the petition of right that "Magna Charta was
situated in different parts of the Islands; it was shown that the women might such a fellow that he will have no sovereign," and after the extension of its
still be imprisoned or restrained of their liberty; and it was shown that if the benefits and securities by the petition of right, bill of rights and habeas corpus
writ was to accomplish its purpose, it must be taken cognizance of and acts, it should now be discovered that evasion of that great clause for the
decided immediately by the appellate court. The failure of the superior court protection of personal liberty, which is the life and soul of the whole
to consider the application and then to grant the writ would have amounted instrument, is so easy as is claimed here. If it is so, it is important that it be
to a denial of the benefits of the writ. determined without delay, that the legislature may apply the proper remedy,
as I can not doubt they would, on the subject being brought to their notice. .
The last argument of the fiscal is more plausible and more difficult to meet. ..
When the writ was prayed for, says counsel, the parties in whose behalf it
was asked were under no restraint; the women, it is claimed, were free in The second proposition — that the statutory provisions are confined to the
Davao, and the jurisdiction of the mayor and the chief of police did not extend case of imprisonment within the state — seems to me to be based upon a
beyond the city limits. At first blush, this is a tenable position. On closer misconception as to the source of our jurisdiction. It was never the case in
examination, acceptance of such dictum is found to be perversive of the first England that the court of king's bench derived its jurisdiction to issue and
principles of the writ of habeas corpus. enforce this writ from the statute. Statutes were not passed to give the right,
but to compel the observance of rights which existed. . . .
A prime specification of an application for a writ of habeas corpus is restraint
of liberty. The essential object and purpose of the writ of habeas corpus is to The important fact to be observed in regard to the mode of procedure upon
inquire into all manner of involuntary restraint as distinguished from this writ is, that it is directed to and served upon, not the person confined, but
voluntary, and to relieve a person therefrom if such restraint is illegal. Any his jailor. It does not reach the former except through the latter. The officer
restraint which will preclude freedom of action is sufficient. The forcible or person who serves it does not unbar the prison doors, and set the prisoner
taking of these women from Manila by officials of that city, who handed free, but the court relieves him by compelling the oppressor to release his
them over to other parties, who deposited them in a distant region, deprived constraint. The whole force of the writ is spent upon the respondent, and if
these women of freedom of locomotion just as effectively as if they had been he fails to obey it, the means to be resorted to for the purposes of compulsion
imprisoned. Placed in Davao without either money or personal belongings, are fine and imprisonment. This is the ordinary mode of affording relief, and
they were prevented from exercising the liberty of going when and where if any other means are resorted to, they are only auxiliary to those which are
they pleased. The restraint of liberty which began in Manila continued until usual. The place of confinement is, therefore, not important to the relief, if
the aggrieved parties were returned to Manila and released or until they freely the guilty party is within reach of process, so that by the power of the court
and truly waived his right. he can be compelled to release his grasp. The difficulty of affording redress
is not increased by the confinement being beyond the limits of the state,
Consider for a moment what an agreement with such a defense would mean. except as greater distance may affect it. The important question is, where the
The chief executive of any municipality in the Philippines could forcibly and power of control exercised? And I am aware of no other remedy. (In the
illegally take a private citizen and place him beyond the boundaries of the matter of Jackson [1867], 15 Mich., 416.)
municipality, and then, when called upon to defend his official action, could
calmly fold his hands and claim that the person was under no restraint and The opinion of Judge Cooley has since been accepted as authoritative by
that he, the official, had no jurisdiction over this other municipality. We other courts. (Rivers vs. Mitchell [1881], 57 Iowa, 193; Breene vs. People
believe the true principle should be that, if the respondent is within the [1911], Colo., 117 Pac. Rep., 1000; Ex parte Young [1892], 50 Fed., 526.)
jurisdiction of the court and has it in his power to obey the order of the court
and thus to undo the wrong that he has inflicted, he should be compelled to The English courts have given careful consideration to the subject. Thus, a
do so. Even if the party to whom the writ is addressed has illegally parted child had been taken out of English by the respondent. A writ of habeas
with the custody of a person before the application for the writ is no reason corpus was issued by the Queen's Bench Division upon the application of the
why the writ should not issue. If the mayor and the chief of police, acting mother and her husband directing the defendant to produce the child. The
under no authority of law, could deport these women from the city of Manila judge at chambers gave defendant until a certain date to produce the child,
to Davao, the same officials must necessarily have the same means to return but he did not do so. His return stated that the child before the issuance of the
them from Davao to Manila. The respondents, within the reach of process, writ had been handed over by him to another; that it was no longer in his
may not be permitted to restrain a fellow citizen of her liberty by forcing her custody or control, and that it was impossible for him to obey the writ. He
to change her domicile and to avow the act with impunity in the courts, while was found in contempt of court. On appeal, the court, through Lord Esher,
the person who has lost her birthright of liberty has no effective recourse. M. R., said:
The great writ of liberty may not thus be easily evaded.
A writ of habeas corpus was ordered to issue, and was issued on January 22.
It must be that some such question has heretofore been presented to the courts That writ commanded the defendant to have the body of the child before a
for decision. Nevertheless, strange as it may seem, a close examination of the judge in chambers at the Royal Courts of Justice immediately after the receipt
authorities fails to reveal any analogous case. Certain decisions of of the writ, together with the cause of her being taken and detained. That is a
respectable courts are however very persuasive in nature. command to bring the child before the judge and must be obeyed, unless
some lawful reason can be shown to excuse the nonproduction of the child.
A question came before the Supreme Court of the State of Michigan at an If it could be shown that by reason of his having lawfully parted with the
early date as to whether or not a writ of habeas corpus would issue from the possession of the child before the issuing of the writ, the defendant had no
Supreme Court to a person within the jurisdiction of the State to bring into longer power to produce the child, that might be an answer; but in the absence
the State a minor child under guardianship in the State, who has been and of any lawful reason he is bound to produce the child, and, if he does not, he
continues to be detained in another State. The membership of the Michigan is in contempt of the Court for not obeying the writ without lawful excuse.
Supreme Court at this time was notable. It was composed of Martin, chief Many efforts have been made in argument to shift the question of contempt
justice, and Cooley, Campbell, and Christiancy, justices. On the question to some anterior period for the purpose of showing that what was done at
presented the court was equally divided. Campbell, J., with whom concurred some time prior to the writ cannot be a contempt. But the question is not as
Martin, C. J., held that the writ should be quashed. Cooley, J., one of the most to what was done before the issue of the writ. The question is whether there
distinguished American judges and law-writers, with whom concurred has been a contempt in disobeying the writ it was issued by not producing
Christiancy, J., held that the writ should issue. Since the opinion of Justice the child in obedience to its commands. (The Queen vs. Bernardo [1889], 23
Campbell was predicated to a large extent on his conception of the English Q. B. D., 305. See also to the same effect the Irish case of In re Matthews, 12
decisions, and since, as will hereafter appear, the English courts have taken Ir. Com. Law Rep. [N. S.], 233; The Queen vs. Barnardo, Gossage's Case
a contrary view, only the following eloquent passages from the opinion of [1890], 24 Q. B. D., 283.)
Justice Cooley are quoted:
A decision coming from the Federal Courts is also of interest. A habeas give the respondents another chance to demonstrate their good faith and to
corpus was directed to the defendant to have before the circuit court of the mitigate their wrong.
District of Columbia three colored persons, with the cause of their detention.
Davis, in his return to the writ, stated on oath that he had purchased the In response to the second order of the court, the respondents appear to have
negroes as slaves in the city of Washington; that, as he believed, they were become more zealous and to have shown a better spirit. Agents were
removed beyond the District of Columbia before the service of the writ of dispatched to Mindanao, placards were posted, the constabulary and the
habeas corpus, and that they were then beyond his control and out of his municipal police joined in rounding up the women, and a steamer with free
custody. The evidence tended to show that Davis had removed the negroes transportation to Manila was provided. While charges and counter-charges
because he suspected they would apply for a writ of habeas corpus. The court in such a bitterly contested case are to be expected, and while a critical
held the return to be evasive and insufficient, and that Davis was bound to reading of the record might reveal a failure of literal fulfillment with our
produce the negroes, and Davis being present in court, and refusing to mandate, we come to conclude that there is a substantial compliance with it.
produce them, ordered that he be committed to the custody of the marshall Our finding to this effect may be influenced somewhat by our sincere desire
until he should produce the negroes, or be otherwise discharged in due course to see this unhappy incident finally closed. If any wrong is now being
of law. The court afterwards ordered that Davis be released upon the perpetrated in Davao, it should receive an executive investigation. If any
production of two of the negroes, for one of the negroes had run away and particular individual is still restrained of her liberty, it can be made the object
been lodged in jail in Maryland. Davis produced the two negroes on the last of separate habeas corpus proceedings.
day of the term. (United States vs. Davis [1839], 5 Cranch C.C., 622, Fed.
Cas. No. 14926. See also Robb vs. Connolly [1883], 111 U.S., 624; Church Since the writ has already been granted, and since we find a substantial
on Habeas, 2nd ed., p. 170.) compliance with it, nothing further in this connection remains to be done.

We find, therefore, both on reason and authority, that no one of the defense The attorney for the petitioners asks that we find in contempt of court Justo
offered by the respondents constituted a legitimate bar to the granting of the Lukban, Mayor of the city of Manila, Anton Hohmann, chief of police of the
writ of habeas corpus. city of Manila, Jose Rodriguez, and Fernando Ordax, members of the police
force of the city of Manila, Modesto Joaquin, the attorney for the Bureau of
There remains to be considered whether the respondent complied with the Labor, Feliciano Yñigo, an hacendero of Davao, and Anacleto Diaz, Fiscal
two orders of the Supreme Court awarding the writ of habeas corpus, and if of the city of Manila.
it be found that they did not, whether the contempt should be punished or be
taken as purged. The power to punish for contempt of court should be exercised on the
preservative and not on the vindictive principle. Only occasionally should
The first order, it will be recalled, directed Justo Lukban, Anton Hohmann, the court invoke its inherent power in order to retain that respect without
Francisco Sales, and Feliciano Yñigo to present the persons named in the writ which the administration of justice must falter or fail. Nevertheless when one
before the court on December 2, 1918. The order was dated November 4, is commanded to produce a certain person and does not do so, and does not
1918. The respondents were thus given ample time, practically one month, offer a valid excuse, a court must, to vindicate its authority, adjudge the
to comply with the writ. As far as the record discloses, the Mayor of the city respondent to be guilty of contempt, and must order him either imprisoned
of Manila waited until the 21st of November before sending a telegram to the or fined. An officer's failure to produce the body of a person in obedience to
provincial governor of Davao. According to the response of the attorney for a writ of habeas corpus when he has power to do so, is a contempt committed
the Bureau of Labor to the telegram of his chief, there were then in Davao in the face of the court. (Ex parte Sterns [1888], 77 Cal., 156; In re Patterson
women who desired to return to Manila, but who should not be permitted to [1888], 99 N. C., 407.)
do so because of having contracted debts. The half-hearted effort naturally
resulted in none of the parties in question being brought before the court on With all the facts and circumstances in mind, and with judicial regard for
the day named. human imperfections, we cannot say that any of the respondents, with the
possible exception of the first named, has flatly disobeyed the court by acting
For the respondents to have fulfilled the court's order, three optional courses in opposition to its authority. Respondents Hohmann, Rodriguez, Ordax, and
were open: (1) They could have produced the bodies of the persons according Joaquin only followed the orders of their chiefs, and while, under the law of
to the command of the writ; or (2) they could have shown by affidavit that public officers, this does not exonerate them entirely, it is nevertheless a
on account of sickness or infirmity those persons could not safely be brought powerful mitigating circumstance. The hacendero Yñigo appears to have
before the court; or (3) they could have presented affidavits to show that the been drawn into the case through a misconstruction by counsel of telegraphic
parties in question or their attorney waived the right to be present. (Code of communications. The city fiscal, Anacleto Diaz, would seem to have done
Criminal Procedure, sec. 87.) They did not produce the bodies of the persons no more than to fulfill his duty as the legal representative of the city
in whose behalf the writ was granted; they did not show impossibility of government. Finding him innocent of any disrespect to the court, his counter-
performance; and they did not present writings that waived the right to be motion to strike from the record the memorandum of attorney for the
present by those interested. Instead a few stereotyped affidavits purporting to petitioners, which brings him into this undesirable position, must be granted.
show that the women were contended with their life in Davao, some of which When all is said and done, as far as this record discloses, the official who was
have since been repudiated by the signers, were appended to the return. That primarily responsible for the unlawful deportation, who ordered the police to
through ordinary diligence a considerable number of the women, at least accomplish the same, who made arrangements for the steamers and the
sixty, could have been brought back to Manila is demonstrated to be found constabulary, who conducted the negotiations with the Bureau of Labor, and
in the municipality of Davao, and that about this number either returned at who later, as the head of the city government, had it within his power to
their own expense or were produced at the second hearing by the respondents. facilitate the return of the unfortunate women to Manila, was Justo Lukban,
the Mayor of the city of Manila. His intention to suppress the social evil was
The court, at the time the return to its first order was made, would have been commendable. His methods were unlawful. His regard for the writ of habeas
warranted summarily in finding the respondents guilty of contempt of court, corpus issued by the court was only tardily and reluctantly acknowledged.
and in sending them to jail until they obeyed the order. Their excuses for the
non-production of the persons were far from sufficient. The, authorities cited It would be possible to turn to the provisions of section 546 of the Code of
herein pertaining to somewhat similar facts all tend to indicate with what Civil Procedure, which relates to the penalty for disobeying the writ, and in
exactitude a habeas corpus writ must be fulfilled. For example, in Gossage's pursuance thereof to require respondent Lukban to forfeit to the parties
case, supra, the Magistrate in referring to an earlier decision of the Court, aggrieved as much as P400 each, which would reach to many thousands of
said: "We thought that, having brought about that state of things by his own pesos, and in addition to deal with him as for a contempt. Some members of
illegal act, he must take the consequences; and we said that he was bound to the court are inclined to this stern view. It would also be possible to find that
use every effort to get the child back; that he must do much more than write since respondent Lukban did comply substantially with the second order of
letters for the purpose; that he must advertise in America, and even if the court, he has purged his contempt of the first order. Some members of the
necessary himself go after the child, and do everything that mortal man could court are inclined to this merciful view. Between the two extremes appears
do in the matter; and that the court would only accept clear proof of an to lie the correct finding. The failure of respondent Lukban to obey the first
absolute impossibility by way of excuse." In other words, the return did not mandate of the court tended to belittle and embarrass the administration of
show that every possible effort to produce the women was made by the justice to such an extent that his later activity may be considered only as
respondents. That the court forebore at this time to take drastic action was extenuating his conduct. A nominal fine will at once command such respect
because it did not wish to see presented to the public gaze the spectacle of a without being unduly oppressive — such an amount is P100.
clash between executive officials and the judiciary, and because it desired to
In resume — as before stated, no further action on the writ of habeas corpus
is necessary. The respondents Hohmann, Rodriguez, Ordax, Joaquin, Yñigo,
and Diaz are found not to be in contempt of court. Respondent Lukban is
found in contempt of court and shall pay into the office of the clerk of the
Supreme Court within five days the sum of one hundred pesos (P100). The
motion of the fiscal of the city of Manila to strike from the record the Replica
al Memorandum de los Recurridos of January 25, 1919, is granted. Costs
shall be taxed against respondents. So ordered.

In concluding this tedious and disagreeable task, may we not be permitted to


express the hope that this decision may serve to bulwark the fortifications of
an orderly government of laws and to protect individual liberty from illegal
encroachment.
MARCOS vs. MANGLAPUS
G.R. No. 88211 September 15, 1989 1. Does the President have the power to bar the return of former President
CORTES, J.: Marcos and family to the Philippines?

Before the Court is a contreversy of grave national importance. While a. Is this a political question?
ostensibly only legal issues are involved, the Court's decision in this case
would undeniably have a profound effect on the political, economic and other 2. Assuming that the President has the power to bar former President Marcos
aspects of national life. and his family from returning to the Philippines, in the interest of "national
security, public safety or public health
We recall that in February 1986, Ferdinand E. Marcos was deposed from the
presidency via the non-violent "people power" revolution and forced into a. Has the President made a finding that the return of former President
exile. In his stead, Corazon C. Aquino was declared President of the Republic Marcos and his family to the Philippines is a clear and present danger to
under a revolutionary government. Her ascension to and consilidation of national security, public safety or public health?
power have not been unchallenged. The failed Manila Hotel coup in 1986 led
by political leaders of Mr. Marcos, the takeover of television station Channel b. Assuming that she has made that finding
7 by rebel troops led by Col. Canlas with the support of "Marcos loyalists"
and the unseccessful plot of the Marcos spouses to surreptitiously return from (1) Have the requirements of due process been complied with in making such
Hawii with mercenaries aboard an aircraft chartered by a Lebanese arms finding?
dealer [Manila Bulletin, January 30, 1987] awakened the nation to the
capacity of the Marcoses to stir trouble even from afar and to the fanaticism (2) Has there been prior notice to petitioners?
and blind loyalty of their followers in the country. The ratification of the 1987
Constitution enshrined the victory of "people power" and also clearly (3) Has there been a hearing?
reinforced the constitutional moorings of Mrs. Aquino's presidency. This did
not, however, stop bloody challenges to the government. On August 28, (4) Assuming that notice and hearing may be dispensed with, has the
1987, Col. Gregorio Honasan, one of the major players in the February President's decision, including the grounds upon which it was based, been
Revolution, led a failed coup that left scores of people, both combatants and made known to petitioners so that they may controvert the same?
civilians, dead. There were several other armed sorties of lesser significance,
but the message they conveyed was the same — a split in the ranks of the c. Is the President's determination that the return of former President Marcos
military establishment that thraetened civilian supremacy over military and and his family to the Philippines is a clear and present danger to national
brought to the fore the realization that civilian government could be at the security, public safety, or public health a political question?
mercy of a fractious military.
d. Assuming that the Court may inquire as to whether the return of former
But the armed threats to the Government were not only found in misguided President Marcos and his family is a clear and present danger to national
elements and among rabid followers of Mr. Marcos. There are also the security, public safety, or public health, have respondents established such
communist insurgency and the seccessionist movement in Mindanao which fact?
gained ground during the rule of Mr. Marcos, to the extent that the
communists have set up a parallel government of their own on the areas they 3. Have the respondents, therefore, in implementing the President's decision
effectively control while the separatist are virtually free to move about in to bar the return of former President Marcos and his family, acted and would
armed bands. There has been no let up on this groups' determination to wrest be acting without jurisdiction, or in excess of jurisdiction, or with grave abuse
power from the govermnent. Not only through resort to arms but also to of discretion, in performing any act which would effectively bar the return of
through the use of propaganda have they been successful in dreating chaos former President Marcos and his family to the Philippines? [Memorandum
and destabilizing the country. for Petitioners, pp. 5-7; Rollo, pp. 234-236.1

Nor are the woes of the Republic purely political. The accumulated foreign The case for petitioners is founded on the assertion that the right of the
debt and the plunder of the nation attributed to Mr. Marcos and his cronies Marcoses to return to the Philippines is guaranteed under the following
left the economy devastated. The efforts at economic recovery, three years provisions of the Bill of Rights, to wit:
after Mrs. Aquino assumed office, have yet to show concrete results in
alleviating the poverty of the masses, while the recovery of the ill-gotten Section 1. No person shall be deprived of life, liberty, or property without
wealth of the Marcoses has remained elusive. due process of law, nor shall any person be denied the equal protection of the
laws.
Now, Mr. Marcos, in his deathbed, has signified his wish to return to the
Philipppines to die. But Mrs. Aquino, considering the dire consequences to xxx xxx xxx
the nation of his return at a time when the stability of government is
threatened from various directions and the economy is just beginning to rise Section 6. The liberty of abode and of changing the same within the limits
and move forward, has stood firmly on the decision to bar the return of Mr. prescribed by law shall not be impaired except upon lawful order of the court.
Marcos and his family. Neither shall the right to travel be impaired except in the interest of national
security, public safety, or public health, as may be provided by law.
The Petition
The petitioners contend that the President is without power to impair the
This case is unique. It should not create a precedent, for the case of a dictator liberty of abode of the Marcoses because only a court may do so "within the
forced out of office and into exile after causing twenty years of political, limits prescribed by law." Nor may the President impair their right to travel
economic and social havoc in the country and who within the short space of because no law has authorized her to do so. They advance the view that
three years seeks to return, is in a class by itself. before the right to travel may be impaired by any authority or agency of the
government, there must be legislation to that effect.
This petition for mandamus and prohibition asks the Courts to order the
respondents to issue travel documents to Mr. Marcos and the immediate The petitioners further assert that under international law, the right of Mr.
members of his family and to enjoin the implementation of the President's Marcos and his family to return to the Philippines is guaranteed.
decision to bar their return to the Philippines.
The Universal Declaration of Human Rights provides:
The Issue
Article 13. (1) Everyone has the right to freedom of movement and residence
Th issue is basically one of power: whether or not, in the exercise of the within the borders of each state.
powers granted by the Constitution, the President may prohibit the Marcoses
from returning to the Philippines. (2) Everyone has the right to leave any country, including his own, and to
return to his country.
According to the petitioners, the resolution of the case would depend on the
resolution of the following issues:
Likewise, the International Covenant on Civil and Political Rights, which Fulgencio batista of Cuba, King Farouk of Egypt, Maximiliano Hernandez
had been ratified by the Philippines, provides: Martinez of El Salvador, and Marcos Perez Jimenez of Venezuela were
among the deposed dictators whose return to their homelands was prevented
Article 12 by their governments. [See Statement of Foreign Affairs Secretary Raul S.
Manglapus, quoted in Memorandum for Respondents, pp. 26-32; Rollo, pp.
1) Everyone lawfully within the territory of a State shall, within that territory, 314-319.]
have the right to liberty of movement and freedom to choose his residence.
The parties are in agreement that the underlying issue is one of the scope of
2) Everyone shall be free to leave any country, including his own. presidential power and its limits. We, however, view this issue in a different
light. Although we give due weight to the parties' formulation of the issues,
3) The above-mentioned rights shall not be subject to any restrictions except we are not bound by its narrow confines in arriving at a solution to the
those which are provided by law, are necessary to protect national security, controversy.
public order (order public), public health or morals or the rights and freedoms
of others, and are consistent with the other rights recognized in the present At the outset, we must state that it would not do to view the case within the
Covenant. confines of the right to travel and the import of the decisions of the U.S.
Supreme Court in the leading cases of Kent v. Dulles [357 U.S. 116, 78 SCt
4) No one shall be arbitrarily deprived of the right to enter his own country. 1113, 2 L Ed. 2d 1204] and Haig v. Agee [453 U.S. 280, 101 SCt 2766, 69 L
Ed. 2d 640) which affirmed the right to travel and recognized exceptions to
On the other hand, the respondents' principal argument is that the issue in this the exercise thereof, respectively.
case involves a political question which is non-justiciable. According to the
Solicitor General: It must be emphasized that the individual right involved is not the right to
travel from the Philippines to other countries or within the Philippines. These
As petitioners couch it, the question involved is simply whether or not are what the right to travel would normally connote. Essentially, the right
petitioners Ferdinand E. Marcos and his family have the right to travel and involved is the right to return to one's country, a totally distinct right under
liberty of abode. Petitioners invoke these constitutional rights in vacuo international law, independent from although related to the right to travel.
without reference to attendant circumstances. Thus, the Universal Declaration of Humans Rights and the International
Covenant on Civil and Political Rights treat the right to freedom of
Respondents submit that in its proper formulation, the issue is whether or not movement and abode within the territory of a state, the right to leave a
petitioners Ferdinand E. Marcos and family have the right to return to the country, and the right to enter one's country as separate and distinct rights.
Philippines and reside here at this time in the face of the determination by the The Declaration speaks of the "right to freedom of movement and residence
President that such return and residence will endanger national security and within the borders of each state" [Art. 13(l)] separately from the "right to
public safety. leave any country, including his own, and to return to his country." [Art.
13(2).] On the other hand, the Covenant guarantees the "right to liberty of
It may be conceded that as formulated by petitioners, the question is not a movement and freedom to choose his residence" [Art. 12(l)] and the right to
political question as it involves merely a determination of what the law "be free to leave any country, including his own." [Art. 12(2)] which rights
provides on the matter and application thereof to petitioners Ferdinand E. may be restricted by such laws as "are necessary to protect national security,
Marcos and family. But when the question is whether the two rights claimed public order, public health or morals or enter qqqs own country" of which
by petitioners Ferdinand E. Marcos and family impinge on or collide with one cannot be "arbitrarily deprived." [Art. 12(4).] It would therefore be
the more primordial and transcendental right of the State to security and inappropriate to construe the limitations to the right to return to one's country
safety of its nationals, the question becomes political and this Honorable in the same context as those pertaining to the liberty of abode and the right
Court can not consider it. to travel.

There are thus gradations to the question, to wit: The right to return to one's country is not among the rights specifically
guaranteed in the Bill of Rights, which treats only of the liberty of abode and
Do petitioners Ferdinand E. Marcos and family have the right to return to the the right to travel, but it is our well-considered view that the right to return
Philippines and reestablish their residence here? This is clearly a justiciable may be considered, as a generally accepted principle of international law and,
question which this Honorable Court can decide. under our Constitution, is part of the law of the land [Art. II, Sec. 2 of the
Constitution.] However, it is distinct and separate from the right to travel and
Do petitioners Ferdinand E. Marcos and family have their right to return to enjoys a different protection under the International Covenant of Civil and
the Philippines and reestablish their residence here even if their return and Political Rights, i.e., against being "arbitrarily deprived" thereof [Art. 12 (4).]
residence here will endanger national security and public safety? this is still
a justiciable question which this Honorable Court can decide. Thus, the rulings in the cases Kent and Haig which refer to the issuance of
passports for the purpose of effectively exercising the right to travel are not
Is there danger to national security and public safety if petitioners Ferdinand determinative of this case and are only tangentially material insofar as they
E. Marcos and family shall return to the Philippines and establish their relate to a conflict between executive action and the exercise of a protected
residence here? This is now a political question which this Honorable Court right. The issue before the Court is novel and without precedent in Philippine,
can not decide for it falls within the exclusive authority and competence of and even in American jurisprudence.
the President of the Philippines. [Memorandum for Respondents, pp. 9-11;
Rollo, pp. 297-299.] Consequently, resolution by the Court of the well-debated issue of whether
or not there can be limitations on the right to travel in the absence of
Respondents argue for the primacy of the right of the State to national legislation to that effect is rendered unnecessary. An appropriate case for its
security over individual rights. In support thereof, they cite Article II of the resolution will have to be awaited.
Constitution, to wit:
Having clarified the substance of the legal issue, we find now a need to
Section 4. The prime duty of the Government is to serve and protect the explain the methodology for its resolution. Our resolution of the issue will
people. The Government may call upon the people to defend the State and, involve a two-tiered approach. We shall first resolve whether or not the
in the fulfillment thereof, all citizens may be required, under conditions President has the power under the Constitution, to bar the Marcoses from
provided by law, to render personal, military, or civil service. returning to the Philippines. Then, we shall determine, pursuant to the
express power of the Court under the Constitution in Article VIII, Section 1,
Section 5. The maintenance of peace and order, the protection of life, liberty, whether or not the President acted arbitrarily or with grave abuse of
and property, and the promotion of the general welfare are essential for the discretion amounting to lack or excess of jurisdiction when she determined
enjoyment by all the people of the blessings of democracy. that the return of the Marcose's to the Philippines poses a serious threat to
national interest and welfare and decided to bar their return.
Respondents also point out that the decision to ban Mr. Marcos and family
from returning to the Philippines for reasons of national security and public Executive Power
safety has international precedents. Rafael Trujillo of the Dominican
Republic, Anastacio Somoza Jr. of Nicaragua, Jorge Ubico of Guatemala,
The 1987 Constitution has fully restored the separation of powers of the three President. Above all, the way each President understood it as his personal
great branches of government. To recall the words of Justice Laurel in obligation to inform and involve the Congress, to earn and hold the
Angara v. Electoral Commission [63 Phil. 139 (1936)], "the Constitution has confidence of the electorate and to render an accounting to the nation and
blocked but with deft strokes and in bold lines, allotment of power to the posterity determined whether he strengthened or weakened the constitutional
executive, the legislative and the judicial departments of the government." order. [At 212- 213.]
[At 157.1 Thus, the 1987 Constitution explicitly provides that "[the
legislative power shall be vested in the Congress of the Philippines" Art VI, We do not say that the presidency is what Mrs. Aquino says it is or what she
Sec. 11, "[t]he executive power shall bevested in the President of the does but, rather, that the consideration of tradition and the development of
Philippines" [Art. VII, Sec. 11, and "[te judicial power shall be vested in one presidential power under the different constitutions are essential for a
Supreme Court and in such lower courts as may be established by law" [Art. complete understanding of the extent of and limitations to the President's
VIII, Sec. 1.] These provisions not only establish a separation of powers by powers under the 1987 Constitution. The 1935 Constitution created a strong
actual division [Angara v. Electoral Commission, supra] but also confer President with explicitly broader powers than the U.S. President. The 1973
plenary legislative, executive and judicial powers subject only to limitations Constitution attempted to modify the system of government into the
provided in the Constitution. For as the Supreme Court in Ocampo v. parliamentary type, with the President as a mere figurehead, but through
Cabangis [15 Phil. 626 (1910)] pointed out "a grant of the legislative power numerous amendments, the President became even more powerful, to the
means a grant of all legislative power; and a grant of the judicial power means point that he was also the de facto Legislature. The 1987 Constitution,
a grant of all the judicial power which may be exercised under the however, brought back the presidential system of government and restored
government." [At 631-632.1 If this can be said of the legislative power which the separation of legislative, executive and judicial powers by their actual
is exercised by two chambers with a combined membership of more than two distribution among three distinct branches of government with provision for
hundred members and of the judicial power which is vested in a hierarchy of checks and balances.
courts, it can equally be said of the executive power which is vested in one
official the President. It would not be accurate, however, to state that "executive power" is the
power to enforce the laws, for the President is head of state as well as head
As stated above, the Constitution provides that "[t]he executive power shall of government and whatever powers inhere in such positions pertain to the
be vested in the President of the Philippines." [Art. VII, Sec. 1]. However, it office unless the Constitution itself withholds it. Furthermore, the
does not define what is meant by executive power" although in the same Constitution itself provides that the execution of the laws is only one of the
article it touches on the exercise of certain powers by the President, i.e., the powers of the President. It also grants the President other powers that do not
power of control over all executive departments, bureaus and offices, the involve the execution of any provision of law, e.g., his power over the
power to execute the laws, the appointing power, the powers under the country's foreign relations.
commander-in-chief clause, the power to grant reprieves, commutations and
pardons, the power to grant amnesty with the concurrence of Congress, the On these premises, we hold the view that although the 1987 Constitution
power to contract or guarantee foreign loans, the power to enter into treaties imposes limitations on the exercise of specific powers of the President, it
or international agreements, the power to submit the budget to Congress, and maintains intact what is traditionally considered as within the scope of
the power to address Congress [Art. VII, Sec. 14-23]. "executive power." Corollarily, the powers of the President cannot be said to
be limited only to the specific powers enumerated in the Constitution. In
The inevitable question then arises: by enumerating certain powers of the other words, executive power is more than the sum of specific powers so
President did the framers of the Constitution intend that the President shall enumerated,
exercise those specific powers and no other? Are these se enumerated powers
the breadth and scope of "executive power"? Petitioners advance the view It has been advanced that whatever power inherent in the government that is
that the President's powers are limited to those specifically enumerated in the neither legislative nor judicial has to be executive. Thus, in the landmark
1987 Constitution. Thus, they assert: "The President has enumerated powers, decision of Springer v. Government of the Philippine Islands, 277 U.S. 189
and what is not enumerated is impliedly denied to her. Inclusion unius est (1928), on the issue of who between the Governor-General of the Philippines
exclusio alterius[Memorandum for Petitioners, p. 4- Rollo p. 233.1 This and the Legislature may vote the shares of stock held by the Government to
argument brings to mind the institution of the U.S. Presidency after which elect directors in the National Coal Company and the Philippine National
ours is legally patterned.** Bank, the U.S. Supreme Court, in upholding the power of the Governor-
General to do so, said:
Corwin, in his monumental volume on the President of the United States
grappled with the same problem. He said: ...Here the members of the legislature who constitute a majority of the
"board" and "committee" respectively, are not charged with the performance
Article II is the most loosely drawn chapter of the Constitution. To those who of any legislative functions or with the doing of anything which is in aid of
think that a constitution ought to settle everything beforehand it should be a performance of any such functions by the legislature. Putting aside for the
nightmare; by the same token, to those who think that constitution makers moment the question whether the duties devolved upon these members are
ought to leave considerable leeway for the future play of political forces, it vested by the Organic Act in the Governor-General, it is clear that they are
should be a vision realized. not legislative in character, and still more clear that they are not judicial. The
fact that they do not fall within the authority of either of these two constitutes
We encounter this characteristic of Article 11 in its opening words: "The logical ground for concluding that they do fall within that of the remaining
executive power shall be vested in a President of the United States of one among which the powers of government are divided ....[At 202-203;
America." . . .. [The President: Office and Powers, 17871957, pp. 3-4.] Emphasis supplied.]

Reviewing how the powers of the U.S. President were exercised by the We are not unmindful of Justice Holmes' strong dissent. But in his enduring
different persons who held the office from Washington to the early 1900's, words of dissent we find reinforcement for the view that it would indeed be
and the swing from the presidency by commission to Lincoln's dictatorship, a folly to construe the powers of a branch of government to embrace only
he concluded that "what the presidency is at any particular moment depends what are specifically mentioned in the Constitution:
in important measure on who is President." [At 30.]
The great ordinances of the Constitution do not establish and divide fields of
This view is shared by Schlesinger who wrote in The Imperial Presidency: black and white. Even the more specific of them are found to terminate in a
penumbra shading gradually from one extreme to the other. ....
For the American Presidency was a peculiarly personal institution. it
remained of course, an agency of government subject to unvarying demands xxx xxx xxx
and duties no remained, of cas President. But, more than most agencies of
government, it changed shape, intensity and ethos according to the man in It does not seem to need argument to show that however we may disguise it
charge. Each President's distinctive temperament and character, his values, by veiling words we do not and cannot carry out the distinction between
standards, style, his habits, expectations, Idiosyncrasies, compulsions, legislative and executive action with mathematical precision and divide the
phobias recast the WhiteHouse and pervaded the entire government. The branches into watertight compartments, were it ever so desirable to do so,
executive branch, said Clark Clifford, was a chameleon, taking its color from which I am far from believing that it is, or that the Constitution requires. [At
the character and personality of the President. The thrust of the office, its 210- 211.]
impact on the constitutional order, therefore altered from President to
The Power Involved laws." [House Resolution No. 1342, Rollo, p. 321.1 The Resolution does not
question the President's power to bar the Marcoses from returning to the
The Constitution declares among the guiding principles that "[t]he prime Philippines, rather, it appeals to the President's sense of compassion to allow
duty of theGovernment is to serve and protect the people" and that "[t]he a man to come home to die in his country.
maintenance of peace and order,the protection of life, liberty, and property,
and the promotion of the general welfare are essential for the enjoyment by What we are saying in effect is that the request or demand of the Marcoses
all the people of the blessings of democracy." [Art. II, Secs. 4 and 5.] to be allowed to return to the Philippines cannot be considered in the light
solely of the constitutional provisions guaranteeing liberty of abode and the
Admittedly, service and protection of the people, the maintenance of peace right to travel, subject to certain exceptions, or of case law which clearly
and order, the protection of life, liberty and property, and the promotion of never contemplated situations even remotely similar to the present one. It
the general welfare are essentially ideals to guide governmental action. But must be treated as a matter that is appropriately addressed to those residual
such does not mean that they are empty words. Thus, in the exercise of unstated powers of the President which are implicit in and correlative to the
presidential functions, in drawing a plan of government, and in directing paramount duty residing in that office to safeguard and protect general
implementing action for these plans, or from another point of view, in making welfare. In that context, such request or demand should submit to the exercise
any decision as President of the Republic, the President has to consider these of a broader discretion on the part of the President to determine whether it
principles, among other things, and adhere to them. must be granted or denied.

Faced with the problem of whether or not the time is right to allow the The Extent of Review
Marcoses to return to the Philippines, the President is, under the Constitution,
constrained to consider these basic principles in arriving at a decision. More Under the Constitution, judicial power includes the duty to determine
than that, having sworn to defend and uphold the Constitution, the President whether or not there has been a grave abuse of discretion amounting to lack
has the obligation under the Constitution to protect the people, promote their or excess of jurisdiction on the part of any branch or instrumentality of the
welfare and advance the national interest. It must be borne in mind that the Government." [Art. VIII, Sec. 1] Given this wording, we cannot agree with
Constitution, aside from being an allocation of power is also a social contract the Solicitor General that the issue constitutes a political question which is
whereby the people have surrendered their sovereign powers to the State for beyond the jurisdiction of the Court to decide.
the common good. Hence, lest the officers of the Government exercising the
powers delegated by the people forget and the servants of the people become The present Constitution limits resort to the political question doctrine and
rulers, the Constitution reminds everyone that "[s]overeignty resides in the broadens the scope of judicial inquiry into areas which the Court, under
people and all government authority emanates from them." [Art. II, Sec. 1.] previous constitutions, would have normally left to the political departments
to decide. But nonetheless there remain issues beyond the Court's jurisdiction
The resolution of the problem is made difficult because the persons who seek the determination of which is exclusively for the President, for Congress or
to return to the country are the deposed dictator and his family at whose door for the people themselves through a plebiscite or referendum. We cannot, for
the travails of the country are laid and from whom billions of dollars believed example, question the President's recognition of a foreign government, no
to be ill-gotten wealth are sought to be recovered. The constitutional matter how premature or improvident such action may appear. We cannot set
guarantees they invoke are neither absolute nor inflexible. For the exercise aside a presidential pardon though it may appear to us that the beneficiary is
of even the preferred freedoms of speech and ofexpression, although couched totally undeserving of the grant. Nor can we amend the Constitution under
in absolute terms, admits of limits and must be adjusted to the requirements the guise of resolving a dispute brought before us because the power is
of equally important public interests [Zaldivar v. Sandiganbayan, G.R. Nos. reserved to the people.
79690-707, October 7, 1981.]
There is nothing in the case before us that precludes our determination
To the President, the problem is one of balancing the general welfare and the thereof on the political question doctrine. The deliberations of the
common good against the exercise of rights of certain individuals. The power Constitutional Commission cited by petitioners show that the framers
involved is the President's residual power to protect the general welfare of intended to widen the scope of judicial review but they did not intend courts
the people. It is founded on the duty of the President, as steward of the people. of justice to settle all actual controversies before them. When political
To paraphrase Theodore Roosevelt, it is not only the power of the President questions are involved, the Constitution limits the determination to whether
but also his duty to do anything not forbidden by the Constitution or the laws or not there has been a grave abuse of discretion amounting to lack or excess
that the needs of the nation demand [See Corwin, supra, at 153]. It is a power of jurisdiction on the part of the official whose action is being questioned. If
borne by the President's duty to preserve and defend the Constitution. It also grave abuse is not established, the Court will not substitute its judgment for
may be viewed as a power implicit in the President's duty to take care that that of the official concerned and decide a matter which by its nature or by
the laws are faithfully executed [see Hyman, The American President, where law is for the latter alone to decide. In this light, it would appear clear that
the author advances the view that an allowance of discretionary power is the second paragraph of Article VIII, Section 1 of the Constitution, defining
unavoidable in any government and is best lodged in the President]. "judicial power," which specifically empowers the courts to determine
whether or not there has been a grave abuse of discretion on the part of any
More particularly, this case calls for the exercise of the President's powers as branch or instrumentality of the government, incorporates in the fundamental
protector of the peace. Rossiter The American Presidency].The power of the law the ruling in Lansang v. Garcia [G.R. No. L-33964, December 11, 1971,
President to keep the peace is not limited merely to exercising the 42 SCRA 4481 that:]
commander-in-chief powers in times of emergency or to leading the State
against external and internal threats to its existence. The President is not only Article VII of the [1935] Constitution vests in the Executive the power to
clothed with extraordinary powers in times of emergency, but is also tasked suspend the privilege of the writ of habeas corpus under specified conditions.
with attending to the day-to-day problems of maintaining peace and order Pursuant to the principle of separation of powers underlying our system of
and ensuring domestic tranquility in times when no foreign foe appears on government, the Executive is supreme within his own sphere. However, the
the horizon. Wide discretion, within the bounds of law, in fulfilling separation of powers, under the Constitution, is not absolute. What is more,
presidential duties in times of peace is not in any way diminished by the it goes hand in hand with the system of checks and balances, under which the
relative want of an emergency specified in the commander-in-chief Executive is supreme, as regards the suspension of the privilege, but only if
provision. For in making the President commander-in-chief the enumeration and when he acts within the sphere alloted to him by the Basic Law, and the
of powers that follow cannot be said to exclude the President's exercising as authority to determine whether or not he has so acted is vested in the Judicial
Commander-in- Chief powers short of the calling of the armed forces, or Department, which, in this respect, is, in turn, constitutionally supreme. In
suspending the privilege of the writ of habeas corpus or declaring martial the exercise of such authority, the function of the Court is merely to check
law, in order to keep the peace, and maintain public order and security. — not to supplant the Executive, or to ascertain merely whether he has gone
beyond the constitutional limits of his jurisdiction, not to exercise the power
That the President has the power under the Constitution to bar the Marcose's vested in him or to determine the wisdom of his act [At 479-480.]
from returning has been recognized by memembers of the Legislature, and is
manifested by the Resolution proposed in the House of Representatives and Accordingly, the question for the Court to determine is whether or not there
signed by 103 of its members urging the President to allow Mr. Marcos to exist factual bases for the President to conclude that it was in the national
return to the Philippines "as a genuine unselfish gesture for true national interest to bar the return of the Marcoses to the Philippines. If such postulates
reconciliation and as irrevocable proof of our collective adherence to do exist, it cannot be said that she has acted, or acts, arbitrarily or that she
uncompromising respect for human rights under the Constitution and our has gravely abused her discretion in deciding to bar their return.
We find that from the pleadings filed by the parties, from their oral
arguments, and the facts revealed during the briefing in chambers by the
Chief of Staff of the Armed Forces of the Philippines and the National
Security Adviser, wherein petitioners and respondents were represented,
there exist factual bases for the President's decision..

The Court cannot close its eyes to present realities and pretend that the
country is not besieged from within by a well-organized communist
insurgency, a separatist movement in Mindanao, rightist conspiracies to grab
power, urban terrorism, the murder with impunity of military men, police
officers and civilian officials, to mention only a few. The documented history
of the efforts of the Marcose's and their followers to destabilize the country,
as earlier narrated in this ponencia bolsters the conclusion that the return of
the Marcoses at this time would only exacerbate and intensify the violence
directed against the State and instigate more chaos.

As divergent and discordant forces, the enemies of the State may be


contained. The military establishment has given assurances that it could
handle the threats posed by particular groups. But it is the catalytic effect of
the return of the Marcoses that may prove to be the proverbial final straw that
would break the camel's back. With these before her, the President cannot be
said to have acted arbitrarily and capriciously and whimsically in
determining that the return of the Marcoses poses a serious threat to the
national interest and welfare and in prohibiting their return.

It will not do to argue that if the return of the Marcoses to the Philippines will
cause the escalation of violence against the State, that would be the time for
the President to step in and exercise the commander-in-chief powers granted
her by the Constitution to suppress or stamp out such violence. The State,
acting through the Government, is not precluded from taking pre- emptive
action against threats to its existence if, though still nascent they are
perceived as apt to become serious and direct. Protection of the people is the
essence of the duty of government. The preservation of the State the fruition
of the people's sovereignty is an obligation in the highest order. The
President, sworn to preserve and defend the Constitution and to see the
faithful execution the laws, cannot shirk from that responsibility.

We cannot also lose sight of the fact that the country is only now beginning
to recover from the hardships brought about by the plunder of the economy
attributed to the Marcoses and their close associates and relatives, many of
whom are still here in the Philippines in a position to destabilize the country,
while the Government has barely scratched the surface, so to speak, in its
efforts to recover the enormous wealth stashed away by the Marcoses in
foreign jurisdictions. Then, We cannot ignore the continually increasing
burden imposed on the economy by the excessive foreign borrowing during
the Marcos regime, which stifles and stagnates development and is one of the
root causes of widespread poverty and all its attendant ills. The resulting
precarious state of our economy is of common knowledge and is easily within
the ambit of judicial notice.

The President has determined that the destabilization caused by the return of
the Marcoses would wipe away the gains achieved during the past few years
and lead to total economic collapse. Given what is within our individual and
common knowledge of the state of the economy, we cannot argue with that
determination.

WHEREFORE, and it being our well-considered opinion that the President


did not act arbitrarily or with grave abuse of discretion in determining that
the return of former President Marcos and his family at the present time and
under present circumstances poses a serious threat to national interest and
welfare and in prohibiting their return to the Philippines, the instant petition
is hereby DISMISSED.

SO ORDERED.
EBRALINAG vs. THE DIVISION SUPERINTENDENT OF Upholding religious freedom as a fundamental right deserving the "highest
SCHOOLS OF CEBU priority and amplest protection among human rights," this Court, in
G.R. No. 95770 December 29, 1995 Ebralinag vs. Division Superintendent of Schools of Cebu4 re-examined our
KAPUNAN, J.: over two decades-old decision in Gerona and reversed expulsion orders made
by the public respondents therein as violative of both the free exercise of
The State moves for a reconsideration of our decision dated March 1, 1993 religion clause and the right of citizens to education under the 1987
granting private respondents' petition for certiorari and prohibition and Constitution.5
annulling the expulsion orders issued by the public respondents therein on
the ground that the said decision created an exemption in favor of the From our decision of March 1, 1993, the public respondents filed a motion
members of the religious sect, the Jehovah's Witnesses, in violation of the for reconsideration on grounds hereinabove stated. After a careful study of
"Establishment Clause" of the Constitution. The Solicitor General, on behalf the grounds adduced in the government's Motion For Reconsideration of our
of the public respondent, furthermore contends that: original decision, however, we find no cogent reason to disturb our earlier
ruling.
The accommodation by this Honorable Court to a demand for special
treatment in favor of a minority sect even on the basis of a claim of religious The religious convictions and beliefs of the members of the religious sect,
freedom may be criticized as granting preference to the religious beliefs of the Jehovah's Witnesses are widely known and are equally widely
said sect in violation of the "non-establishment guarantee" provision of the disseminated in numerous books, magazines, brochures and leaflets
Constitution. Surely, the decision of the Court constitutes a special favor distributed by their members in their house to house distribution efforts and
which immunizes religious believers such as Jehovah's Witnesses to the law in many public places. Their refusal to render obeisance to any form or
and the DECS rules and regulations by interposing the claim that the conduct symbol which smacks of idolatry is based on their sincere belief in the
required by law and the rules and regulation (sic) are violative of their biblical injunction found in Exodus 20:4,5, against worshipping forms or
religious beliefs. The decision therefore is susceptible to the very criticism idols other than God himself. The basic assumption in their universal refusal
that the grant of exemption is a violation of the "non-establishment" to salute the flags of the countries in which they are found is that such a salute
provision of the Constitution. constitutes an act of religious devotion forbidden by God's law. This
assumption, while "bizarre" to others is firmly anchored in several biblical
Furthermore, to grant an exemption to a specific religious minority poses a passages.6
risk of collision course with the "equal protection of the laws" clause in
respect of the non-exempt, and, in public schools, a collision course with the And yet, while members of Jehovah's Witnesses, on the basis of religious
"non-establishment guarantee." convictions, refuse to perform an act (or acts) which they consider proscribed
by the Bible, they contend that such refusal should not be taken to indicate
Additionally the public respondent insists that this Court adopt a "neutral disrespect for the symbols of the country or evidence that they are wanting
stance" by reverting to its holding in Gerona declaring the flag as being in patriotism and nationalism. They point out that as citizens, they have an
devoid of any religious significance. He stresses that the issue here is not excellent record as law abiding members of society even if they do not
curtailment of religious belief but regulation of the exercise of religious demonstrate their refusal to conform to the assailed orders by overt acts of
belief. Finally, he maintains that the State's interests in the case at bench are conformity. On the contrary, they aver that they show their respect through
constitutional and legal obligations to implement the law and the less demonstrative methods manifesting their allegiance, by their simple
constitutional mandate to inculcate in the youth patriotism and nationalism obedience to the country's laws,7 by not engaging in antigovernment
and to encourage their involvement in public and civic affairs, referring to activities of any kind,8 and by paying their taxes and dues to society as self-
the test devised by the United States Supreme Court in U.S. vs. O'Brien.1 sufficient members of the community.9 While they refuse to salute the flag,
they are willing to stand quietly and peacefully at attention, hands on their
II side, in order not to disrupt the ceremony or disturb those who believe
differently.10
All the petitioners in the original case2 were minor school children, and
members of the sect, Jehovah's Witnesses (assisted by their parents) who The religious beliefs, practices and convictions of the members of the sect as
were expelled from their classes by various public school authorities in Cebu a minority are bound to be seen by others as odd and different and at
for refusing to salute the flag, sing the national anthem and recite the patriotic divergence with the complex requirements of contemporary societies,
pledge as required by Republic Act No. 1265 of July 11, 1955 and by particularly those societies which require certain practices as manifestations
Department Order No. 8, dated July 21, 1955 issued by the Department of of loyalty and patriotic behavior. Against those who believe that coerced
Education. Aimed primarily at private educational institutions which did not loyalty and unity are mere shadows of patriotism, the tendency to exact "a
observe the flag ceremony exercises, Republic Act No. 1265 penalizes all hydraulic insistence on conformity to majoritarian standards,"11 is seductive
educational institutions for failure or refusal to observe the flag ceremony to the bureaucratic mindset as a shortcut to patriotism.
with public censure on first offense and cancellation of the recognition or
permit on second offense. No doubt, the State possesses what the Solicitor General describes as the
responsibility "to inculcate in the minds of the youth the values of patriotism
The implementing regulations issued by the Department of Education and nationalism and to encourage their involvement in public and civic
thereafter detailed the manner of observance of the same. Immediately affairs." The teaching of these values ranks at the very apex of education's
pursuant to these orders, school officials in Masbate expelled children "high responsibility" of shaping up the minds of the youth in those principles
belonging to the sect of the Jehovah's Witnesses from school for failing or which would mold them into responsible and productive members of our
refusing to comply with the flag ceremony requirement. Sustaining these society. However, the government's interest in molding the young into
expulsion orders, this Court in the 1959 case of Gerona vs. Secretary of patriotic and civic spirited citizens is "not totally free from a balancing
Education3 held that: process"12 when it intrudes into other fundamental rights such as those
specifically protected by the Free Exercise Clause, the constitutional right to
The flag is not an image but a symbol of the Republic of the Philippines, an education and the unassailable interest of parents to guide the religious
emblem of national sovereignty, of national unity and cohesion and of upbringing of their children in accordance with the dictates of their
freedom and liberty which it and the Constitution guarantee and protect. conscience and their sincere religious beliefs.13 Recognizing these values,
Considering the complete separation of church and state in our system of Justice Carolina Grino-Aquino, the writer of the original opinion,
government, the flag is utterly devoid of any religious significance. Saluting underscored that a generation of Filipinos which cuts its teeth on the Bill of
the flag consequently does not involve any religious ceremony. . . . Rights would find abhorrent the idea that one may be compelled, on pain of
expulsion, to salute the flag sing the national anthem and recite the patriotic
After all, the determination of whether a certain ritual is or is not a religious pledge during a flag ceremony.14 "This coercion of conscience has no place
ceremony must rest with the courts. It cannot be left to a religious group or in a free society".15
sect, much less to a follower of said group or sect; otherwise, there would be
confusion and misunderstanding for there might be as many interpretations The State's contentions are therefore, unacceptable, for no less fundamental
and meanings to be given to a certain ritual or ceremony as there are religious than the right to take part is the right to stand apart.16 In the context of the
groups or sects or followers. instant case, the freedom of religion enshrined in the Constitution should be
seen as the rule, not the exception. To view the constitutional guarantee in
the manner suggested by the petitioners would be to denigrate the status of a
preferred freedom and to relegate it to the level of an abstract principle devoid To the extent to which members of the Jehovah's Witnesses sect assiduously
of any substance and meaning in the lives of those for whom the protection pursue their belief in the flag's religious symbolic meaning, the State cannot,
is addressed. As to the contention that the exemption accorded by our without thereby transgressing constitutionally protected boundaries, impose
decision benefits a privileged few, it is enough to re-emphasize that "the the contrary view on the pretext of sustaining a policy designed to foster the
constitutional protection of religious freedom terminated disabilities, it did supposedly far-reaching goal of instilling patriotism among the youth. While
not create new privileges. It gave religious equality, not civil immunity."17 conceding to the idea — adverted to by the Solicitor General — that certain
The essence of the free exercise clause is freedom from conformity to methods of religious expression may be prohibited26 to serve legitimate
religious dogma, not freedom from conformity to law because of religious societal purposes, refusal to participate in the flag ceremony hardly
dogma.18 Moreover, the suggestion implicit in the State's pleadings to the constitutes a form of religious expression so offensive and noxious as to
effect that the flag ceremony requirement would be equally and evenly prompt legitimate State intervention. It is worth repeating that the absence of
applied to all citizens regardless of sect or religion and does not thereby a demonstrable danger of a kind which the State is empowered to protect
discriminate against any particular sect or denomination escapes the fact that militates against the extreme disciplinary methods undertaken by school
"[a] regulation, neutral on its face, may in its application, nonetheless offend authorities in trying to enforce regulations designed to compel attendance in
the constitutional requirement for governmental neutrality if it unduly flag ceremonies. Refusal of the children to participate in the flag salute
burdens the free exercise of religion."19 ceremony would not interfere with or deny the rights of other school children
to do so. It bears repeating that their absence from the ceremony hardly
III constitutes a danger so grave and imminent as to warrant the state's
intervention.
The ostensible interest shown by petitioners in preserving the flag as the
symbol of the nation appears to be integrally related to petitioner's Finally, the respondents' insistence on the validity of the actions taken by the
disagreement with the message conveyed by the refusal of members of the government on the basis of their averment that "a government regulation of
Jehovah's Witness sect to salute the flag or participate actively in flag expressive conduct is sufficiently justified if it is within the constitutional
ceremonies on religious grounds.20 Where the governmental interest clearly power of the government (and) furthers an important and substantial
appears to be unrelated to the suppression of an idea, a religious doctrine or government interest"27 misses the whole point of the test devised by the
practice or an expression or form of expression, this Court will not find it United States Supreme Court in O'Brien, cited by respondent, because the
difficult to sustain a regulation. However, regulations involving this area are Court therein was emphatic in stating that "the government interest (should
generally held against the most exacting standards, and the zone of protection be) unrelated to the suppression of free expression." We have already stated
accorded by the Constitution cannot be violated, except upon a showing of a that the interest in regulation in the case at bench was clearly related to the
clear and present danger of a substantive evil which the state has a right to suppression of an expression directly connected with the freedom of religion
protect.21 Stated differently, in the case of a regulation which appears to and that respondents have not shown to our satisfaction that the restriction
abridge a right to which the fundamental law accords high significance it is was prompted by a compelling interest in public order which the state has a
the regulation, not the act (or refusal to act), which is the exception and which right to protect. Moreover, if we were to refer (as respondents did by referring
requires the court's strictest scrutiny. In the case at bench, the government to the test in O'Brien) to the standards devised by the US Supreme Court in
has not shown that refusal to do the acts of conformity exacted by the assailed determining the validity or extent of restrictive regulations impinging on the
orders, which respondents point out attained legislative cachet in the freedoms of the mind, then the O'Brien standard is hardly appropriate
Administrative Code of 1987, would pose a clear and present danger of a because the standard devised in O'Brien only applies if the State's regulation
danger so serious and imminent, that it would prompt legitimate State is not related to communicative conduct. If a relationship exists, a more
intervention. demanding standard is applied.28

In a case involving the Flag Protection Act of 1989, the U.S. Supreme Court The responsibility of inculcating the values of patriotism, nationalism, good
held that the "State's asserted interest in preserving the fag as a symbol of citizenship, and moral uprightness is a responsibility shared by the State with
nationhood and national unity was an interest related to the suppression of parents and other societal institutions such as religious sects and
free expression . . . because the State's concern with protecting the flag's denominations. The manner in which such values are demonstrated in a
symbolic meaning is implicated only when a person's treatment of the flag plural society occurs in ways so variable that government cannot make
communicates some message. 22 While the very concept of ordered liberty claims to the exclusivity of its methods of inculcating patriotism so all-
precludes this Court from allowing every individual to subjectively define encompassing in scope as to leave no room for appropriate parental or
his own standards on matters of conformity in which society, as a whole has religious influences. Provided that those influences do not pose a clear and
important interests, the records of the case and the long history of flag salute present danger of a substantive evil to society and its institutions, expressions
cases abundantly supports the religious quality of the claims adduced by the of diverse beliefs, no matter how upsetting they may seem to the majority,
members of the sect Jehovah's Witnesses. Their treatment of flag as a are the price we pay for the freedoms we enjoy.
religious symbol is well-founded and well-documented and is based on
grounds religious principle. The message conveyed by their refusal to WHEREFORE, premises considered, the instant Motion is hereby DENIED.
participate in the flag ceremony is religious, shared by the entire community
of Jehovah's Witnesses and is intimately related to their theocratic beliefs and SO ORDERED.
convictions. The subsequent expulsion of members of the sect on the basis
of the regulations assailed in the original petitions was therefore clearly
directed against religious practice. It is obvious that the assailed orders and
memoranda would gravely endanger the free exercise of the religious beliefs
of the members of the sect and their minor children.

Furthermore, the view that the flag is not a religious but a neutral, secular
symbol expresses a majoritarian view intended to stifle the expression of
the belief that an act of saluting the flag might sometimes be — to some
individuals — so offensive as to be worth their giving up another
constitutional right — the right to education. Individuals or groups of
individuals get from a symbol the meaning they put to it.23 Compelling
members of a religious sect to believe otherwise on the pain of denying minor
children the right to an education is a futile and unconscionable detour
towards instilling virtues of loyalty and patriotism which are best instilled
and communicated by painstaking and non-coercive methods. Coerced
loyalties, after all, only serve to inspire the opposite. The methods utilized to
impose them breed resentment and dissent. Those who attempt to coerce
uniformity of sentiment soon find out that the only path towards achieving
unity is by way of suppressing dissent.24 In the end, such attempts only find
the "unanimity of the graveyard."25
SANIDAD vs. THE COMMISSION ON ELECTIONS It is stated further by respondent that Resolution 2167 does not absolutely
G.R. No. 90878 January 29, 1990 bar petitioner from expressing his views and/or from campaigning for or
MEDIALDEA, J.: against the Organic Act. He may still express his views or campaign for or
against the act through the Comelec space and airtime. This is provided under
This is a petition for certiorari assailing the constitutionality of Section 19 of Sections 90 and 92 of BP 881:
Comelec Resolution No. 2167 on the ground that it violates the constitutional
guarantees of the freedom of expression and of the press. Section 90. Comelec Space. — Commission shall procure space in at least
one newspaper of general circulation in every province or city: Provided,
On October 23, 1989, Republic Act No. 6766, entitled "AN ACT however, That in the absence of said newspaper, publication shall be done in
PROVIDING FOR AN ORGANIC ACT FOR THE CORDILLERA any other magazine or periodical in said province or city, which shall be
AUTONOMOUS REGION" was enacted into law. Pursuant to said law, the known as "Comelec Space" wherein candidates can announce their
City of Baguio and the Cordilleras which consist of the provinces of Benguet, candidacy. Said space shall be allocated, free of charge equally and
Mountain Province, Ifugao, Abra and Kalinga-Apayao, all comprising the impartially within the area in which the newspaper is circulated.
Cordillera Autonomous Region, shall take part in a plebiscite for the
ratification of said Organic Act originally scheduled last December 27, 1989 Section 92. Comelec Time. — The Commission shall procure radio and
which was, however, reset to January 30, 1990 by virtue of Comelec television time to be known as "Comelec Time" which shall be allocated
Resolution No. 2226 dated December 27, 1989. equally and impartially among the candidates within the area of coverage of
all radio and television stations. For this purpose, the franchise of all radio
The Commission on Elections, by virtue of the power vested by the 1987 broadcasting and television stations are hereby amended so as to provide
Constitution, the Omnibus Election Code (BP 881), said R.A. 6766 and other radio or television time, free of charge, during the period of the campaign.
pertinent election laws, promulgated Resolution No. 2167, to govern the
conduct of the plebiscite on the said Organic Act for the Cordillera Respondent Comelec has relied much on Article IX-C of the 1987
Autonomous Region. Constitution and Section 11 of R.A. 6646 as the basis for the promulgation
of the questioned Section 19 of Comelec Resolution 2167.
In a petition dated November 20, 1989, herein petitioner Pablito V. Sanidad,
who claims to be a newspaper columnist of the "OVERVIEW" for the Article IX-C of the 1987 Constitution provides:
BAGUIO MIDLAND COURIER, a weekly newspaper circulated in the City
of Baguio and the Cordilleras, assailed the constitutionality of Section 19 of The Commission may, during the election period, supervise or regulate the
Comelec Resolution No. 2167, which provides: enjoyment or utilization of all franchises or permits for the operation of
transportation and other public utilities, media of communication or
Section 19. Prohibition on columnists, commentators or announcers. — information, all grants, special privileges, or concessions granted by the
During the plebiscite campaign period, on the day before and on the Government or any subdivision, agency or instrumentality thereof, including
plebiscite day, no mass media columnist, commentator, announcer or any government-owned or controlled corporation or its subsidiary. Such
personality shall use his column or radio or television time to campaign for supervision or regulation shall aim to ensure equal opportunity, time, and
or against the plebiscite issues. space, and the right to reply, including reasonable, equal rates therefor, for
public information campaigns and forums among candidates in connection
It is alleged by petitioner that said provision is void and unconstitutional with the objective of holding free, orderly, honest, peaceful and credible
because it violates the constitutional guarantees of the freedom of expression elections.
and of the press enshrined in the Constitution.
Similarly, Section 11 of Republic Act No. 6646 (The Electoral Reform Law
Unlike a regular news reporter or news correspondent who merely reports of 1987) likewise provides:
the news, petitioner maintains that as a columnist, his column obviously and
necessarily contains and reflects his opinions, views and beliefs on any issue Prohibited forms of election Propaganda. — In addition to the forms of
or subject about which he writes. Petitioner believes that said provision of election propaganda prohibited under Section 85 of Batas Pambansa Blg.
COMELEC Resolution No. 2167 constitutes a prior restraint on his 881, it shall be unlawful: ...
constitutionally-guaranteed freedom of the press and further imposes
subsequent punishment for those who may violate it because it contains a (b) for any newspaper, radio, broadcasting or television station, or other mass
penal provision, as follows: media, or any person making use of the mass media to sell or to give free of
charge print space or air time for campaign or other political purposes except
Article XIII, Section 122, Election Offenses and Banned Acts or Activities. to the Commission as provided under Sections 90 and 92 of Batas Pambansa
— Except to the extent that the same may not be applicable plebiscite. the Blg. 881. Any mass media columnist, commentator, announcer, or
banned acts/activities and offenses defined in and penalized by the Omnibus personality who is a candidate for any elective office shall take a leave of
Election Code ('Sections 261, 262, 263 and Article' XXII, B.P. Blg. 881) and absence from his work as such during the campaign period. (Emphasis ours)
the pertinent provisions of R.A. No. 6646 shall be aplicable to the plebiscite
governed by this Resolution. However, it is clear from Art. IX-C of the 1987 Constitution that what was
granted to the Comelec was the power to supervise and regulate the use and
Petitioner likewise maintains that if media practitioners were to express their enjoyment of franchises, permits or other grants issued for the operation of
views, beliefs and opinions on the issue submitted to a plebiscite, it would in transportation or other public utilities, media of communication or
fact help in the government drive and desire to disseminate information, and information to the end that equal opportunity, time and space, and the right
hear, as well as ventilate, all sides of the issue. to reply, including reasonable, equal rates therefor, for public information
campaigns and forums among candidates are ensured. The evil sought to be
On November 28, 1989, We issued a temporary restraining order enjoining prevented by this provision is the possibility that a franchise holder may favor
respondent Commission on Elections from enforcing and implementing or give any undue advantage to a candidate in terms of advertising space or
Section 19 of Resolution No. 2167. We also required the respondent to radio or television time. This is also the reason why a "columnist,
comment on the petition. commentator, announcer or personality, who is a candidate for any elective
office is required to take a leave of absence from his work during the
On January 9, 1990, respondent Commission on Elections, through the campaign period (2nd par. Section 11(b) R.A. 6646). It cannot be gainsaid
Office of the Solicitor General filed its Comment. that a columnist or commentator who is also a candidate would be more
exposed to the voters to the prejudice of other candidates unless required to
Respondent Comelec maintains that the questioned provision of Comelec take a leave of absence.
Resolution No. 2167 is not violative of the constitutional guarantees of the
freedom of expression and of the press. Rather it is a valid implementation However, neither Article IX-C of the Constitution nor Section 11 (b), 2nd
of the power of the Comelec to supervise and regulate media during election par. of R.A. 6646 can be construed to mean that the Comelec has also been
or plebiscite periods as enunciated in Article IX-C, Section 4 of the 1987 granted the right to supervise and regulate the exercise by media practitioners
Constitution of the Republic of the Philippines. themselves of their right to expression during plebiscite periods. Media
practitioners exercising their freedom of expression during plebiscite periods
are neither the franchise holders nor the candidates. In fact, there are no
candidates involved in a plebiscite. Therefore, Section 19 of Comelec
Resolution No. 2167 has no statutory basis.

In the case of Badoy, Jr. v. Comelec, L-32546, Oct. 16, 1970, where the
constitutionality of the prohibition of certain forms of election propaganda
was assailed, We ruled therein that the prohibition is a valid exercise of the
police power of the state "to prevent the perversion and prostitution of the
electoral apparatus and of the denial of equal protection of the laws." The
evil sought to be prevented in an election which led to Our ruling in that case
does not obtain in a plebiscite. In a plebiscite, votes are taken in an area on
some special political matter unlike in an election where votes are cast in
favor of specific persons for some office. In other words, the electorate is
asked to vote for or against issues, not candidates in a plebiscite.

Anent respondent Comelec's argument that Section 19 of Comelec


Resolution 2167 does not absolutely bar petitioner-columnist from
expressing his views and/or from campaigning for or against the organic act
because he may do so through the Comelec space and/or Comelec
radio/television time, the same is not meritorious. While the limitation does
not absolutely bar petitioner's freedom of expression, it is still a restriction
on his choice of the forum where he may express his view. No reason was
advanced by respondent to justify such abridgement. We hold that this form
of regulation is tantamount to a restriction of petitioner's freedom of
expression for no justifiable reason.

Plebiscite issues are matters of public concern and importance. The people's
right to be informed and to be able to freely and intelligently make a decision
would be better served by access to an unabridged discussion of the issues,
including the forum. The people affected by the issues presented in a
plebiscite should not be unduly burdened by restrictions on the forum where
the right to expression may be exercised. Comelec spaces and Comelec radio
time may provide a forum for expression but they do not guarantee full
dissemination of information to the public concerned because they are
limited to either specific portions in newspapers or to specific radio or
television times.

ACCORDINGLY, the instant petition is GRANTED. Section 19 of Comelec


Resolution No. 2167 is declared null and void and unconstitutional. The
restraining order herein issued is hereby made permanent.

SO ORDERED.
AYER PRODUCTIONS PTY. LTD. vs. CAPULONG The six hour series is a McElroy and McElroy co-production with Home Box
G.R. No. 82380 April 29, 1988 Office in American, the Australian Broadcast Corporation in Australia and
FELICIANO, J.: Zenith Productions in the United Kingdom

Petitioner Hal McElroy an Australian film maker, and his movie production The proposed motion picture would be essentially a re-enact. ment of the
company, Petitioner Ayer Productions pty Ltd. (Ayer Productions), 1 events that made possible the EDSA revolution; it is designed to be viewed
envisioned, sometime in 1987, the for commercial viewing and for Philippine in a six-hour mini-series television play, presented in a "docu-drama" style,
and international release, the histolic peaceful struggle of the Filipinos at creating four (4) fictional characters interwoven with real events, and
EDSA (Epifanio de los Santos Avenue). Petitioners discussed this Project utilizing actual documentary footage as background.
with local movie producer Lope V. Juban who suggested th they consult with
the appropriate government agencies and also with General Fidel V. Ramos On 21 December 1987, private respondent Enrile replied that "[he] would not
and Senator Juan Ponce Enrile, who had played major roles in the events and will not approve of the use, appropriation, reproduction and/or exhibition
proposed to be filmed. of his name, or picture, or that of any member of his family in any cinema or
television production, film or other medium for advertising or commercial
The proposed motion picture entitled "The Four Day Revolution" was exploitation" and further advised petitioners that 'in the production, airing,
endorsed by the Movie Television Review and Classification Board as wel showing, distribution or exhibition of said or similar film, no reference
as the other government agencies consulted. General Fidel Ramos also whatsoever (whether written, verbal or visual) should not be made to [him]
signified his approval of the intended film production. or any member of his family, much less to any matter purely personal to
them.
In a letter dated 16 December 1987, petitioner Hal McElroy informed private
respondent Juan Ponce Enrile about the projected motion picture enclosing a It appears that petitioners acceded to this demand and the name of private
synopsis of it, the full text of which is set out below: respondent Enrile was deleted from the movie script, and petitioners
proceeded to film the projected motion picture.
The Four Day Revolution is a six hour mini-series about People Power—a
unique event in modern history that-made possible the Peaceful revolution in On 23 February 1988, private respondent filed a Complaint with application
the Philippines in 1986. for Temporary Restraining Order and Wilt of Pretion with the Regional Trial
Court of Makati, docketed as Civil Case No. 88-151 in Branch 134 thereof,
Faced with the task of dramatising these rerkble events, screenwriter David seeking to enjoin petitioners from producing the movie "The Four Day
Williamson and history Prof Al McCoy have chosen a "docu-drama" style Revolution". The complaint alleged that petitioners' production of the mini-
and created [four] fictitious characters to trace the revolution from the death series without private respondent's consent and over his objection, constitutes
of Senator Aquino, to the Feb revolution and the fleeing of Marcos from the an obvious violation of his right of privacy. On 24 February 1988, the trial
country. court issued ex-parte a Temporary Restraining Order and set for hearing the
application for preliminary injunction.
These character stories have been woven through the real events to help our
huge international audience understand this ordinary period inFilipino On 9 March 1988, Hal McElroy flied a Motion to Dismiss with Opposition
history. to the Petition for Preliminary Injunction contending that the mini-series fim
would not involve the private life of Juan Ponce Enrile nor that of his family
First, there's Tony O'Neil, an American television journalist working for and that a preliminary injunction would amount to a prior restraint on their
major network. Tony reflects the average American attitude to the right of free expression. Petitioner Ayer Productions also filed its own
Phihppinence —once a colony, now the home of crucially important military Motion to Dismiss alleging lack of cause of action as the mini-series had not
bases. Although Tony is aware of the corruption and of Marcos' yet been completed.
megalomania, for him, there appears to be no alternative to Marcos except
the Communists. In an Order 2 dated 16 March 1988, respondent court issued a writ of
Preliminary Injunction against the petitioners, the dispositive portion of
Next, Angie Fox a fiery Australian photo-journalist. A 'new girl in town,' she which reads thus:
is quickly caught up in the events as it becomes dear that the time has come
for a change. Through Angle and her relationship with one of the Reform WHEREFORE, let a writ of preliminary injunction be issued, ordering
Army Movement Colonels (a fictitious character), we follow the developing defendants, and all persons and entities employed or under contract with
discontent in the armed forces. Their dislike for General Ver, their strong them, including actors, actresses and members of the production staff and
loyalty to Defense Minister Enrile, and ultimately their defection from crew as well as all persons and entities acting on defendants' behalf, to cease
Marcos. and desist from producing and filming the mini-series entitled 'The Four Day
Revolution" and from making any reference whatsoever to plaintiff or his
The fourth fictitious character is Ben Balano, a middle-aged editor of a family and from creating any fictitious character in lieu of plaintiff which
Manila newspaper who despises the Marcos regime and is a supporter an nevertheless is based on, or bears rent substantial or marked resemblance or
promoter of Cory Aquino. Ben has two daughters, Cehea left wing lawyer similarity to, or is otherwise Identifiable with, plaintiff in the production and
who is a secret member of the New People's Army, and Eva--a -P.R. girl, any similar film or photoplay, until further orders from this Court, upon
politically moderate and very much in love with Tony. Ultimately, she must plaintiff's filing of a bond in the amount of P 2,000,000.00, to answer for
choose between her love and the revolution. whatever damages defendants may suffer by reason of the injunction if the
Court should finally decide that plaintiff was not entitled thereto.
Through the interviews and experiences of these central characters, we show
the complex nature of Filipino society, and thintertwining series of events xxx xxx xxx
and characters that triggered these remarkable changes. Through them also,
we meet all of the principal characters and experience directly dramatic (Emphasis supplied)
recreation of the revolution. The story incorporates actual documentary
footage filmed during the period which we hope will capture the unique On 22 March 1988, petitioner Ayer Productions came to this Court by a
atmosphere and forces that combined to overthrow President Marcos. Petition for certiorari dated 21 March 1988 with an urgent prayer for
Preliminary Injunction or Restraining Order, which petition was docketed as
David Williamson is Australia's leading playwright with some 14 hugely G.R. No. L-82380.
successful plays to his credit(Don's Party,' 'The Club,' Travelling North) and
11 feature films (The Year of Living Dangerously,' Gallipoli,' 'Phar Lap'). A day later, or on 23 March 1988, petitiioner Hal McElroy also filed separate
Petition for certiorari with Urgent Prayer for a Restraining Order or
Professor McCoy (University of New South Wales) is an American historian Preliminary Injunction, dated 22 March 1988, docketed as G.R. No. L-82398.
with a deep understanding of the Philippines, who has worked on the research
for this project for some 18 months. Together with Davi Wilhamgon they By a Resolution dated 24 March 1988, the petitions were consolidated and
have developed a script we believe accurately depicts the complex issues and private respondent was required to file a consolidated Answer. Further, in the
events that occurred during th period . same Resolution, the Court granted a Temporary Restraining Order partially
enjoining the implementation of the respondent Judge's Order of 16 March
1988 and the Writ of Preliminary Injunction issued therein, and allowing the his men were tried and convicted. 11 In the judgment of the lower court
petitioners to resume producing and filming those portions of the projected enforcing the licensing agreement against the licensee who had produced the
mini-series which do not make any reference to private respondent or his motion picture and exhibited it but refused to pay the stipulated royalties, the
family or to any fictitious character based on or respondent. Court, through Justice Melencio-Herrera, said:

Private respondent seasonably filed his Consolidated Answer on 6 April 1988 Neither do we agree with petitioner's subon that the Licensing Agreement is
invoking in the main a right of privacy. null and void for lack of, or for having an illegal cause or consideration, while
it is true that petitioner bad pled the rights to the book entitled "The Moises
I Padilla Story," that did not dispense with the need for prior consent and
authority from the deceased heirs to portray publicly episodes in said
The constitutional and legal issues raised by the present Petitions are sharply deceased's life and in that of his mother and the member of his family. As
drawn. Petitioners' claim that in producing and "The Four Day Revolution," held in Schuyler v. Curtis, ([1895],147 NY 434,42 NE 31 LRA 286.49 Am
they are exercising their freedom of speech and of expression protected under St Rep 671), 'a privilege may be given the surviving relatives of a deperson
our Constitution. Private respondent, upon the other hand, asserts a right of to protect his memory, but the privilege wts for the benefit of the living, to
privacy and claims that the production and filming of the projected mini- protect their feelings and to preventa violation of their own rights in the
series would constitute an unlawful intrusion into his privacy which he is character and memory of the deceased.'
entitled to enjoy.
Petitioners averment that private respondent did not have any property right
Considering first petitioners' claim to freedom of speech and of expression over the life of Moises Padilla since the latter was a public figure, is neither
the Court would once more stress that this freedom includes the freedom to well taken. Being a public figure ipso facto does not automatically destroy in
film and produce motion pictures and to exhibit such motion pictures in toto a person's right to privacy. The right to invade a person's privacy to
theaters or to diffuse them through television. In our day and age, motion disseminate public information does not extend to a fictional or novelized
pictures are a univesally utilized vehicle of communication and medium Of representation of a person, no matter how public a he or she may be (Garner
expression. Along with the press, radio and television, motion pictures v. Triangle Publications, DCNY 97 F. Supp., SU 549 [1951]). In the case at
constitute a principal medium of mass communication for information, bar, while it is true that petitioner exerted efforts to present a true-to-life Story
education and entertainment. In Gonzales v. Katigbak, 3 former Chief Justice Of Moises Padilla, petitioner admits that he included a little romance in the
Fernando, speaking for the Court, explained: film because without it, it would be a drab story of torture and brutality. 12

1. Motion pictures are important both as a medium for the communication of In Lagunzad, the Court had need, as we have in the instant case, to deal with
Ideas and the expression of the artistic impulse. Their effect on the perception contraposed claims to freedom of speech and of expression and to privacy.
by our people of issues and public officials or public figures as well as the Lagunzad the licensee in effect claimed, in the name of freedom of speech
pre cultural traits is considerable. Nor as pointed out in Burstyn v. Wilson and expression, a right to produce a motion picture biography at least partly
(343 US 495 [19421) is the Importance of motion pictures as an organ of "fictionalized" of Moises Padilla without the consent of and without paying
public opinion lessened by the fact that they are designed to entertain as well pre-agreed royalties to the widow and family of Padilla. In rejecting the
as to inform' (Ibid, 501). There is no clear dividing line between what licensee's claim, the Court said:
involves knowledge and what affords pleasure. If such a distinction were
sustained, there is a diminution of the basic right to free expression. ...4 Lastly, neither do we find merit in petitioners contention that the Licensing
Agreement infringes on the constitutional right of freedom of speech and of
This freedom is available in our country both to locally-owned and to the press, in that, as a citizen and as a newspaperman, he had the right to
foreign-owned motion picture companies. Furthermore the circumstance that express his thoughts in film on the public life of Moises Padilla without prior
the production of motion picture films is a commercial activity expected to restraint.The right freedom of expression, indeed, occupies a preferred
yield monetary profit, is not a disqualification for availing of freedom of position in the "hierarchy of civil liberties" (Philippine Blooming Mills
speech and of expression. In our community as in many other countries, Employees Organization v. Philippine Blooming Mills Co., Inc., 51 SCRA
media facilities are owned either by the government or the private sector but 191 [1963]). It is not, however, without limitations. As held in Gonzales v.
the private sector-owned media facilities commonly require to be sustained Commission on Elections, 27 SCRA 835, 858 [1960]:
by being devoted in whole or in pailt to revenue producing activities. Indeed,
commercial media constitute the bulk of such facilities available in our xxx xxx xxx
country and hence to exclude commercially owned and operated media from
the exerciseof constitutionally protected om of speech and of expression can The prevailing doctine is that the clear and present danger rule is such a
only result in the drastic contraction of such constitutional liberties in our limitation. Another criterion for permissible limitation on freedom of speech
country. and the press, which includes such vehicles of the mass media as radio,
television and the movies, is the "balancing of interest test" (Chief Justice
The counter-balancing of private respondent is to a right of privacy. It was Enrique M. Fernando on the Bill of Rights, 1970 ed. p. 79). The principle
demonstrated sometime ago by the then Dean Irene R. Cortes that our law, "requires a court to take conscious and detailed consideration of the interplay
constitutional and statutory, does include a right of privacy. 5 It is left to case of interests observable in given situation or type of situation" (Separation
law, however, to mark out the precise scope and content of this right in Opinion of the late Chief Justice Castro in Gonzales v. Commission on
differing types of particular situations. The right of privacy or "the right to Elections, supra, p. 899).
be let alone," 6 like the right of free expression, is not an absolute right. A
limited intrusion into a person's privacy has long been regarded as In the case at bar, the interests observable are the right to privacy asserted by
permissible where that person is a public figure and the information sought respondent and the right of freedom of expression invoked by petitioner.
to be elicited from him or to be published about him constitute of apublic taking into account the interplay of those interests, we hold that under the
character. 7 Succinctly put, the right of privacy cannot be invoked resist particular circumstances presented, and considering the obligations assumed
publication and dissemination of matters of public interest. 8 The interest in the Licensing Agreement entered into by petitioner, the validity of such
sought to be protected by the right of privacy is the right to be free from agreement will have to be upheld particularly because the limits of freedom
unwarranted publicity, from the wrongful publicizing of the private affairs of expression are reached when expression touches upon matters of
and activities of an individual which are outside the realm of legitimate essentially private concern." 13
public concern. 9
Whether the "balancing of interests test" or the clear and present danger test"
Lagunzad v. Vda. de Gonzales, 10 on which private respondent relies be applied in respect of the instant Petitions, the Court believes that a
heavily, recognized a right to privacy in a context which included a claim to different conclusion must here be reached: The production and filming by
freedom of speech and of expression. Lagunzad involved a suit fortion petitioners of the projected motion picture "The Four Day Revolution" does
picture producer as licensee and the widow and family of the late Moises not, in the circumstances of this case, constitute an unlawful intrusion upon
Padilla as licensors. This agreement gave the licensee the right to produce a private respondent's "right of privacy."
motion Picture Portraying the life of Moises Padilla, a mayoralty candidate
of the Nacionalista Party for the Municipality of Magallon, Negros 1. It may be observed at the outset that what is involved in the instant case is
Occidental during the November 1951 elections and for whose murder, a prior and direct restraint on the part of the respondent Judge upon the
Governor Rafael Lacson, a member of the Liberal Party then in power and exercise of speech and of expression by petitioners. The respondent Judge
has restrained petitioners from filming and producing the entire proposed information to tell it. "News" includes all events and items of information
motion picture. It is important to note that in Lagunzad, there was no prior which are out of the ordinary hum-drum routine, and which have 'that
restrain of any kind imposed upon the movie producer who in fact completed indefinable quality of information which arouses public attention.' To a very
and exhibited the film biography of Moises Padilla. Because of the speech great extent the press, with its experience or instinct as to what its readers
and of expression, a weighty presumption of invalidity vitiates. 14 The will want, has succeeded in making its own definination of news, as a glance
invalidity of a measure of prior restraint doesnot, of course, mean that no at any morning newspaper will sufficiently indicate. It includes homicide and
subsequent liability may lawfully be imposed upon a person claiming to othe crimes, arrests and police raides, suicides, marriages and divorces,
exercise such constitutional freedoms. The respondent Judge should have accidents, a death from the use of narcotics, a woman with a rare disease, the
stayed his hand, instead of issuing an ex-parte Temporary Restraining Order birth of a child to a twelve year old girl, the reappearance of one supposed to
one day after filing of a complaint by the private respondent and issuing a have been murdered years ago, and undoubtedly many other similar matters
Preliminary Injunction twenty (20) days later; for the projected motion of genuine, if more or less deplorable, popular appeal.
picture was as yet uncompleted and hence not exhibited to any audience.
Neither private respondent nor the respondent trial Judge knew what the The privilege of enlightening the public was not, however, limited, to the
completed film would precisely look like. There was, in other words, no dissemination of news in the scene of current events. It extended also to
"clear and present danger" of any violation of any right to privacy that private information or education, or even entertainment and amusement, by books,
respondent could lawfully assert. articles, pictures, films and broadcasts concerning interesting phases of
human activity in general, as well as the reproduction of the public scene in
2. The subject matter of "The Four Day Revolution" relates to the non-bloody newsreels and travelogues. In determining where to draw the line, the courts
change of government that took place at Epifanio de los Santos Avenue in were invited to exercise a species of censorship over what the public may be
February 1986, and the trian of events which led up to that denouement. permitted to read; and they were understandably liberal in allowing the
Clearly, such subject matter is one of public interest and concern. Indeed, it benefit of the doubt. 15
is, petitioners' argue, of international interest. The subject thus relates to a
highly critical stage in the history of this countryand as such, must be Private respondent is a "public figure" precisely because, inter alia, of his
regarded as having passed into the public domain and as an appropriate participation as a principal actor in the culminating events of the change of
subject for speech and expression and coverage by any form of mass media. government in February 1986. Because his participation therein was major
The subject mater, as set out in the synopsis provided by the petitioners and in character, a film reenactment of the peaceful revolution that fails to make
quoted above, does not relate to the individual life and certainly not to the reference to the role played by private respondent would be grossly
private life of private respondent Ponce Enrile. Unlike in Lagunzad, which unhistorical. The right of privacy of a "public figure" is necessarily narrower
concerned the life story of Moises Padilla necessarily including at least his than that of an ordinary citizen. Private respondent has not retired into the
immediate family, what we have here is not a film biography, more or less seclusion of simple private citizenship. he continues to be a "public figure."
fictionalized, of private respondent Ponce Enrile. "The Four Day Revolution" After a successful political campaign during which his participation in the
is not principally about, nor is it focused upon, the man Juan Ponce Enrile' EDSA Revolution was directly or indirectly referred to in the press, radio and
but it is compelled, if it is to be historical, to refer to the role played by Juan television, he sits in a very public place, the Senate of the Philippines.
Ponce Enrile in the precipitating and the constituent events of the change of
government in February 1986. 5. The line of equilibrium in the specific context of the instant case between
the constitutional freedom of speech and of expression and the right of
3. The extent of the instrusion upon the life of private respondent Juan Ponce privacy, may be marked out in terms of a requirement that the proposed
Enrile that would be entailed by the production and exhibition of "The Four motion picture must be fairly truthful and historical in its presentation of
Day Revolution" would, therefore, be limited in character. The extent of that events. There must, in other words, be no knowing or reckless disregard of
intrusion, as this Court understands the synopsis of the proposed film, may truth in depicting the participation of private respondent in the EDSA
be generally described as such intrusion as is reasonably necessary to keep Revolution. 16 There must, further, be no presentation of the private life of
that film a truthful historical account. Private respondent does not claim that the unwilling private respondent and certainly no revelation of intimate or
petitioners threatened to depict in "The Four Day Revolution" any part of the embarrassing personal facts. 17 The proposed motion picture should not
private life of private respondent or that of any member of his family. enter into what Mme. Justice Melencio-Herrera in Lagunzad referred to as
"matters of essentially private concern." 18 To the extent that "The Four Day
4. At all relevant times, during which the momentous events, clearly of public Revolution" limits itself in portraying the participation of private respondent
concern, that petitioners propose to film were taking place, private in the EDSA Revolution to those events which are directly and reasonably
respondent was what Profs. Prosser and Keeton have referred to as a "public related to the public facts of the EDSA Revolution, the intrusion into private
figure:" respondent's privacy cannot be regarded as unreasonable and actionable.
Such portrayal may be carried out even without a license from private
A public figure has been defined as a person who, by his accomplishments, respondent.
fame, or mode of living, or by adopting a profession or calling which gives
the public a legitimate interest in his doings, his affairs, and his character, has II
become a 'public personage.' He is, in other words, a celebrity. Obviously to
be included in this category are those who have achieved some degree of In a Manifestation dated 30 March 1988, petitioner Hal McElroy informed
reputation by appearing before the public, as in the case of an actor, a this Court that a Temporary Restraining Order dated 25 March 1988, was
professional baseball player, a pugilist, or any other entertainment. The list issued by Judge Teofilo Guadiz of the Regional Trial Court of Makati,
is, however, broader than this. It includes public officers, famous inventors Branch 147, in Civil Case No. 88-413, entitled "Gregorio B. Honasan vs.
and explorers, war heroes and even ordinary soldiers, an infant prodigy, and Ayer Productions Pty. Ltd., McElroy Film Productions, Hal McElroy, Lope
no less a personage than the Grand Exalted Ruler of a lodge. It includes, in Juban and PMP Motion for Pictures Production" enjoining him and his
short, anyone who has arrived at a position where public attention is focused production company from further filimg any scene of the projected mini-
upon him as a person. series film. Petitioner alleged that Honasan's complaint was a "scissors and
paste" pleading, cut out straight grom the complaint of private respondent
Such public figures were held to have lost, to some extent at least, their tight Ponce Enrile in Civil Case No. 88-151. Petitioner Ayer Productions, in a
to privacy. Three reasons were given, more or less indiscrimately, in the separate Manifestation dated 4 April 1988, brought to the attention of the
decisions" that they had sought publicity and consented to it, and so could Court the same information given by petitoner Hal McElroy, reiterating that
not complaint when they received it; that their personalities and their affairs the complaint of Gregorio B. Honasan was substantially identical to that filed
has already public, and could no longer be regarded as their own private by private respondent herein and stating that in refusing to join Honasan in
business; and that the press had a privilege, under the Constitution, to inform Civil Case No. 88-151, counsel for private respondent, with whom counsel
the public about those who have become legitimate matters of public interest. for Gregorio Honasan are apparently associated, deliberately engaged in
On one or another of these grounds, and sometimes all, it was held that there "forum shopping."
was no liability when they were given additional publicity, as to matters
legitimately within the scope of the public interest they had aroused. Private respondent filed a Counter-Manifestation on 13 April 1988 stating
that the "slight similarity" between private respondent's complaint and that
The privilege of giving publicity to news, and other matters of public interest, on Honasan in the construction of their legal basis of the right to privacy as
was held to arise out of the desire and the right of the public to know what is a component of the cause of action is understandable considering that court
going on in the world, and the freedom of the press and other agencies of pleadings are public records; that private respondent's cause of action for
invasion of privacy is separate and distinct from that of Honasan's although
they arose from the same tortious act of petitioners' that the rule on
permissive joinder of parties is not mandatory and that, the cited cases on
"forum shopping" were not in point because the parties here and those in
Civil Case No. 88-413 are not identical.

For reasons that by now have become clear, it is not necessary for the Court
to deal with the question of whether or not the lawyers of private respondent
Ponce Enrile have engaged in "forum shopping." It is, however, important to
dispose to the complaint filed by former Colonel Honasan who, having
refused to subject himself to the legal processes of the Republic and having
become once again in fugitive from justice, must be deemed to have forfeited
any right the might have had to protect his privacy through court processes.

WHEREFORE,

a) the Petitions for Certiorari are GRANTED DUE COURSE, and the Order
dated 16 March 1988 of respondent trial court granting a Writ of Preliminary
Injunction is hereby SET ASIDE. The limited Temporary Restraining Order
granted by this Court on 24 March 1988 is hereby MODIFIED by enjoining
unqualifiedly the implementation of respondent Judge's Order of 16 March
1988 and made PERMANENT, and

b) Treating the Manifestations of petitioners dated 30 March 1988 and 4


April 1988 as separate Petitions for Certiorari with Prayer for Preliminary
Injunction or Restraining Order, the Court, in the exercise of its plenary and
supervisory jurisdiction, hereby REQUIRES Judge Teofilo Guadiz of the
Regional Trial Court of Makati, Branch 147, forthwith to DISMISS Civil
Case No. 88-413 and accordingly to SET ASIDE and DISSOLVE his
Temporary Restraining Order dated 25 March 1988 and any Preliminary
Injunction that may have been issued by him.

No pronouncement as to costs.

SO ORDERED.
VALMONTE vs. BELMONTE, JR. My opinion in this regard is that a confidential relationship exists between
G.R. No. 74930 February 13, 1989 the GSIS and all those who borrow from it, whoever they may be; that the
CORTES, J.: GSIS has a duty to its customers to preserve this confidentiality; and that it
would not be proper for the GSIS to breach this confidentiality unless so
Petitioners in this special civil action for mandamus with preliminary ordered by the courts.
injunction invoke their right to information and pray that respondent be
directed: As a violation of this confidentiality may mar the image of the GSIS as a
reputable financial institution, I regret very much that at this time we cannot
respond positively to your request.

(a) to furnish petitioners the list of the names of the Batasang Pambansa Very truly yours,
members belonging to the UNIDO and PDP-Laban who were able to secure
clean loans immediately before the February 7 election thru the (Sgd.) MEYNARDO A. TIRO
intercession/marginal note of the then First Lady Imelda Marcos; and/or Deputy General Counsel
[Rollo, p. 40.]
(b) to furnish petitioners with certified true copies of the documents
evidencing their respective loans; and/or On June 20, 1986, apparently not having yet received the reply of the
Government Service and Insurance System (GSIS) Deputy General Counsel,
(c) to allow petitioners access to the public records for the subject petitioner Valmonte wrote respondent another letter, saying that for failure
information. (Petition, pp. 4-5; paragraphing supplied.] to receive a reply, "(W)e are now considering ourselves free to do whatever
action necessary within the premises to pursue our desired objective in
The controversy arose when petitioner Valmonte wrote respondent Belmonte pursuance of public interest." [Rollo, p. 8.]
the following letter:
On June 26, 1986, Valmonte, joined by the other petitioners, filed the instant
June 4, 1986 suit.

Hon. Feliciano Belmonte On July 19, 1986, the Daily Express carried a news item reporting that 137
GSIS General Manager former members of the defunct interim and regular Batasang Pambansa,
Arroceros, Manila including ten (10) opposition members, were granted housing loans by the
GSIS [Rollo, p. 41.]
Sir:
Separate comments were filed by respondent Belmonte and the Solicitor
As a lawyer, member of the media and plain citizen of our Republic, I am General. After petitioners filed a consolidated reply, the petition was given
requesting that I be furnished with the list of names of the opposition due course and the parties were required to file their memoranda. The parties
members of (the) Batasang Pambansa who were able to secure a clean loan having complied, the case was deemed submitted for decision.
of P2 million each on guarranty (sic) of Mrs. Imelda Marcos. We understand
that OIC Mel Lopez of Manila was one of those aforesaid MPs. Likewise, In his comment respondent raises procedural objections to the issuance of a
may we be furnished with the certified true copies of the documents writ of mandamus, among which is that petitioners have failed to exhaust
evidencing their loan. Expenses in connection herewith shall be borne by us. administrative remedies.

If we could not secure the above documents could we have access to them? Respondent claims that actions of the GSIS General Manager are reviewable
by the Board of Trustees of the GSIS. Petitioners, however, did not seek relief
We are premising the above request on the following provision of the from the GSIS Board of Trustees. It is therefore asserted that since
Freedom Constitution of the present regime. administrative remedies were not exhausted, then petitioners have no cause
of action.
The right of the people to information on matters of public concern shall be
recognized. Access to official records, and to documents and papers To this objection, petitioners claim that they have raised a purely legal issue,
pertaining to official acts, transactions or decisions, shall be afforded the viz., whether or not they are entitled to the documents sought, by virtue of
citizen subject to such limitation as may be provided by law. (Art. IV, Sec. their constitutional right to information. Hence, it is argued that this case falls
6). under one of the exceptions to the principle of exhaustion of administrative
remedies.
We trust that within five (5) days from receipt hereof we will receive your
favorable response on the matter. Among the settled principles in administrative law is that before a party can
be allowed to resort to the courts, he is expected to have exhausted all means
Very truly yours, of administrative redress available under the law. The courts for reasons of
law, comity and convenience will not entertain a case unless the available
(Sgd.) RICARDO C. VALMONTE administrative remedies have been resorted to and the appropriate authorities
have been given opportunity to act and correct the errors committed in the
[Rollo, p. 7.] administrative forum. However, the principle of exhaustion of administrative
remedies is subject to settled exceptions, among which is when only a
To the aforesaid letter, the Deputy General Counsel of the GSIS replied: question of law is involved [Pascual v. Provincial Board, 106 Phil. 466
(1959); Aguilar v. Valencia, et al., G.R. No. L-30396, July 30, 1971, 40
June 17, 1986 SCRA 210; Malabanan v. Ramento, G.R. No. L-2270, May 21, 1984, 129
SCRA 359.] The issue raised by petitioners, which requires the interpretation
Atty. Ricardo C. Valmonte of the scope of the constitutional right to information, is one which can be
108 E. Benin Street passed upon by the regular courts more competently than the GSIS or its
Caloocan City Board of Trustees, involving as it does a purely legal question. Thus, the
exception of this case from the application of the general rule on exhaustion
Dear Compañero: of administrative remedies is warranted. Having disposed of this procedural
issue, We now address ourselves to the issue of whether or not mandamus
Possibly because he must have thought that it contained serious legal hes to compel respondent to perform the acts sought by petitioners to be done,
implications, President & General Manager Feliciano Belmonte, Jr. referred in pursuance of their right to information.
to me for study and reply your letter to him of June 4, 1986 requesting a list
of the opposition members of Batasang Pambansa who were able to secure a We shall deal first with the second and third alternative acts sought to be
clean loan of P2 million each on guaranty of Mrs. Imelda Marcos. done, both of which involve the issue of whether or not petitioners are
entitled to access to the documents evidencing loans granted by the GSIS.
This is not the first time that the Court is confronted with a controversy The Court has always grappled with the meanings of the terms "public
directly involving the constitutional right to information. In Tañada v. interest" and "public concern". As observed in Legazpi:
Tuvera, G.R. No. 63915, April 24,1985, 136 SCRA 27 and in the recent case
of Legaspi v. Civil Service Commission, G.R. No. 72119, May 29, 1987,150 In determining whether or not a particular information is of public concern
SCRA 530, the Court upheld the people's constitutional right to be informed there is no rigid test which can be applied. "Public concern" like "public
of matters of public interest and ordered the government agencies concerned interest" is a term that eludes exact definition. Both terms embrace a broad
to act as prayed for by the petitioners. spectrum of subjects which the public may want to know, either because
these directly affect their lives, or simply because such matters naturally
The pertinent provision under the 1987 Constitution is Art. 111, Sec. 7 which arouse the interest of an ordinary citezen. In the final analysis, it is for the
states: courts to determine on a case by case basis whether the matter at issue is of
interest or importance, as it relates to or affects the public. [Ibid. at p. 541]
The right of the people to information on matters of public concern shall be
recognized. Access to official records, and to documents, and papers In the Tañada case the public concern deemed covered by the constitutional
pertaining to official acts, transactions, or decisions, as well as to government right to information was the need for adequate notice to the public of the
research data used as basis for policy development, shall be afforded the various laws which are to regulate the actions and conduct of citezens. In
citizen, subject to such limitations as may be provided by law. Legaspi, it was the "legitimate concern of citezensof ensure that government
positions requiring civil service eligibility are occupied only by persons who
The right of access to information was also recognized in the 1973 are eligibles" [Supra at p. 539.]
Constitution, Art. IV Sec. 6 of which provided:
The information sought by petitioners in this case is the truth of reports that
The right of the people to information on 'matters of public concern shall be certain Members of the Batasang Pambansa belonging to the opposition were
recognized. Access to official records, and to documents and papers able to secure "clean" loans from the GSIS immediately before the February
pertaining to official acts, transactions, or decisions, shall be afforded the 7, 1986 election through the intercession of th eformer First Lady, Mrs.
citizen subject to such limitations as may be provided by law. Imelda Marcos.

An informed citizenry with access to the diverse currents in political, moral The GSIS is a trustee of contributions from the government and its employees
and artistic thought and data relative to them, and the free exchange of ideas and the administrator of various insurance programs for the benefit of the
and discussion of issues thereon, is vital to the democratic government latter. Undeniably, its funds assume a public character. More particularly,
envisioned under our Constitution. The cornerstone of this republican system Secs. 5(b) and 46 of P.D. 1146, as amended (the Revised Government
of government is delegation of power by the people to the State. In this Service Insurance Act of 1977), provide for annual appropriations to pay the
system, governmental agencies and institutions operate within the limits of contributions, premiums, interest and other amounts payable to GSIS by the
the authority conferred by the people. Denied access to information on the government, as employer, as well as the obligations which the Republic of
inner workings of government, the citizenry can become prey to the whims the Philippines assumes or guarantees to pay. Considering the nature of its
and caprices of those to whom the power had been delegated. The postulate funds, the GSIS is expected to manage its resources with utmost prudence
of public office as a public trust, institutionalized in the Constitution (in Art. and in strict compliance with the pertinent laws or rules and regulations.
XI, Sec. 1) to protect the people from abuse of governmental power, would Thus, one of the reasons that prompted the revision of the old GSIS law (C.A.
certainly be were empty words if access to such information of public No. 186, as amended) was the necessity "to preserve at all times the actuarial
concern is denied, except under limitations prescribed by implementing solvency of the funds administered by the System" [Second Whereas Clause,
legislation adopted pursuant to the Constitution. P.D. No. 1146.] Consequently, as respondent himself admits, the GSIS "is
not supposed to grant 'clean loans.'" [Comment, p. 8.] It is therefore the
Petitioners are practitioners in media. As such, they have both the right to legitimate concern of the public to ensure that these funds are managed
gather and the obligation to check the accuracy of information the properly with the end in view of maximizing the benefits that accrue to the
disseminate. For them, the freedom of the press and of speech is not only insured government employees. Moreover, the supposed borrowers were
critical, but vital to the exercise of their professions. The right of access to Members of the defunct Batasang Pambansa who themselves appropriated
information ensures that these freedoms are not rendered nugatory by the funds for the GSIS and were therefore expected to be the first to see to it that
government's monopolizing pertinent information. For an essential element the GSIS performed its tasks with the greatest degree of fidelity and that an
of these freedoms is to keep open a continuing dialogue or process of its transactions were above board.
communication between the government and the people. It is in the interest
of the State that the channels for free political discussion be maintained to In sum, the public nature of the loanable funds of the GSIS and the public
the end that the government may perceive and be responsive to the people's office held by the alleged borrowers make the information sought clearly a
will. Yet, this open dialogue can be effective only to the extent that the matter of public interest and concern.
citizenry is informed and thus able to formulate its will intelligently. Only
when the participants in the discussion are aware of the issues and have A second requisite must be met before the right to information may be
access to information relating thereto can such bear fruit. enforced through mandamus proceedings, viz., that the information sought
must not be among those excluded by law.
The right to information is an essential premise of a meaningful right to
speech and expression. But this is not to say that the right to information is Respondent maintains that a confidential relationship exists between the
merely an adjunct of and therefore restricted in application by the exercise of GSIS and its borrowers. It is argued that a policy of confidentiality restricts
the freedoms of speech and of the press. Far from it. The right to information the indiscriminate dissemination of information.
goes hand-in-hand with the constitutional policies of full public disclosure *
and honesty in the public service. ** It is meant to enhance the widening role Yet, respondent has failed to cite any law granting the GSIS the privilege of
of the citizenry in governmental decision-making as well as in checking confidentiality as regards the documents subject of this petition. His position
abuse in government. is apparently based merely on considerations of policy. The judiciary does
not settle policy issues. The Court can only declare what the law is, and not
Yet, like all the constitutional guarantees, the right to information is not what the law should be. Under our system of government, policy issues are
absolute. As stated in Legaspi, the people's right to information is limited to within the domain of the political branches of the government, and of the
"matters of public concern," and is further "subject to such limitations as may people themselves as the repository of all State power.
be provided by law." Similarly, the State's policy of full disclosure is limited
to "transactions involving public interest," and is "subject to reasonable Respondent however contends that in view of the right to privacy which is
conditions prescribed by law." equally protected by the Constitution and by existing laws, the documents
evidencing loan transactions of the GSIS must be deemed outside the ambit
Hence, before mandamus may issue, it must be clear that the information of the right to information.
sought is of "public interest" or "public concern," and is not exempted by law
from the operation of the constitutional guarantee [Legazpi v. Civil Service There can be no doubt that right to privacy is constitutionally protected. In
Commission, supra, at p. 542.] the landmark case of Morfe v. Mutuc [130 Phil. 415 (1968), 22 SCRA 424],
this Court, speaking through then Mr. Justice Fernando, stated:
... The right to privacy as such is accorded recognition independently of its transactions entered into by them within the coverage of the State policy of
identification with liberty; in itself, it is fully deserving of constitutional fun public disclosure is manifest from the records of the proceedings:
protection. The language of Prof. Emerson is particularly apt: "The concept
of limited government has always included the idea that governmental xxx xxx xxx
powers stop short of certain intrusions into the personal life of the citizen.
This is indeed one of the basic distinctions between absolute and limited THE PRESIDING OFFICER (Mr. Colayco).
government. UItimate and pervasive control of the individual, in all aspects
of his life, is the hallmark of the absolute. state, In contrast, a system of Commissioner Suarez is recognized.
limited government safeguards a private sector, which belongs to the
individual, firmly distinguishing it from the public sector, which the state can MR. SUAREZ. Thank you. May I ask the Gentleman a few question?
control. Protection of this private sector — protection, in other words, of the
dignity and integrity of the individual — has become increasingly important MR. OPLE. Very gladly.
as modem society has developed. All the forces of technological age —
industrialization, urbanization, and organization — operate to narrow the MR. SUAREZ. Thank you.
area of privacy and facilitate intrusion into it. In modern terms, the capacity
to maintain and support this enclave of private life marks the difference When we declare a "policy of full public disclosure of all its transactions" —
between a democratic and a totalitarian society." [at pp. 444-445.] referring to the transactions of the State — and when we say the "State"
which I suppose would include all of the various agencies, departments,
When the information requested from the government intrudes into the ministries and instrumentalities of the government....
privacy of a citizen, a potential conflict between the rights to information and
to privacy may arise. However, the competing interests of these rights need MR. OPLE. Yes, and individual public officers, Mr. Presiding Officer.
not be resolved in this case. Apparent from the above-quoted statement of
the Court in Morfe is that the right to privacy belongs to the individual in his MR. SUAREZ. Including government-owned and controlled corporations.
private capacity, and not to public and governmental agencies like the GSIS.
Moreover, the right cannot be invoked by juridical entities like the GSIS. As MR. OPLE. That is correct, Mr. Presiding Officer.
held in the case of Vassar College v. Loose Wills Biscuit Co. [197 F. 982
(1912)], a corporation has no right of privacy in its name since the entire MR. SUAREZ. And when we say "transactions" which should be
basis of the right to privacy is an injury to the feelings and sensibilities of the distinguished from contracts, agreements, or treaties or whatever, does the
party and a corporation would have no such ground for relief. Gentleman refer to the steps leading to the consummation of the contract, or
does he refer to the contract itself?
Neither can the GSIS through its General Manager, the respondent, invoke
the right to privacy of its borrowers. The right is purely personal in nature MR. OPLE. The "transactions" used here I suppose is generic and, therefore,
[Cf. Atkinson v. John Doherty & Co., 121 Mich 372, 80 N.W. 285, 46 L.RA. it can cover both steps leading to a contract, and already a consummated
219 (1899); Schuyler v. Curtis, 147 N.Y. 434, 42 N.E. 22, 31 L.R.A. 286 contract, Mr. Presiding Officer.
(1895)), and hence may be invoked only by the person whose privacy is
claimed to be violated. MR. SUAREZ. This contemplates inclusion of negotiations leading to the
consummation of the transaction.
It may be observed, however, that in the instant case, the concerned
borrowers themselves may not succeed if they choose to invoke their right to MR. OPLE. Yes, subject only to reasonable safeguards on the national
privacy, considering the public offices they were holding at the time the loans interest.
were alleged to have been granted. It cannot be denied that because of the
interest they generate and their newsworthiness, public figures, most MR. SUAREZ. Thank you. [V Record of the Constitutional Commission 24-
especially those holding responsible positions in government, enjoy a more 25.] (Emphasis supplied.)
limited right to privacy as compared to ordinary individuals, their actions
being subject to closer public scrutiny [Cf. Ayer Productions Pty. Ltd. v. Considering the intent of the framers of the Constitution which, though not
Capulong, G.R. Nos. 82380 and 82398, April 29, 1988; See also Cohen v. binding upon the Court, are nevertheless persuasive, and considering further
Marx, 211 P. 2d 321 (1949).] that government-owned and controlled corporations, whether performing
proprietary or governmental functions are accountable to the people, the
Respondent next asserts that the documents evidencing the loan transactions Court is convinced that transactions entered into by the GSIS, a government-
of the GSIS are private in nature and hence, are not covered by the controlled corporation created by special legislation are within the ambit of
Constitutional right to information on matters of public concern which the people's right to be informed pursuant to the constitutional policy of
guarantees "(a)ccess to official records, and to documents, and papers transparency in government dealings.
pertaining to official acts, transactions, or decisions" only.
In fine, petitioners are entitled to access to the documents evidencing loans
It is argued that the records of the GSIS, a government corporation granted by the GSIS, subject to reasonable regulations that the latter may
performing proprietary functions, are outside the coverage of the people's promulgate relating to the manner and hours of examination, to the end that
right of access to official records. damage to or loss of the records may be avoided, that undue interference with
the duties of the custodian of the records may be prevented and that the right
It is further contended that since the loan function of the GSIS is merely of other persons entitled to inspect the records may be insured [Legaspi v.
incidental to its insurance function, then its loan transactions are not covered Civil Service Commission, supra at p. 538, quoting Subido v. Ozaeta, 80 Phil.
by the constitutional policy of full public disclosure and the right to 383, 387.] The petition, as to the second and third alternative acts sought to
information which is applicable only to "official" transactions. be done by petitioners, is meritorious.

First of all, the "constituent — ministrant" dichotomy characterizing However, the same cannot be said with regard to the first act sought by
government function has long been repudiated. In ACCFA v. Confederation petitioners, i.e., "to furnish petitioners the list of the names of the Batasang
of Unions and Government Corporations and Offices (G.R. Nos. L-21484 Pambansa members belonging to the UNIDO and PDP-Laban who were able
and L-23605, November 29, 1969, 30 SCRA 6441, the Court said that the to secure clean loans immediately before the February 7 election thru the
government, whether carrying out its sovereign attributes or running some intercession/marginal note of the then First Lady Imelda Marcos."
business, discharges the same function of service to the people.
Although citizens are afforded the right to information and, pursuant thereto,
Consequently, that the GSIS, in granting the loans, was exercising a are entitled to "access to official records," the Constitution does not accord
proprietary function would not justify the exclusion of the transactions from them a right to compel custodians of official records to prepare lists,
the coverage and scope of the right to information. abstracts, summaries and the like in their desire to acquire information on
matters of public concern.
Moreover, the intent of the members of the Constitutional Commission of
1986, to include government-owned and controlled corporations and It must be stressed that it is essential for a writ of mandamus to issue that the
applicant has a well-defined, clear and certain legal right to the thing
demanded and that it is the imperative duty of defendant to perform the act
required. The corresponding duty of the respondent to perform the required
act must be clear and specific [Lemi v. Valencia, G.R. No. L-20768,
November 29,1968,126 SCRA 203; Ocampo v. Subido, G.R. No. L-28344,
August 27, 1976, 72 SCRA 443.] The request of the petitioners fails to meet
this standard, there being no duty on the part of respondent to prepare the list
requested.

WHEREFORE, the instant petition is hereby granted and respondent General


Manager of the Government Service Insurance System is ORDERED to
allow petitioners access to documents and records evidencing loans granted
to Members of the former Batasang Pambansa, as petitioners may specify,
subject to reasonable regulations as to the time and manner of inspection, not
incompatible with this decision, as the GSIS may deem necessary.

SO ORDERED.
REYES vs. BAGATSING were toupled in a single guarantee with the and to petition the rights of the
G.R. No. L-65366 November 9, 1983 people peaceably to assemble and to petition the government for redress of
FERNANDO, C.J.: grievances. All these rights, while not Identical, are inseparable. the every
case, therefo re there is a limitation placed on the exercise of this right, the
This Court, in this case of first impression, at least as to some aspects, is judiciary is called upon to examine the effects of the challenged
called upon to delineate the boundaries of the protected area of the cognate governmental actuation. The sole justification for a limitation on the exercise
rights to free speech and peaceable assembly, 1 against an alleged intrusion of this right, so fundamental to the maintenance of democratic institutions, is
by respondent Mayor Ramon Bagatsing. Petitioner, retired Justice JB L. the danger, of a character both grave and imminent, of a serious evil to public
Reyes, on behalf of the Anti-Bases Coalition sought a permit from the City safety, public morals, public health, or any other legitimate public interest.
of Manila to hold a peaceful march and rally on October 26, 1983 from 2:00 21
to 5:00 in the afternoon, starting from the Luneta, a public park, to the gates
of the United States Embassy, hardly two blocks away. Once there, and in an 2. Nowhere is the rationale that underlies the freedom of expression and
open space of public property, a short program would be held. 2 During the peaceable assembly better expressed than in this excerpt from an opinion of
course of the oral argument, 3 it was stated that after the delivery of two brief Justice Frankfurter: "It must never be forgotten, however, that the Bill of
speeches, a petition based on the resolution adopted on the last day by the Rights was the child of the Enlightenment. Back of the guaranty of free
International Conference for General Disbarmament, World Peace and the speech lay faith in the power of an appeal to reason by all the peaceful means
Removal of All Foreign Military Bases held in Manila, would be presented for gaining access to the mind. It was in order to avert force and explosions
to a representative of the Embassy or any of its personnel who may be there due to restrictions upon rational modes of communication that the guaranty
so that it may be delivered to the United States Ambassador. The march of free speech was given a generous scope. But utterance in a context of
would be attended by the local and foreign participants of such conference. violence can lose its significance as an appeal to reason and become part of
There was likewise an assurance in the petition that in the exercise of the an instrument of force. Such utterance was not meant to be sheltered by the
constitutional rights to free speech and assembly, all the necessary steps Constitution." 22 What was rightfully stressed is the abandonment of reason,
would be taken by it "to ensure a peaceful march and rally." 4 the utterance, whether verbal or printed, being in a context of violence. It
must always be remembered that this right likewise provides for a safety
The filing of this suit for mandamus with alternative prayer for writ of valve, allowing parties the opportunity to give vent to their-views, even if
preliminary mandatory injunction on October 20, 1983 was due to the fact contrary to the prevailing climate of opinion. For if the peaceful means of
that as of that date, petitioner had not been informed of any action taken on communication cannot be availed of, resort to non-peaceful means may be
his request on behalf of the organization to hold a rally. On October 25, 1983, the only alternative. Nor is this the sole reason for the expression of dissent.
the answer of respondent Mayor was filed on his behalf by Assistant Solicitor It means more than just the right to be heard of the person who feels
General Eduardo G. Montenegro. 5 It turned out that on October 19, such aggrieved or who is dissatisfied with things as they are. Its value may lie in
permit was denied. Petitioner was unaware of such a fact as the denial was the fact that there may be something worth hearing from the dissenter. That
sent by ordinary mail. The reason for refusing a permit was due to police is to ensure a true ferment of Ideas. There are, of course, well-defined limits.
intelligence reports which strongly militate against the advisability of issuing What is guaranteed is peaceable assembly. One may not advocate disorder in
such permit at this time and at the place applied for." 6 To be more specific, the name of protest, much less preach rebellion under the cloak of dissent.
reference was made to persistent intelligence reports affirm[ing] the plans of The Constitution frowns on disorder or tumult attending a rally or assembly.
subversive/criminal elements to infiltrate and/or disrupt any assembly or resort to force is ruled out and outbreaks of violence to be avoided. The
congregations where a large number of people is expected to attend." 7 utmost calm though is not required. As pointed out in an early Philippine
Respondent Mayor suggested, however, in accordance with the case, penned in 1907 to be precise, United States v. Apurado: 23 "It is rather
recommendation of the police authorities, that "a permit may be issued for to be expected that more or less disorder will mark the public assembly of
the rally if it is to be held at the Rizal Coliseum or any other enclosed area the people to protest against grievances whether real or imaginary, because
where the safety of the participants themselves and the general public may on such occasions feeling is always wrought to a high pitch of excitement,
be ensured." 8 and the greater the grievance and the more intense the feeling, the less
perfect, as a rule, will be the disciplinary control of the leaders over their
The oral argument was heard on October 25, 1983, the very same day the irresponsible followers." 24 It bears repeating that for the constitutional right
answer was filed. The Court then deliberated on the matter. That same to be invoked, riotous conduct, injury to property, and acts of vandalism must
afternoon, a minute resolution was issued by the Court granting the be avoided, To give free rein to one's destructive urges is to call for
mandatory injunction prayed for on the ground that there was no showing of condemnation. It is to make a mockery of the high estate occupied by
the existence of a clear and present danger of a substantive evil that could intellectual liberty in our scheme of values.
justify the denial of a permit. On this point, the Court was unanimous, but
there was a dissent by Justice Aquino on the ground that the holding of a rally 3. There can be no legal objection, absent the existence of a clear and present
in front of the US Embassy would be violative of Ordinance No. 7295 of the danger of a substantive evil, on the choice of Luneta as the place where the
City of Manila. The last sentence of such minute resolution reads: "This peace rally would start. The Philippines is committed to the view expressed
resolution is without prejudice to a more extended opinion." 9 Hence this in the plurality opinion, of 1939 vintage, of Justice Roberts in Hague v. CIO:
detailed exposition of the Court's stand on the matter. 25 Whenever the title of streets and parks may rest, they have immemorially
been held in trust for the use of the public and, time out of mind, have been
1. It is thus clear that the Court is called upon to protect the exercise of the used for purposes of assembly, communicating thoughts between citizens,
cognate rights to free speech and peaceful assembly, arising from the denial and discussing public questions. Such use of the streets and public places has,
of a permit. The Constitution is quite explicit: "No law shall be passed from ancient times, been a part of the privileges, immunities, rights, and
abridging the freedom of speech, or of the press, or the right of the people liberties of citizens. The privilege of a citizen of the United States to use the
peaceably to assemble and petition the Government for redress of streets and parks for communication of views on national questions may be
grievances." 10 Free speech, like free press, may be Identified with the liberty regulated in the interest of all; it is not absolute, but relative, and must be
to discuss publicly and truthfully any matter of public concern without exercised in subordination to the general comfort and convenience, and in
censorship or punishment. 11 There is to be then no previous restraint on the consonance with peace and good order; but it must not, in the guise of
communication of views or subsequent liability whether in libel suits, 12 regulation, be abridged or denied. 26 The above excerpt was quoted with
prosecution for sedition, 13 or action for damages, 14 or contempt approval in Primicias v. Fugoso. 27 Primicias made explicit what was
proceedings 15 unless there be a clear and present danger of a substantive implicit in Municipality of Cavite v. Rojas," 28 a 1915 decision, where this
evil that [the State] has a right to prevent." 16 Freedom of assembly connotes Court categorically affirmed that plazas or parks and streets are outside the
the right people to meet peaceably for consultation and discussion of matters commerce of man and thus nullified a contract that leased Plaza Soledad of
Of public concern.17 It is entitled to be accorded the utmost deference and plaintiff-municipality. Reference was made to such plaza "being a
respect. It is hot to be limited, much less denied, except on a showing, as 's promenade for public use," 29 which certainly is not the only purpose that it
the case with freedom of expression, of a clear and present danger of a could serve. To repeat, there can be no valid reason why a permit should not
substantive evil that the state has a right to prevent. 18 Even prior to the 1935 be granted for the or oposed march and rally starting from a public dark that
Constitution, Justice Maicolm had occasion to stress that it is a necessary is the Luneta.
consequence of our republican institutions and complements the right of free
speech. 19 To paraphrase opinion of Justice Rutledge speaking for the 4. Neither can there be any valid objection to the use of the streets, to the
majority of the American Supreme Court Thomas v. Collins, 20 it was not gates of the US Embassy, hardly two block-away at the Roxas Boulevard.
by accident or coincidence that the right to freedom of speech and of the press Primicias v. Fugoso has resolved any lurking doubt on the matter. In holding
that the then Mayor Fugoso of the City of Manila should grant a permit for a Universal Declaration of Human Rights. 35 The participants to such
public meeting at Plaza Miranda in Quiapo, this Court categorically declared: assembly, composed primarily of those in attendance at the International
"Our conclusion finds support in the decision in the case of Willis Cox vs. Conference for General Disbarmament, World Peace and the Removal of All
State of New Hampshire, 312 U.S., 569. In that case, the statute of New Foreign Military Bases would start from the Luneta. proceeding through
Hampshire P. L. chap. 145, section 2, providing that 'no parade or procession Roxas Boulevard to the gates of the United States Embassy located at the
upon any ground abutting thereon, shall 'De permitted unless a special license same street. To repeat, it is settled law that as to public places, especially so
therefor shall first be explained from the selectmen of the town or from as to parks and streets, there is freedom of access. Nor is their use dependent
licensing committee,' was construed by the Supreme Court of New on who is the applicant for the permit, whether an individual or a group. If it
Hampshire as not conferring upon the licensing board unfettered discretion were, then the freedom of access becomes discriminatory access, giving rise
to refuse to grant the license, and held valid. And the Supreme Court of the to an equal protection question. The principle under American doctrines was
United States, in its decision (1941) penned by Chief Justice Hughes given utterance by Chief Justice Hughes in these words: "The question, if the
affirming the judgment of the State Supreme Court, held that 'a statute rights of free speech and peaceable assembly are to be preserved, is not as to
requiring persons using the public streets for a parade or procession to the auspices under which the meeting is held but as to its purpose; not as to
procure a special license therefor from the local authorities is not an The relations of the speakers, but whether their utterances transcend the
unconstitutional abridgment of the rights of assembly or of freedom of bounds of the freedom of speech which the Constitution protects." 36 There
speech and press, where, as the statute is construed by the state courts, the could be danger to public peace and safety if such a gathering were marked
licensing authorities are strictly limited, in the issuance of licenses, to a by turbulence. That would deprive it of its peaceful character. Even then,
consideration of the time, place, and manner of the parade or procession, with only the guilty parties should be held accountable. It is true that the licensing
a view to conserving the public convenience and of affording an opportunity official, here respondent Mayor, is not devoid of discretion in determining
to provide proper policing, and are not invested with arbitrary discretion to whether or not a permit would be granted. It is not, however, unfettered
issue or refuse license, ... " 30 Nor should the point made by Chief Justice discretion. While prudence requires that there be a realistic appraisal not of
Hughes in a subsequent portion of the opinion be ignored, "Civil liberties, as what may possibly occur but of what may probably occur, given all the
guaranteed by the Constitution, imply the existence of an organized society relevant circumstances, still the assumption — especially so where the
maintaining public order without which liberty itself would be lost in the assembly is scheduled for a specific public — place is that the permit must
excesses of unrestricted abuses. The authority of a municipality to impose be for the assembly being held there. The exercise of such a right, in the
regulations in order to assure the safety and convenience of the people in the language of Justice Roberts, speaking for the American Supreme Court, is
use of public highways has never been regarded as inconsistent with civil not to be "abridged on the plea that it may be exercised in some other place."
liberties but rather as one of the means of safeguarding the good order upon 37
which they ultimately depend. The control of travel on the streets of cities is
the most familiar illustration of this recognition of social need. Where a 7. In fairness to respondent Mayor, he acted on the belief that Navarro v.
restriction of the use of highways in that relation is designed to promote the Villegas 38 and Pagkakaisa ng Manggagawang Pilipino (PMP.) v. Bagatsing,
public convenience in the interest of all, it cannot be disregarded by the 39 called for application. While the General rule is that a permit should
attempted exercise of some civil right which in other circumstances would recognize the right of the applicants to hold their assembly at a public place
be entitled to protection." 31 of their choice, another place may be designated by the licensing authority if
it be shown that there is a clear and present danger of a substantive evil if no
5. There is a novel aspect to this case, If the rally were confined to Luneta, such change were made. In the Navarro and the Pagkakaisa decisions, this
no question, as noted, would have arisen. So, too, if the march would end at Court was persuaded that the clear and present danger test was satisfied. The
another park. As previously mentioned though, there would be a short present situation is quite different. Hence the decision reached by the Court.
program upon reaching the public space between the two gates of the United The mere assertion that subversives may infiltrate the ranks of the
States Embassy at Roxas Boulevard. That would be followed by the handing demonstrators does not suffice. Not that it should be overlooked. There was
over of a petition based on the resolution adopted at the closing session of in this case, however, the assurance of General Narciso Cabrera,
the Anti-Bases Coalition. The Philippines is a signatory of the Vienna Superintendent, Western Police District, Metropolitan Police Force, that the
Convention on Diplomatic Relations adopted in 1961. It was concurred in by police force is in a position to cope with such emergency should it arise That
the then Philippine Senate on May 3, 1965 and the instrument of ratification is to comply with its duty to extend protection to the participants of such
was signed by the President on October 11, 1965, and was thereafter peaceable assembly. Also from him came the commendable admission that
deposited with the Secretary General of the United Nations on November 15. there were the least five previous demonstrations at the Bayview hotel Area
As of that date then, it was binding on the Philippines. The second paragraph and Plaza Ferguson in front of the United States Embassy where no untoward
of the Article 22 reads: "2. The receiving State is under a special duty to take event occurred. It was made clear by petitioner, through counsel, that no act
appropriate steps to protect the premises of the mission against any intrusion offensive to the dignity of the United States Mission in the Philippines would
or damage and to prevent any disturbance of the peace of the mission or take place and that, as mentioned at the outset of this opinion, "all the
impairment of its dignity. " 32 The Constitution "adopts the generally necessary steps would be taken by it 'to ensure a peaceful march and rally.' "
accepted principles of international law as part of the law of the land. ..." 33 40 Assistant Solicitor General Montenegro expressed the view that the
To the extent that the Vienna Convention is a restatement of the generally presence of policemen may in itself be a provocation. It is a sufficient answer
accepted principles of international law, it should be a part of the law of the that they should stay at a discreet distance, but ever ready and alert to cope
land. 34 That being the case, if there were a clear and present danger of any with any contingency. There is no need to repeat what was pointed out by
intrusion or damage, or disturbance of the peace of the mission, or Chief Justice Hughes in Cox that precisely, it is the duty of the city authorities
impairment of its dignity, there would be a justification for the denial of the to provide the proper police protection to those exercising their right to
permit insofar as the terminal point would be the Embassy. Moreover, peaceable assembly and freedom of expression.
respondent Mayor relied on Ordinance No. 7295 of the City of Manila
prohibiting the holding or staging of rallies or demonstrations within a radius 8. By way of a summary The applicants for a permit to hold an assembly
of five hundred (500) feet from any foreign mission or chancery and for other should inform the licensing authority of the date, the public place where and
purposes. Unless the ordinance is nullified, or declared ultra vires, its the time when it will take place. If it were a private place, only the consent
invocation as a defense is understandable but not decisive, in view of the of the owner or the one entitled to its legal possession is required. Such
primacy accorded the constitutional rights of free speech and peaceable application should be filed well ahead in time to enable the public official
assembly. Even if shown then to be applicable, that question the confronts concerned to appraise whether there may be valid objections to the grant of
this Court. the permit or to its grant but at another public place. It is an indispensable
condition to such refusal or modification that the clear and present danger
6. There is merit to the observation that except as to the novel aspects of a test be the standard for the decision reached. If he is of the view that there is
litigation, the judgment must be confined within the limits of previous such an imminent and grave danger of a substantive evil, the applicants must
decisions. The law declared on past occasions is, on the whole, a safe guide, be heard on the matter. Thereafter, his decision, whether favorable or
So it has been here. Hence, as noted, on the afternoon of the hearing, October adverse, must be transmitted to them at the earliest opportunity. Thus if so
25, 1983, this Court issued the minute resolution granting the mandatory minded, then, can have recourse to the proper judicial authority. Free speech
injunction allowing the proposed march and rally scheduled for the next day. and peaceable assembly, along with the other intellectual freedoms, are
That conclusion was inevitable ill the absence of a clear and present danger highly ranked in our scheme of constitutional values. It cannot be too strongly
of a substantive, evil to a legitimate public interest. There was no justification stressed that on the judiciary, — even more so than on the other departments
then to deny the exercise of the constitutional rights of tree speech and — rests the grave and delicate responsibility of assuring respect for and
peaceable assembly. These rights are assured by our Constitution and the deference to such preferred rights. No verbal formula, no sanctifying phrase
can, of course, dispense with what has been so felicitiously termed by Justice
Holmes "as the sovereign prerogative of judgment." Nonetheless, the
presumption must be to incline the weight of the scales of justice on the side
of such rights, enjoying as they do precedence and primacy. Clearly then, to
the extent that there may be inconsistencies between this resolution and that
of Navarro v. Villegas, that case is pro tanto modified. So it was made clear
in the original resolution of October 25, 1983.

9. Respondent Mayor posed the issue of the applicability of Ordinance No.


7295 of the City of Manila prohibiting the holding or staging of rallies or
demonstrations within a radius of five hundred (500) feet from any foreign
mission or chancery and for other purposes. It is to be admitted that it finds
support In the previously quoted Article 22 of the Vienna Convention on
Diplomatic Relations. There was no showing, however, that the distance
between the chancery and the embassy gate is less than 500 feet. Even if it
could be shown that such a condition is satisfied. it does not follow that
respondent Mayor could legally act the way he did. The validity of his denial
of the permit sought could still be challenged. It could be argued that a case
of unconstitutional application of such ordinance to the exercise of the right
of peaceable assembly presents itself. As in this case there was no proof that
the distance is less than 500 feet, the need to pass on that issue was obviated,
Should it come, then the qualification and observation of Justices Makasiar
and Plana certainly cannot be summarily brushed aside. The high estate
accorded the rights to free speech and peaceable assembly demands nothing
less.

10. Ordinarily, the remedy in cases of this character is to set aside the denial
or the modification of the permit sought and order the respondent official, to
grant it. Nonetheless, as there was urgency in this case, the proposed march
and rally being scheduled for the next day after the hearing, this Court. in the
exercise of its conceded authority, granted the mandatory injunction in the
resolution of October 25, 1983. It may be noted that the peaceful character
of the peace march and rally on October 26 was not marred by any untoward
incident. So it has been in other assemblies held elsewhere. It is quite
reassuring such that both on the part of the national government and the
citizens, reason and moderation have prevailed. That is as it should be.

WHEREFORE, the mandatory injunction prayed for is granted. No costs.


VICTORIANO vs. ELIZALDE ROPE WORKERS' UNION Bill of Rights, and thus becomes obnoxious to Article III, Section 1 (6) of the
G.R. No. L-25246 September 12, 1974 1935 Constitution. 6
ZALDIVAR, J.:p
Secondly, the Union contended that Republic Act No. 3350 is
Appeal to this Court on purely questions of law from the decision of the Court unconstitutional for impairing the obligation of contracts in that, while the
of First Instance of Manila in its Civil Case No. 58894. Union is obliged to comply with its collective bargaining agreement
containing a "closed shop provision," the Act relieves the employer from its
The undisputed facts that spawned the instant case follow: reciprocal obligation of cooperating in the maintenance of union membership
as a condition of employment; and that said Act, furthermore, impairs the
Benjamin Victoriano (hereinafter referred to as Appellee), a member of the Union's rights as it deprives the union of dues from members who, under the
religious sect known as the "Iglesia ni Cristo", had been in the employ of the Act, are relieved from the obligation to continue as such members.7
Elizalde Rope Factory, Inc. (hereinafter referred to as Company) since 1958.
As such employee, he was a member of the Elizalde Rope Workers' Union Thirdly, the Union contended that Republic Act No. 3350 discriminatorily
(hereinafter referred to as Union) which had with the Company a collective favors those religious sects which ban their members from joining labor
bargaining agreement containing a closed shop provision which reads as unions, in violation of Article Ill, Section 1 (7) of the 1935 Constitution; and
follows: while said Act unduly protects certain religious sects, it leaves no rights or
protection to labor organizations.8
Membership in the Union shall be required as a condition of employment for
all permanent employees workers covered by this Agreement. Fourthly, Republic Act No. 3350, asserted the Union, violates the
constitutional provision that "no religious test shall be required for the
The collective bargaining agreement expired on March 3, 1964 but was exercise of a civil right," in that the laborer's exercise of his civil right to join
renewed the following day, March 4, 1964. associations for purposes not contrary to law has to be determined under the
Act by his affiliation with a religious sect; that conversely, if a worker has to
Under Section 4(a), paragraph 4, of Republic Act No. 875, prior to its sever his religious connection with a sect that prohibits membership in a labor
amendment by Republic Act No. 3350, the employer was not precluded organization in order to be able to join a labor organization, said Act would
"from making an agreement with a labor organization to require as a violate religious freedom.9
condition of employment membership therein, if such labor organization is
the representative of the employees." On June 18, 1961, however, Republic Fifthly, the Union contended that Republic Act No. 3350, violates the "equal
Act No. 3350 was enacted, introducing an amendment to — paragraph (4) protection of laws" clause of the Constitution, it being a discriminately
subsection (a) of section 4 of Republic Act No. 875, as follows: ... "but such legislation, inasmuch as by exempting from the operation of closed shop
agreement shall not cover members of any religious sects which prohibit agreement the members of the "Iglesia ni Cristo", it has granted said members
affiliation of their members in any such labor organization". undue advantages over their fellow workers, for while the Act exempts them
from union obligation and liability, it nevertheless entitles them at the same
Being a member of a religious sect that prohibits the affiliation of its time to the enjoyment of all concessions, benefits and other emoluments that
members with any labor organization, Appellee presented his resignation to the union might secure from the employer. 10
appellant Union in 1962, and when no action was taken thereon, he reiterated
his resignation on September 3, 1974. Thereupon, the Union wrote a formal Sixthly, the Union contended that Republic Act No. 3350 violates the
letter to the Company asking the latter to separate Appellee from the service constitutional provision regarding the promotion of social justice. 11
in view of the fact that he was resigning from the Union as a member. The
management of the Company in turn notified Appellee and his counsel that Appellant Union, furthermore, asserted that a "closed shop provision" in a
unless the Appellee could achieve a satisfactory arrangement with the Union, collective bargaining agreement cannot be considered violative of religious
the Company would be constrained to dismiss him from the service. This freedom, as to call for the amendment introduced by Republic Act No. 3350;
prompted Appellee to file an action for injunction, docketed as Civil Case 12 and that unless Republic Act No. 3350 is declared unconstitutional, trade
No. 58894 in the Court of First Instance of Manila to enjoin the Company unionism in this country would be wiped out as employers would prefer to
and the Union from dismissing Appellee.1 In its answer, the Union invoked hire or employ members of the Iglesia ni Cristo in order to do away with
the "union security clause" of the collective bargaining agreement; assailed labor organizations. 13
the constitutionality of Republic Act No. 3350; and contended that the Court
had no jurisdiction over the case, pursuant to Republic Act No. 875, Sections Appellee, assailing appellant's arguments, contended that Republic Act No.
24 and 9 (d) and (e).2 Upon the facts agreed upon by the parties during the 3350 does not violate the right to form lawful associations, for the right to
pre-trial conference, the Court a quo rendered its decision on August 26, join associations includes the right not to join or to resign from a labor
1965, the dispositive portion of which reads: organization, if one's conscience does not allow his membership therein, and
the Act has given substance to such right by prohibiting the compulsion of
IN VIEW OF THE FOREGOING, judgment is rendered enjoining the workers to join labor organizations; 14 that said Act does not impair the
defendant Elizalde Rope Factory, Inc. from dismissing the plaintiff from his obligation of contracts for said law formed part of, and was incorporated into,
present employment and sentencing the defendant Elizalde Rope Workers' the terms of the closed shop agreement; 15 that the Act does not violate the
Union to pay the plaintiff P500 for attorney's fees and the costs of this establishment of religion clause or separation of Church and State, for
action.3 Congress, in enacting said law, merely accommodated the religious needs of
those workers whose religion prohibits its members from joining labor
From this decision, the Union appealed directly to this Court on purely unions, and balanced the collective rights of organized labor with the
questions of law, assigning the following errors: constitutional right of an individual to freely exercise his chosen religion;
that the constitutional right to the free exercise of one's religion has primacy
I. That the lower court erred when it did not rule that Republic Act No. 3350 and preference over union security measures which are merely contractual
is unconstitutional. 16 ; that said Act does not violate the constitutional provision of equal
protection, for the classification of workers under the Act depending on their
II. That the lower court erred when it sentenced appellant herein to pay religious tenets is based on substantial distinction, is germane to the purpose
plaintiff the sum of P500 as attorney's fees and the cost thereof. of the law, and applies to all the members of a given class; 17 that said Act,
finally, does not violate the social justice policy of the Constitution, for said
In support of the alleged unconstitutionality of Republic Act No. 3350, the Act was enacted precisely to equalize employment opportunities for all
Union contented, firstly, that the Act infringes on the fundamental right to citizens in the midst of the diversities of their religious beliefs." 18
form lawful associations; that "the very phraseology of said Republic Act
3350, that membership in a labor organization is banned to all those I. Before We proceed to the discussion of the first assigned error, it is
belonging to such religious sect prohibiting affiliation with any labor necessary to premise that there are some thoroughly established principles
organization"4 , "prohibits all the members of a given religious sect from which must be followed in all cases where questions of constitutionality as
joining any labor union if such sect prohibits affiliations of their members obtains in the instant case are involved. All presumptions are indulged in
thereto"5 ; and, consequently, deprives said members of their constitutional favor of constitutionality; one who attacks a statute, alleging
right to form or join lawful associations or organizations guaranteed by the unconstitutionality must prove its invalidity beyond a reasonable doubt, that
a law may work hardship does not render it unconstitutional; that if any
reasonable basis may be conceived which supports the statute, it will be collective bargaining union. It is clear, therefore, that the assailed Act, far
upheld, and the challenger must negate all possible bases; that the courts are from infringing the constitutional provision on freedom of association,
not concerned with the wisdom, justice, policy, or expediency of a statute; upholds and reinforces it. It does not prohibit the members of said religious
and that a liberal interpretation of the constitution in favor of the sects from affiliating with labor unions. It still leaves to said members the
constitutionality of legislation should be adopted. 19 liberty and the power to affiliate, or not to affiliate, with labor unions. If,
notwithstanding their religious beliefs, the members of said religious sects
1. Appellant Union's contention that Republic Act No. 3350 prohibits and prefer to sign up with the labor union, they can do so. If in deference and
bans the members of such religious sects that forbid affiliation of their fealty to their religious faith, they refuse to sign up, they can do so; the law
members with labor unions from joining labor unions appears nowhere in the does not coerce them to join; neither does the law prohibit them from joining;
wording of Republic Act No. 3350; neither can the same be deduced by and neither may the employer or labor union compel them to join. Republic
necessary implication therefrom. It is not surprising, therefore, that appellant, Act No. 3350, therefore, does not violate the constitutional provision on
having thus misread the Act, committed the error of contending that said Act freedom of association.
is obnoxious to the constitutional provision on freedom of association.
2. Appellant Union also contends that the Act is unconstitutional for
Both the Constitution and Republic Act No. 875 recognize freedom of impairing the obligation of its contract, specifically, the "union security
association. Section 1 (6) of Article III of the Constitution of 1935, as well clause" embodied in its Collective Bargaining Agreement with the Company,
as Section 7 of Article IV of the Constitution of 1973, provide that the right by virtue of which "membership in the union was required as a condition for
to form associations or societies for purposes not contrary to law shall not be employment for all permanent employees workers". This agreement was
abridged. Section 3 of Republic Act No. 875 provides that employees shall already in existence at the time Republic Act No. 3350 was enacted on June
have the right to self-organization and to form, join of assist labor 18, 1961, and it cannot, therefore, be deemed to have been incorporated into
organizations of their own choosing for the purpose of collective bargaining the agreement. But by reason of this amendment, Appellee, as well as others
and to engage in concerted activities for the purpose of collective bargaining similarly situated, could no longer be dismissed from his job even if he
and other mutual aid or protection. What the Constitution and the Industrial should cease to be a member, or disaffiliate from the Union, and the
Peace Act recognize and guarantee is the "right" to form or join associations. Company could continue employing him notwithstanding his disaffiliation
Notwithstanding the different theories propounded by the different schools from the Union. The Act, therefore, introduced a change into the express
of jurisprudence regarding the nature and contents of a "right", it can be terms of the union security clause; the Company was partly absolved by law
safely said that whatever theory one subscribes to, a right comprehends at from the contractual obligation it had with the Union of employing only
least two broad notions, namely: first, liberty or freedom, i.e., the absence of Union members in permanent positions, It cannot be denied, therefore, that
legal restraint, whereby an employee may act for himself without being there was indeed an impairment of said union security clause.
prevented by law; and second, power, whereby an employee may, as he
pleases, join or refrain from Joining an association. It is, therefore, the According to Black, any statute which introduces a change into the express
employee who should decide for himself whether he should join or not an terms of the contract, or its legal construction, or its validity, or its discharge,
association; and should he choose to join, he himself makes up his mind as or the remedy for its enforcement, impairs the contract. The extent of the
to which association he would join; and even after he has joined, he still change is not material. It is not a question of degree or manner or cause, but
retains the liberty and the power to leave and cancel his membership with of encroaching in any respect on its obligation or dispensing with any part of
said organization at any time. 20 It is clear, therefore, that the right to join a its force. There is an impairment of the contract if either party is absolved by
union includes the right to abstain from joining any union. 21 Inasmuch as law from its performance. 22 Impairment has also been predicated on laws
what both the Constitution and the Industrial Peace Act have recognized, and which, without destroying contracts, derogate from substantial contractual
guaranteed to the employee, is the "right" to join associations of his choice, rights. 23
it would be absurd to say that the law also imposes, in the same breath, upon
the employee the duty to join associations. The law does not enjoin an It should not be overlooked, however, that the prohibition to impair the
employee to sign up with any association. obligation of contracts is not absolute and unqualified. The prohibition is
general, affording a broad outline and requiring construction to fill in the
The right to refrain from joining labor organizations recognized by Section 3 details. The prohibition is not to be read with literal exactness like a
of the Industrial Peace Act is, however, limited. The legal protection granted mathematical formula, for it prohibits unreasonable impairment only. 24 In
to such right to refrain from joining is withdrawn by operation of law, where spite of the constitutional prohibition, the State continues to possess authority
a labor union and an employer have agreed on a closed shop, by virtue of to safeguard the vital interests of its people. Legislation appropriate to
which the employer may employ only member of the collective bargaining safeguarding said interests may modify or abrogate contracts already in
union, and the employees must continue to be members of the union for the effect. 25 For not only are existing laws read into contracts in order to fix the
duration of the contract in order to keep their jobs. Thus Section 4 (a) (4) of obligations as between the parties, but the reservation of essential attributes
the Industrial Peace Act, before its amendment by Republic Act No. 3350, of sovereign power is also read into contracts as a postulate of the legal order.
provides that although it would be an unfair labor practice for an employer All contracts made with reference to any matter that is subject to regulation
"to discriminate in regard to hire or tenure of employment or any term or under the police power must be understood as made in reference to the
condition of employment to encourage or discourage membership in any possible exercise of that power. 26 Otherwise, important and valuable
labor organization" the employer is, however, not precluded "from making reforms may be precluded by the simple device of entering into contracts for
an agreement with a labor organization to require as a condition of the purpose of doing that which otherwise may be prohibited. The policy of
employment membership therein, if such labor organization is the protecting contracts against impairment presupposes the maintenance of a
representative of the employees". By virtue, therefore, of a closed shop government by virtue of which contractual relations are worthwhile a
agreement, before the enactment of Republic Act No. 3350, if any person, government which retains adequate authority to secure the peace and good
regardless of his religious beliefs, wishes to be employed or to keep his order of society. The contract clause of the Constitution must, therefore, be
employment, he must become a member of the collective bargaining union. not only in harmony with, but also in subordination to, in appropriate
Hence, the right of said employee not to join the labor union is curtailed and instances, the reserved power of the state to safeguard the vital interests of
withdrawn. the people. It follows that not all legislations, which have the effect of
impairing a contract, are obnoxious to the constitutional prohibition as to
To that all-embracing coverage of the closed shop arrangement, Republic Act impairment, and a statute passed in the legitimate exercise of police power,
No. 3350 introduced an exception, when it added to Section 4 (a) (4) of the although it incidentally destroys existing contract rights, must be upheld by
Industrial Peace Act the following proviso: "but such agreement shall not the courts. This has special application to contracts regulating relations
cover members of any religious sects which prohibit affiliation of their between capital and labor which are not merely contractual, and said labor
members in any such labor organization". Republic Act No. 3350 merely contracts, for being impressed with public interest, must yield to the common
excludes ipso jure from the application and coverage of the closed shop good. 27
agreement the employees belonging to any religious sects which prohibit
affiliation of their members with any labor organization. What the exception In several occasions this Court declared that the prohibition against impairing
provides, therefore, is that members of said religious sects cannot be the obligations of contracts has no application to statutes relating to public
compelled or coerced to join labor unions even when said unions have closed subjects within the domain of the general legislative powers of the state
shop agreements with the employers; that in spite of any closed shop involving public welfare. 28 Thus, this Court also held that the Blue Sunday
agreement, members of said religious sects cannot be refused employment or Law was not an infringement of the obligation of a contract that required the
dismissed from their jobs on the sole ground that they are not members of the employer to furnish work on Sundays to his employees, the law having been
enacted to secure the well-being and happiness of the laboring class, and
being, furthermore, a legitimate exercise of the police power. 29 The constitutional provision into only prohibits legislation for the support of
any religious tenets or the modes of worship of any sect, thus forestalling
In order to determine whether legislation unconstitutionally impairs contract compulsion by law of the acceptance of any creed or the practice of any form
obligations, no unchanging yardstick, applicable at all times and under all of worship, 35 but also assures the free exercise of one's chosen form of
circumstances, by which the validity of each statute may be measured or religion within limits of utmost amplitude. It has been said that the religion
determined, has been fashioned, but every case must be determined upon its clauses of the Constitution are all designed to protect the broadest possible
own circumstances. Legislation impairing the obligation of contracts can be liberty of conscience, to allow each man to believe as his conscience directs,
sustained when it is enacted for the promotion of the general good of the to profess his beliefs, and to live as he believes he ought to live, consistent
people, and when the means adopted to secure that end are reasonable. Both with the liberty of others and with the common good. 36 Any legislation
the end sought and the means adopted must be legitimate, i.e., within the whose effect or purpose is to impede the observance of one or all religions,
scope of the reserved power of the state construed in harmony with the or to discriminate invidiously between the religions, is invalid, even though
constitutional limitation of that power. 30 the burden may be characterized as being only indirect. 37 But if the stage
regulates conduct by enacting, within its power, a general law which has for
What then was the purpose sought to be achieved by Republic Act No. 3350? its purpose and effect to advance the state's secular goals, the statute is valid
Its purpose was to insure freedom of belief and religion, and to promote the despite its indirect burden on religious observance, unless the state can
general welfare by preventing discrimination against those members of accomplish its purpose without imposing such burden. 38
religious sects which prohibit their members from joining labor unions,
confirming thereby their natural, statutory and constitutional right to work, In Aglipay v. Ruiz 39 , this Court had occasion to state that the government
the fruits of which work are usually the only means whereby they can should not be precluded from pursuing valid objectives secular in character
maintain their own life and the life of their dependents. It cannot be gainsaid even if the incidental result would be favorable to a religion or sect. It has
that said purpose is legitimate. likewise been held that the statute, in order to withstand the strictures of
constitutional prohibition, must have a secular legislative purpose and a
The questioned Act also provides protection to members of said religious primary effect that neither advances nor inhibits religion. 40 Assessed by
sects against two aggregates of group strength from which the individual these criteria, Republic Act No. 3350 cannot be said to violate the
needs protection. The individual employee, at various times in his working constitutional inhibition of the "no-establishment" (of religion) clause of the
life, is confronted by two aggregates of power — collective labor, directed Constitution.
by a union, and collective capital, directed by management. The union, an
institution developed to organize labor into a collective force and thus protect The purpose of Republic Act No. 3350 is secular, worldly, and temporal, not
the individual employee from the power of collective capital, is, spiritual or religious or holy and eternal. It was intended to serve the secular
paradoxically, both the champion of employee rights, and a new source of purpose of advancing the constitutional right to the free exercise of religion,
their frustration. Moreover, when the Union interacts with management, it by averting that certain persons be refused work, or be dismissed from work,
produces yet a third aggregate of group strength from which the individual or be dispossessed of their right to work and of being impeded to pursue a
also needs protection — the collective bargaining relationship. 31 modest means of livelihood, by reason of union security agreements. To help
its citizens to find gainful employment whereby they can make a living to
The aforementioned purpose of the amendatory law is clearly seen in the support themselves and their families is a valid objective of the state. In fact,
Explanatory Note to House Bill No. 5859, which later became Republic Act the state is enjoined, in the 1935 Constitution, to afford protection to labor,
No. 3350, as follows: and regulate the relations between labor and capital and industry. 41 More so
now in the 1973 Constitution where it is mandated that "the State shall afford
It would be unthinkable indeed to refuse employing a person who, on account protection to labor, promote full employment and equality in employment,
of his religious beliefs and convictions, cannot accept membership in a labor ensure equal work opportunities regardless of sex, race or creed and regulate
organization although he possesses all the qualifications for the job. This is the relation between workers and employers. 42
tantamount to punishing such person for believing in a doctrine he has a right
under the law to believe in. The law would not allow discrimination to The primary effects of the exemption from closed shop agreements in favor
flourish to the detriment of those whose religion discards membership in any of members of religious sects that prohibit their members from affiliating
labor organization. Likewise, the law would not commend the deprivation of with a labor organization, is the protection of said employees against the
their right to work and pursue a modest means of livelihood, without in any aggregate force of the collective bargaining agreement, and relieving certain
manner violating their religious faith and/or belief. 32 citizens of a burden on their religious beliefs; and by eliminating to a certain
extent economic insecurity due to unemployment, which is a serious menace
It cannot be denied, furthermore, that the means adopted by the Act to to the health, morals, and welfare of the people of the State, the Act also
achieve that purpose — exempting the members of said religious sects from promotes the well-being of society. It is our view that the exemption from
coverage of union security agreements — is reasonable. the effects of closed shop agreement does not directly advance, or diminish,
the interests of any particular religion. Although the exemption may benefit
It may not be amiss to point out here that the free exercise of religious those who are members of religious sects that prohibit their members from
profession or belief is superior to contract rights. In case of conflict, the latter joining labor unions, the benefit upon the religious sects is merely incidental
must, therefore, yield to the former. The Supreme Court of the United States and indirect. The "establishment clause" (of religion) does not ban regulation
has also declared on several occasions that the rights in the First Amendment, on conduct whose reason or effect merely happens to coincide or harmonize
which include freedom of religion, enjoy a preferred position in the with the tenets of some or all religions. 43 The free exercise clause of the
constitutional system. 33 Religious freedom, although not unlimited, is a Constitution has been interpreted to require that religious exercise be
fundamental personal right and liberty, 34 and has a preferred position in the preferentially aided. 44
hierarchy of values. Contractual rights, therefore, must yield to freedom of
religion. It is only where unavoidably necessary to prevent an immediate and We believe that in enacting Republic Act No. 3350, Congress acted
grave danger to the security and welfare of the community that infringement consistently with the spirit of the constitutional provision. It acted merely to
of religious freedom may be justified, and only to the smallest extent relieve the exercise of religion, by certain persons, of a burden that is imposed
necessary to avoid the danger. by union security agreements. It was Congress itself that imposed that burden
when it enacted the Industrial Peace Act (Republic Act 875), and, certainly,
3. In further support of its contention that Republic Act No. 3350 is Congress, if it so deems advisable, could take away the same burden. It is
unconstitutional, appellant Union averred that said Act discriminates in favor certain that not every conscience can be accommodated by all the laws of the
of members of said religious sects in violation of Section 1 (7) of Article Ill land; but when general laws conflict with scrupples of conscience,
of the 1935 Constitution, and which is now Section 8 of Article IV of the exemptions ought to be granted unless some "compelling state interest"
1973 Constitution, which provides: intervenes. 45 In the instant case, We see no such compelling state interest to
withhold exemption.
No law shall be made respecting an establishment of religion, or prohibiting
the free exercise thereof, and the free exercise and enjoyment of religious Appellant bewails that while Republic Act No. 3350 protects members of
profession and worship, without discrimination and preference, shall forever certain religious sects, it leaves no right to, and is silent as to the protection
be allowed. No religious test shall be required for the exercise of civil or of, labor organizations. The purpose of Republic Act No. 3350 was not to
political rights. grant rights to labor unions. The rights of labor unions are amply provided
for in Republic Act No. 875 and the new Labor Code. As to the lamented In the exercise of its power to make classifications for the purpose of enacting
silence of the Act regarding the rights and protection of labor unions, suffice laws over matters within its jurisdiction, the state is recognized as enjoying
it to say, first, that the validity of a statute is determined by its provisions, not a wide range of discretion. 56 It is not necessary that the classification be
by its silence 46 ; and, second, the fact that the law may work hardship does based on scientific or marked differences of things or in their relation. 57
not render it unconstitutional. 47 Neither is it necessary that the classification be made with mathematical
nicety. 58 Hence legislative classification may in many cases properly rest
It would not be amiss to state, regarding this matter, that to compel persons on narrow distinctions, 59 for the equal protection guaranty does not preclude
to join and remain members of a union to keep their jobs in violation of their the legislature from recognizing degrees of evil or harm, and legislation is
religious scrupples, would hurt, rather than help, labor unions, Congress has addressed to evils as they may appear.
seen it fit to exempt religious objectors lest their resistance spread to other
workers, for religious objections have contagious potentialities more than We believe that Republic Act No. 3350 satisfies the aforementioned
political and philosophic objections. requirements. The Act classifies employees and workers, as to the effect and
coverage of union shop security agreements, into those who by reason of
Furthermore, let it be noted that coerced unity and loyalty even to the country, their religious beliefs and convictions cannot sign up with a labor union, and
and a fortiori to a labor — union assuming that such unity and loyalty can be those whose religion does not prohibit membership in labor unions. Tile
attained through coercion — is not a goal that is constitutionally obtainable classification rests on real or substantial, not merely imaginary or whimsical,
at the expense of religious liberty. 48 A desirable end cannot be promoted by distinctions. There is such real distinction in the beliefs, feelings and
prohibited means. sentiments of employees. Employees do not believe in the same religious
faith and different religions differ in their dogmas and cannons. Religious
4. Appellants' fourth contention, that Republic Act No. 3350 violates the beliefs, manifestations and practices, though they are found in all places, and
constitutional prohibition against requiring a religious test for the exercise of in all times, take so many varied forms as to be almost beyond imagination.
a civil right or a political right, is not well taken. The Act does not require as There are many views that comprise the broad spectrum of religious beliefs
a qualification, or condition, for joining any lawful association membership among the people. There are diverse manners in which beliefs, equally
in any particular religion or in any religious sect; neither does the Act require paramount in the lives of their possessors, may be articulated. Today the
affiliation with a religious sect that prohibits its members from joining a labor country is far more heterogenous in religion than before, differences in
union as a condition or qualification for withdrawing from a labor union. religion do exist, and these differences are important and should not be
Joining or withdrawing from a labor union requires a positive act. Republic ignored.
Act No. 3350 only exempts members with such religious affiliation from the
coverage of closed shop agreements. So, under this Act, a religious objector Even from the phychological point of view, the classification is based on real
is not required to do a positive act — to exercise the right to join or to resign and important differences. Religious beliefs are not mere beliefs, mere ideas
from the union. He is exempted ipso jure without need of any positive act on existing only in the mind, for they carry with them practical consequences
his part. A conscientious religious objector need not perform a positive act and are the motives of certain rules. of human conduct and the justification
or exercise the right of resigning from the labor union — he is exempted from of certain acts. 60 Religious sentiment makes a man view things and events
the coverage of any closed shop agreement that a labor union may have in their relation to his God. It gives to human life its distinctive character, its
entered into. How then can there be a religious test required for the exercise tone, its happiness or unhappiness its enjoyment or irksomeness. Usually, a
of a right when no right need be exercised? strong and passionate desire is involved in a religious belief. To certain
persons, no single factor of their experience is more important to them than
We have said that it was within the police power of the State to enact their religion, or their not having any religion. Because of differences in
Republic Act No. 3350, and that its purpose was legal and in consonance religious belief and sentiments, a very poor person may consider himself
with the Constitution. It is never an illegal evasion of a constitutional better than the rich, and the man who even lacks the necessities of life may
provision or prohibition to accomplish a desired result, which is lawful in be more cheerful than the one who has all possible luxuries. Due to their
itself, by discovering or following a legal way to do it. 49 religious beliefs people, like the martyrs, became resigned to the inevitable
and accepted cheerfully even the most painful and excruciating pains.
5. Appellant avers as its fifth ground that Republic Act No. 3350 is a Because of differences in religious beliefs, the world has witnessed turmoil,
discriminatory legislation, inasmuch as it grants to the members of certain civil strife, persecution, hatred, bloodshed and war, generated to a large
religious sects undue advantages over other workers, thus violating Section extent by members of sects who were intolerant of other religious beliefs.
1 of Article III of the 1935 Constitution which forbids the denial to any The classification, introduced by Republic Act No. 3350, therefore, rests on
person of the equal protection of the laws. 50 substantial distinctions.

The guaranty of equal protection of the laws is not a guaranty of equality in The classification introduced by said Act is also germane to its purpose. The
the application of the laws upon all citizens of the state. It is not, therefore, a purpose of the law is precisely to avoid those who cannot, because of their
requirement, in order to avoid the constitutional prohibition against religious belief, join labor unions, from being deprived of their right to work
inequality, that every man, woman and child should be affected alike by a and from being dismissed from their work because of union shop security
statute. Equality of operation of statutes does not mean indiscriminate agreements.
operation on persons merely as such, but on persons according to the
circumstances surrounding them. It guarantees equality, not identity of rights. Republic Act No. 3350, furthermore, is not limited in its application to
The Constitution does not require that things which are different in fact be conditions existing at the time of its enactment. The law does not provide that
treated in law as though they were the same. The equal protection clause does it is to be effective for a certain period of time only. It is intended to apply
not forbid discrimination as to things that are different. 51 It does not prohibit for all times as long as the conditions to which the law is applicable exist. As
legislation which is limited either in the object to which it is directed or by long as there are closed shop agreements between an employer and a labor
the territory within which it is to operate. union, and there are employees who are prohibited by their religion from
affiliating with labor unions, their exemption from the coverage of said
The equal protection of the laws clause of the Constitution allows agreements continues.
classification. Classification in law, as in the other departments of knowledge
or practice, is the grouping of things in speculation or practice because they Finally, the Act applies equally to all members of said religious sects; this is
agree with one another in certain particulars. A law is not invalid because of evident from its provision. The fact that the law grants a privilege to members
simple inequality. 52 The very idea of classification is that of inequality, so of said religious sects cannot by itself render the Act unconstitutional, for as
that it goes without saying that the mere fact of inequality in no manner We have adverted to, the Act only restores to them their freedom of
determines the matter of constitutionality. 53 All that is required of a valid association which closed shop agreements have taken away, and puts them
classification is that it be reasonable, which means that the classification in the same plane as the other workers who are not prohibited by their religion
should be based on substantial distinctions which make for real differences; from joining labor unions. The circumstance, that the other employees,
that it must be germane to the purpose of the law; that it must not be limited because they are differently situated, are not granted the same privilege, does
to existing conditions only; and that it must apply equally to each member of not render the law unconstitutional, for every classification allowed by the
the class. 54 This Court has held that the standard is satisfied if the Constitution by its nature involves inequality.
classification or distinction is based on a reasonable foundation or rational
basis and is not palpably arbitrary. 55 The mere fact that the legislative classification may result in actual inequality
is not violative of the right to equal protection, for every classification of
persons or things for regulation by law produces inequality in some degree, and that the Union, by its act, inflicted intentional harm on Appellee; that
but the law is not thereby rendered invalid. A classification otherwise since Appellee was compelled to institute an action to protect his right to
reasonable does not offend the constitution simply because in practice it work, appellant could legally be ordered to pay attorney's fees under Articles
results in some inequality. 61 Anent this matter, it has been said that 1704 and 2208 of the Civil Code. 73
whenever it is apparent from the scope of the law that its object is for the
benefit of the public and the means by which the benefit is to be obtained are The second paragraph of Section 24 of Republic Act No. 875 which is relied
of public character, the law will be upheld even though incidental advantage upon by appellant provides that:
may occur to individuals beyond those enjoyed by the general public. 62
No suit, action or other proceedings shall be maintainable in any court against
6. Appellant's further contention that Republic Act No. 3350 violates the a labor organization or any officer or member thereof for any act done by or
constitutional provision on social justice is also baseless. Social justice is on behalf of such organization in furtherance of an industrial dispute to which
intended to promote the welfare of all the people. 63 Republic Act No. 3350 it is a party, on the ground only that such act induces some other person to
promotes that welfare insofar as it looks after the welfare of those who, break a contract of employment or that it is in restraint of trade or interferes
because of their religious belief, cannot join labor unions; the Act prevents with the trade, business or employment of some other person or with the right
their being deprived of work and of the means of livelihood. In determining of some other person to dispose of his capital or labor. (Emphasis supplied)
whether any particular measure is for public advantage, it is not necessary
that the entire state be directly benefited — it is sufficient that a portion of That there was a labor dispute in the instant case cannot be disputed for
the state be benefited thereby. appellant sought the discharge of respondent by virtue of the closed shop
agreement and under Section 2 (j) of Republic Act No. 875 a question
Social justice also means the adoption by the Government of measures involving tenure of employment is included in the term "labor dispute". 74
calculated to insure economic stability of all component elements of society, The discharge or the act of seeking it is the labor dispute itself. It being the
through the maintenance of a proper economic and social equilibrium in the labor dispute itself, that very same act of the Union in asking the employer
inter-relations of the members of the community. 64 Republic Act No. 3350 to dismiss Appellee cannot be "an act done ... in furtherance of an industrial
insures economic stability to the members of a religious sect, like the Iglesia dispute". The mere fact that appellant is a labor union does not necessarily
ni Cristo, who are also component elements of society, for it insures security mean that all its acts are in furtherance of an industrial dispute. 75 Appellant
in their employment, notwithstanding their failure to join a labor union Union, therefore, cannot invoke in its favor Section 24 of Republic Act No.
having a closed shop agreement with the employer. The Act also advances 875. This case is not intertwined with any unfair labor practice case existing
the proper economic and social equilibrium between labor unions and at the time when Appellee filed his complaint before the lower court.
employees who cannot join labor unions, for it exempts the latter from the
compelling necessity of joining labor unions that have closed shop Neither does Article 2208 of the Civil Code, invoked by the Union, serve as
agreements and equalizes, in so far as opportunity to work is concerned, those its shield. The article provides that attorney's fees and expenses of litigation
whose religion prohibits membership in labor unions with those whose may be awarded "when the defendant's act or omission has compelled the
religion does not prohibit said membership. Social justice does not imply plaintiff ... to incur expenses to protect his interest"; and "in any other case
social equality, because social inequality will always exist as long as social where the court deems it just and equitable that attorney's fees and expenses
relations depend on personal or subjective proclivities. Social justice does of litigation should be recovered". In the instant case, it cannot be gainsaid
not require legal equality because legal equality, being a relative term, is that appellant Union's act in demanding Appellee's dismissal caused
necessarily premised on differentiations based on personal or natural Appellee to incur expenses to prevent his being dismissed from his job. Costs
conditions. 65 Social justice guarantees equality of opportunity 66 , and this according to Section 1, Rule 142, of the Rules of Court, shall be allowed as
is precisely what Republic Act No. 3350 proposes to accomplish — it gives a matter of course to the prevailing party.
laborers, irrespective of their religious scrupples, equal opportunity for work.
WHEREFORE, the instant appeal is dismissed, and the decision, dated
7. As its last ground, appellant contends that the amendment introduced by August 26, 1965, of the Court of First Instance of Manila, in its Civil Case
Republic Act No. 3350 is not called for — in other words, the Act is not No. 58894, appealed from is affirmed, with costs against appellant Union. It
proper, necessary or desirable. Anent this matter, it has been held that a is so ordered.
statute which is not necessary is not, for that reason, unconstitutional; that in
determining the constitutional validity of legislation, the courts are
unconcerned with issues as to the necessity for the enactment of the
legislation in question. 67 Courts do inquire into the wisdom of laws. 68
Moreover, legislatures, being chosen by the people, are presumed to
understand and correctly appreciate the needs of the people, and it may
change the laws accordingly. 69 The fear is entertained by appellant that
unless the Act is declared unconstitutional, employers will prefer employing
members of religious sects that prohibit their members from joining labor
unions, and thus be a fatal blow to unionism. We do not agree. The threat to
unionism will depend on the number of employees who are members of the
religious sects that control the demands of the labor market. But there is
really no occasion now to go further and anticipate problems We cannot
judge with the material now before Us. At any rate, the validity of a statute
is to be determined from its general purpose and its efficacy to accomplish
the end desired, not from its effects on a particular case. 70 The essential
basis for the exercise of power, and not a mere incidental result arising from
its exertion, is the criterion by which the validity of a statute is to be
measured. 71

II. We now pass on the second assignment of error, in support of which the
Union argued that the decision of the trial court ordering the Union to pay
P500 for attorney's fees directly contravenes Section 24 of Republic Act No.
875, for the instant action involves an industrial dispute wherein the Union
was a party, and said Union merely acted in the exercise of its rights under
the union shop provision of its existing collective bargaining contract with
the Company; that said order also contravenes Article 2208 of the Civil Code;
that, furthermore, Appellee was never actually dismissed by the defendant
Company and did not therefore suffer any damage at all . 72

In refuting appellant Union's arguments, Appellee claimed that in the instant


case there was really no industrial dispute involved in the attempt to compel
Appellee to maintain its membership in the union under pain of dismissal,

You might also like